Você está na página 1de 265

SC Strategy

Sentence Correction

Note: You are looking for the BEST ANSWER and not the perfect answer. Pick the answer that is superior to all the other answer choices whether or not ou agree with the la out or diction. Sentences will not alwa s onl ha!e one error. The will "ostl ha!e two errors# so read all the answers carefull .

Three main areas of language proficiency: $. '. *. Grammar% in e!er sentence there is a gra""atical error that needs to &e resol!ed. Conciseness% Sentences "ust e(press the co"plete idea in a few words. This "a not alwa s "ean the shortest sentence. )t could &e the one that is long &ut detailed free of redundant words Correct diction% so"e +uestions will assess diction usage which is our choice of an appropriate word or words. Word "ust &e "eaningful to the conte(t

Basic Grammar

Noun% is a part of speech that na"es a person place or thing Common noun% na"es co""on things such as ,at# -o!ernor# .igh school /a &asic identifier. Proper Noun% Na"e specific people# place or things and are capitali0ed. E(. 1ido# -o!ernor 1edell 2ell Count nouns% these can &e counted e(a"ple# one pen# two dogs Non count Nouns% can not &e counted e(a"ple# gra!it # grass# wood / ou can not sa grass3s4 these are things that can &e "easured5 Collective nouns% these are singular ,ount Nouns that identif a -roup e(a"ple facult # choir# co""ittee# audience Pronoun% is a person place or thing that replaces a noun e(a"ple He, she, It, they, their. The word that a pronoun replaces is called an Antecedent.

Types of Pronouns>: $. '. *. 7. 8. Personal pronouns% these refer to a specific person or thing such as: she# it# the 6e"onstrati!e pronoun% The point to a noun that is near& in ti"e or space E(a"ple: This# That# These# Those E(a"ple: Who# Who"# Which# What E(a"ple: Who# Who"# Which# That )nterrogati!e Pronouns% this t pe of pronoun asks +uestions. Relati!e Pronouns% The relate. The connect a phrase to an antecedent. )ndefinite Pronouns% These refer to a person or thing that is identified# &ut not specificall . E(a"ple: E!er &od # few# each#

'

so"e&od # an thing 9. ;. Refle(i!e Pronouns% These pronouns reflect &ack onto the noun. E(a"ple: : self# itself# oursel!es. )ntensi!e Pronouns% These e"phasi0e the antecedent. The take the e(act sa"e for" as refle(i!e pronouns &ut the follow the noun "ore closel . E(a"ple: ) " self would prefer to eat now. /<ook at how close " self is to =)>5 i. The office "anager herself said???

Verbs% a word that shows action $. '. Sentences "ust hold at least one !er&. E(a"ple% ) went to eat @er&als: Ane t pe of !er&al is an infiniti!e which attaches the word =TA> to the !er&. E(a"ple% To eat# To drink. These are therefore nouns. Prepositions Prepositions Bwords used to link a noun or pronoun to other words Prepositions ne!er occur alone and are alwa s in prepositional phrases as the &egin with a preposition and end with a noun. The descri&e ti"e /at, by, during5 place / above, on, within5 and "o!e"ent/ to, towards5. d!ectives" A word that descri&es# or "odifies a noun or a pronoun. E(a"ple% The chopp water caused the s"all &oat to turn o!er. dverbs% words that "odifies a !er&. The end with a /y# or /ly# E(a"ple: Ca"ie ran +uite slowl up the field. Con!unction% a word which links words or phrases ,oordinating conDunction are used to Doin nouns# pronouns# !er&s# prepositional phrases and adDecti!es. T pes are B and, but, or, yet, for, nor, so. What the do is also Doin sentences. E(a"ple: ,oke or pepsi. Rock and roll. ,orrelati!e conDunction% E(a"ples /either$$or, Netiher$$nor, Not only$$but also, whether$$or, as$$as,not%but,both%and# ) can either take the &us or dri!e " car.

Parts of speech

Sub!ect and Predicate& Subject is the part of the sentence that perfor"s the Action of the !er& in that sentence. Predicate is the part of the sentence that contains the !er& as well as the o&Dects and the phrases controlled & the !er& ,o"pound su&Dect has two nouns or pronouns that perfor" the sa"e action in the sentence. E(a"ple: Elaina and 6ean ate food together

SC Strategy

'irect and (ndirect ob!ect The direct ob!ect is often confused for the sub!ect$ 'irect ob!ect% it is the noun or pronoun in the predicate that receives the action of the !er& or feels the effects of the !er&. E(a"ple: -rant/su&Dect5 hit the &all/direct o&Dect5 out of the park (ndirect ob!ect% )s the noun or pronoun in the predicate that is indirectl affected & the !er&. E(a"ple% :o" /su&Dect5 ga!e "e /indirect o&Dect5 " allowance/direct o&Dect5. Pg 7E.

Phrases& This is a group of words that do not contain a su&Dect and a !er&. )t has either or the other# &ut not &oth. Clause This contains &oth a su&Dect and a !er& (ndependent clause could act as a sentence. 'ependant clauses can not stand & the"sel!es.

Although he was acquitted of embezzlement charges, mike lost his job and his home

(Dependant clause)

(Independent clause)

)f a dependant clause appears in a sentence# it will alwa s ha!e an independent clause on which to lean.

)inding the main sub!ect and verb The Sub!ect is that noun or pronoun that perfor"s the action of the !er&. The &est approach to finding the "ain Su&Dect is to find the !er& first and see what is affected & it. *The main verb must be in the independent clause of the sentence, not in the dependant clause. The main subject is never in dependant clauses or phrases separated by a comma. It should be in the Independent clause. after taking a preparation course

E(a"ple: Although her grade point a!erage was falling# ,henn 3s SAT score rose *this is the independent clause#$

'o not rewrite the sentence on your note board or pad, as it is a waste of time$ Practice mentally eliminating unneeded phrases and clauses in order to find the main sub!ect$

(n order to locate an error in the sentence correction section, find the main sub!ect and verb and then test for agreement, tense, and form etc$

+rrors (nvolving V+,BS

Sub!ect and Verb agreement& The su&Dect and its !er& "ust agree with each other. Plural su&Dect with Plural @er&# and !ice !ersa. :an singular !er&s end in &-s. or -es..

The four types of sentence constructions to confuse you in this area & Phrase between the sub!ect and the verb% a long phrase is inserted &etween the "ain su&Dect and !er&. +/ample: The feline leuke"ia !irus /su&Dect5# characteri0ed & a loss of appetite# weight loss and poor cat conditions# are /!er&. )t should &e is5 ra"pant a"ong cats. The sub!ect follows the verb% :ost sentences in English language are arranged so that the "ain su&Dect co"es &efore the !er&. The -:AT will tr to "ess ou up & putting the !er& &efore the su&Dect. +/ample& There are /!er&5 "an reasons /su&Dect5 for the ta( increase. This is considered an e(pleti!e construction and the onl &egin with: / There is ,! It is ,! Here is .pg 64 This is an indicator% when ou see sentences &eginning with those words ou know the !er& is co"ing &efore the su&Dect and that is wrong gra""ar. Compound Sub!ect% These are plural and recei!e plural !er&s. E(a"ple: Erin and Kara sing together. ,o"pound su&Dects on the -:AT will often consist of one singular su&Dect and one plural su&Dect so that either for" of the !er& is plausi&le. 6A NAT 1A<< )N T.)S TRAP. Sentences with Each and Every in front of a co"pound su&Dect "ust ha!e a singular !er& &ecause each and e!er are singular$ +/ample: Each child# teenager# and adult was wearing a seat &elt on the &us. (ndefinite Pronouns as the sub!ect B A pronoun that does not refer to an one person or thing such as: So"eone# So"e&od # an one# e!er &od .

Singular

lural (more than one thing)

!oth ("ither singular or plural# depends on the word affected b$ the %erb

an$bod$ an$one

both few

all an$

SC Strategy
each either e%er$bod$ e%er$one neither nobod$ no one somebod$ someone man$ se%eral more most none some

(rregular verbs are an !er&s that do not add B ed to create the past for" of the !er& E(a"ple arose or had arisen# take or took# fl or flew. Alwa s choose the answer choice that uses the acti!e !oice while correcting gra""atical errors. Perfect tense: Fse of 0ad 1 the verb% there "ust &e two action /!er&s5 in a sentence# where one takes place &efore the other in order to use G0ad.3 E(a"ple: ) had walked to the store and &ack & the ti"e ou got off the phone. ,an not ha!e the use of .A6 twice in a sentence. Fse of 0ave or 0as H the !er&% shows so"ething that has happened in the past that "a continue. The choice &etween ha!e or has depends on the !er& for". E(a"ple: ) ha!e walked to worked for a "onth.

Shift in V+,B T+NS+ )n a sentence with two e!ents occurring at two different ti"es# it is i"perati!e to use two verb tenses to show the order in which the action takes place as is in sentences where two actions are taking place the sa"e ti"e.

)n sentences with two e!ents taking place# e!aluate whether the e!ents are si"ultaneous or separated & ti"e. Then check the !er&s to "ake sure that the tenses con!e accurac .

Sentences with -(). "ust ha!e the word -were. following it. The presence of the word )f should cause ou to look for a conditional !er&

+/ample: )f the &otanist was /is5 right# the plants# failure to produce &uds is caused & so"ething other than the place"ent of the seeds and the "oisture le!el of the soil.

+rrors with Nouns and Pronouns

Noun greement& Nouns "ust agree in nu"&er to the nouns the are referencing. Plural to plural and singular to singular. $. +/ample: ill and !ean &elie!ed that if the worked hard enough their drea" of &eco"ing a professional skater will &e a success. /wrong5. )t should &e skater/s5.

Pronouns Personal Pronouns% you, he, she, it, they. These are related to a particular person or thing. )ndefinite Pronouns% all, everyone, each, somebody and something. The do not refer to an particular thing or person Three specific pronoun errors: 2$ Pronoun and ntecedent agreement& Pronouns "ust agree in gender# nu"&er# and person with their antecedent. find a pronoun in a sentence ,elative Pronouns& These relate groups of words to another noun or pronoun. E(a"ples are% /who# who"# that# which# whoe!er# who"e!er5. A clause that &egins with a relati!e pronoun should &e placed as close to the noun it is "odif ing. When referring to people# use 304, 3045# and When referring to a thing use W.),. or T.AT. The "ost co""on error "ade is when a pronoun used to refer to a T.)N- is used to refer to a PERSAN and !ice !ersa. +/ample& The tea" who /should &e That5 ) follow Dust signed a "ulti"illion dollar contract with the &est ho"e run hitter. 2now when to use W.A /used when it is doing the action# or change it to .E in our "ind5 or W.A: /used when ha!ing the action done to it or change it to .): in our "ind5. i""ediatel identif the antecedent +/$ The "an lost his wallet. )f ou

6$

mbiguous and (mplied Pronouns& This is where the reader is left wondering who or what the pronoun is referring to. )t looks as though it

refers to "ore than one thing.

SC Strategy
E(a"ple B R an called Seth# he went to !isit 2atina. /Who does the .E refer to5. Pg $'7

5odifiers

The are recogni0ed & the use of a co""a. There are ad!er&ial "odifiers that "odif the verb and ad!ective "odifiers that "odif a noun or pronoun. 6angling "odifier: this is where the noun is not "entioned in the sentence structure and therefore has to &e placed in ne(t to the "odifier phrase. This usuall occur in the introductory phrase. +/ample& 6ri!ing to 1lorida# the dog needed to stop often to pee. )t should sa Gwe needed to stop often so the dog could pee3 after the co""a. :isplaced "odifier: This occurs in relative clauses that &egin with "hich or That This is where the "odifier is placed in the wrong area of the sentence and has to &e placed ne(t to the noun. E(a"ple: Yogi Rock# which is a rock on "ars# was na"ed after the good old doctor. Ad!er&ial "odifiers do not ha!e to &e positioned nearest the !er& it is "odif ing. )t sta s where it is. GNearl # Cust# Anl # Barle 3 B are so"e ad!er&ial "odif ing words.

7uantifiers&
Countable Words Non countable words (measure)

&umbers 'an$ )ew

Amount much(more *ittle(least

The number of 8is singular number of" is plural +rror.s involving con!unctions

Coordinating Con!unctions / nd, but, or, yet, for, no, so# Pa close attention to the use of conDunctions like: Neither.. nor# either.. or# not onl ? &ut also# not.. &ut# whether.. or# as? as# &oth.. and. The usage of these for"s "ake the su&Dects singular. Yet# -i!en that ou will ha!e "ore than one su&Dect# if one of the" is plural the wa to choose the right !er& for" is & looking at the su&Dect that co"es close to the end part of conDunctions lie either or. The part

closest to the -or. dictates the tense of the !er&. +/ample: Neither Coe nor his friends are / it should not &e is5 going to the &each. The reason for are is that friends is close to the !er& and it is plural.

Subordinating con!unction These con!unctions connect a dependant clause to an independent clause$ Subordination means we have made one of the sentences subordinate to the main clause$ The dependent clause can no longer stand on its own
after although as if as if as though because before e%en if e%en though how if now that pro%ided since so that than that though till unless until when whene%er where where%er while

Errors with this t pe of conDunction occur when an inappropriate conDunction is used. )t3s all a&out the relationship these words create &etween the clauses. E(a"ple: ) cried although /&ecause5 ) was sad /Wrong5.

+rrors in Construction Comparisons& Three t pes of co"parison errors are: Comparitive degree: This deals with the intensit con!e ed & the adDecti!es or ad!er&s. When ou speak a&out two things# use the Ger3 for" of the word. When ou are speaking a&out three or more things# use the Gest3 for" of the !er&

one object warm dark

Two Objects warmer draker

Three or more objects warmest darkest

SC Strategy
sunn$ wildl$ more sunn$ more wildl$ most sunn$ most wildl$

Fse :ore and :ost for ad!er&s. An descripti!e words that end in =Y> "ust ha!e :ARE or :AST &efore it. (ncomplete and (deological comparisons& Be sure to 9now the two ob!ect compared are ali9e$ :ou can not compare a truc9 to a house$ +/ample& ;i9e most des9s at wor9, Spence has his laden with Spence and the des9 or Spence.s des9s at wor9%%$ ;(<+ vs$ S ;i9e B is used to show si"ilarit and "ust &e followed & a noun. +/ample: ) /su&Dect5 look like " sister/noun5. s% is used to show an e(a"ple or introduce dependant clauses. +/ample& : dog is !er skittish# as ou "a e(pect a rescued ani"al to &e. pictures$ *what is compared here, des9 and other des9s#$ Correct answer is" Spence.s des9, li9e most

Comparison key words like unlike as++as greater than less than shorter than more than least

Parallel Structure& Patterns "ust &e !isi&le and "atching. The errors with parallel structure "a occur with parts of speech: !er&s# nouns and adDecti!es. Parralel #erbs% if ou encounter a -:AT sentence with two !er&s separated & a conDunction# check to ensure that the !er&s are parallel. +/ample& ) dro!e the entire trip# &ut &r an clai"ed to &e "ore tired.

$E

Parralel $ouns% Parallel Prepositions% Prepositions "ust &e used & either all "e"&ers# or & the first "e"&er of the series. +/ample& You can succeed on the -:AT & reading# & stud ing# and & taking a class. Parallel Conjunctions% /either. or# neither? nor5 Notice that each word pr phrase after the first conDunction "atches the for"at of the !er& or phrase after the second conDunction. E(a"ple: We accept either cash or "one order Parallel Comparisons% You "ust co"pare two things or "ore. E(a"ple: Ci" Thorpe# enDo ed pla ing foot&all more than he ran /should &e en!oyed running5 track and field# &ut it was winning the gold "edal that did hi" "uch pleasure.

Semicolons& *pg2=># This is used to Doin two closel related independent clauses. Re"e"&er an independent clause can stand alone as a sentence. +/ample& The night &efore the SAT# 2en sta ed up until "idnight4 he suspected this was the reason he did poorl on the test. Never use a semicolon with a dependant clause use a con!unction$ E(a"ple: & The night &efore the SAT# 2en sta ed up until "idnight# 30(C0 )S W.Y .E 6)6 PAAR<Y. (f you see a semicolon in a ?uestion on the G5 T, immediately chec9 that the clause on both sides of the semicolon are independent$ :ou can either fi/ the problem by finding an answer with a (ndependent clause, or use a subordinating con!unction +/ample: 6espite the fact that it has a duck shaped &ill and la s eggs# the uni+ue "a""al in Australia. nswers: bird@ rather Bird, but rather *Correct# Bird@ rather that of Bird@ it is that of )n this case none of the answer choices had a good se"icolon answer so the ne(t su&ordinating conDunction and there was onl one answer. step is to find one with a plat pus is not a &ird4 rather the "ost

(dioms *pg 2A2#

(f is used for "ultiple decisions# 3hether is used for onl two choices to &e "ade.

SC Strategy
The choice &etween A"ong and Between is alwa s tested. mong is for "ore that three things co"pared. Between is for two things co"pared

$$

5ore than doubled is preferred to 5ore than twice$ Than by is preferred to instead of ,ather than is preferred to But not Credited with is preferred to Credited as Twice as is preferred to double as Because is preferred to due to the fact 5ore than doubled is preferred to (ncreased by more than twice$ ,ather than is preferred to (nstead of Probably is preferred to 5ay be Because is preferred to 4n account ,ising costs is preferred to ,aising cost Note& See other idioms in the 5anhattan boo9

+rrors (nvolving Style

3ordy ;anguage& You will encounter word sentences in which ou "ust choose a concise correction. Two t pes of wordiness to a!oid are 34,': +BP,+SS(4NS and ,edundant +BP,+SS(4NS$ a$ 34,': +BP,+SS(4NS
Wordy E pressions after the conclusion of at this point in time despite the fact that e,cessi%e number of in order to Concise Correction after now although because of to

$'

in the e%ent that is in a position to regardless of the fact that

if can although

b$ ,+'CN' NT +BP,+SS(4NS This is where unnecessary repetition detracts from a sentence$ void these below&
ad%ance planning all $ear round annuall$ each $ear biograph$ of his life close pro,imit$ customar$ habit end result essential requirement e,actl$ identical forward progress free gift group together honest trust joint cooperation meet together necessar$ requirement new breakthrough past histor$ postpone until later reduced down repeat again re%erse back rising increase sharing the same temporar$ loan usual habit -ealth$ millionaire+

1.

A calendar stick carved centuries ago by the Winnebago tribe may provide the first evidence that the North American Indians have developed advanced full-year calendars basing them on systematic astronomical observation. A! that the North American Indians have developed advanced full-year calendars basing them "! of the North American Indians #ho have developed advanced full-year calendars and based them $! of the development of advanced full-year calendars by North American Indians% basing them &! of the North American Indians and their development of advanced full-year calendars based '! that the North American Indians developed advanced full-year calendars based

(.

A 1)*( agreement bet#een $anada and the +nited ,tates reduced the amount of phosphates that

SC Strategy

$*

municipalities had been allo#ed to dump into the -reat .akes. A! reduced the amount of phosphates that municipalities had been allo#ed to dump "! reduced the phosphate amount that municipalities had been dumping $! reduces the phosphate amount municipalities have been allo#ed to dump &! reduced the amount of phosphates that municipalities are allo#ed to dump '! reduces the amount of phosphates allo#ed for dumping by municipalities /. A collection of /0 poems by 1hillis Wheatley% a slave% #as published in the 1**23s% the first book by a "lack #oman and it #as only the second published by an American #oman. A! it #as only the second published by an American #oman "! it #as only the second that an American #oman published $! the second one only published by an American #oman &! the second one only that an American #oman published '! only the second published by an American #oman 4. A common disability in test pilots is hearing impairment% a conse5uence of sitting too close to large 6et engines for long periods of time. A! a conse5uence of sitting too close to large 6et engines for long periods of time "! a conse5uence from sitting for long periods of time too near to large 6et engines $! a conse5uence #hich resulted from sitting too close to large 6et engines for long periods of time &! damaged from sitting too near to large 6et engines for long periods of time '! damaged because they sat too close to large 6et engines for long periods of time 7. A controversial figure throughout most of his public life% the "lack leader 8arcus -arvey advocated that some "lacks return to Africa% the land that% to him% symboli9ed the possibility of freedom. A! that some "lacks return to Africa% the land that% to him% symboli9ed the possibility of freedom "! that some "lacks return to the African land symboli9ing the possibility of freedom to him $! that some "lacks return to Africa #hich #as the land #hich symboli9ed the possibility of freedom to him &! some "lack3s returning to Africa #hich #as the land that to him symboli9ed the possibility of freedom '! some "lack3s return to the land symboli9ing the possibility of freedom to him% Africa :. A fire in an enclosed space burns #ith the aid of reflected radiation that preheats the fuel% making ignition much easier and flames spreading more 5uickly. A! flames spreading "! flame spreads

$7

$! flames are caused to spread &! causing flames to spread '! causing spreading of the flames

*.

A firm that speciali9es in the analysis of hand#riting claims from a one-page #riting sample that it can assess more than three hundred personality traits% including enthusiasm% imagination% and ambition. A! from a one-page #riting sample that it can assess "! from a one-page #riting sample it has the ability of assessing $! the ability% from a one-page #riting sample% of assessing &! to be able% from a one-page #riting sample% to assess '! being able to assess% from a one-page #riting sample%

0.

A huge flying reptile that died out #ith the dinosaurs some :7 million years ago% the ;uet9alcoatlus had a #ingspan of /: feet% believed to be the largest flying creature the #orld has ever seen. A! believed to be "! and that is believed to be $! and it is believed to have been &! #hich #as% it is believed% '! #hich is believed to be

).

A .abor &epartment study states that the numbers of #omen employed outside the home gre# by more than a thirty-five percent increase in the past decade and accounted for more than si<ty-t#o percent of the total gro#th in the civilian #ork force. A! numbers of #omen employed outside the home gre# by more than a thirty-five percent increase "! numbers of #omen employed outside the home gre# more than thirty-five percent $! numbers of #omen employed outside the home #ere raised by more than thirty-five percent &! number of #omen employed outside the home increased by more than thirty-five percent '! number of #omen employed outside the home #as raised by more than a thirty-five percent increase

12. A large rise in the number of housing starts in the coming year should boost ne# construction dollars by several billion dollars% making the construction industry3s economic health much more robust than five years ago. A! making the construction industry3s economic health much more robust than five years ago

SC Strategy

$8

"! and make the construction industry3s economic health much more robust than five years ago $! making the construction industry3s economic health much more robust than it #as five years ago &! to make the construction industry3s economic health much more robust than five years ago '! in making the construction industry3s economic health much more robust than it as five years ago 11. A letter by 8ark =#ain% #ritten in the same year as =he Adventures of >uckleberry ?inn #ere published% reveals that =#ain provided financial assistance to one of the first "lack students at @ale .a# ,chool. A! A letter by 8ark =#ain% #ritten in the same year as =he Adventures of >uckleberry ?inn #ere published% "! A letter by 8ark =#ain% #ritten in the same year of publication as =he Adventures of >uckleberry ?inn% $! A letter by 8ark =#ain% #ritten in the same year that =he Adventures of >uckleberry ?inn #as published% &! 8ark =#ain #rote a letter in the same year as he published =he Adventures of >uckleberry ?inn that '! 8ark =#ain #rote a letter in the same year of publication as =he Adventures of >uckleberry ?inn that 1(. A little under a million years ago% the briny #aters of the "altic ,ea began flooding into the cold North AtlanticA geologists are still debating #hether the flood #as gradual or created a cataclysm. A! #hether the flood #as gradual or created a cataclysm "! if the flood #as gradual or created a cataclysm $! about #hether the flood #as gradual or cataclysmic &! #hether the flood #as gradual or cataclysmic '! #hether the flood #as gradual or it created a cataclysm 1/. A ma6ority of the international 6ournalists surveyed vie# nuclear po#er stations as unsafe at present but that they #ill% or could% be made sufficiently safe in the future. A! that they #ill% or could% "! that they #ould% or could% $! they #ill be or could &! think that they #ill be or could '! think the po#er stations #ould or could 14. A migraine typically afflicts one side of the head% lasts for hours or days% and may recur as infre5uently as once every other month or often% as daily. A! as infre5uently as once every other month or often% as

$9

"! as infre5uently as once every other month or as often as $! infre5uently% as often as once every other month% or often% like &! infre5uently% like once every other month% or often% like '! infre5uently% like once every other month% or as often as 17. A ne# phenomena% #hich is visible at 8anagua3s ma6or intersections% are #aves of vendors and beggars% #hich include many children and mob cars at the stoplights. A! A ne# phenomena% #hich is visible at 8anagua3s ma6or intersections% are #aves of vendors and beggars% #hich include many children and "! Bisible at 8anagua3s ma6or intersections are #aves of vendors and beggars #ith many children% ne# phenomena that $! A ne# phenomenon visible at 8anagua3s ma6or intersections is #aves of vendors and beggars% many of them children% #ho &! 1henomenally ne# #aves of vendors% beggars% and many children are visible at 8anagua3s ma6or intersections% #hich '! A #ave of vendors and beggars% many of #hom are children% are visible at 8anagua3s ma6or intersections% #here they are a ne# phenomenon and 1:. A number of linguists contend that all of the thousands of languages spoken by the #orld3s five billion people can be traced back to a common root language. A! that all of the thousands of languages spoken by the #orld3s five billion people can be traced "! that the #orld3s five billion people speak thousands of languages of #hich all can be traced $! the #orld3s five billion people speak thousands of languages #hich are all traceable &! all of the thousands of languages spoken by the #orld3s five billion people to be traceable '! the ability to trace all of the thousands of languages that are spoken by the #orld3s five billion people 1*. A patient accusing a doctor of malpractice #ill find it difficult to prove damage if there is a lack of some other doctor to testify about proper medical procedures. A! if there is a lack of some other doctor to testify "! unless there #ill be another doctor to testify $! #ithout another doctor3s testimony &! should there be no testimony from some other doctor '! lacking another doctor to testify 10. A peculiar feature of the embryonic mammalian circulatory system is that in the area of the heart the cells adhere to one another% beating in unison and adopting speciali9ed orientations e<clusive of one another. A! beating in unison and adopting "! they beat in unison #hile adopting

SC Strategy

$;

$! beat in unison% and adopt &! beating in unison yet adopting '! even though they beat in unison and adopt 1). A 1resident entering the final t#o years of a second term is likely to be at a severe disadvantage and is often unable to carry out a legislative program. A! likely to be at a severe disadvantage and is often unable to "! likely severely disadvantaged and often unable to $! liable to be severely disadvantaged and cannot often &! liable that he or she is at a severe disadvantage and cannot often '! at a severe disadvantage% often likely to be unable that he or she can (2. A prolific architect #ho #orked from the turn of the century until the late 1)723s% Culia 8organ designed nearly 022 buildings in $alifornia% perhaps most notably William Dandolph >earst3s monumental estate at ,an ,imeon. A! Culia 8organ designed nearly 022 buildings in $alifornia% perhaps most notably William Dandolph >earst3s monumental estate at ,an ,imeon "! perhaps the most notable of the nearly 022 buildings in $alifornia designed by Culia 8organ #as William Dandolph >earst3s monumental estate at ,an ,imeon $! of the nearly 022 buildings in $alifornia designed by Culia 8organ% perhaps the most notable #as William Dandolph >earst3s monumental estate at ,an ,imeon &! nearly 022 buildings in $alifornia #ere designed by Culia 8organ% of #hich William Dandolph >earst3s monumental estate at ,an ,imeon is perhaps the most notable '! William Dandolph >earst3s monumental estate at ,an ,imeon is perhaps the most notable of the nearly 022 buildings in $alifornia designed by Culia 8organ (1. A proposal has been made to trim the horns from rhinoceroses to discourage poachersE the 5uestion is #hether tourists #ill continue to visit game parks and see rhinoceroses after their horns are trimmed. A! #hether tourists #ill continue to visit game parks and see rhinoceroses after their horns are "! #hether tourists #ill continue to visit game parks to see one once their horns are $! #hether tourists #ill continue to visit game parks to see rhinoceroses once the animals3 horns have been &! if tourists #ill continue to visit game parks and see rhinoceroses once the animals3 horns are '! if tourists #ill continue to visit game parks to see one after the animals3 horns have been ((. A recent national study of the public schools sho#s that there are no# one microcomputer for every thirty-t#o pupils% four times as many than there #ere four years ago. A! there are no# one microcomputer for every thirty-t#o pupils% four times as many than there #ere

$I

"! there is no# one microcomputer for every thirty-t#o pupils% four times as many than there #ere $! there is no# one microcomputer for every thirty-t#o pupils% four times as many as there #ere &! every thirty-t#o pupils no# have one microcomputer% four times as many than there #ere '! every thirty-t#o pupils no# has one microcomputer% four times as many as (/. A recent Ne# @ork =imes editorial critici9ed the city3s election board for% first of all% failing to replace outmoded voting machines prone to breakdo#ns% and secondarily% for their failure to investigate allegations of corruption involving board members. A! secondarily% for their failure to "! secondly% for their failure to $! secondly% that they failed and did not &! second% that they failed to '! second% for failing to (4. A recent study has found that #ithin the past fe# years% many doctors had elected early retirement rather than face the threats of la#suits and the rising costs of malpractice insurance. A! had elected early retirement rather than face "! had elected early retirement instead of facing $! have elected retiring early instead of facing &! have elected to retire early rather than facing '! have elected to retire early rather than face (7. A recent study of ancient clay deposits has provided ne# evidence supporting the theory of global forest fires ignited by a meteorite impact that contributed to the e<tinction of the dinosaurs and many other creatures some :7 million years ago. A! supporting the theory of global forest fires ignited by a meteorite impact that "! supporting the theory that global forest fires ignited by a meteorite impact $! that supports the theory of global forest fires that #ere ignited by a meteorite impact and that &! in support of the theory that global forest fires #ere ignited by a meteorite impact and that '! of support for the theory of a meteorite impact that ignited global forest fires and (:. A recording system #as so secretly installed and operated in the Fennedy Gval Gffice that even =heodore $. ,orensen% the White >ouse counsel% did not kno# it e<isted. A! A recording system #as so secretly installed and operated in the Fennedy Gval Gffice that "! ,o secret #as a recording system installation and operation in the Fennedy Gval Gffice $! It #as so secret that a recording system #as installed and operated in the Fennedy Gval Gffice &! A recording system that #as so secretly installed and operated in the Fennedy Gval Gffice '! Installed and operated so secretly in the Fennedy Gval Gffice #as a recording system that

SC Strategy

$J

(*. A report by the American Academy for the Advancement of ,cience has concluded that much of the currently uncontrolled dio<ins to #hich North Americans are e<posed comes from the incineration of #astes. A! much of the currently uncontrolled dio<ins to #hich North Americans are e<posed comes "! much of the currently uncontrolled dio<ins that North Americans are e<posed to come $! much of the dio<ins that are currently uncontrolled and that North Americans are e<posed to comes &! many of the dio<ins that are currently uncontrolled and North Americans are e<posed to come '! many of the currently uncontrolled dio<ins to #hich North Americans are e<posed come (0. A representative of the Women3s "ureau of the +nited ,tates &epartment of .abor contends that employers #ho offer benefits #hich permit that employees can balance home and #ork responsibilities better% reali9ing gains in attendance% recruiting% and retention. A! #hich permit that employees can balance home and #ork responsibilities better% reali9ing "! #hich permit employees balancing home and #ork responsibilities better #ill reali9e $! that permit employees to balance the responsibilities of home and #ork better #ill reali9e &! that permit employees a better balance bet#een the responsibilities of home and #ork% thus reali9ing '! such that employees are permitted a balance bet#een home and #ork responsibilities% and they #ill reali9e (). A shy% religious-minded publisher #ho had married a duke3s daughter% >arold 8acmillan3s rise to the position of 1rime 8inister in 1)7* surprised many% though $hurchill had since the 1)/2s been e<tolling 8acmillan3s courage. A! >arold 8acmillan3s rise to the position of 1rime 8inister in 1)7* surprised many "! >arold 8acmillan3s rise in 1)7* to the position of 1rime 8inister surprised many $! >arold 8acmillan3s becoming 1rime 8inister in 1)7* surprised many &! >arold 8acmillan surprised many by rising to the position of 1rime 8inister in 1)7* '! the position of 1rime 8inister attained by >arold 8acmillan in 1)7* surprised many /2. A special Capanese green tea called genmai-cha contains bro#n rice and is considered as a delicacy fit for a gourmet by most Capanese% though it is virtually unavailable outside @okohama. A! A special Capanese green tea called genmai-cha contains bro#n rice and is considered as a delicacy fit for a gourmet by most Capanese% though it is virtually unavailable outside @okohama. "! $onsidered to be a delicacy fit for a gourmet by most Capanese% genmai-cha is a special green tea that contains bro#n rice% virtually unavailable outside @okohama. $! A special Capanese green tea called genmai-cha contains bro#n rice and is considered a gourmet delicacy by most Capanese% though it is virtually unavailable outside @okohama.

'E

&! 8ost Capanese consider genmai-cha% a special green tea #hich contains bro#n rice% as a delicacy virtually unavailable outside @okohama. '! =hough virtually unavailable outside @okohama% most Capanese consider genmai-cha% a special green tea that contains bro#n rice% a gourmet delicacy. /1. A star #ill compress itself into a #hite d#arf% a neutron star% or a black hole after it passes through a red giant stage% depending on mass. A! A star #ill compress itself into a #hite d#arf% a neutron star% or a black hole after it passes through a red giant stage% depending on mass. "! After passing through a red giant stage% depending on its mass% a star #ill compress itself into a #hite d#arf% a neutron star% or a black hole. $! After passing through a red giant stage% a star3s mass #ill determine if it compresses itself into a #hite d#arf% a neutron star% or a black hole. &! 8ass determines #hether a star% after passing through the red giant stage% #ill compress itself into a #hite d#arf% a neutron star% or a black hole. '! =he mass of a star% after passing through the red giant stage% #ill determine #hether it compresses itself into a #hite d#arf% a neutron star% or a black hole. /(. A study commissioned by the &epartment of Agriculture sho#ed that if calves e<ercise and associated #ith other calves% they #ill re5uire less medication and gain #eight 5uicker than do those raised in confinement. A! associated #ith other calves% they #ill re5uire less medication and gain #eight 5uicker than do "! associated #ith other calves% they re5uire less medication and gain #eight 5uicker than $! associate #ith other calves% they re5uired less medication and #ill gain #eight 5uicker than do &! associate #ith other calves% they have re5uired less medication and #ill gain #eight more 5uickly than do '! associate #ith other calves% they re5uire less medication and gain #eight more 5uickly than //. A substance derived from the 8adagascar peri#inkle% #hich has proved useful in decreasing mortality among young leukemia patients% is cultivated in $hina as part of a program to integrate traditional herbal medicine into a contemporary system of health care. A! A substance derived from the 8adagascar peri#inkle% #hich has proved useful in decreasing mortality among young leukemia patients% "! A derivative% #hich has proved useful in decreasing mortality among young leukemia patients% of the 8adagascar peri#inkle% $! A 8adagascar peri#inkle derivative% #hich has proved useful in decreasing mortality among young leukemia patients% &! =he 8adagascar peri#inkle has a derivative #hich has proved useful in decreasing mortality among young leukemia patients% that '! =he 8adagascar peri#inkle% a derivative of #hich has proved useful in decreasing mortality

SC Strategy

'$

among young leukemia patients% /4. A #ildlife e<pert predicts that the reintroduction of the caribou into northern 8innesota #ould fail if the density of the timber #olf population in that region is more numerous than one #olf for every /) s5uare miles. A! #ould fail if the density of the timber #olf population in that region is more numerous than "! #ould fail provided the density of the timber #olf population in that region is more than $! should fail if the timber #olf density in that region #as greater than &! #ill fail if the density of the timber #olf population in that region is greater than '! #ill fail if the timber #olf density in that region #ere more numerous than /7. According to a panel of health officials% there has been a great deal of confusion in the medical profession about #hether obesity is a biological disorder posing serious health risks or a condition more related to appearance than to health. A! about #hether obesity is a biological disorder posing serious health risks or a condition more related to appearance than to "! #ith respect to obesity being a biological disorder posing serious health risks or if it is related more to appearance than $! over #hether or not obesity is a biological disorder posing serious health risks or it is a condition more related to appearance than to &! about obesity and if it is a biological disorder posing serious health risks or a condition related to appearance more than to '! concerning #hether obesity is a biological disorder posing serious health risks or it is a condition related to appearance more than /:. According to a recent poll% o#ning and living in a freestanding house on its o#n land is still a goal of a ma6ority of young adults% like that of earlier generations. A! like that of earlier generations "! as that for earlier generations $! 6ust as earlier generations did &! as have earlier generations '! as it #as of earlier generations /*. According to a recent study by Dutgers +niversity% the number of #omen in state legislatures has gro#n in every election since 1):0. A! the number of #omen in state legislatures has gro#n "! the number of #omen #ho are in state legislatures have gro#n $! there has been gro#th in the number of #omen in state legislatures &! a gro#ing number of #omen have been in state legislatures '! #omen have been gro#ing in number in state legislatures

''

/0. According to a recent study% the elderly in the +nited ,tates are four times more likely to give regular financial aid to their children as to receive it from them. A! the elderly in the +nited ,tates are four times more likely to give regular financial aid to their children as "! the elderly in the +nited ,tates are four times as likely to give regular financial aid to their children as it is for them $! the elderly in the +nited ,tates are four times more likely to give regular financial aid to their children than &! it is four times more likely for the elderly in the +nited ,tates to give regular financial aid to their children than they are '! it is four times as likely that the elderly in the +nited ,tates #ill give their children regular financial aid as they are /). According to a ruling by the state supreme court% the o#ner of polluted land is liable for the cleanup of the property even if the o#ner did not have the responsibility that pollution occurred before the title changed hands. A! the o#ner did not have the responsibility that pollution "! the o#ner is not responsible for pollution that $! it #as not the o#ner3s responsibility that pollution #ould have &! the responsibility of the o#ner is not that pollution '! the responsibility #as not the o#ner3s that pollution #ould have 42. According to a study by the $arnegie ?oundation for the Advancement of =eaching% companies in the +nited ,tates are providing 6ob training and general education for nearly eight million people% about e5uivalent to the enrollment of the nation3s four-year colleges and universities. A! e5uivalent to the enrollment of "! the e5uivalent of those enrolled in $! e5ual to those #ho are enrolled in &! as many as the enrollment of '! as many as are enrolled in 41. According to a study published by &r. 8yrna Weissman% only one percent of Americans born before 1)27 had suffered ma6or depression by the age of seventy-fiveE of those born since 1)77% si< percent had become depressed by age t#enty-four. A! only one percent of Americans born before 1)27 had suffered ma6or depression by the age of seventy-fiveE of those born since 1)77% si< percent had become depressed by age t#enty-four "! only one percent of Americans born before 1)27 suffer ma6or depression by the age of seventy-fiveE if they are born since 1)77% si< percent become depressed by age t#enty-four $! of Americans born before 1)27% only one percent of them have suffered ma6or depression by

SC Strategy

'*

age seventy-five% but si< percent of those born since 1)77 do by the age of t#enty-four &! ma6or depression is suffered by the age of seventy-five by only one percent of Americans born before 1)27% and by age t#enty-four by the si< percent born since 1)77 '! Americans born before 1)27 suffer ma6or depression by the age of seventy-five only one percent of the time% but si< percent of those born since 1)77 did so by age t#enty-four 4(. According to a survey of graduating medical students conducted by the Association of American 8edical $olleges% minority graduates are nearly four times more likely than are other graduates in planning to practice in socioeconomically deprived areas. A! minority graduates are nearly four times more likely than are other graduates in planning to practice "! minority graduates are nearly four times more likely than other graduates #ho plan on practicing $! minority graduates are nearly four times as likely as other graduates to plan on practicing &! it is nearly four times more likely that minority graduates rather than other graduates #ill plan to practice '! it is nearly four times as likely for minority graduates than other graduates to plan to practice 4/. According to "ooker =. Whatley3s recent analysis% planting the same crops as are planted on large farms #ill lead to economic disaster for the small farmer% #ho should plan a succession of highvalue crops that #ill provide a year-round cash flo#. A! planting the same crops as are planted on large farms #ill lead to economic disaster for the small farmer% #ho "! it #ill lead to economic disaster for the small farmer to plant the same crops as on the large farmsE they $! economic disaster #ill result from planting the same crops as large farms to the small farmer% #ho &! economic disaster for the small farmer #ill result from planting the same crops as on the large farmsE they '! the small farmer planting the same crops as are planted on large farms #ill lead to economic disasterE they 44. According to >enry &avid =horeau% the reason a ma6ority is allo#ed to rule is not that it is more likely to be right% but because it is stronger. A! the reason a ma6ority is allo#ed to rule is not that it is more likely to be right% but because it is stronger "! a ma6ority is allo#ed to rule not because it is more likely to be right% but because it is stronger $! the reason for ma6ority rule is not because they are more likely to be right% they are stronger &! the ma6ority is allo#ed to rule because of its strength% not because it is more likely to be right '! the reason #hy the ma6ority rules is that it is strong% not because it is likely to be right

'7

47. According to his o#n account% ?rederic-Auguste "artholdi% the sculptor of the ,tatue of .iberty% modeled the face of the statue like his mother3s and the body like his #ife3s. A! modeled the face of the statue like his mother3s and the body like his #ife3s "! modeled the face of the statue after that of his mother and the body after that of his #ife $! modeled the face of the statue like his mother and the body like his #ife &! made the face of the statue after his mother and the body after his #ife '! made the face of the statue look like his mother and the body look like his #ife 4:. According to Interstudy% a nonprofit organi9ation that studies health maintenance organi9ations >8G3s!% they estimate that% in comparison to last year% #hen only /: percent of the nation3s :2* >8G3s #as profitable% this year */ percent #ill be. A! they estimate that% in comparison to last year% #hen only /: percent of the nation3s :2* >8G3s #as profitable% this year */ percent #ill be "! compared to only /: percent of the nation3s :2* >8G3s being profitable last year% they estimate */ percent #ould be this year $! only /: percent of the nation3s :2* >8G3s #ere profitable last yearE it estimates that this year */ percent #ill be &! it estimates */ percent of the nation3s :2* >8G3s #ould be profitable this yearE last year that #as only /: percent '! only /: percent of the nation3s :2* >8G3s last year #ere profitable% #hereas they estimate it this year to be */ percent 4*. According to scientists at the +niversity of $alifornia% the pattern of changes that have occurred in human &NA over the millennia indicate the possibility that everyone alive today might be descended from a single female ancestor #ho lived in Africa sometime bet#een 142%222 and (02%222 years ago. A! indicate the possibility that everyone alive today might be descended from a single female ancestor #ho "! indicate that everyone alive today might possibly be a descendant of a single female ancestor #ho had $! may indicate that everyone alive today has descended from a single female ancestor #ho had &! indicates that everyone alive today may be a descendant of a single female ancestor #ho '! indicates that everyone alive today might be a descendant from a single female ancestor #ho 40. According to some analysts% #hatever its merits% the proposal to ta< a#ay all capital gains on short-term investments #ould% if enacted% have a disastrous effect on Wall ,treet trading and employment. A! its merits% the proposal to ta< "! its merits may be% the proposal of ta<ing

SC Strategy

'8

$! its merits as a proposal% ta<ing &! the proposal3s merits% to ta< '! the proposal3s merits are% ta<ing 4). According to some economists% Capan is in danger of plunging into a depression that% #ith doubledigit unemployment% could severely strain a society that regards lifetime employment as a virtual right of citi9enship. A! that% #ith double-digit unemployment% could severely strain "! that% because of double-digit unemployment% could be a severe strain for $! #ith double-digit unemployment% and it could severely strain &! #ith double-digit unemployment and could be a severe strain '! #ith double-digit unemployment and could severely strain 72. According to some economists% the Culy decrease in unemployment so that it #as the lo#est in t#o years suggests that the gradual improvement in the 6ob market is continuing. A! so that it #as the lo#est in t#o years "! so that it #as the lo#est t#o-year rate $! to #hat #ould be the lo#est in t#o years &! to a t#o-year lo# level '! to the lo#est level in t#o years 71. According to surveys by the National Institute on &rug Abuse% about (2 percent of young adults used cocaine in 1)*)% doubling those reported in the 1)** survey. A! doubling those reported in the 1)** survey "! to double the number the 1)** survey reported $! t#ice those the 1)** survey reported &! t#ice as much as those reported in the 1)** survey '! t#ice the number reported in the 1)** survey 7(. According to the "etter "usiness "ureau% if you fail to advertise the highest price in a range of prices for a service or product as prominently as that of the lo#est% it violates the Ne# @ork $onsumer 1rotection .a#. A! if you fail to advertise the highest price in a range of prices for a service or product as prominently as that of the lo#est% it "! if one fails to advertise the highest price in a range of prices for a service or product as prominently as the lo#est price% it $! failure to advertise the highest price in a range of prices for a service or product as prominently as the lo#est &! failure to advertise as prominently the highest price in a range of prices for a service or

'9

product as the lo#est '! failing to advertise as prominently the highest price in a range of prices for a service or products as that of the lo#est 7/. According to the National 1asta Association% per-capita consumption of pasta in the +nited ,tates% #hich has already been approaching 1) pounds a year% #ill achieve /2 pounds a year by the t#enty-first century. A! According to the National 1asta Association% per-capita consumption of pasta in the +nited ,tates% #hich has already been approaching 1) pounds a year% #ill achieve /2 pounds a year by the t#enty-first century. "! Already approaching 1) pounds a year in the +nited ,tates% the National 1asta Association predicts that per-capita consumption of pasta #ill reach /2 pounds a year by the t#enty-first century. $! =he National 1asta association predicts by the t#enty-first century that per-capita consumption of pasta in the +nited ,tates% #hich is already approaching 1) pounds a year% #ill reach /2 pounds a year. &! "y the t#enty-first century% the National 1asta Association predicts that per-capita consumption of pasta in the +nited ,tates% having already approached 1) pounds a year% #ill reach /2 pounds a year. '! According to the National 1asta Association% per-capita consumption of pasta in the +nited ,tates is already approaching 1) pounds a year and #ill reach /2 pounds a year by the t#enty-first century. 74. According to the professor3s philosophy% the antidote to envy is one3s o#n #ork% al#ays one3s o#n #orkA not thinking about it% not assessing it% but simply doing it. A! one3s o#n #ork% al#ays one3s o#n #orkA not thinking about it% not assessing it% but simply doing it "! al#ays #orkE because you don3t think about it or assess it% you 6ust do it $! al#ays one3s o#n #orkA not thinking about or assessing it% but simply to do it &! not to think or assess% but doing one3s o#n #ork '! neither to think about one3s o#n #ork nor to assess it% it is al#ays simply doing it 77. According to +nited ,tates Air ?orce officials% a cannon shooting dead chickens at airplanes has proved helpful to demonstrate #hat kind of damage can result #hen 6ets fly into a flock of large birds. A! shooting dead chickens at airplanes has proved helpful to demonstrate "! shooting dead chickens at airplanes has proved itself helpful as a demonstration of $! shooting dead chickens at airplanes proves itself helpful as demonstrating &! that shoots dead chickens at airplanes proves itself helpful to demonstrate '! that shoots dead chickens at airplanes has proved helpful in demonstrating

SC Strategy

';

7:. Acid rain and sno# result from the chemical reactions bet#een industrial emissions of sulfur dio<ide and nitrogen o<ides #ith atmospheric #ater vapor to produce highly corrosive sulfuric and nitric acids. A! #ith atmospheric #ater vapor to produce highly corrosive sulfuric and nitric acids "! #ith atmospheric #ater vapor producing highly corrosive sulfuric and nitric acids $! and atmospheric #ater vapor #hich has produced highly corrosive sulfuric and nitric acids &! and atmospheric #ater vapor #hich have produced sulfuric and nitric acids #hich are highly corrosive '! and atmospheric #ater vapor to produce highly corrosive sulfuric and nitric acids 7*. Added to the increase in hourly #ages re5uested last Culy% the railroad employees are no# seeking an e<panded program of retirement benefits. A! Added to the increase in hourly #ages re5uested last Culy% the railroad employees are no# seeking an e<panded program of retirement benefits. "! Added to the increase in hourly #ages #hich had been re5uested last Culy% the employees of the railroad are no# seeking an e<panded program of retirement benefits. $! =he railroad employees are no# seeking an e<panded program of retirement benefits added to the increase in hourly #ages that #ere re5uested last Culy. &! In addition to the increase in hourly #ages that #ere re5uested last Culy% the railroad employees are no# seeking an e<panded program of retirement benefits. '! In addition to the increase in hourly #ages re5uested last Culy% the employees of the railroad are no# seeking an e<panded program of retirement benefits. 70. Adult survivors of child abuse traditionally have had little or no chance that they could get their symptoms recogni9ed and treated. A! that they could get their symptoms recogni9ed and treated "! to recogni9e and treat their symptoms $! of getting their symptoms recogni9ed and treated &! of recogni9ing and treating symptoms '! of getting his or her symptoms recogni9ed and treated 7). Affording strategic pro<imity to the ,trait of -ibraltar% 8orocco #as also of interest to the ?rench throughout the first half of the t#entieth century because they assumed that if they did not hold it% their grip on Algeria #as al#ays insecure. A! if they did not hold it% their grip on Algeria #as al#ays insecure "! #ithout it their grip on Algeria #ould never be secure $! their grip on Algeria #as not ever secure if they did not hold it &! #ithout that% they could never be secure about their grip on Algeria '! never #ould their grip on Algeria be secure if they did not hold it

'I

:2. After a fe# #eeks3 e<perience% apprentice 6e#elers can usually begin to discriminate% though not #ith absolute certainty% genuine diamonds from imitation diamonds. A! genuine diamonds from imitation diamonds "! genuine diamonds apart from imitations $! bet#een genuine diamonds and imitation diamonds &! among genuine diamonds and imitation diamonds '! #hether diamonds are imitation or genuine :1. After crude oil% natural gas is the +nited ,tates second biggest fuel source and supplied almost e<clusively from reserves in North America. A! After crude oil% natural gas is the +nited ,tates second biggest fuel source and supplied almost e<clusively from reserves in North America. "! Natural gas% after crude oil the +nited ,tates second biggest fuel source% supplied almost e<clusively from reserves in North America. $! "eing supplied almost e<clusively from reserves in North America% natural gas% the +nited ,tates second biggest fuel source after crude oil. &! Natural gas% the +nited ,tates3 second biggest fuel source after crude oil% is supplied almost e<clusively from reserves in North America. '! Natural gas is supplied almost e<clusively from reserves in North America% being the +nited ,tates3 second biggest fuel source after crude oil. :(. After gradual declension do#n to about /) hours in 1)*2% the #ork#eek in the +nited ,tates has steadily increased to the point that the average #orker no# puts in an estimated 1:4 e<tra hours of paid labor a year. A! After gradual declension do#n "! ?ollo#ing a gradual declension do#n $! After gradual declining do#n &! After gradually declining '! ?ollo#ing gradually declining :/. After Culy% anyone disposing of or servicing refrigerators must capture the chlorofluorocarbons in the refrigerant chemicals. A! anyone disposing of or servicing "! those #ho dispose or service $! anyone disposing of or #ho services &! the disposal or repair of '! someone #ho disposes or repairs :4. After ;ueen Isabella asked Admiral $olumbus to describe the island of >ispaniola no# >aiti!% #hich #as ne#ly discovered% he had reached for a sheet of paper% crumpled it% and said% It looks

SC Strategy

'J

like thatHbeyond the mountains% more mountains. A! After ;ueen Isabella asked Admiral $olumbus to describe the island of >ispaniola no# >aiti!% #hich #as ne#ly discovered% he had reached "! Gn being asked to describe the ne# discovery of the island of >ispaniola no# >aiti! by ;ueen Isabella% Admiral $olumbus% reaching $! ;ueen Isabella asked Admiral $olumbus to describe the ne#ly discovered island of >ispaniola no# >aiti!% then he reached &! When asked by ;ueen Isabella to describe the ne#ly discovered island of >ispaniola no# >aiti!% Admiral $olumbus reached '! After ;ueen Isabella had asked Admiral $olumbus to describe the discovery of the island of >ispaniola no# >aiti!% he had reached :7. After suffering I( billion in losses and (7%222 layoffs% the nation3s semiconductor industry% #hich makes chips that run everything from computers and spy satellites to dish#ashers% appears to have made a long-a#aited recovery. A! computers and spy satellites to dish#ashers% appears to have "! computers% spy satellites% and dish#ashers% appears having $! computers% spy satellites% and dish#ashers% appears that it has &! computers and spy satellites to dish#ashers% appears that it has '! computers and spy satellites as #ell as dish#ashers% appears to have ::. After the Arab con5uest of 'gypt in A.&. :42% Arabic became the dominant language of the 'gyptians% replacing older languages and #riting systems. A! became the dominant language of the 'gyptians% replacing older languages "! became the dominant language of the 'gyptians% replacing languages that #ere older $! becomes the dominant language of the 'gyptians and it replaced older languages &! becomes the dominant language of the 'gyptians and it replaced languages that #ere older '! becomes the dominant language of the 'gyptians% having replaced languages that #ere older :*. After the $ivil War% contemporaries of >arriet =ubman3s maintained that she has all of the 5ualities of a great leader% coolness in the face of danger% an e<cellent sense of strategy% and an ability to plan in minute detail. A! =ubman3s maintained that she has "! =ubman3s maintain that she had $! =ubman3s have maintained that she had &! =ubman maintained that she had '! =ubman had maintained that she has :0. After the $olonial period3s 72 percent mortality rate% life e<pectancy improved for children% but as late as the nineteenth century about one child in three died before reaching the age of si<.

*E

A! After the $olonial period3s 72 percent mortality rate% life e<pectancy improved for children% but "! 'ven though children3s life e<pectancy% #hich improved over the $olonial period3s 72 percent mortality rate% $! Although life e<pectancy for children improved after the $olonial period% during #hich the mortality rate #as 72 percent% &! While there #as an improvement in life e<pectancy for children after the 72 percent mortality rate of the $olonial period% still '! &espite children3s life e<pectancy improvement from the $olonial period3s 72 percent mortality rate% :). After the Bietnam #ar "ettye -ranther% a +.,. Army nurse% continued her efforts on behalf of in6ured Bietnamese children% providing medical care% helping to reunite estranged families% and the establishment of a fund for the children3s future education. A! the establishment of a fund for the children3s future education "! the establishing of a fund for the future education of children $! establishing a fund for the children3s future education &! establishing a fund for the childrens3 future education '! the establishment of a fund for the childrens3 future education *2. After this year3s record- shattering Canuary performance in 8adison ,5uare -arden% the ensemble #ere touted as the country3s best ne# group in decadesE no critic or revie#er had anything but praise for the young musicians. A! the ensemble #ere touted as the country3s "! the ensemble #as touted as the country3s $! the country touted the ensemble like the &! touting the ensemble as the country3s '! they #ere touting the ensemble as the country3s *1. Aging is a property of all animals that reach a fi<ed si9e at maturity% and the variations in life spans among different species are far greater as that among individuals from the same speciesA a fruit fly is ancient at 42 days% a mouse at / years% a horse at /2% a man at 122% and some tortoises at 172. A! among different species are far greater as that among individuals from "! among different species are far greater than that among individuals from $! among different species are far greater than those among individuals of &! bet#een different species are far more than that bet#een individuals of '! bet#een different species are greater by far than is that bet#een individuals from *(. Aho% a Fio#a matriarch% held festivals in her home% they featured the preparation of great

SC Strategy

*$

5uantities of ceremonial food% the #earing of many layers of colorful clothing adorned #ith silver% and the recounting of traditional tribal 6okes and stories. A! Aho% a Fio#a matriarch% held festivals in her home% they featured "! ?estivals #ere held in Aho% a Fio#a matriarch3s home% #hich featured $! Aho% #ho #as a Fio#a matriarch in her home% held festivals featuring &! In her home% Aho% a Fio#a matriarch% held festivals that featured '! Aho% a Fio#a matriarch% held festivals in her home that featured */. Alaska regularly deposits some of its profits from the sale of oil into a special fund% #ith the intention to sustain the state3s economy after the e<haustion of its oil reserves. A! fund% #ith the intention to sustain the state3s economy after the e<haustion of its oil reserves "! fund% the intention of #hich is to sustain the state3s economy after they have e<hausted their oil reserves $! fund intended to sustain the state3s economy after oil reserves are e<hausted &! fund intended to sustain the state3s economy after e<hausting its oil reserves '! fund that they intend to sustain the state3s economy after oil reserves are e<hausted *4. All-terrain vehicles have allo#ed vacationers to reach many previously inaccessible areas% but they have also been blamed for causing hundreds of deaths% in6ury to thousands% and seriously damaging the nation3s recreational areas. A! deaths% in6ury to thousands% and seriously damaging "! deaths and in6uring thousands% and serious damage to $! deaths% thousands #ho are in6ured% as #ell as seriously damaging &! deaths and thousands of in6uries% as #ell as doing serious damage to '! deaths% thousands are in6ured% and they do serious damage to *7. Along #ith the drop in producer prices announced yesterday% the strong retail sales figures released today seem like it is indicative that the economy% although gro#ing slo#ly% is not nearing a recession. A! like it is indicative that "! as if to indicate $! to indicate that &! indicative of '! like an indication of *:. Although all the proceedings of the $ommunist party conference held in 8osco# #ere not carried live% ,oviet audiences have seen a great deal of coverage. A! all the proceedings of the $ommunist party conference held in 8osco# #ere not carried live "! all the $ommunist party conference3s 8osco# proceedings #ere not carried live

*'

$! all the $ommunist party conference 8osco# proceedings have not been carried alive &! not all the $ommunist party conference 8osco# proceedings have been carried alive '! not all the proceedings of the $ommunist party conference held in 8osco# #ere carried live **. Although aspirin irritates the stomach% it can be avoided if the aspirin tablet is given a coating that #ill not dissolve until the tablet reaches the intestine. A! Although aspirin irritates the stomach% it "! =he irritation of the stomach caused by aspirin $! =he fact that aspirin causes irritation of the stomach &! Aspirin causes stomach irritation% although it '! Aspirin irritates the stomach% #hich *0. Although do9ens of Ne# @ork3s small museums are either devoted to local history or various ethnic groups% there are many one-of-a-kind museums from 8anhattan to the "ron< that are open for e<ploration on summer #eekends. A! Although do9ens of Ne# @ork3s small museums are either devoted to local history or various ethnic groups% there are "! Although do9ens of Ne# @ork3s small museums are devoted to local history or various ethnic groups% $! &o9ens of Ne# @ork3s small museums are devoted to local history or various ethnic groups% but there are &! &o9ens of Ne# @ork3s small museums are devoted to local history or various ethnic groups% and there are also '! &evoted to local history or various ethnic groups% do9ens of Ne# @ork3s small museums and also *). Although early soap operas #ere first aired on evening radio in the 1)(23s% they had moved to the daytime hours of the 1)/23s #hen the evening schedule became cro#ded #ith comedians and variety sho#s. A! #ere first aired on evening radio in the 1)(23s% they had moved to the daytime hours of the 1)/23s "! #ere first aired on evening radio in the 1)(23s% they #ere moved to the daytime hours in the 1)/23s $! #ere aired first on evening radio in the 1)(23s% moving to the daytime hours in the 1)/23s &! #ere aired first in the evening on 1)(23s radio% they moved to the daytime hours of the 1)/23s '! aired on evening radio first in the 1)(23s% they #ere moved to the 1)/23s in the daytime hours 02. Although films about the American West depict coyotes as solitary animals ho#ling mournfully on the tops of distant hills% in reality these gregarious creatures live in stable groups that occupy the same territory for long periods.

SC Strategy

**

A! films about the American West depict coyotes as solitary animals ho#ling mournfully on the tops of distant hills "! in films about the American West coyotes are depicted to be solitary animals that ho#l mournfully on the tops of distant hills $! coyotes are depicted as solitary animals ho#ling mournfully on the tops of distant hills in films about the American West &! films about the American West depict coyotes as if they #ere solitary% mournfully ho#ling animals on the tops of distant hills '! films about the American West depict coyotes to be solitary and mournfully ho#ling animals on the tops of distant hills 01. Although fruit can no longer gro# once it is picked% it continues for some time to respire% taking in o<ygen and giving off carbon dio<ide% similar to the #ay human beings breathe. A! similar to the #ay human beings breathe "! similarly to human beings #ho are breathing $! 6ust like the breathing of human beings &! as human beings #hen breathing '! 6ust as human beings do #hen they breathe 0(. Although he is as gifted as% if not more gifted than% many of his colleagues% he is e<tremely modest and his poetry is unpublished. A! Although he is as gifted as% if not more gifted than% many of his colleagues% he is e<tremely modest and his poetry is unpublished. "! Although he is as gifted% if not more gifted% than many of his colleagues% he is e<tremely modest and #ith his poetry remaining unpublished. $! Although he is as gifted as% if not more gifted than% many of his colleagues% he is e<tremely modest and #ill not publish his poetry. &! &espite his being gifted% if not more gifted than his colleagues% he is e<tremely modest and #ill not publish his poetry. '! "eing a gifted as% or more gifted than% many of his colleagues% he is e<tremely modest and his poetry is unpublished. 0/. Although it claims to delve into political issues% television can be superficial such as #hen each of the three ma6or net#orks broadcast e<actly the same statement from a political candidate. JJJ A! superficial such as #hen each of the three ma6or net#orks "! superficial% as can sometimes occur if all of the three ma6or net#orks $! superficial if the three ma6or net#orks all &! superficial #henever each of the three ma6or net#orks '! superficial% as #hen the three ma6or net#orks each

*7

04. Although it #as e<pected that #orkers under forty #ould sho# hostility to the plan% the research report indicates that both younger and the older people approve of governmental appropriations for ,ocial ,ecurity. A! younger and the older people "! younger people and the older $! the younger and the older people &! younger and older people '! people #ho are younger and those #ho are older 07. Although it #as once funded entirely by the government% the Bictoria and Albert 8useum #as one of the first of "ritain3s national museums seeking support from corporations and private donors and to increase income by increasing attendance. A! one of the first of "ritain3s national museums seeking support from "! one of "ritain3s first national museums seeking support of $! among "ritain3s first national museums to seek support of &! among the first of "ritain3s national museums to seek support from '! among "ritain3s first national museums that have sought the support of 0:. Although 6ust inside the orbit of Cupiter% amateur astronomers #ith good telescopes should be able to see the comet #ithin the ne<t fe# #eeks. A! Although 6ust inside the orbit of "! Although it is 6ust inside the orbit of $! Cust inside the orbit of &! Grbiting 6ust inside '! >aving orbited 6ust inside 0*. Although many art patrons can readily differentiate a good debenture from an undesirable one% they are much less e<pert in distinguishing good paintings and poor ones% authentic art and fakes. A! much less e<pert in distinguishing good paintings and poor ones% authentic art and "! far less e<pert in distinguishing good paintings from poor ones% authentic art from $! much less e<pert #hen it comes to distinguishing good paintings and poor ones% authentic art from &! far less e<pert in distinguishing good paintings and poor ones% authentic art and '! far less the e<pert #hen it comes to distinguishing bet#een good painting% poor ones% authentic art% and 00. Although many Whites% noting the presence of some "lacks in the middle class% think that the time for enforcing civil rights measures is past% "lacks generally are a#are that the figures for average

SC Strategy

*8

income and unemployment sho# as #ide of a radical discrepancy as ever. A! that the figures for average income and unemployment sho# as #ide of "! that average-income and unemployment figures sho# as #ide $! that the average-income and unemployment figures are sho#ing as #ide of &! of average-income and unemployment figures that sho# as #ide of '! of figures for average income and unemployment sho#ing as #ide 0). Although 8s. "akara had previously emphasi9ed that she could not speak for other "lack people% she ventured to do so on this one occasion because she firmly believed that many minority people% likely most% #ould agree #ith her. A! do so on this one occasion because she firmly believed that many minority people% likely most% #ould agree "! speak on this one occasion since she firmly believed that many minority people% likely most% #ould have agreed $! so speak on this one occasion due to her firmly believing that many minority people% even most% #ould likely agree &! do so on this one occasion because she firmly believed that many minority people% if not most% #ould agree '! do so on this one occasion since she firmly believed many minority people% and even most% #ould likely agree )2. Although Napoleon3s army entered Dussia #ith far more supplies than they had in their previous campaigns% it had provisions for only t#enty-four days. A! they had in their previous campaigns "! their previous campaigns had had $! they had for any previous campaign &! in their previous campaigns '! for any previous campaign )1. Although one link in the chain #as demonstrated to be #eak% but not sufficiently so to re5uire the recall of the automobile. A! demonstrated to be #eak% but not sufficiently so to re5uire "! demonstrated as #eak% but it #as not sufficiently so that it re5uired $! demonstrably #eak% but not sufficiently so to re5uire &! demonstrably #eak% it #as not so #eak as to re5uire '! demonstrably #eak% it #as not #eak enough that it re5uired )(. Although partially destroyed% the archaeologists #ere able to infer from #hat remained of the inscription that the priest Konainos #as buried in the crypt.

*9

A! Although partially destroyed% the archaeologists #ere able to infer "! Although partially destroyed% the archaeologists had inferred $! Although it had been partially destroyed% the archaeologists #ere able to infer &! 1artially destroyed though it had been% the archaeologists had been able to infer '! &estroyed partially% the archaeologists #ere able to infer )/. Although schistosomiasis is not often fatal% it is so debilitating that it has become an economic drain on many developing countries. A! it is so debilitating that it has become an economic "! it is of such debilitation% it has become an economical $! so debilitating is it as to become an economic &! such is its debilitation% it becomes an economical '! there is so much debilitation that it has become an economical )4. Although she had signed a pledge of abstinence #hile being an adolescent% ?rances Willard #as /7 years old before she chose to become a temperance activist. A! #hile being an adolescent "! #hile in adolescence $! at the time of her being adolescent &! as being in adolescence '! as an adolescent )7. Although some officials noted that using machines for farming in $hina costs more than traditional hand cultivation% the mechani9ation of farming in the village of .ong "o# doubled the corn yield #hile the previous year3s costs #ere cut in half. A! mechani9ation of farming in the village of .ong "o# doubled the corn yield #hile the previous year3s costs #ere cut in half "! mechani9ation of farming in the village of .ong "o# doubled the corn yield #hile cutting costs to half those of the previous year $! mechani9ation of farming in the village of .ong "o# doubled the corn yield as cost #ere cut to half of the previous year3s &! farming mechani9ation in the village of .ong "o# doubled the corn yield as it cut in half the previous year3s costs '! farming mechani9ation in the village of .ong "o# doubled the corn yield #hile costs #ere cut to half that of the previous year

):. Although the bite of bro#n recluse spiders are rarely fatal% they cause chronic flesh #ounds% posing the greatest danger to the infant and elderly% #ho are particularly vulnerable to its poison.

SC Strategy

*;

A! bro#n recluse spiders are rarely fatal% they cause chronic flesh #ounds% posing the greatest danger to the infant and elderly% #ho are particularly vulnerable to its "! bro#n recluse spiders are rarely fatal% they cause chronic flesh #ounds and pose the greatest danger to the infant and elderly% #ho are particularly vulnerable to their $! the bro#n recluse spider is rarely fatal% it causes chronic flesh #ounds% posing the greatest danger to the infant and elderly% #ho are particularly vulnerable to their &! the bro#n recluse spider is rarely fatal% it causes chronic flesh #ounds and poses the greatest danger to infants and the elderly% #ho are particularly vulnerable to its '! the bro#n recluse spider is rarely fatal% they cause chronic flesh #ounds% posing the greatest danger to the infant and elderly% #ho are particularly vulnerable to its )*. Although the coordination of monetary policy can help facilitate the orderly financing of e<isting imbalances% it is unlikely that its effect on their si9e is significant in the absence of an appropriate fiscal ad6ustment. A! it is unlikely that its effect on their si9e is significant "! it is unlikely that the si9e of their effect #ould be significant $! affecting their si9es are not likely to be significant &! the significance of their effect on its si9e is unlikely '! its effect on their si9e is not likely to be significant )0. Although the lesser cornstalk borer is #idely distributed% control of them is necessary only in the ,outh. A! the lesser cornstalk borer is #idely distributed% control of them is "! #idely distributed% measures to control the lesser cornstalk borer are $! #idely distributed% lesser cornstalk borer control is &! the lesser cornstalk borer is #idely distributed% measures to control it are '! it is #idely distributed% control of the lesser cornstalk borer is )). Although the manager agreed to a more fle<ible #ork schedule% he said that it must be posted on the bulletin board so that both management and labor #ill kno# #hat everyone is assigned to do. A! he said that it must be posted on the bulletin board so that both management and labor #ill kno# #hat everyone is "! he said it had to be posted on the bulletin board so that both management and labor kno#s #hat everyone is $! he said that they #ould have to post the assignments on the bulletin board so that management and labor kne# #hat everyone #as &! he said that the schedule #ould have to be posted on the bulletin board so that both management and labor #ould kno# #hat everyone #as '! saying that the schedule had to be posted on the bulletin board so that both management and

*I

labor #ould kno# #hat everyone had been 122. Although the phenomenon of #ithdra#al has al#ays been the crucial physiological .eng for distinguishing addictive from nonaddictive drugs% it has become increasingly evident that not all regular heroin users e<perience #ithdra#al symptoms. A! addictive from "! addictive and $! addictive or &! bet#een addictive or '! among addictive or 121. Although the ,upreme $ourt ruled as long ago as 1002 that "lacks could not be e<cluded outright from 6ury service% nearly a century of case-by-case ad6udication has been necessary to develop and enforce the principle that all 6uries must be dra#n from a fair cross section of the community. A! has been necessary to develop and enforce the principle that all 6uries must be "! #as necessary for developing and enforcing the principle of all 6uries being $! #as to be necessary in developing and enforcing the principle of all 6uries to be &! is necessary to develop and enforce the principle that all 6uries must be '! #ill be necessary for developing and enforcing the principle of all 6uries being 12(. Although the term psychopath is popularly applied to an especially brutal criminal% in psychology it is someone #ho is apparently incapable of feeling compassion or the pangs of conscience. A! it is someone #ho is "! it is a person $! they are people #ho are &! it refers to someone #ho is '! it is in reference to people 12/. American productivity is declining in relation to 'urope3sE the energy e<pended per unit of production in the +nited ,tates is as much as t#ice that e<pended in West -ermany. A! as much as t#ice that e<pended in West -ermany "! as much as t#ice that of West -ermany3s e<penditure $! up to t#o times of West -ermany3s e<penditure &! up to t#o times #hat West -ermans e<pended '! up to double the West -erman e<penditure 124. Among the cossacks% vegetable farming #as once so despised that it #as forbidden on pain of death.

SC Strategy

*J

A! so despised that it #as "! so despised to be $! so despised it had been &! despised enough that it #as '! despised enough as to be 127. Among the emotions on display in the negotiating room #ere anger for repeatedly raising the issue over and over again and preventing the ra# #ounds from earlier battles from ever beginning to heal. A! #ere anger for repeatedly raising the issue over and over again and preventing the ra# #ounds from earlier battles from ever beginning to heal "! #as anger for repeatedly raising the issue and preventing the ra# #ounds from earlier battles from ever beginning to heal $! #ere anger over repeatedly raising the issue and preventing the ra# #ounds from earlier battles to begin healing &! #as anger about the issue% #hich #as raised over and over% and preventing the #ounds from earlier battles% still ra#% to begin healing '! #ere anger about the issue% #hich #as raised repeatedly% and preventing the ra# #ounds from earlier battles to begin to heal 12:. Among the ob6ects found in the e<cavated temple #ere small terra-cotta effigies left by supplicants #ho #ere either asking the goddess "ona &ea3s aid in healing physical and mental ills or thanking her for such help. A! in healing physical and mental ills or thanking her for such help "! in healing physical and mental ills and to thank her for helping $! in healing physical and mental ills% and thanking her for helping &! to heal physical and mental ills or to thank her for such help '! to heal physical and mental ills or thanking her for such help 12*. Among the reasons for the decline of Ne# 'ngland agriculture in the last three decades #ere the high cost of land% the pressure of housing and commercial development% and basing a marketing and distribution system on importing produce from ?lorida and $alifornia. A! basing a marketing and distribution system on importing produce from ?lorida and $alifornia "! basing a marketing and distribution system on the imported produce of ?lorida and $alifornia $! basing a system of marketing and distribution on the import of produce from ?lorida and $alifornia &! a marketing and distribution system based on importing produce from ?lorida and $alifornia '! a marketing and distribution system importing produce from ?lorida and $alifornia as its base 120. An array of ta< incentives has led to a boom in the construction of ne# office buildingsE so

7E

abundant has capital been for commercial real estate that investors regularly scour the country for areas in #hich to build. A! so abundant has capital been for commercial real estate that "! capital has been so abundant for commercial real estate% so that $! the abundance of capital for commercial real estate has been such% &! such has the abundance of capital been for commercial real estate that '! such has been an abundance of capital for commercial real estate% 12). An artistic presence of the first order% one fre5uently ranked #ith 1icasso% ,travinsky% and Cames Coyce% 8artha -raham #as acclaimed as a great dancer long before her innovative master#orks made her the most honored of American choreographers. A! 8artha -raham #as acclaimed as "! 8artha -raham #as acclaimed to be $! 8artha -raham3s acclaim is as &! 8artha -raham3s acclaim to be '! 8artha -raham3s acclaim #as in being 112. An attempt to ratify the '5ual Dights Amendment% begun almost t#o decades ago% has been unsuccessful despite efforts by many important groups% including the National Grgani9ation for Women. A! to ratify the '5ual Dights Amendment% begun almost t#o decades ago% "! begun almost t#o decades ago% for ratifying the '5ual Dights Amendment $! begun for ratifying the '5ual Dights Amendment almost t#o decades ago &! at ratifying the '5ual Dights Amendment% begun almost t#o decades ago% '! that has begun almost t#o decades ago to ratify the '5ual Dights Amendment 111. An inventory e5ual to )2 days sales is as much as even the strongest businesses carry% and then only as a #ay to anticipate higher prices or ensure against shortages. A! as much as even "! so much as even $! even so much as &! even as much that '! even so much that 11(. An unusually strong cyclist can% it is hoped% provide enough po#er to set a ne# distance record for human-po#ered aircraft in 8I=3s diaphanous construction of graphite fiber and plastic. A! can% it is hoped% provide enough po#er to set "! it is hoped% can provide enough po#er that #ill set $! hopefully can provide enough po#er% this #ill set

SC Strategy

7$

&! is hopeful to set '! hopes setting 11/. Analysts blamed 8ay3s sluggish retail sales on une<citing merchandise as #ell as the #eather% colder and #etter than #as usual in some regions% #hich slo#ed sales of barbecue grills and la#n furniture. A! colder and #etter than #as usual in some regions% #hich slo#ed "! #hich #as colder and #etter than usual in some regions% slo#ing $! since it #as colder and #etter than usually in some regions% #hich slo#ed &! being colder and #etter than usually in some regions% slo#ing '! having been colder and #etter than #as usual in some regions and slo#ed 114. Ancient Domans found it therapeutic to bathe in cold milk% in stra#berries that had been crushed% or in bathtubs filled #ith black caviar. A! to bathe in cold milk% in stra#berries that had been crushed% or in bathtubs filled #ith black caviar "! that they bathe in cold milk% in stra#berries that had been crushed% or in caviar that #as black $! to bathe in cold milk% crushed stra#berries% or black caviar &! that they bathe in cold milk% crushed stra#berries% or black caviar '! to bathe in milk % stra#berries% or caviar 117. Any medical test #ill sometimes fail to detect a condition #hen it is present and indicate that there is one #hen it is not. A! a condition #hen it is present and indicate that there is one "! #hen a condition is present and indicate that there is one $! a condition #hen it is present and indicate that it is present &! #hen a condition is present and indicate its presence '! the presence of a condition #hen it is there and indicate its presence 11:. Archaeologists in Ireland believe that a recently discovered chalice% #hich dates from the eighth century% #as probably buried to keep from being stolen by invaders. A! to keep from "! to keep it from $! to avoid &! in order that it #ould avoid '! in order to keep from 11*. Architects and stonemasons% huge palace and temple clusters #ere built by the 8aya #ithout benefit of the #heel or animal transport. A! huge palace and temple clusters #ere built by the 8aya #ithout benefit of the #heel or animal

7'

transport "! #ithout the benefits of animal transport or the #heel% huge palace and temple clusters #ere built by the 8aya $! the 8aya built huge palace and temple clusters #ithout the benefit of animal transport or the #heel &! there #ere built% #ithout the benefit of the #heel or animal transport% huge palace and temple clusters by the 8aya '! #ere the 8aya #ho% #ithout the benefit of the #heel or animal transport% built huge palace and temple clusters 110. Art museums do not usually think of their collections as capital or consider the interest income that #ould be generated if a portion of the capital #ould have been invested in another form. A! be generated if a portion of the capital #ould have been "! have been generated if a portion of the capital #ould have been $! be generated if a portion of the capital #ere &! be generated if a portion of the capital #as '! be generated if a portion of the capital had been 11). Artificial intelligence emerged during the late 1)723s as an academic discipline based on the assumption that computers are able to be programmed to think like people. A! are able to be programmed to think like people "! #ere able to be programmed to think as people $! can be programmed to think as people can &! could be programmed to think like people '! are capable of being programmed to think like people do 1(2. As a baby emerges from the darkness of the #omb #ith a rudimentary sense of vision% it #ould be rated about (2L722% or legally blind if it #ere an adult #ith such vision. A! As a baby emerges from the darkness of the #omb #ith a rudimentary sense of vision% it #ould be rated about (2L722% or legally blind if it #ere an adult #ith such vision. "! A baby emerges from the darkness of the #omb #ith a rudimentary sense of vision that #ould be rated about (2L722% or legally blind as an adult. $! As a baby emerges from the darkness of the #omb% its rudimentary sense of vision #ould be rated about (2L722E 5ualifying it to be legally blind if an adult. &! A baby emerges from the darkness of the #omb #ith a rudimentary sense of vision that #ould be rated about (2L722E an adult #ith such vision #ould be deemed legally blind. '! As a baby emerges from the darkness of the #omb% its rudimentary sense of vision% #hich #ould be deemed legally blind for an adult% #ould be rated about (2L722.

SC Strategy

7*

1(1.

As a result of medical advances% many people that might at one time have died as children of such infections as diphtheria% pneumonia% or rheumatic fever no# live #ell into old age. A! that might at one time have died as children "! #ho might once have died in childhood $! that as children might once have died &! #ho in childhood might have at one time died '! #ho% #hen they #ere children% might at one time have died

1((.

As a result of the continuing decline in the birth rate% less people #ill enter the labor force in the 1)023s than did in the 1):23s and 1)*23s% a t#enty year period during #hich people born after the #ar s#elled the ranks of #orkers. A! less people #ill enter the labor force in the 1)023s than "! less people #ill be entering the labor force in the 1)023s as $! fe#er people #ill enter the labor force in the 1)023s as &! fe#er people #ill be entering the labor force in the 1)023s as '! fe#er people #ill enter the labor force in the 1)023s than

1(/.

As a result of the ground-breaking #ork of "arbara 8c$lintock% many scientists no# believe that all of the information encoded in 72%222 to 122%222 of the different genes found in a human cell are contained in merely three percent of the cell3s &NA. A! 72%222 to 122%222 of the different genes found in a human cell are contained in merely "! 72%222 to 122%222 of the human cell3s different genes are contained in a mere $! the 72%222 to 122%222 different genes found in human cells are contained in merely &! 72%222 to 122%222 of human cell3s different genes is contained in merely '! the 72%222 to 122%222 different genes found in a human cell is contained in a mere

1(4.

As business gro#s more comple<% students ma6oring in speciali9ed areas like those of finance and marketing have been becoming increasingly successful in the 6ob market. A! ma6oring in speciali9ed areas like those of finance and marketing have been becoming increasingly "! #ho ma6or in such speciali9ed areas as finance and marketing are becoming more and more $! #ho ma6ored in speciali9ed areas such as those of finance and marketing are being increasingly &! #ho ma6or in speciali9ed areas like those of finance and marketing have been becoming more and more '! having ma6ored in such speciali9ed areas as finance and marketing are being increasingly

1(7.

As contrasted #ith the honeybee% the yello# 6acket can sting repeatedly #ithout dying and carries a potent venom that can cause intense pain.

77

A! As contrasted #ith the honeybee "! In contrast to the honeybee3s $! +nlike the sting of the honeybee &! +nlike that of the honeybee '! +nlike the honeybee 1(:. As envisioned by researchers% commercial farming of lobsters #ill enable fisheries to sell the shellfish year-round% taking advantage of off-season demand% standardi9e its si9es and colors% and to predict sales volume in advance. A! taking advantage of off-season demand% standardi9e "! taking advantage of off-season demand% to standardi9e $! taking advantage of off-season demand% standardi9ing &! take advantage of off-season demand% standardi9ing '! take advantage of off-season demand% to standardi9e 1(*. As >urricane >ugo approached the Atlantic coast% it increased dramatically in strength% becoming the tenth most intense hurricane to hit the +nited ,tates mainland in the t#entieth century and most intense since $amille in 1):). A! most intense since $amille in 1):) "! most intense after $amille in 1):) $! the most intense since $amille in 1):) &! the most intense after 1):)% #hich had $amille '! since 1):) and $amille% the most intense 1(0. As it becomes more fre5uent to have spouses #ho both #ork outside the home% companies are beginning to help in finding ne# employment for the spouses of transferred employees. A! it becomes more fre5uent to have spouses #ho both #ork outside the home "! it becomes more fre5uent to have couples both #orking outside the home $! it becomes more common that both husband and #ife should be #orking outside the home &! it becomes more common for both husband and #ife to #ork outside the home '! couples in #hich both of the spouses #orking outside the home become more common 1(). As litigation gro#s more comple<% the need that e<perts e<plain technical issues becomes more apparent. A! that e<perts e<plain technical issues becomes "! for e<perts to e<plain technical issues became $! for e<perts to e<plain technical issues becomes &! that technical issues be e<plained by e<perts became '! that there be e<planations of technical issues by e<perts has become

SC Strategy

78

1/2.

As many as /22 of the *(2 paintings attributed to Dembrandt may actually be the #orks of his students or other admirers. A! the *(2 paintings attributed to Dembrandt may "! the *(2 paintings attributed to be Dembrandt3s might $! the *(2 paintings that #ere attributed to be by Dembrandt may &! the *(2 Dembrandt paintings that #ere once attributed to him might '! Dembrandt3s paintings% although *(2 #ere once attributed to him% may

1/1.

As measured by the $ommerce &epartment% corporate profits peaked in the fourth 5uarter of 1)00 and have slipped since then% as many companies have been unable to pass on higher costs. A! and have slipped since then% as many companies have been unable to pass on higher costs "! and have slipped since then% the reason being because many companies have been unable to pass on higher costs $! and slipped since then% many companies being unable to pass on higher costs &! but% many companies unable to pass on higher costs% they have slipped since then '! yet are slipping since then% because many companies #ere unable to pass on higher costs

1/(.

As more and more people invest their money in savings certificates or money-market funds in order to earn higher interest% they are abandoning traditional lo#-interest investment havens such as passbook accounts and life insurance policies. A! As more and more people invest their money "! While people have more and more been investing their money $! As money is more and more invested by people &! 8ore and more% #hen investors put their money '! While% more and more% investors have been putting their money

1//.

As part of their therapy% young polio victims learning to live #ith their disabilities #ere helped to practice falling during the 1)72s% so that they could learn to fall #ithout being hurt. A! As part of their therapy% young polio victims learning to live #ith their disabilities #ere helped to practice falling during the 1)72s "! As part of their therapy% young polio victims learning to live during the 1)72s #ith their disabilities #ere helped to practice falling $! @oung polio victims learning to live during the 1)72s #ith their disabilities #ere helped to practice falling as part of their therapy &! @oung polio victims learning to live #ith their disabilities #ere helped to practice falling during the 1)72s as part of their therapy '! &uring the 1)72s% as part of their therapy% young polio victims learning to live #ith their disabilities #ere helped to practice falling

1/4.

As rare as something becomes% be it a baseball card or a musical recording or a postage stamp%

79

the more avidly it is sought by collectors. A! As rare as something becomes% be it "! As rare as something becomes% #hether it is $! As something becomes rarer and rarer% like &! =he rarer something becomes% like '! =he rarer something becomes% #hether it is 1/7. As recently as 1)72% tuberculosis #as never curable unless se5uestered in sanitariumsE today% the drug Isonia9id has made such treatment obsolete. A! unless se5uestered "! #ithout se5uestering $! #ithout being se5uestered &! unless it #as se5uestered '! unless patients #ere se5uestered 1/:. As researchers continue to probe the highly e<pressive vocal and postural language of #olves% their close resemblance to dogs has become ever more striking. A! their close resemblance to dogs has become "! the closeness of their resemblance to dogs has become $! the close resemblance bet#een them and dogs has become &! the close resemblance bet#een #olves and dogs becomes '! the close resemblance of #olves #ith dogs becomes 1/*. As Dussell "anks suggests% it is a lack of grand ideas that have only left #riters #ith semiotics% hermeneutics% and deconstruction. A! As Dussell "anks suggests% it is a lack of grand ideas that have only left #riters #ith "! Writers% Dussell "anks suggests% #ho lack of grand ideas and leave only $! Dussell "anks suggests that a lack of grand ideas has left #riters #ith only &! It is Dussell "anks3 suggestion that a lack of grand ideas have left #riters only #ith '! It is only a lack of grand ideas% Dussell "anks suggests% #hich leave #riters #ith 1/0. As sales of cars and light trucks made in North America #ere declining 1/.: percent in late ?ebruary% many analysts conclude that evidence of a recovering automotive market remains slight. A! As sales of cars and light trucks made in North America #ere declining 1/.: percent in late ?ebruary% many analysts conclude "! ,ince sales of cars and light trucks made in North America declined 1/.: percent in late ?ebruary% and many analysts conclude $! With sales of cars and light trucks made in North America declining 1/.: percent in late ?ebruary% #ith many analysts concluding

SC Strategy

7;

&! "ecause sales of cars and light trucks made in North America declined 1/.: percent in late ?ebruary% many analysts concluded '! "ecause of sales of cars and light trucks made in North America declining 1/.: percent in late ?ebruary% therefore% many analysts concluded 1/). As the etched lines on computer memory chips have become thinner and the chips3 circuits more comple<% both the po#er of the chips and the electronic devices they drive have vastly increased. A! the chips3 circuits more comple<% both the po#er of the chips and the electronic devices they drive have "! the chips3 circuits more comple<% the po#er of both the chips and the electronic devices they drive has $! the chips3 circuits are more comple<% both the po#er of the chips and the electronic devices they drive has &! their circuits are more comple<% the po#er of both the chips and the electronic devices they drive have '! their circuits more comple<% both the po#er of the chips and the electronic devices they drive have 142. As the housing affordability gap #idens% middle-income families are especially hard-hit% and these families can no longer 5ualify to buy homes% and rising rental rates force them to use far more than the standard (7 percent of their incomes for housing% leaving them #ith no e5uity or ta< #rite-offs to offset the e<penditures. A! and these families can no longer 5ualify to buy homes% and "! since these families can no longer afford to buy homes% furthermore $! for these families can no longer afford to buy homes% yet &! and these families can no longer afford to buy homesE ho#ever% '! and these families can no longer afford to buy homes% for

141.

As the price of gasoline rises% #hich makes substituting alcohol distilled from cereal grain attractive% the prices of bread and livestock feed are sure to increase. A! #hich makes substituting alcohol distilled from cereal grain attractive "! #hich makes substituting the distillation of alcohol from cereal grain attractive $! #hich makes distilling alcohol from cereal grain an attractive substitute &! making an attractive substitution of alcohol distilled from cereal grain '! making alcohol distilled from cereal grain an attractive substitute

14(.

As +.,. nuclear attack submarines pro#l their familiar haunts deep #ithin the oceans of the #orld these days% they increasingly are engaged in missions far different from the tasks for #hich

7I

they #ere built and for #hich their cre#s #ere trained over the last forty years. A! they increasingly are engaged in missions far different from the tasks for #hich they #ere built "! the missions they increasingly engage in are far different than the tasks they #ere built for $! they engage increasingly in missions that differ significantly from the tasks they #ere built for &! the missions they engage in are increasingly different than the tasks for #hich they #ere built '! they increasingly engage in missions far different than the tasks for #hich they #ere built 14/. As virtually all the nation3s 72 busiest airports are% Ne# @ork3s #ere built for an age of propellers% before 6et planes #eighing 022%222 pounds needed over t#o miles of run#ay. A! As virtually all the nation3s 72 busiest airports are "! As #ith virtually all of the nation3s 72 busiest airports $! .ike virtually all of the nation3s 72 busiest airports &! .ike the cities #here virtually all the nation3s 72 busiest airports are '! .ike other cities #here virtually all the nation3s 72 busiest airports are 144. As #ell as heat and light% the ,un is the source of a continuous stream of atomic particles kno#n as the solar #ind. . A! As #ell as heat and light% the ,un is the source of a continuous stream "! "esides heat and light% also the ,un is the source of a continuous stream $! "esides heat and light% the ,un is also the source of a continuous streaming &! =he ,un is the source not only of heat and light% but also of a continuous stream '! =he ,un is the source of not only heat and light but% as #ell% of a continuous streaming 147. Astronomers at the 1alomar Gbservatory have discovered a distant supernova e<plosion% one that they believe is a type previously unkno#n to science. A! that they believe is "! that they believe it to be $! they believe that it is of &! they believe that is '! they believe to be of 14:. Astronomers studying the ne#ly discovered star say that it provides clues about our gala<y3s origin% that it may supply data about ho# fast our gala<y is e<panding% and moreover it is perhaps the most distant star in the entire 8ilky Way. A! about ho# fast our gala<y is e<panding% and moreover it is perhaps "! regarding the speed our gala<y e<pands atE moreover% they think it may be $! about ho# fast our gala<y e<pands and perhaps be &! on the gala<y3s e<pansion rate and perhaps

SC Strategy

7J

'! about ho# fast our gala<y is e<panding% and that it is perhaps 14*. At a recent session% the ?rench government has decided that 1aris needs a second% larger opera house to complement the famous 1aris Gpera. A! has decided that 1aris needs "! decided that 1aris needs $! has decided that 1aris #ill need &! decided that 1aris has a need of '! has decided that 1aris needed 140. At a time #hen it #as unusual to do it% &orothy ,terling #rote about such ma6or figures of "lack history as >arriet =ubman and W. '. ". &u "ois. A! it #as unusual to do it "! it #as unusual to do so $! doing that #as unusual &! that #as an unusual thing to be doing '! it #as not usual to do 14). At ground level% o9one is a harmful pollutant% but in the stratosphere it shields the 'arth from the most biologically harmful radiation emitted by the ,un% radiation in the ultraviolet band of the spectrum. A! in the stratosphere "! in the stratosphere% in #hich $! it is in the stratosphere in #hich &! in the stratosphere #here '! it is in the stratosphere and 172. At ,hiprock% Ne# 8e<ico% a perennially po#erful girls3 high school basketball team has become a path to college for some and a source of pride for a community #here the household incomes of 4) percent of them are belo# the poverty level. A! #here the household incomes of 4) percent of them are "! #here they have 4) percent of the household incomes $! #here 4) percent of the household incomes are &! #hich has 4) percent of the household incomes '! in #hich 4) percent of them have household incomes 171. At the beginning of the year% the city allocated I172 million to cover the increase in #ages that it e<pected to approve as a result of negotiations #ith the municipal labor unions. A! increase in #ages that it e<pected to approve as a result of negotiations "! increased #ages it e<pected to approve from negotiations

8E

$! increasing #ages e<pected to be approved as a result of negotiating &! negotiated increases in #ages it e<pected to approve '! increases e<pected to be approved in #ages from negotiating 17(. At the invitation of F#ame Nkrumah% W. '. ". &u "ois settled in -harla and became the editor of the 'ncyclopedia Africana. A! At the invitation of "! When he #as invited by $! An invitation coming from &! An invitation that came from '! After having an invitation by 17/. At the time of the 8e<ican agrarian revolution% the most radical faction% that of Kapata and his follo#ers% proposed a return to communal o#nership of land% to #hat had been a pre-$olumbian form of o#nership respected by the ,paniards. A! land% to #hat had been a pre-$olumbian form of o#nership respected by the ,paniards "! land% a form of o#nership of the pre-$olumbians and respected by the ,paniards $! land% respected by the ,paniards and a pre-$olumbian form of o#nership &! land in #hich a pre-$olumbian form of o#nership #as respected by the ,paniards '! land that had been a pre-$olumbian form of o#nership respected by the ,paniards 174. Automation has undermined the traditional position of labor as much by robbing #orkers of the special ,kills that #ere once their most important strength than by the elimination of 6obs. A! than by the elimination of 6obs "! rather than by the elimination of a 6ob $! than by eliminating 6obs &! as by the elimination of a 6ob '! as by eliminating 6obs 177. Aviation officials have not only failed to determine the cause of the crash% but they have also ignored demands by the pilots union that the investigation should be e<pedited. A! crash% but they have also ignored demands by the pilots union that the investigation should be e<pedited "! crash but also ignored demands by the pilots union that the investigation be e<pedited $! crash but have ignored demands for e<pediting the investigation by the pilots3 union &! crash% but they are also ignoring the pilots3 union3s demands to e<pedite the investigation '! crashA the demands by the pilots3 union that the investigation should be e<pedited have also been ignored 17:. "alding is much more common among White males than males of other races.

SC Strategy

8$

A! than "! than among $! than is so of &! compared to '! in comparison #ith 17*. "al9ac drank more than fifty cups of coffee a day and died of caffeine poisoningE furthermore% caffeine did not seem to bother ,amuel Cohnson% the great #riter and le<icographer% #ho #as reported to have drunk t#enty-five cups of tea at one sitting. A! furthermore% caffeine did not seem to bother "! ho#ever% caffeine did not seem to bother $! ho#ever% caffeine did not seem to have bothered &! furthermore% caffeine did not seem to have bothered '! in addition% caffeine did not seem to bother 170. "ankers re5uire that the financial information presented to them by mortgage applicants be complete and follo# a prescribed format. A! be complete and follo# a prescribed format "! is complete and it follo#s a prescribed format $! be complete and a prescribed format is follo#ed &! to be complete and a prescribed format be follo#ing '! be completed% and it follo#ed a prescribed format 17). "ased on accounts of various ancient #riters% scholars have painted a sketchy picture of the activities of an all-female cult that% perhaps as early as the si<th century ".$.% #orshipped a goddess kno#n in .atin as "ona &ea% the good goddess. A! "ased on accounts of various ancient #riters "! "asing it on various ancient #riters3 accounts $! With accounts of various ancient #riters used for a basis &! "y the accounts of various ancient #riters they used '! +sing accounts of various ancient #riters 1:2. "ased on pinto beans and corn tortillas% the 1ima Indians have a diet that derives *2 percent of its protein from vegetable sources and only /2 percent from animal foods% the reverse of the typical North American diet. A! 1ima Indians have a diet that derives "! 1ima Indians in their diet derive $! diet of the 1ima Indians derives &! diets of the 1ima Indians have derived

8'

'! diet of the 1ima Indians% deriving 1:1. "ased on the gro#th rates of large modern reptiles such as the -alapagos tortoise and e<aminations of fossils of 6uvenile dinosaurs% scientists estimate that the largest dinosaurs probably lived to be bet#een 122 and (22 years old. A! "ased on the gro#th rates of large modern reptiles such as "! Gn the basis of gro#th rates of large modern reptiles such as $! "ased on the gro#th rates of large modern reptiles like &! Gn the basis of gro#th rates of large modern reptiles% like those of '! "ased on such gro#th rates as those of large modern reptiles like 1:(. "eatri< 1otter% in her book illustrations% carefully coordinating them #ith her narratives% capitali9ed on her keen observation and love of the natural #orld. A! "eatri< 1otter% in her book illustrations% carefully coordinating them #ith her narratives% "! In her book illustrations% carefully coordinating them #ith her narratives% "eatri< 1otter $! In her book illustrations% #hich she carefully coordinated #ith her narratives% "eatri< 1otter &! $arefully coordinated #ith her narratives% "eatri< 1otter% in her book illustrations '! "eatri< 1otter% in her book illustrations% carefully coordinated them #ith her narratives and 1:/. "ecause >alley3s comet changes orbit slightly during the seventy-si<-year interval bet#een passing close to 'arth% it may veer onto a collision course #ith a planet sometime in the distant future. A! bet#een passing "! of passing $! bet#een its passes &! of its passes '! as it passes 1:4. "ecause natural gas is composed mostly of methane% a simple hydrocarbon% vehicles po#ered by natural gas emit less of certain pollutants than the burning of gasoline or diesel fuel. A! less of certain pollutants than the burning of gasoline or diesel fuel "! fe#er of certain pollutants than burning gasoline or diesel fuel do $! less of certain pollutants than gasoline or diesel fuel &! fe#er of certain pollutants than does burning gasoline or diesel fuel '! less of certain pollutants than those burning gasoline or diesel fuel 1:7. "ecause of the business community3s uncertainty about the 1resident3s position in regard to the issue of the budget deficit% an unanticipated rise in interest rates has occurred. A! in regard to the issue of the budget deficit% an unanticipated rise in interest rates has "! on the deficit% an unanticipated rise in interest rates has

SC Strategy

8*

$! regarding the budgetary deficit% an unanticipated rise in interest rates have &! on the deficit% an unanticipated rise in interest rates have '! regarding the deficit% an unanticipated rise in interest rats have 1::. "ecause of the enormous research and development e<penditures re5uired to survive in the electronics industry% an industry marked by rapid innovation and volatile demand% such firms tend to be very large. I A! to survive "! of firms to survive $! for surviving &! for survival '! for firms3 survival 1:*. "ecause paper of all kinds is the biggest single component of municipal trash% many municipalities have tried recycling to reduce the cost of trash disposal. A! "ecause paper of all kinds is the biggest single component of municipal trash% many municipalities have tried recycling to reduce the cost of trash disposal. "! "ecause paper of all kinds is the biggest single component in municipal trash% many municipalities tried to recycle so that the cost of trash disposal is reduced. $! "ecause paper of all kinds are the biggest single components in municipal trash% many municipalities have tried to recycle to reduce the cost of trash disposal. &! All kinds of paper are the biggest single components of municipal trash% and so many municipalities have tried recycling to reduce the cost of trash disposal. '! All kinds of paper is the biggest single component of municipal trash% so many municipalities have tried to recycle so that the cost of trash disposal could be reduced. 1:0. "ecause the 'arth3s crust is more solid there and thus better able to transmit shock #aves% an earth5uake of a given magnitude typically devastates an area 122 times greater in the eastern +nited ,tates than it does in the West. A! of a given magnitude typically devastates an area 122 times greater in the eastern +nited ,tates than it does in the West "! of a given magnitude #ill typically devastate 122 times the area if it occurs in the eastern +nited ,tates instead of the West $! #ill typically devastate 122 times the area in the eastern +nited ,tates than one of the comparable magnitude occurring in the West &! in the eastern +nited ,tates #ill typically devastate an area 122 times greater than #ill a 5uake of comparable magnitude occurring in the West '! that occurs in the eastern +nited ,tates #ill typically devastate 122 times more area than if it occurred #ith comparable magnitude in the West

87

1:).

"ecause the enemy3s ne# ship is the 5uietest and it is therefore the most elusive submarine% it is being increasingly vie#ed by the military as a threat to security. A! and it is therefore the most elusive submarine% it is being increasingly vie#ed "! it is therefore the most elusive of submarines% and it has increased the vie# $! and therefore the most elusive submarine% it is being increasingly vie#ed &! and therefore it is the most elusive of submarines% there is an increasing vie# '! therefore being the most elusive of submarines% it is increasingly vie#ed

1*2.

"ecause the financial revie# covered only the last fe# fiscal years% and therefore the investigators #ere unable to determine the e<tent to possible earlier overpayments. A! and therefore the investigators #ere unable to determine the e<tent to possible "! so therefore the investigators #ere not capable of determining the possible e<tent of $! therefore the investigators #ere unable to determine the possible e<tent of &! the investigators #ere not capable of determining the possible e<tent of '! the investigators #ere unable to determine the e<tent of possible

1*1.

"ecause young children do not organi9e their attention or perceptions systematically% like adults% they may notice and remember details that their elders ignore. A! like adults "! unlike an adult $! as adults &! as adults do '! as an adult

1*(.

"efore the $ivil War% >arriet =ubman% herself an escaped slave% returned again and again to 8aryland to guide other slaves along the +nderground Dailroad to freedom. A! herself an escaped slave% returned again and again to 8aryland to guide "! being an escaped slave herself% returned again and again to 8aryland so as to guide $! an escaped slave herself% returned again and again to 8aryland for guiding &! herself as an escaped slave% returned again and again to 8aryland so as to be the guide of '! #ho had been herself as an escaped slave% returned again and again to 8aryland for the guiding of

1*/.

"eing a +nited ,tates citi9en since 1)00 and born in $alcutta in 1)42% author "harati 8ukher6ee has lived in 'ngland and $anada% and first came to the +nited ,tates in 1):1 to study at the Io#a Writers3 Workshop. A! "eing a +nited ,tates citi9en since 1)00 and born in $alcutta in 1)42% author "harati 8ukher6ee has "! >aving been a +nited ,tates citi9en since 1)00% she #as born in $alcutta in 1)42E author

SC Strategy

88

"harati 8ukher6ee $! "orn in $alcutta in 1)42% author "harati 8ukher6ee became a +nited ,tates citi9en in 1)00E she has &! "eing born in $alcutta in 1)42 and having been a +nited ,tates citi9en since 1)00% author "harati 8ukher6ee '! >aving been born in $alcutta in 1)42 and being a +nited ,tates citi9en since 1)00% author "harati 8ukher6ee

1*4.

"elieved to originate from a small area on their foreheads% elephants emit lo#-fre5uency sounds that may be used as a secret language to communicate #ith other members of the herd. A! "elieved to originate from a small area on their foreheads% elephants emit lo#-fre5uency sounds that may be used "! 'lephants emit lo#-fre5uency sounds that are believed to originate from a small area on their foreheads% and they may use this $! 'lephants emit lo#-fre5uency sounds% believed to originate from a small area on their foreheads% that they may use &! Griginating% it is believed% from a small area on their foreheadsE elephants emit lo#-fre5uency sounds they may use '! Griginating% it is believed% from a small area on their foreheads% lo#-fre5uency sounds are emitted by elephants that may be used

1*7.

"esides yielding such psychological re#ards as relief from stress% deep rela<ation% if practiced regularly% can strengthen the immune system and produce a host of other physiological benefits. A! deep rela<ation% if practiced regularly% can strengthen the immune system "! one can strengthen the immune system through deep rela<ation% if it is practiced regularly $! the immune system can be strengthened as a result of deep rela<ation if practiced regularly &! #hen deep rela<ation is practiced regularly% the immune system can be strengthened '! #hen practiced regularly% the results of deep rela<ation can be to strengthen the immune system

1*:.

"eyond the immediate cash flo# crisis that the museum faces% its survival depends on if it can broaden its membership and leave its cramped 5uarters for a site #here it can store and e<hibit its more than 1(%222 artifacts. A! if it can broaden its membership and leave "! #hether it can broaden its membership and leave $! #hether or not it has the capability to broaden its membership and can leave &! its ability for broadening its membership and leaving

89

'! the ability for it to broaden its membership and leave 1**. "ihar is India3s poorest state% #ith an annual per capita income of I111% lo#er than in the most impoverished countries of the #orld. A! lo#er than in "! lo#er than that of $! and lo#er than that of &! #hich is lo#er than in '! #hich is lo#er than it is in 1*0. "ill Walton continued on playing% even though he had in6uries that recurred over and over again% al#ays hoping to return back to his position as a regular starter in the game he loved. A! on playing% even though he had in6uries that recurred over and over again% al#ays hoping to return back "! playing% in spite of recurrent in6uries% al#ays hoping to return $! playing% though in6ured over and over% and he #as al#ays hoping to return back &! on playing% even #ith in6uries that recurred% and al#ays hoped to return '! to play% despite recurring in6uries% hoping that the return 1*). "iologists believe that they have found one of the substances that tell individual genes both #hen to become active and #hen to remain 5uiescent in the earliest phases of an embryo3s development. A! tell individual genes both #hen to become active and #hen to remain "! tell individual genes both at #hich time they should become active or should remain $! tells individual genes both #hen to become active or remain &! tells individual genes both #hen to become active or #hen to remain '! #ill tell an individual gene both about #hen it should become active and remain 102. "luegrass musician "ill 8onroe% #hose repertory% vie#s on musical collaboration% and vocal style #ere influential on generations of bluegrass artists% #as also an inspiration to many musicians% that included 'lvis 1resley and Cerry -arcia% #hose music differed significantly from his o#n. A! #ere influential on generations of bluegrass artists% #as also an inspiration to many musicians% that included 'lvis 1resley and Cerry -arcia% #hose music differed significantly from "! influenced generations of bluegrass artists% also inspired many musicians% including 'lvis 1resley and Cerry -arcia% #hose music differed significantly from $! #as influential to generations of bluegrass artists% #as also inspirational to many musicians% that included 'lvis 1resley and Cerry -arcia% #hose music #as different significantly in comparison to &! #as influential to generations of bluegrass artists% also inspired many musicians% #ho included

SC Strategy

8;

'lvis 1resley and Cerry -arcia% the music of #hom differed significantly #hen compared to '! #ere an influence on generations of bluegrass artists% #as also an inspiration to many musicians% including 'lvis 1resley and Cerry -arcia% #hose music #as significantly different from that of 101. "ob Wilber became ,idney "echet3s student and protMgM #hen he #as nineteen and% for a fe# years in the 1)423s% came as close to being a carbon copy of the 6a99 virtuoso in performance as anyone has ever come. A! as anyone has ever come "! as anyone ever had been $! as anyone ever had done &! that anyone ever did '! that anyone ever came 10(. "ringing the ?ord 8otor $ompany back from the verge of bankruptcy shortly after the ,econd World War #as a special governmentally sanctioned price increase during a period of #age and price controls. A! "ringing the ?ord 8otor $ompany back from the verge of bankruptcy shortly after the ,econd World War #as a special governmentally sanctioned price increase during a period of #age and price controls. "! What brought the ?ord 8otor $ompany back from the verge of bankruptcy shortly after the ,econd World War #as a special price increase that the government sanctioned during a period of #age and price controls. $! =hat #hich brought the ford 8otor $ompany back from the verge of bankruptcy shortly after the ,econd World War #as a special governmentally sanctioned price increase during a period of #age and price controls. &! What has brought the ?ord 8otor $ompany back from the verge of bankruptcy shortly after the ,econd World War #as a special price increase that the government sanctioned during a period of #ages and price controls. '! =o bring the ?ord 8otor $ompany back from the verge of bankruptcy shortly after the ,econd World War% there #as a special price increase during a period of #ages and price controls that government sanctioned. 10/. "ufo marinus toads% fierce predators that #ill eat frogs% li9ards% and even small birds% are native to ,outh America but #ere introduced into ?lorida during the 1)/23s in an attempt to control pests in the state3s vast sugarcane fields. A! are native to ,outh America but #ere introduced into ?lorida during the 1)/23s in an attempt to control "! are native in ,outh America but #ere introduced into ?lorida during the 1)/23s as attempts to control

8I

$! are natives of ,outh America but #ere introduced into ?lorida during the 1)/23s in an attempt at controlling &! had been native to ,outh America but #ere introduced to ?lorida during the 1)/23s as an attempt at controlling '! had been natives of ,outh America but #ere introduced to ?lorida during the 1)/23s as attempts at controlling 104. "uilding large ne# hospitals in the bistate area #ould constitute a #asteful use of resources% on the basis of avoidance of duplicated facilities alone. A! on the basis of avoidance of duplicated facilities alone "! on the grounds of avoiding duplicated facilities alone $! solely in that duplicated facilities should be avoided &! #hile the duplication of facilities should be avoided '! if only because the duplication of facilities should be avoided

107.

"y 1)14% ten of the #estern states had granted #omen the right to vote% but only one in the 'ast. A! only one in the 'ast "! only one eastern state $! in the 'ast there #as only one state &! in the 'ast only one state did '! only one in the 'ast had

10:.

"y a vote of ) to 2% the ,upreme $ourt a#arded the $entral Intelligence Agency broad discretionary po#ers enabling it to #ithhold from the public the identities of its sources of intelligence information. A! enabling it to #ithhold from the public "! for it to #ithhold from the public $! for #ithholding disclosure to the public of &! that enable them to #ithhold from public disclosure '! that they can #ithhold public disclosure of

10*.

"y analy9ing the advanced olfactory apparatus of 1leistocene chordates% paleo9oologists have discovered a link bet#een the brain3s regions of scent discrimination and its regions of long-term memory storage% a link that could prove invaluable in the treatment of amnesia victims. A! paleo9oologists have discovered a link bet#een the brain3s regions of scent discrimination and its regions of long-term memory storage

SC Strategy

8J

"! a link bet#een the brain3s regions of scent discrimination and its regions of long-term memory storage has been discovered by paleo9oologists $! there is a link that paleo9oologists have discovered bet#een the brain3s regions of scent discrimination and its regions of long-term memory storage &! the discovery of a link bet#een the brain3s regions of scent discrimination and its regions of long-term memory storage #as made by paleo9oologists '! the brain3s regions of scent discrimination and long-term memory storage have a link that #as discovered by paleo9oologists 100. "y installing special electric pumps% farmers3 houses could be heated by the #armth from co#s3 milk% according to one agricultural engineer. A! farmers3 houses could be heated by the #armth from co#s3 milk% according to one agricultural engineer "! the #armth from co#s3 milk could be used by farmers to heat their houses% according to one agricultural engineer $! one agricultural engineer reports that farmers could use the #armth from co#s3 milk to heat their houses &! farmers% according to one agricultural engineer% could use the #armth from co#s3 milk to heat their houses '! one agricultural engineer reports that farmers3 houses could be heated by the #armth from co#s3 milk 10). "y offering lo#er prices and a menu of personal communications options% such as caller identification and voice mail% the ne# telecommunications company has not only captured customers from other phone companies but also forced them to offer competitive prices. A! has not only captured customers from other phone companies but also forced them "! has not only captured customers from other phone companies% but it also forced them $! has not only captured customers from other phone companies but also forced these companies &! not only has captured customers from other phone companies but also these companies have been forced '! not only captured customers from other phone companies% but it also has forced them 1)2. "y sho#ing that ,outh Africa does not have a free market and is in fact a kind of collectivist #elfare state for Whites only% ,o#ell argues that American conservatives have no valid ideological grounds to be in sympathy #ith the 1retoria regime. A! to be in sympathy #ith "! to sympathi9e #ith $! for sympathi9ing #ith &! that they should sympathi9e #ith '! that they should have sympathy for

9E

1)1.

"y studying the primitive visual systems of single-cell a5uatic organisms% biophysicists have discovered a striking similarity bet#een algae and co#s% a similarity that indicates the common evolutionary origin of plants and animalsA both algae and co#s produce a light-sensitive protein called rhodopsin. A! biophysicists have discovered a striking similarity bet#een algae and co#s "! a striking similarity bet#een algae and co#s has been discovered by biophysicists $! there is a striking similarity that biophysicists have discovered bet#een algae and co#s &! the discovery of a striking similarity bet#een algae and co#s #as made by biophysicists '! algae and co#s have a striking similarity that #as discovered by biophysicists

1)(.

"y the mid-seventeenth century% Amsterdam had built a ne# to#n hall so large that only ,t. 1eter3s in Dome% the 'scorial in ,pain% and the 1ala99a &ucale in Benice could rival it for scale or magnificence. A! could rival it for "! #ere the rivals of it in their $! #ere its rival as to &! could be its rivals in their '! #ere rivaling its

1)/.

"yron possessed po#ers of observation that #ould have made him a great anthropologist and that makes his letters as a group the rival of the best novels of the time. A! makes his letters as a group the rival of "! makes his letters as a group one to rival $! makes his letters a group rivaling &! make his letters as a group the rival of '! make his letters a group #hich is the rival of

1)4.

$a6uns speak a dialect brought to southern .ouisiana by the four thousand Acadians #ho migrated there in 1*77E their language is basically seventeenth-century ?rench to #hich has been added 'nglish% ,panish and Italian #ords. A! to #hich has been added 'nglish% ,panish and Italian #ords "! added to #hich is 'nglish% ,panish% and Italian #ords $! to #hich 'nglish% ,panish% and Italian #ords have been added &! #ith 'nglish% ,panish% and Italian #ords having been added to it '! and% in addition% 'nglish% ,panish% and Italian #ords are added

1)7.

$alifornia3s child-support payments are as high or higher than other states. A! as high or higher than other states "! at least as high as any states

SC Strategy

9$

$! as high as% or higher than% those of any other state &! higher than any state3s% or 6ust as high '! higher% or as high as% other states 1):. $amille $laudet #orked continuously through the 10023s and early 10)23s #ith the sculptor Auguste DodinE since there are very fe# signed #orks of hers% the inescapable conclusion seems to be one of $laudet conceiving and e<ecuting part of Dodin3s enormous production of that period. A! inescapable conclusion seems to be one of $laudet conceiving and e<ecuting part of Dodin3s enormous production of that period "! conclusion of $laudet conceiving and e<ecuting part of Dodin3s enormous production of that period seems inescapable $! conclusion seems inescapable that part of Dodin3s enormous production of that period #as conceived and e<ecuted by $laudet &! conclusion of part of Dodin3s enormous production of that period having been conceived and e<ecuted by $laudet seems inescapable '! seemingly inescapable conclusion is that $laudet #ould have conceived and e<ecuted part of Dodin3s enormous production of that period 1)*. $amus broke #ith ,artre in a bitter dispute over the nature of ,talinism. A! in a bitter dispute over "! over bitterly disputing $! after there #as a bitter dispute over &! after having bitterly disputed about '! over a bitter dispute about 1)0. $anadian scientists have calculated that one human being should be struck every nine years by a meteorite% #hile each year si<teen buildings can be e<pected to sustain damage from such ob6ects. A! one human being should be struck every nine years by a meteorite "! a human being should be struck by a meteorite once in every nine years $! a meteorite #ill strike one human being once in every nine years &! every nine years a human being #ill be struck by a meteorite '! every nine years a human being should be struck by a meteorite

1)).

$ar o#ners #ho inflate their tires properly can substantially boost their vehicles3 fuel efficiency% since the increase in car-road friction can #aste up to five percent of car fuel by underinflation. A! $ar o#ners #ho inflate their tires properly can substantially boost their vehicles3 fuel efficiency% since the increase in car-road friction can #aste up to five percent of car fuel by

9'

underinflation. "! "ecause the underinflation of tires can #aste up to five percent of a car3s fuel by increasing car-road friction% car o#ners can substantially boost their vehicles3 fuel efficiency by properly inflating the tires. $! =heir vehicles3 fuel efficiency is substantially boosted by car o#ners through the proper inflation of tires that% #hen underinflated% can #aste up to five percent of car fuel by an increase in car-road friction. &! =he proper inflation of tires by car o#ners% due to the fact that underinflation can #aste up to five percent of a car3s fuel by the increase of car-road friction% can substantially boost their fuel efficiency. '! "ecause up to five percent of a car3s fuel are #asted through the increases in car-road friction #hen the tires are underinflated% car o#ners properly inflating tires can substantially boost their fuel efficiency. (22. $arbon-14 dating reveals that the megalithic monuments in "rittany are nearly (%222 years as old as any of their supposed 8editerranean predecessors. A! as old as any of their supposed "! older than any of their supposed $! as old as their supposed &! older than any of their supposedly '! as old as their supposedly (21. $ardiopulmonary resuscitation should begin bet#een one to four minutes after a cardiac arrest in order to be a success. A! $ardiopulmonary resuscitation should begin bet#een one to four minutes after a cardiac arrest in order to be a success. "! "et#een one and four minutes after a cardiac arrest% cardiopulmonary resuscitation should begin to be successful. $! ,uccessful cardiopulmonary resuscitation should be begun from one to four minutes after a cardiac arrest occurs. &! If it is to be successful% a cardiac arrest should be treated #ith cardiopulmonary resuscitation one to four minutes after#ard. '! =o be successful% cardiopulmonary resuscitation should begin #ithin one to four minutes after a cardiac arrest. (2(. $arnivorous mammals can endure #hat #ould other#ise be lethal levels of body heat because they have a heat-e<change net#ork #hich kept the brain from getting too hot. A! #hich kept "! that keeps $! #hich has kept

SC Strategy

9*

&! that has been keeping '! having kept (2/. $arpenters% dentists% se#ing machine operators% needlepointers% piano players% and indeed anyone #ho #orks #ith their hands for long hours can get carpal tunnel syndrome. A! anyone #ho #orks "! anyone #orking $! #orkers &! those for #hom #ork is '! any people #ho #ork (24. $artographers have long struggled #ith the problem of having the spherical 'arth to dra# on a flat sheet of paper. A! having the spherical 'arth to dra# on a flat sheet of paper "! having a flat sheet of paper on #hich to dra# the spherical 'arth $! ho# can one dra# the spherical 'arth on a flat sheet of paper &! ho# they could use a flat sheet of paper to dra# the spherical 'arth '! ho# to dra# the spherical 'arth on a flat sheet of paper (27. $ertain pesticides can become ineffective if used repeatedly in the same placeE one reason is suggested by the finding that there are much larger populations of pesticide-degrading microbes in soils #ith a relatively long history of pesticide use than in soils that are free of such chemicals. A! $ertain pesticides can become ineffective if used repeatedly in the same placeE one reason is suggested by the finding that there are much larger populations of pesticide-degrading microbes in soils #ith a relatively long history of pesticide use than in soils that are free of such chemicals. "! If used repeatedly in the same place% one reason that certain pesticides can become ineffective is suggested by the finding that there are much larger populations of pesticide-degrading microbes in soils #ith a relatively long history of pesticide use than in soils that are free of such chemicals. $! If used repeatedly in the same place% one reason certain pesticides can become ineffective is suggested by the finding that much larger populations of pesticide-degrading microbes are found in soils #ith a relatively long history of pesticide use than those that are free of such chemicals. &! =he finding that there are much larger populations of pesticide-degrading microbes in soils #ith a relatively long history of pesticide use than in soils that are free of such chemicals is suggestive of one reason% if used repeatedly in the same place% certain pesticides can become ineffective. '! =he finding of much larger populations of pesticide-degrading microbes in soils #ith a relatively long history of pesticide use than in those that are free of such chemicals suggests

97

one reason certain pesticides can become ineffective if used repeatedly in the same place. (2:. $harles .indbergh% for his attempt at a solo transatlantic flight% #as very reluctant to have any e<tra #eight on his plane% he therefore refused to carry even a pound of mail% despite being offered I1%222 to do so. A! $harles .indbergh% for his attempt at a solo transatlantic flight% #as very reluctant to have any e<tra #eight on his plane% he therefore "! When $harles .indbergh #as attempting his solo transatlantic flight% being very reluctant to have any e<tra #eight on his plane% he $! ,ince he #as very reluctant to carry any e<tra #eight on his plane #hen he #as attempting his solo transatlantic flight% so $harles .indbergh &! "eing very reluctant to carry any e<tra #eight on his plane #hen he attempted his solo transatlantic flight #as the reason that $harles .indbergh '! Bery reluctant to have any e<tra #eight on his plane #hen he attempted his solo transatlantic flight% $harles .indbergh (2*. $harlotte 1erkins -ilman% a late nineteenth-century feminist% called for urban apartment houses including child-care facilities and clustered suburban houses including communal eating and social facilities. A! including child-care facilities and clustered suburban houses including communal eating and social facilities "! that included child-care facilities% and for clustered suburban houses to include communal eating and social facilities $! #ith child-care facilities included and for clustered suburban houses to include communal eating and social facilities &! that included child-care facilities and for clustered suburban houses #ith communal eating and social facilities '! to include child-care facilities and for clustered suburban houses #ith communal eating and social facilities included (20. $hicago% #here industrial gro#th in the nineteenth century #as more rapid than any other American city% #as plagued by labor troubles like the 1ullman ,trikes of 10)4. A! #here industrial gro#th in the nineteenth century #as more rapid than any other American city "! #hich had industrial gro#th in the nineteenth century more rapid than that of other American cities $! #hich had gro#th industrially more rapid than any other American city in the nineteenth century &! #hose industrial gro#th in the nineteenth century #as more rapid than any other American city

SC Strategy

98

J '! #hose industrial gro#th in the nineteenth century #as more rapid than that of any other American city (2). $hild prodigies are marked not so much by their skills but instead by the fact that these skills are fully developed at a very early age. A! but instead "! rather than $! than &! as '! so much as (12. $hinese% the most ancient of living #riting systems% consists of tens of thousands of ideographic characters% each character a miniature calligraphic composition inside its o#n s5uare frame. A! each character a miniature calligraphic composition inside its "! all the characters a miniature calligraphic composition inside their $! all the characters a miniature calligraphic composition inside its &! every character a miniature calligraphic composition inside their '! each character a miniature calligraphic composition inside their (11. $hronic lo#-level leaking and the routine discharge of drilling mud and mineral salts present considerable environmental risk during offshore oil drilling. A! $hronic lo#-level leaking and the routine discharge of drilling mud and mineral salts present considerable environmental risk during offshore oil drilling. "! =he reason offshore oil drilling presents a considerably environmental risk is because of chronic lo#-level leaking and the routine discharge of drilling mud and mineral salts. $! A considerable risk to the environment is presented during offshore oil drilling% #here lo#level leaks are chronic and the routine discharge of mud and mineral salts. &! Gffshore oil drilling presents a considerable risk to the environment due to the fact of chronic lo#-level leaks% and mud and mineral salts are routinely discharged. '! $hronic lo#-level leaking% along #ith the routine discharge of drilling mud and mineral salts% are #hat make offshore oil drilling environmentally risky. (1(. $iting evidence that the carbon dio<ide content of the atmosphere has increased more than seven percent in the last /2 years% many scientists have e<pressed a fear of destroying forests and continued use of fossil fuels #ill cause an irreversible shift in 'arth3s climatic pattern. A! many scientists have e<pressed a fear of destroying forests and continued use of "! many scientists have e<pressed a fear that destruction of forests and continued use of $! many scientists have e<pressed a fear that destruction of forests and continually using &! a fear that many scientists have e<pressed is that destroying forests and continually using

99

'! a fear that many scientists have e<pressed is that destruction of forests and continual using of (1/. $lark and ?lorence Wallace% a husband-and-#ife medical team% #orked steadily and efficiently through the night% but sipping their coffee the ne<t morning% she noticed that he seemed disoriented. A! sipping their coffee the ne<t morning% she noticed that he seemed disoriented "! sipping their coffee the ne<t morning% he seemed to be disoriented% she noticed $! as they sipped their coffee the ne<t morning% she noticed that he seemed disoriented &! as they #ere sipping their coffee the ne<t morning% he seemed% she noticed% disoriented '! he seemed disoriented% she noticed% sipping their coffee the ne<t morning (14. $limatic shifts are so gradual as to be indistinguishable at first from ordinary fluctuations in the #eather. A! so gradual as to be indistinguishable "! so gradual they can be indistinguishable $! so gradual that they are unable to be distinguished &! gradual enough not to be distinguishable '! gradual enough so that one cannot distinguish them (17. $omparing the ;uechans #ith other Native Americans of the ,outh#est% the ;uechans #ere singularly uninterested in the accumulation of material #ealth or in the crafting of elaborate pottery and basketry. A! $omparing the ;uechans #ith other Native Americans of the ,outh#est% the ;uechans "! When you compare the ;uechans to other Native Americans of the ,outh#est% they $! When other Native Americans of the ,outh#est are compared to the ;uechans% they &! $omparison to other Native Americans of the ,outh#est sho#s that the ;uechans '! $ompared #ith other Native Americans of the ,outh#est% the ;uechans (1:. $omputers are becoming faster% more po#erful% and more reliable% and so too are modems% they are the devices to allo# t#o or more computers to share information over regular telephone lines. A! so too are modems% they are the devices to allo# "! so too are modems% the devices that allo# $! so too modems% the devices allo#ing &! also modems% they are the devices that allo# '! also modems% #hich are the devices to allo# (1*. $oncerned at the increase in accident fatalities% =ennessee adopted a child-passenger protection la# re5uiring the parents of children under four years of age to be restrained in a child safety seat.

SC Strategy

9;

A! the parents of children under four years of age to be restrained in a child safety seat "! the restraint of parents of children under four years of age in a child safety seat $! that parents restrain children under four years of age in a child safety seat &! that children be restrained under four years of age in a child safety seat by their parents '! children to be restrained under four years of age by their parents in a child safety seat (10. $onfused by the many strata and substrata of ancient civili9ations overlying one another% ,chliemann3s e<cavations of the fabled city of Ilium% the ancient =roy% #ere temporarily called to a halt. A! ,chliemann3s e<cavations of the fabled city of Ilium% the ancient =roy% #ere temporarily called to a halt "! ,chliemann3s e<cavations of the fabled city of Ilium% the ancient =roy% temporarily halted $! ,chliemann temporarily halted his e<cavations of the fabled city of Ilium% the ancient =roy &! e<cavations of the fabled city of Ilium% the ancient =roy% #ere temporarily halted by ,chliemann '! e<cavations of the fabled city of Ilium% and of the ancient =roy% #ere temporarily halted by ,chliemann (1). $ongress is debating a bill re5uiring certain employers provide #orkers #ith unpaid leave so as to care for sick or ne#born children. A! provide #orkers #ith unpaid leave so as to "! to provide #orkers #ith unpaid leave so as to $! provide #orkers #ith unpaid leave in order that they &! to provide #orkers #ith unpaid leave so that they can '! provide #orkers #ith unpaid leave and ((2. $onstance >orner% chief of the +nited ,tates government3s personnel agency% has recommended that the use of any dangerous or illegal drug in the five years prior to application for a 6ob be grounds for not hiring an applicant. A! the use of any dangerous or illegal drug in the five years prior to application for a 6ob be grounds for not hiring an applicant "! any dangerous or illegal drug% if used in the five years prior to applying for a 6ob% should be grounds not to hire the applicant $! an applicant3s use of any dangerous or illegal drug in the five years prior to application for a 6ob be grounds not to hire them &! an applicant3s use of any dangerous or illegal drug in the five years prior to applying for a 6ob are grounds that they not be hired '! for five years prior to applying for a 6ob% an applicant3s use of any dangerous or illegal drug be grounds for not hiring them

9I

((1.

$onsumers may not think of household cleaning products to be ha9ardous substances% but many of them can be harmful to health% especially if they are used improperly. I A! $onsumers may not think of household cleaning products to be "! $onsumers may not think of household cleaning products being $! A consumer may not think of their household cleaning products being &! A consumer may not think of household cleaning products as '! >ousehold cleaning products may not be thought of% by consumers% as

(((.

$ontrary to earlier con6ectures% it may be that increased atmospheric carbon dio<ide as a result of burning fossil fuels #ould cool the globe by reducing the amount of solar energy absorbed by sno#. A! increased atmospheric carbon dio<ide as a result of burning "! increased atmospheric carbon dio<ide resulting from the combustion of $! increasing carbon dio<ide in the atmosphere resulting from the combustion of &! carbon dio<ide in the atmosphere that increase from burning '! atmospheric carbon dio<ide that increased from burning

((/.

$ontrary to popular belief% victors in the ancient -reek Glympic -ames received cash pri9es in addition to their laurel #reaths. A! $ontrary to "! In contrast #ith $! Gpposite of &! +nlike '! In spite of

((4.

$ontrary to popular opinion% the movement to#ard a service economy is leading neither to lo#er standards of living% more of an une5ual distribution of income% or displacing the physical production of goods. A! leading neither to lo#er standards of living% more of an une5ual distribution of income% or "! leading neither to lo#er standards of living nor a more une5ual distribution of income% or $! not leading to either lo#er standards of living nor to more of an une5ual distribution of income% and neither is it &! not leading to lo#er standards of living% more of an une5ual distribution of income% and it is not '! not leading to lo#er standards of living or to a more une5ual distribution of income% nor is it

((7.

$onvinced at last of the soundness of her advice% the villagers tried crop rotation% built crude sanitary facilities% and even the use of goat3s milk for the making of cheese. A! the use of goat3s milk for the making of cheese

SC Strategy

9J

"! used goat3s milk to make cheese $! the use of goat3s milk in cheese making &! making cheese from goat3s milk '! goat3s milk to make cheese ((:. $ooperative apartment houses have the peculiar distinction of being d#ellings that must also operate as businesses. A! of being d#ellings that must also operate as businesses "! of d#ellings that must also operate like business $! that they are d#ellings that must operate like business &! that% as d#ellings% they must also operate like businesses '! to be a d#elling that must also operate as a business ((*. $oronary angiography% a sophisticated method for diagnosing coronary disease involving the introduction of a dye into the arteries of the heart% is no# administered selectively% because it uses <-rays to observe cardiac function. A! for diagnosing coronary disease involving the introduction of a dye into the arteries of the heart% is no# administered selectively% because it uses "! for diagnosing coronary disease involving the introduction of a dye into the arteries of heart% is no# administered selectively% because of using $! for diagnosing coronary disease% involves the introduction of dye into the arteries of the heart and is no# administered selectively% because it uses &! to diagnose coronary disease that involves the introduction of a dye into the arteries of the heart% is no# administered selectively% because it uses '! to diagnose coronary disease involving the introduction of a dye into the arteries of the heart% #hich is no# administered selectively% uses ((0. $orporations #ill soon be re5uired to report to the government #hether they have the necessary reserves to pay the pension benefits earned by their employees and that the information be published in annual reports to shareholders. A! earned by their employees and that the information be published "! that their employees earned and that the information be published $! that #as earned by their employees #ith the information being published &! earned by their employees% information that must also be published '! earned by their employees and published the information ((). $rises in international diplomacy do not al#ays result from maliceE for nations% like individuals% can find themselves locked into difficult positions% unable to back do#n. A! do not al#ays result from maliceE for nations% like individuals% can find "! do not al#ays results from maliceE nations% 6ust as individuals% finding

;E

$! do not al#ays results from maliceE nations% such as individuals% can find &! aren3t al#ays the results of maliceE nations in the same #ay that individuals can find '! aren3t resulting al#ays from maliceE 6ust like individuals #ho can find (/2. $ritics of the trend to#ard privately operated prisons consider corrections facilities to be an integral part of the criminal 6ustice system and 5uestion if profits should be made from incarceration. A! to be an integral part of the criminal 6ustice system and 5uestion if "! as an integral part of the criminal 6ustice system and they 5uestion if $! as being an integral part of the criminal 6ustice system and 5uestion #hether &! an integral part of the criminal 6ustice system and 5uestion #hether '! are an integral part of the criminal 6ustice system% and they 5uestion #hether (/1. $ut-paper design% a popular pastime of $olonial #omen% became an art form in the hands of Abigail .efferts .loyd% a Devolutionary War heroine. A! design% a popular pastime of colonial #omen% became an art form in the hands of "! design% a popular $olonial #omen3s pastime% and it became an art form in the hands of $! design #as a popular pastime for $olonial #omen% then it became an art form in the hands of &! design is a popular pastime of $olonial #omen that has become an art form by '! design% the popular $olonial pastime% became an art form for (/(. &ance fans kne# =amara -eva as a soloist in several "allanchine #orks% a dancer #ho introduced his choreography to the +nited ,tates% and as a star in "road#ay theater. A! a dancer #ho introduced "! as a dancer #hich introduced $! the dancer to introduce &! #ho introduced '! as the dancer #ho introduced (//. &ar#in #as not the first to advance a theory of evolutionE his tremendous originality lay in the fact that he proposed the idea of natural selection as the means by #hich evolution #orked. A! lay in the fact that he proposed the idea "! lay in the fact of his proposing the idea $! laid in the fact of his proposing the idea &! laid in his proposal '! lay in his proposal (/4. &eclining values for farm e5uipment and land% the collateral against #hich farmers borro# to get through the harvest season% is going to force many lenders to tighten or deny credit this spring. I

SC Strategy

;$

A! the collateral against #hich farmers borro# to get through the harvest season% is "! #hich farmers use as collateral to borro# against to get through the harvest season% is $! the collateral #hich is borro#ed against by farmers to get through the harvest season% is &! #hich farmers use as collateral to borro# against to get through the harvest season% are '! the collateral against #hich farmers borro# to get through the harvest season% are (/7. &efense attorneys have occasionally argued that their clients3 misconduct stemmed from a reaction to something ingested% but in attributing criminal or delin5uent behavior to some food allergy% the perpetrators are in effect told that they are not responsible for their actions. A! in attributing criminal or delin5uent behavior to some food allergy "! if criminal or delin5uent behavior is attributed to an allergy to some food $! in attributing behavior that is criminal or delin5uent to an allergy to some food &! if some food allergy is attributed as the cause of criminal or delin5uent behavior '! in attributing a food allergy as the cause of criminal or delin5uent behavior (/:. &egler does more than merely summari9ing e<isting researchE he constructs a coherent picture of t#o centuries of studies dealing #ith the changing roles of #omen. A! &egler does more than merely summari9ing "! &egler3s study is more than a mere summari9ing of $! &egler has done more than a mere summari9ing of &! &egler3s study has done more than summari9e merely '! &egler does more than merely summari9e (/*. &elighted by the reported earnings for the first 5uarter of the fiscal year% it #as decided by the company manager to give her staff a raise. A! it #as decided by the company manager to give her staff a raise "! the decision of the company manager #as to give her staff a raise $! the company manager decided to give her staff a raise &! the staff #as given a raise by the company manager '! a raise #as given to the staff by the company manager (/0. &ental caries and gingivitis can be e<acerbated not only by the foods patients eat but also by #hen the patients eat them. A! not only by the foods patients eat but also by #hen the patients eat them "! by not only the foods patients eat but also by #hen the patients eat them $! not only by the foods patients eat but also by time #hen the foods are eaten &! by not only the foods that are eaten by patients but also by the times the foods are eaten '! not only by #hat patients eat but also by #hen they eat it

;'

(/).

&espite its attractiveness% investing abroad can still pose big risks% ranging from the potential for political instability in some countries to the shortage of regulations to protect investors and a serious lack of information about investments in others. A! to the shortage of regulations to protect investors and a serious lack of information about investments in others "! to the shortage of regulations to protect investors and in others a serious lack of information about investments $! and the shortage of regulations to protect investors and a serious lack of information about investments in others &! and the shortage of regulations to protect investors to a serious lack of information about investments in others '! to the shortage of regulations to protect investors in others and a serious lack of information about investments

(42.

&espite no proof that the consumption of any particular foods reverse hardening of the arteries% studies indicate that refraining from eating certain foods could help reverse blockage of coronary arteries% the blood vessels that feed the heart. A! &espite no proof that the consumption of any particular foods reverse hardening of the arteries% studies indicate that refraining from eating certain foods could "! &espite no foods having been proved to reverse hardening of the arteries #hen consumed% studies indicate that refraining from eating certain foods can $! Although the consumption of no particular foods have been proved to reverse hardening of the arteries% studies indicate that to refrain from eating certain foods could &! Although not proved that the consumption of any foods reverse hardening of the arteries% studies indicate that refraining from eating certain foods can '! Although it has not been proved that the consumption of any particular food #ill reverse hardening of the arteries% studies indicate that refraining from eating certain foods can

(41.

&espite protests from some #aste-disposal companies% state health officials have ordered the levels of bacteria in sea#ater at popular beaches to be measured and that the results be published. I A! the levels of bacteria in sea#ater at popular beaches to be measured and that the results be "! that sea#ater at popular beaches should be measured for their levels of bacteria% #ith the results being $! the measure of levels of bacteria in sea#ater at popular beaches and the results to be &! sea#ater measured at popular beaches for levels of bacteria% #ith their results '! that the levels of bacteria in sea#ater at popular beaches be measured and the results

(4(.

&espite the recent election of a #oman to the office of prime minister% the status of #omen in 1akistan is little changed from ho# it #as in the last century.

SC Strategy

;*

A! is little changed from ho# it #as "! is a little change from ho# it #as $! has changed little &! has changed little from ho# it has been '! is little changed from the #ay it #as (4/. &eveloping nations in various parts of the #orld have amassed I*22 billion in debtsE at stake% should a significant number of these debts be repudiated% is the solvency of some of the #orld3s largest multinational banks. A! should a significant number of these debts be repudiated% is "! should a significant number of these debts be repudiated% are $! should they repudiate a significant number of these debts% are &! if there is a repudiation of a significant number of these debts% #ould be '! if a significant number of these debts #ill be repudiated% is (44. &irt roads may evoke the bucolic simplicity of another century% but financially strained to#nships point out that dirt roads cost t#ice as much as maintaining paved roads. A! dirt roads cost t#ice as much as maintaining paved roads "! dirt roads cost t#ice as much to maintain as paved roads do $! maintaining dirt roads costs t#ice as much as paved roads do &! maintaining dirt roads costs t#ice as much as it does for paved roads '! to maintain dirt roads costs t#ice as much as for paved roads (47. &iscrimination in #ages paid in occupations that are predominantly male over the predominantly female have given rise to substantial differentials bet#een the #age of housepainters and secretaries and bet#een the #ages of parking-lot attendants and library assistants. A! paid in occupations that are predominantly male over the predominantly female have "! paid in occupations that are predominantly make over those that are predominantly female have $! that favors predominantly male occupations over the predominantly female have &! that favors predominantly male occupations over those that are predominantly female has '! paid in predominantly male occupations over the predominantly female has (4:. &isplays of the aurora borealis% or northern lights% can heat the atmosphere over the arctic enough to affect the tra6ectories of ballistic missiles% induce electric currents that can cause blackouts in some areas and corrosion in north-south pipelines. A! to affect the tra6ectories of ballistic missiles% induce "! that the tra6ectories of ballistic missiles are affected% induce

;7

$! that it affects the tra6ectories of ballistic missiles% induces &! that the tra6ectories of ballistic missiles are affected and induces '! to affect the tra6ectories of ballistic missiles and induce (4*. &istinguished architecture re5uires the e<penditure of large sums of money% even if it is by no means certain that the e<penditure of large sums of money produce distinguished architecture. A! even if it is by no means certain that the e<penditure of large sums of money produce "! even if it is by no means certain that the e<penditure of large sums of money #ill produce $! even though there is no certainty that the e<penditure of money in large sums produces &! even though it is by no means certain that the e<penditure of large sums of money produces '! though there is no certainty as to the e<penditure of money in large sums producing (40. &octors generally agree that such factors as cigarette smoking% eating rich foods high in fats% and alcohol consumption not only do damage by themselves but also aggravate genetic predispositions to#ard certain diseases. A! not only do damage by themselves but also aggravate "! do damage by themselves but also are aggravating to $! are damaging by themselves but also are aggravating &! not only do damage by themselves% they are also aggravating to '! are doing damage by themselves% and they are also aggravating (4). &octors still kno# little about ho# the .isteria bacterium is spread and #hy the disease it causes% listeriosis% afflicts some people in a contaminated area though it spares many others. A! though it spares "! #here it spares $! despite sparing &! #hen sparing '! #hile sparing (72. &omestic automobile manufacturers have invested millions of dollars into research to develop cars more gasoline-efficient even than presently on the road. A! into research to develop cars more gasoline-efficient even than presently on the road "! into research for developing even more gasoline-efficient cars on the road than at present $! for research for cars to be developed that are more gasoline-efficient even than presently on the road &! in research to develop cars even more gasoline-efficient than those at present on the road '! in research for developing cars that are even more gasoline-efficient than presently on the road (71. &o#n9oning% 9oning that typically results in the reduction of housing density% allo#s for more open space in areas #here little #ater or services e<ist.

SC Strategy

;8

A! little #ater or services e<ist "! little #ater or services e<ists $! fe# services and little #ater e<ists &! there is little #ater or services available '! there are fe# services and little available #ater (7(. &r. >akuta3s research among >ispanic children in the +nited ,tates indicates that the more the children use both ,panish and 'nglish% their intellectual advantage is greater in skills underlying reading ability and nonverbal logic. A! their intellectual advantage is greater in skills underlying reading ability and nonverbal logic "! their intellectual advantage is the greater in skills underlaying reading ability and nonverbal logic $! the greater their intellectual advantage in skills underlying reading ability and nonverbal logic &! in skills that underlay reading ability and nonverbal logic% their intellectual advantage is the greater '! in skills underlying reading ability and nonverbal logic% the greater intellectual advantage is theirs (7/. &r. >egsted argues that 6ust as polio vaccine is given to every person to protect the fe# #ho might actually contract polio% mass dietary change is needed to protect the significant number #ho are susceptible to the life-threatening effects of press eating habits. A! 6ust as polio vaccine is given to every person to protect the fe# #ho might actually contract polio "! like polio vaccine% #hich is given to every person to protect the fe# #ho might contract polio $! similar to polio vaccine #hich is given to every person in order to protect the fe# #ho might actually contract polio &! #hile% to protect the fe# #ho might actually contract polio% polio vaccine is given to every person '! similar to the giving of polio vaccine to every person in order to protect the fe# #ho might contract polio actually (74. &r. ,ayre3s lecture recounted several little-kno#n episodes in the relations bet#een nations that illustrates #hat is #rong #ith alliances and treaties that do not have popular support. A! relations bet#een nations that illustrates "! relation of one nation #ith another that illustrates $! relations bet#een nations that illustrate &! relation of one nation #ith another and illustrate '! relations of nations that illustrates (77. &r. =onega#a #on the Nobel 1ri9e for discovering ho# the body can constantly change its

;9

genes to fashion a seeming unlimited number of antibodies% each specifically targeted at an invading microbe or foreign substance. A! seeming unlimited number of antibodies% each specifically targeted at "! seeming unlimited number of antibodies% each targeted specifically to $! seeming unlimited number of antibodies% all specifically targeted at &! seemingly unlimited number of antibodies% all of them targeted specifically to '! seemingly unlimited number of antibodies% each targeted specifically at (7:. &uring an era #hen interracial tensions in the +nited ,tates have run high% Dosa 1arks became a 5uiet% unassuming symbol of the continued struggle for human dignity. A! have run high "! ran high $! had run highly &! run high '! #ere running highly (7*. &uring an ice age% the buildup of ice at the poles and the drop in #ater levels near the e5uator speed up the 'arth3s rotation% like a spinning figure skater #hose speed increases #hen her arms are dra#n in. A! like a spinning figure skater #hose speed increases #hen her arms are dra#n in "! like the increased speed of a figure skater #hen her arms are dra#n in $! like a figure skater #ho increases speed #hile spinning #ith her arms dra#n in &! 6ust as a spinning figure skater #ho increases speed by dra#ing in her arms '! 6ust as a spinning figure skater increases speed by dra#ing in her arms (70. &uring her lecture the speaker used map to clarify directional terms% for not everyone in attendance #as kno#ledgeable that #inds are designated by the direction from #hich they come. A! for not everyone in attendance #as kno#ledgeable "! for everyone in attendance did not kno# $! #ith everyone in attendance not kno#ing &! #ith everyone attending not kno#ledgeable '! for not everyone attending kne# (7). &uring Doosevelt3s years in office "lack Americans began voting for &emocrats rather than Depublicans in national elections% but "lack support for &emocrats at the state and local levels developed only after #hen civil rights legislation #as supported by >arry =ruman. A! developed only after #hen civil rights legislation #as supported by >arry =ruman "! developed only after #hen >arry =ruman supported civil rights legislation $! developed only after >arry =ruman3s support of civil rights legislation

SC Strategy

;;

&! develops only at the time after the supporting of civil rights legislation by >arry =ruman '! developed only after there being >arry =ruman3s support of civil rights legislation (:2. &uring the 1)023s appro<imately I72 billion in private investment capital is estimated to have left 8e<ico and added to the strain on the country3s debt-ridden economy. A! &uring the 1)023s appro<imately I72 billion in private investment capital is estimated to have left 8e<ico and added "! &uring the 1)023s it is estimated that appro<imately I72 billion in private investment capital left 8e<ico and added $! It is estimated that there #as appro<imately I72 billion in private investment capital that left 8e<ico during the 1)023s and added &! It is estimated that during the 1)023s appro<imately I72 billion in private investment capital left 8e<ico% adding '! Appro<imately I72 billion in private investment capital is estimated as having left 8e<ico during the 1)023s% adding (:1. &uring the 1)02s it became clear that soliciting private funds #as far more efficient for environmentalists #ho sought financial aid than to go to state or federal agencies. A! that soliciting private funds #as far more efficient for environmentalists #ho sought financial aid "! that for environmentalists #ho sought financial aid% it #as far more efficient to solicit private funds $! that for environmentalists seeking financial aid% private organi9ations #ere far more efficient to go to &! for environmentalists seeking financial aid% going to private organi9ations #as far more efficient '! for environmentalists #ho sought financial aid% private organi9ations #ere far more efficient (:(. &uring the early years of 'uropean settlement on a continent that #as vie#ed as #ilderness by the ne#comers% Native Americans% intimately kno#ing the ecology of the land% #ere a help in the rescuing of many 1ilgrims and pioneers from hardship% or even death. A! Native Americans% intimately kno#ing the ecology of the land% #ere a help in the rescuing of "! Native Americans kne# the ecology and the land intimately and this enabled them to help in the rescue of $! Native Americans% #ith their intimate kno#ledge of the ecology of the land% helped to rescue &! having intimate kno#ledge of the ecology of the land% Native Americans helped the rescue of '! kno#ing intimately the ecology of the land% Native Americans helped to rescue (:/. &uring the first nine months of 1)*)% te<tbook publishers incurred substantial costs for creating products that% due to a decline in public funding for instructional material% never #ere

;I

sold. A! funding for instructional material% never #ere "! funding for instructional material% never #as $! funding% the instructional material% #as never &! funding for instructional material% the products #ere never '! funding% they #ere never (:4. &uring the first one hundred fifty years of the e<istence of this republic% no one e<pected the press #as fairE ne#spapers #ere mostly shrill% scurrilous% and partisan. A! #as "! to be $! of being &! should be '! had to be (:7. &uring the first year after the corporate reorgani9ation% no one considered the management #as #ell-organi9edE managers #ere largely untrained and directionless. A! #as #ell-organi9ed "! #ell-organi9ed $! #ere #ell-organi9ed &! seemed to be #ell-organi9ed '! seemed #ell-organi9ed (::. &uring the -reat &epression% industrial output fell by nearly fifty percent from its peak in 1)() do#n to its nadir in 1)//. A! &uring the -reat &epression% industrial output fell by nearly fifty percent from its peak in 1)() do#n to its nadir in 1)//. "! &uring the -reat &epression% industrial output fell by nearly fifty percent from its peak in 1)() to its nadir in 1)//. $! At the time of the -reat &epression% industrial output fell by almost fifty percent from its 1)() peak do#n to its 1)// nadir. &! At the time of the -reat &epression% industrial output fell from its peak in 1)()% by nearly fifty percent% to it nadir in 1)//. '! &uring the -reat &epression% industrial output fell from its peak in 1)() to its nadir in 1)// by nearly fifty percent. (:*. &uring the nineteenth century 'mily 'den and ?anny 1arks 6ourneyed throughout India% sketching and keeping 6ournals forming the basis of ne#s reports about the princely states #here they had visited.

SC Strategy

;J

A! forming the basis of ne#s reports about the princely states #here they had "! that #ere forming the basis of ne#s reports about the princely states $! to form the basis of ne#s reports about the princely states #hich they have &! #hich had formed the basis of ne#s reports about the princely states #here they had '! that formed the basis of ne#s reports about the princely states they (:0. &uring the recession of 1)*/% home mortgage foreclosures resulted in tens of thousands of Americans being evicted from homes that they can no longer afford monthly payments. A! that they can "! that they could $! on #hich they can &! because they can '! for #hich they could (:). &uring the Denaissance% scholars #ere uncertain as to the location of =roy% and by the eighteenth century many historians doubted that =roy had ever e<isted. A! doubted that =roy had ever e<isted "! doubt that =roy has ever e<isted $! #ere in doubt as to the e<istence of =roy &! #ere doubtful concerning =roy3s e<istence '! had doubts about =roy3s ever e<isting (*2. 'ach of >eming#ay3s #ivesH>adley Dichardson% 1auline 1feiffer% 8artha -elhorn% and 8ary WelshH#ere strong and interesting #omen% very different from the often pallid #omen #ho populate his novels. A! 'ach of >eming#ay3s #ivesH>adley Dichardson% 1auline 1feiffer% 8artha -elhorn% and 8ary WelshH#ere strong and interesting #omen% "! >adley Dichardson% 1auline 1feiffer% 8artha -elhorn% and 8ary WelshHeach of them >eming#ay3s #ivesH#ere strong and interesting #omen% $! >eming#ay3s #ivesH>adley Dichardson% 1auline 1feiffer% 8artha -elhorn% and 8ary Welsh H#ere all strong and interesting #omen% &! ,trong and interesting #omenH>adley Dichardson% 1auline 1feiffer% 8artha -elhorn% and 8ary WelshHeach a #ife of >eming#ay% #as '! ,trong and interesting #omenH>adley Dichardson% 1auline 1feiffer% 8artha -elhorn% and 8ary WelshHevery one of >eming#ay3s #ives #ere (*1. 'aten in the 8editerranean countries% northern 'uropeans vie#ed the tomato #ith suspicion% for they assumed it had poisonous properties because of its relationship to deadly nightshade. A! northern 'uropeans vie#ed the tomato #ith suspicion% for they

IE

"! northern 'uropeans #ere suspicious of the tomato% and they $! the tomato #as vie#ed #ith suspicion by northern 'uropeans% #ho &! the tomato #as suspicious to northern 'uropeans% and it #as '! the tomato #as vie#ed #ith suspicion by northern 'uropeans% it being (*(. 'ating salt#ater fish may significantly reduce the risk of heart attacks and also aid for sufferers of rheumatoid arthritis and asthma% according to three research studies published in the Ne# 'ngland Cournal of 8edicine. A! significantly reduce the risk of heart attacks and also aid for "! be significant in reducing the risk of heart attacks and aid for $! significantly reduce the risk of heart attacks and aid &! cause a significant reduction in the risk of heart attacks and aid to '! significantly reduce the risk of heart attacks as #ell as aiding (*/. 'fforts to e5uali9e the funds available to school districts% a ma6or goal of education reformers and many states in the 1)*23s% has not significantly reduced the gaps e<isting bet#een the richest and poorest districts. A! has not significantly reduced the gaps e<isting "! has not been significant in reducing the gap that e<ists $! has not made a significant reduction in the gap that e<ists &! have not significantly reduced the gap that e<ists '! have not been significant in a reduction of the gaps e<isting (*4. 'gyptians are credited as having pioneered embalming methods as long ago as (:72 ".$. A! as having "! #ith having $! to have &! as the ones #ho '! for being the ones #ho (*7. 'l Nino% the periodic abnormal #arming of the sea surface off 1eru% a phenomenon in #hich changes in the ocean and atmosphere combine allo#ing the #arm #ater that has accumulated in the #estern 1acific to flo# back to the east. A! a phenomenon in #hich changes in the ocean and atmosphere combine allo#ing the #arm #ater that has accumulated "! a phenomenon #here changes in the ocean and atmosphere are combining to allo# the #arm #ater that is accumulating $! a phenomenon in #hich ocean and atmosphere changes combine and #hich allo#s the #arm #ater that is accumulated

SC Strategy

I$

&! is a phenomenon in #hich changes in the ocean and atmosphere combine to allo# the #arm #ater that has accumulated '! is a phenomenon #here ocean and atmosphere changes are combining and allo# the #arm #ater accumulating (*:. 'rasmus 8ontanus% a seventeenth-century farce #ritten by .ud#ig >olberg% both predates and resembles 8oliere3s =artuffe and is therefore thought to be one of 8oliere3s sources. A! both predates and resembles "! it both predates and resembles $! both predated and resembles &! has both predated and resembled '! because it both predated and resembled (**. 'urope3s travel industry is suffering as a result of a sluggish economy% a stretch of bad #eather% as #ell as the chilling effects of terrorist activity that is persistent. A! as #ell as the chilling effects of terrorist activity that is persistent "! and the chilling effect of terrorist activity that is persistent $! but persistent terrorist activity has had a chilling effect too &! and the chilling effects of persistent terrorist activity '! as #ell as the chilling effects of terrorist activity that persists (*0. 'uropeans have long kno#n that eating 5uail sometimes makes the eater ill% but only recently has it been established that the illness is caused by a to<in present in the 5uail3s body only under certain conditions. A! 'uropeans have long kno#n that eating 5uail sometimes makes "! 'uropeans have long kno#n 5uail eating is sometimes able to make $! 'ating 5uail has long been kno#n to 'uropeans to sometimes make &! It has long been kno#n to 'uropeans that 5uail eating #ill sometimes make '! It has long been kno#n to 'uropeans that 5uail% #hen it is eaten% has sometimes made (*). 'ven as they never forgave the $rusaders #ho overran their homeland% the ,yrians have never absolved the ?rench for taking territory from them. A! 'ven as they never forgave "! While they never forgave $! Cust like they never forgave &! ,imilarly to not forgiving '! In spite of their never forgiving (02. 'ven astronomers #ere ama9ed at the success of the Neptune flyby% #hich produced a photograph of a previously undetected moonE this is likely to result in increased governmental

I'

support for the hitherto neglected +.,. space program. A! this is likely to result in "! that #ill cause $! and #hich is likely to result in &! this success is likely to result in '! it is likely to result #ith (01. 'ven their most ardent champions concede that no less than a technical or scientific breakthrough is necessary before solar cells can meet the goal of providing one percent of the nation3s energy needs. A! that no less than a technical or scientific breakthrough is necessary "! that nothing other than a technical or scientific breakthrough is needed $! that a technical or scientific breakthrough is necessary &! the necessity for an occurrence of a technical or scientific breakthrough '! the necessity for a technical or scientific breakthrough occurring (0(. 'ven though "Mla "artNk3s music has proved less popular than Igor ,travinsky3s and less influential than Arnold ,chonberg3s% it is no less important. A! ,travinsky3s and less influential than Arnold ,chonberg3s% it "! ,travinsky3s and less influential than Arnold ,chonberg3s% he $! ,travinsky3s is and less influential than Arnold ,chonberg3s is% it &! ,travinsky and not as influential as Arnold ,chonberg% he '! ,travinsky and not as influential as Arnold ,chonberg% it (0/. 'ven though its per capita food supply hardly increased during t#o decades% stringent rationing and planned distribution have allo#ed the 1eople3s Depublic of $hina to ensure nutritional levels of (%222 calories per person per day for its population. A! 'ven though its per capita food supply hardly increased during "! 'ven though its per capita food supply has hardly increased in $! &espite its per capita food supply hardly increasing over &! &espite there being hardly any increase in its per capita food supply during '! Although there is hardly any increase in per capita food supply for (04. 'ven though the direct costs of malpractice disputes amounts to a sum lo#er than one percent of the I741 billion the nation spent on health care last year% doctors say fear of la#suits plays ma6or role in health-care inflation. A! amounts to a sum lo#er "! amounts to less $! amounted to less

SC Strategy

I*

&! amounted to lo#er '! amounted to a lo#er sum (07. 'ven though the state has spent ten years and seven million dollars planning a reservoir along the +bi Diver% the pro6ect #ill have to be abandoned as a result of the river becoming so heavily polluted. A! #ill have to be abandoned as a result of the river becoming so heavily polluted "! is to be abandoned on account of the heavy pollution #hich the river received $! had to be abandoned because the river had received such heavy pollution &! has to be abandoned because of the river and its heavy pollution '! must be abandoned because the river has become so heavily polluted (0:. 'ven today% a century after 1asteur developed the first vaccine% rabies almost al#ays kills its victims unless inoculated in the earliest stages of the disease. A! its victims unless inoculated "! its victims unless they are inoculated $! its victims unless inoculation is done &! the victims unless there is an inoculation '! the victims unless inoculated (0*. 'veryone participating in the early sociological study committed a crucial methodological error by failing to fully consider alternative #ays of classifying their data. A! by failing to fully consider alternative #ays of classifying their "! by failing fully to consider alternative #ays of classifying their $! they failed to consider alternative #ays to classify their &! by not fully considering alternative #ays to classify the '! by not considering fully alternative #ays of classifying his or her (00. '<cept for a concert performance that the composer himself staged in 1)11% ,cott Coplin3s ragtime opera =reemonisha #as not produced until 1)*(% si<ty-one years after its completion. A! '<cept for a concert performance that the composer himself staged "! '<cept for a concert performance #ith the composer himself staging it $! "esides a concert performance being staged by the composer himself &! '<cepting a concert performance that the composer himself staged '! With the e<ception of a concert performance #ith the staging done by the composer himself (0). '<ecutives and federal officials say that the use of crack and cocaine is gro#ing rapidly among #orkers% significantly compounding the effects of drug and alcohol abuse% #hich already are a cost to business of more than I122 billion a year. A! significantly compounding the effects of drug and alcohol abuse% #hich already are a cost to

I7

business of "! significantly compounding the effects of drug and alcohol abuse% #hich already cost business $! significantly compounding the effects of drug and alcohol abuse% already #ith business costs of &! significant in compounding the effects of drug and alcohol abuse% and already costing business '! significant in compounding the effects of drug and alcohol abuse% and already costs business ()2. ?aced #ith an estimated I( billion budget gap% the city3s mayor proposed a nearly 1* percent reduction in the amount allocated the previous year to maintain the city3s ma6or cultural institutions and to subsidi9e hundreds of local arts groups. A! proposed a nearly 1* percent reduction in the amount allocated the previous year to maintain the city3s ma6or cultural institutions and to subsidi9e "! proposed a reduction from the previous year of nearly 1* percent in the amount it #as allocating to maintain the city3s ma6or cultural institutions and for subsidi9ing $! proposed to reduce% by nearly 1* percent% the amount from the previous year that #as allocated for the maintenance of the city3s ma6or cultural institutions and to subsidi9e &! has proposed a reduction from the previous year of nearly 1* percent of the amount it #as allocating for maintaining the city3s ma6or cultural institutions% and to subsidi9e '! #as proposing that the amount they #ere allocating be reduced by nearly 1* percent from the previous year for maintaining the city3s ma6or cultural institutions and for the subsidi9ation ()1. ?actory outlet stores% operated by manufacturers% are usually located miles from do#nto#n and regional shopping centers so as not directly to be competitive against department stores in the same trading area. A! so as not directly to be competitive against "! in order for them not to have direct competition #ith $! so that they do not compete directly #ith &! in order that they are not directly competitive against '! for the purpose of not competing directly #ith ()(. ?ederal authorities involved in the investigation have found the local #itnesses are difficult to locate% reticent% and are suspicious of strangers. A! the local #itnesses are difficult to locate% reticent% and are "! local #itnesses to be difficult to locate% reticent% and are $! that local #itnesses are difficult to locate% reticent% and &! local #itnesses are difficult to locate and reticent% and they are '! that local #itnesses are difficult to locate and reticent% and they are ()/. ?ederal incentives no# encourage investing capital in commercial office buildings despite

SC Strategy

I8

vacancy rates in e<isting structures that are e<ceptionally high and no demand for ne# construction. A! investing capital in commercial office buildings despite vacancy rates in e<isting structures that are e<ceptionally high and "! capital investment in commercial office buildings% even though vacancy rates in e<isting structures are e<ceptionally high and there is $! capital to be invested in commercial office buildings even though there are e<ceptionally high vacancy rates in e<isting structures #ith &! investing capital in commercial office buildings even though the vacancy rates are e<ceptionally high in e<isting structures #ith '! capital investment in commercial office buildings despite vacancy rates in e<isting structures that are e<ceptionally high% and although there is ()4. ?ederal legislation establishing a fund for the cleanup of sites damaged by to<ic chemicals permits compensating state governments for damage to their natural resources but does not allo# claims for in6ury to people. A! compensating state governments for damage to "! compensating state governments for the damaging of $! giving state governments compensation for damaging &! giving compensation to state governments for the damage of '! the giving of compensation to state governments for damaging ()7. ?ederally imposed restrictions on ho# much they may pay small savers has made difficulties for savings banks as they are competing #ith such unregulated investment vehicles as money market certificates. A! has made difficulties for savings banks as they are competing #ith such "! has made difficulties for savings banks competing #ith such $! have made difficulties for savings banks as they are competing #ith &! have made it difficult for savings banks to compete #ith such '! have made it difficult for savings banks as they are competing #ith such ():. ?e# people reali9e that the chance of accidental in6ury or death may be as great or greater in the safety of their o#n homes than in a plane or on the road. A! may be as great or greater in the safety of their o#n homes than "! is at least as great or greater in the safety of their o#n homes than $! might be so great or greater in the safety of their o#n home as &! may be at least as great in the safety of their o#n homes as '! can be at least so great in the safety of their o#n home as ()*. ?ifty-t#o percent of +nited ,tates high school graduates go on to college% compared #ith

I9

$anada3s thirty-five percent and -reat "ritain% Capan% and West -ermany3s fifteen percent. A! ?ifty-t#o percent of +nited ,tates high school graduates go on to college% compared #ith $anada3s thirty-five percent and -reat "ritain% Capan% and West -ermany3s fifteen percent. "! ?ifty-t#o percent of +nited ,tates high school graduates go on to collegeE in $anada it is thirty-five percent and in -reat "ritain% Capan% and West -ermany it is fifteen percent. $! In the +nited ,tates% ?ifty-t#o percent of high school graduates go on to college% compared #ith thirty-five percent in $anada and fifteen percent in -reat "ritain% Capan% and West -ermany. &! =he percentage of high school graduates in the +nited ,tates #ho go on to college is fiftyt#o% compared #ith $anada3s thirty-five percent% -reat "ritain3s fifteen% Capan3s fifteen% and West -ermany3s fifteen. '! =he percentage of +nited ,tates high school graduates going on to college is fifty-t#o that in $anada is thirty-five% and that in -reat "ritain% Capan% and West -ermany is fifteen. ()0. ?ive fledgling sea eagles left their nests in #estern ,cotland this summer% bringing to /4 the number of #ild birds successfully raised since transplants from Nor#ay began in 1)*7. A! bringing "! and brings $! and it brings &! and it brought '! and brought ()). ?lorida #ill gain another 5uarter-million 6obs this year alone% many of them in high-paying fields like electronics and banking% making the state3s economy far more diversified than ten years ago. A! high-paying fields like electronics and banking% making the state3s economy far more diversified than "! high-paying fields like electronics and banking% and making the state3s economy far more diversified than its economy $! high-paying fields such as electronics and banking% to make the state3s economy far more diversified than &! such high-paying fields as electronics and banking% making the state3s economy far more diversified than it #as '! such high-paying fields as electronics and banking% and make the state3s economy far more diversified than it #as /22. ?ollo#ing the destruction of the space shuttle $hallenger% investigators concluded that many key people employed by the National Aeronautics and ,pace Administration and its contractors #ork an e<cessive amount of overtime that has the potential of causing errors in 6udgment. A! overtime that has the potential of causing

SC Strategy

I;

"! overtime that has the potential to cause $! overtime that potentially can cause &! overtime% a practice that has the potential for causing '! overtime% a practice that can% potentially% cause /21. ?ollo#ing the nutrition board3s advice on salt consumption #ould mean a virtual end of the use of salt in cooking% an avoidance of obviously salty foods% and reducing the reliance on processed foods that contain significant amounts of often hidden sodium. A! reducing the reliance on processed foods that contain significant amounts of often hidden sodium "! reducing the reliance on processed foods containing often hidden but significant amounts of sodium $! a reduction of the reliance on processed foods% containing as they do often hidden sodium in significant amounts &! a reduced reliance on the significant amounts of hidden sodium often contained in processed foods '! a reduced reliance on processed foods that contain significant but often hidden amounts of sodium /2(. ?or all his professed disdain of such activities% Auden #as an inveterate literary gossip. A! ?or all his professed disdain of such activities "! >aving al#ays professed disdain for such activities $! All such activities #ere% he professed% disdained% and &! 1rofessing that all such activities #ere disdained '! In spite of professions of disdaining all such activities /2/. ?or almost a hundred years after having its beginning in 1*00% 'ngland e<iled some 1:2%222 criminals to Australia. A! ?or almost a hundred years after having its beginning in 1*00 "! "eginning in 1*00 for a period of a hundred years $! "eginning a period of almost a hundred years% in 1*00 &! &uring a hundred years% a period beginning in 1*00 '! Gver a period of a hundred years beginning in 1*00 /24. ?or many people% household labor remains demanding even if able to afford household appliances their grandparents #ould find a miracle. A! even if able to afford household appliances their grandparents #ould find a miracle "! despite being able to afford household appliances their grandparents #ould find a miracle $! even if they can afford household appliances their grandparents #ould have found miraculous

II

&! although they could afford household appliances their grandparents #ould find miraculous '! even if they are able to afford household appliances #hich #ould have been a miracle to their grandparents /27. ?or many travelers% charter vacations often turn out to cost considerably more than they originally seemed. A! they originally seemed "! they originally seem to $! they seemingly #ould cost originally &! it seemed originally '! it originally seemed they #ould /2:. ?or members of the seventeenth-century Ashanti nation in Africa% animal-hide shields #ith #ooden frames #ere essential items of military e5uipment% a method to protect #arriors against enemy arro#s and spears. A! a method to protect "! as a method protecting $! protecting &! as a protection of '! to protect /2*. ?or most consumers% the price of automobile insurance continues to rise annually% even if free of damage claims and moving violations. A! even if "! despite being $! even if they are &! although they may be '! even if remaining /20. ?or protection from the summer sun% the 8o6ave lived in open-sided% flat-topped d#ellings kno#n as shades% each a roof of poles and arro#head supported by posts set in a rectangle. A! each a roof of poles and arro#head "! each a roof of poles and arro#head that are being $! #ith each being a roof of poles and arro#head &! #ith roofs of poles and arro#head to be '! #ith roofs of poles and arro#head that are /2). ?or some birds the sense of smell appears to play a role in navigation% since pigeons #ith surgically removed olfactory nerves #ere found to have increased difficulties in homing. A! #ere found to have increased difficulties

SC Strategy

IJ

"! have been found to have increased difficulty $! #ere found to have increasing difficulty &! had been found to have increased difficulties '! have been found to have increasing difficulties /12. ?or some reason the ne# consultant treats his clients like idiots% talking to them like they #ere mentally deficient and incapable of understanding more than the simplest ideas. A! like idiots% talking to them like they "! as if they #ere idiots% talking to them like they $! like idiots% talking to them as if they &! as idiots% talking to them like they '! like idiots #ho /11. ?oreign investors% because of their gro#ing confidence in their capability for making profitable investments in the +nited ,tates% have been led to move from passive involvement in commercial real estate partnerships to active development of their o#n increasingly ambitious pro6ects. A! ?oreign investors% because of their gro#ing confidence in their capability for making profitable investments in the +nited ,tates% have been led "! ?oreign investors% gro#ing confident about their capability for making profitable investments in the +nited ,tates% has led them $! -ro#ing confidence in their ability to make profitable investments in the +nited ,tates has led foreign investors &! -ro#ing confidence in their ability for making profitable investments in the +nited ,tates have led foreign investors '! -ro#ing confident about their capabilities for making profitable investments in the +nited ,tates% foreign investors have been led /1(. ?ormulas for cash flo# and the ratio of debt to e5uity do not apply to ne# small businesses in the same #ay as they do to established big businesses% because they are gro#ing and are seldom in e5uilibrium. A! ?ormulas for cash flo# and the ratio of debt to e5uity do not apply to ne# small businesses in the same #ay as they do to established big businesses% because they are gro#ing and are seldom in e5uilibrium. "! "ecause they are gro#ing and are seldom in e5uilibrium% formulas for cash flo# and the ratio of debt to e5uity do not apply to ne# small businesses in the same #ay as they do to established big businesses. $! "ecause they are gro#ing and are seldom in e5uilibrium% ne# small businesses are not sub6ect to the same applicability of formulas for cash flo# and the ratio of debt to e5uity as established big businesses.

JE

&! "ecause ne# small businesses are gro#ing and are seldom in e5uilibrium% formulas for cash flo# and the ratio of debt to e5uity do not apply to them in the same #ay as to established big businesses. '! Ne# small businesses are not sub6ect to the applicability of formulas for cash flo# and the ratio of debt to e5uity in the same #ay as established big businesses% because they are gro#ing and are seldom in e5uilibrium. /1/. ?ound throughout $entral and ,outh America% sloths hang from trees by long rubbery limbs and sleep fifteen hours a day% moving infre5uently enough that t#o species of algae gro# on its coat and bet#een its toes. A! sloths hang from trees by long rubbery limbs and sleep fifteen hours a day% moving infre5uently enough "! sloths hang from trees by long rubbery limbs% they sleep fifteen hours a day% and #ith such infre5uent movements $! sloths use their long rubbery limbs to hang from trees% sleep fifteen hours a day% and move so infre5uently &! the sloth hangs from trees by its long rubbery limbs% sleeping fifteen hours a day and moving so infre5uently '! the sloth hangs from trees by its long rubbery limbs% sleeps fifteen hours a day% and it moves infre5uently enough /14. ?ounded in 1)0/% the maga9ine increased its circulation more than double since then% and its advertising. A! increased its circulation more than double since then% "! has since increased its circulation more than double% $! has since more than doubled its circulation &! since then more than doubled its circulation '! more than doubled its circulation since then /17. ?ramed by traitorous colleagues% Alfred &reyfus #as imprisoned for t#elve years before there #as e<oneration and his freedom. A! there #as e<oneration and his freedom "! he #as to be e<onerated #ith freedom $! being e<onerated and freed &! e<oneration and his freedom '! being freed% having been e<onerated /1:. ?rances Wright3s book on America contrasted the republicanism of the +nited ,tates #ith #hat she sa# as the aristocratic and corrupt institutions of 'ngland. A! #ith #hat she sa# as

SC Strategy

J$

"! #ith that #hich she sa# to be $! to that she sa# being &! and that #hich she sa# as '! and #hat she sa# to be /1*. ?reedman3s survey sho#ed that people living in small to#ns and rural areas consider themselves no happier than do people living in big cities. A! no happier than do people living "! not any happier than do people living $! not any happier than do people #ho live &! no happier than are people #ho are living '! not as happy as are people #ho live /10. ?rom 1):7 on% @ugoslavia3s standard of living has soared% but unemployment and prices too. A! but unemployment and prices too "! and also unemployment and prices $! but so have unemployment and prices &! and so also unemployment and prices '! but so did unemployment and prices /1). ?rom 1)0( to 1)0* sales of ne# small boats increased bet#een five and ten percent annually. A! ?rom 1)0( to 1)0* sales of ne# small boats increased bet#een five and ten percent annually. "! ?ive to ten percent is the annual increase in sales of ne# small boats in the years 1)0( to 1)0*. $! ,ales of ne# small boats have increased annually five and ten percent in the years 1)0( to 1)0*. &! Annually an increase of five to ten percent has occurred bet#een 1)0( and 1)0* in the sales of ne# small boats. '! Gccurring from 1)0( to 1)0* #as an annual increase of five and ten percent in the sales of ne# small boats. /(2. ?rom the bark of the paper birch tree the 8enomini crafted a canoe about t#enty feet long and t#o feet #ide% #ith small ribs and rails of cedar% #hich could carry four persons or eight hundred pounds of baggage so light that a person could easily portage it around impeding rapids. A! baggage so light "! baggage being so light $! baggage% yet being so light &! baggage% and so light '! baggage yet #as so light

J'

/(1.

?rom the earliest days of the tribe% kinship determined the #ay in #hich the G6ib#a society organi9ed its labor% provided access to its resources% and defined rights and obligations involved in the distribution and consumption of those resources. A! and defined rights and obligations involved in the distribution and consumption of those resources "! defining rights and obligations involved in their distribution and consumption $! and defined rights and obligations as they #ere involved in its distribution and consumption &! #hose rights and obligations #ere defined in their distribution and consumption '! the distribution and consumption of them defined by rights and obligations

/((.

?rom the time of its defeat by the -ermans in 1)42 until its liberation in 1)44% ?rance #as a bitter and divided countryE a kind of civil #ar raged in the Bichy government bet#een those #ho #anted to collaborate #ith the Na9is #ith those #ho opposed them. A! bet#een those #ho #anted to collaborate #ith the Na9is #ith those #ho opposed "! bet#een those #ho #anted to collaborate #ith the Na9is and those #ho opposed $! bet#een those #anting to collaborate #ith the Na9is #ith those opposing &! among those #ho #anted to collaborate #ith the Na9is and those #ho opposed '! among those #anting to collaborate #ith the Na9is #ith those opposing

/(/.

-alileo #as convinced that natural phenomena% as manifestations of the la#s of physics% #ould appear the same to someone on the deck of a ship moving smoothly and uniformly through the #ater as a person standing on land. A! #ater as a "! #ater as to a $! #aterE 6ust as it #ould to &! #ater% as it #ould to the '! #aterE 6ust as to the

/(4.

-all3s hypothesis of there being different mental functions locali9ed in different parts of the brain is #idely accepted today. A! of there being different mental functions locali9ed in different parts of the brain is #idely accepted today "! of different mental functions that are locali9ed in different parts of the brain is #idely accepted today $! that different mental functions are locali9ed in different parts of the brain is #idely accepted today &! #hich is that there are different mental functions locali9ed in different parts of the brain is #idely accepted today '! #hich is #idely accepted today is that there are different mental functions locali9ed in

SC Strategy

J*

different parts of the brain /(7. -eologists believe that the "ering land bridge% over #hich human beings are thought to have first entered the Americans% disappeared about 14%222 years ago #hen massive glaciers melted and caused the sea level to rise several hundred feet #orld#ide. A! are thought to have first entered "! #ere thought first to enter $! #ere thought at first to enter &! are thought of as first entering '! #ere thought to first enter /(:. -eologists believe that the #arning signs for a ma6or earth5uake may include sudden fluctuations in local seismic activity% tilting and other deformations of the 'arth3s crust% changing the measured stain across a fault 9one% and varying the electrical properties of underground rocks. A! changing the measured strain across a fault 9one% and varying "! changing measurements of the strain across a fault 9one% and varying $! changing the strain as measured across a fault 9one% and variations of &! changes in the measured strain across a fault 9one% and variations in '! changes in measurements of the strain across a fault 9one% and variations among /(*. -eorge ,and Aurore .ucile &upin! #as one of the first 'uropean #riters to consider the rural poor to be legitimate sub6ects for literature and portray these #ith sympathy and respect in her novels. A! to be legitimate sub6ects for literature and portray these "! should be legitimate sub6ects for literature and portray these $! as being legitimate sub6ects for literature and portraying them &! as if they #ere legitimate sub6ects for literature and portray them '! legitimate sub6ects for literature and to portray them /(0. -reen anole li9ards% familiar to schoolchildren as chameleons% have recently become familiar to biologists as an e<cellent animal for laboratory studies of the interaction bet#een stimuli #ith hormones. A! an e<cellent animal for laboratory studies of the interaction bet#een stimuli #ith "! an e<cellent animal for laboratory studies of interaction of stimuli and $! being e<cellent animals for laboratory studies of the interaction bet#een stimuli #ith &! e<cellent animals for laboratory studies of the interaction bet#een stimuli #ith '! e<cellent animals for laboratory studies of the interaction of stimuli and /(). -ro#ing competitive pressures may be encouraging auditors to bend the rules in favor of clientsE auditors may% for instance% allo# a 5uestionable loan to remain on the books in order to

J7

maintain a bank3s profits on paper. A! clientsE auditors may% for instance% allo# "! clients% as an instance% to allo# $! clients% like to allo# &! clients% such as to be allo#ing '! clientsE #hich might% as an instance% be the allo#ing of //2. >aving the right hand and arm being crippled by a sniper3s bullet during the ?irst World War% >orace 1ippin% a "lack American painter% #orked by holding the brush in his right hand and guiding its movements #ith his left. A! >aving the right hand and arm being crippled by a sniper3s bullet during the ?irst World War "! In spite of his right hand and arm being crippled by a sniper3s bullet during the ?irst World War $! "ecause there had been a sniper3s bullet during the ?irst World War that crippled his right hand and arm &! =he right hand and arm being crippled by a sniper3s bullet during the ?irst World War '! >is right hand and arm crippled by a sniper3s bullet during the ?irst World War //1. >ealth officials estimate that /7 million Africans are in danger of contracting trypanosomiasis% or African sleeping sickness% a parasitic disease spread by the bites of tsetse flies. A! are in danger of contracting "! are in danger to contract $! have a danger of contracting &! are endangered by contraction '! have a danger that they #ill contract //(. >is studies of ice-polished rocks in his Alpine homeland% far outside the range of present-day glaciers% led .ouis Agassi9 in 10/* to propose the concept of an age in #hich great ice sheets had e<isted in no# currently temperate areas. A! in #hich great ice sheets had e<isted in no# currently temperate areas "! in #hich great ice sheets e<isted in #hat are no# temperate areas $! #hen great ice sheets e<isted #here there #ere areas no# temperate &! #hen great ice sheets had e<isted in current temperate areas '! #hen great ice sheets e<isted in areas no# that are temperate ///. >ouseflies that hatch in summer live only about three #eeks% but those that emerge in the cooler days of fall often live longer than si< months. A! #eeks% but those that emerge in the cooler days of fall often live

SC Strategy

J8

"! #eeks% but those that emerge in the cooler days of fall often live as long or $! #eeks% #hich is different from those that emerge in the cooler days of fall and often live &! #eeksE then those that emerge in the cooler days of fall often live as long as or '! #eeksE this is different from those that emerge in the cooler days of fall% #ho often live //4. >o#ever much +nited ,tates voters may agree that there is #aste in government and that the government as a #hole spends beyond its means% it is difficult to find broad support for a movement to#ard a minimal state. A! >o#ever much +nited ,tates voters may agree that "! &espite the agreement among +nited ,tates voters to the fact $! Although +nited ,tates voters agree &! 'ven though +nited ,tates voters may agree '! =here is agreement among +nited ,tates voters that

//7.

>umans have been damaging the environment for centuries by overcutting trees and farming too intensively% and though some protective measures% like the establishment of national forests and #ildlife sanctuaries% having been taken decades ago% great increases in population and in the intensity of industriali9ation are causing a #orld#ide ecological crisis. A! though some protective measures% like the establishment of national forests and #ildlife sanctuaries% having been taken decades ago% great increases in population "! though some protective measures% such as the establishment of national forests and #ildlife sanctuaries% #ere taken decades ago% great increases in population $! though some protective measures% such as establishing national forests and #ildlife sanctuaries% having been taken decades ago% great population increases &! #ith some protective measures% like establishing national forests and #ildlife sanctuaries that #ere taken decades ago% great increases in population '! #ith some protective measures% such as the establishment of national forests and #ildlife sanctuaries% having been taken decades ago% great population increases

//:.

Ideally% the professional career diplomat should help in the ongoing maintenance of an effective American foreign policy despite changes in administration. A! in the ongoing maintenance of "! in the maintaining of $! maintain &! to maintain and continue '! the maintenance of

//*.

Idioglossia is a phenomenon% incompletely understood at best% #here t#o persons develop a uni5ue and private language #ith highly original vocabulary and synta<.

J9

A! #here t#o persons develop a uni5ue and private language #ith "! #hen t#o persons develop a uni5ue and private language having $! in #hich t#o persons have uni5ue and private language development #ith &! having t#o persons #ho develop a uni5ue and private language that has '! in #hich t#o persons develop a uni5ue and private language #ith

//0.

If a single strain of plant is used for a given crop over a #ide area% a practice fostered by modern seed-marketing methods% it increases the likelihood that the impact of a single crop disease or pest #ill be disastrous. A! If a single strain of plant is used for a given crop over a #ide area% a practice fostered by modern seed-marketing methods% it "! If a single strain of plant is used for a given crop over a #ide area% as is fostered by modern seed-marketing methods% it $! A practice fostered by modern seed-marketing methods% a single strain of plant used for a given crop over a #ide area &! A single strain of plant used for a given crop over a #ide area% a practice fostered by modern seed-marketing methods% '! =he use of single strain of plant for a given crop over a #ide area% a practice fostered by modern seed-marketing methods%

//).

If additional deposits of oil are found% it #ill e<pand the amount that can be used as fuel and reduce the price of oil% even if the deposits are not immediately tapped. A! it #ill e<pand the amount that can be used as fuel and reduce the price of oil "! the amount that is able to be used as fuel #ill e<pand and the price of oil be reduced $! it #ill cause an increase in the amount that is able to be used as fuel and a reduction in the price of oil &! the amount that can be used as fuel #ill increase and the price of oil #ill drop '! it #ill increase the amount of oil that can be used as fuel and cause a drop in the price

/42.

If anyone at Inter$om ?inancial Advisers #ould have anticipated% or even suspected% the impending sale of the Foniko kelp processing plant% they #ould have advised o#ners of Foniko stock to unload all shares immediately. A! If anyone at Inter$om ?inancial Advisers #ould have anticipated "! >ad anyone at Inter$om ?inancial Advisers anticipated $! If any people at Inter$om ?inancial Advisers #ould have anticipated &! If any people at Inter$om ?inancial Advisers had anticipated '! If anybody at Inter$om ?inancial Advisers anticipated

SC Strategy

J;

/41.

If &r. Wade #as right% any apparent connection of the eating of highly processed foods and e<celling at sports is purely coincidental. A! If &r. Wade #as right% any apparent connection of the eating of "! ,hould &r. Wade be right% any apparent connection of eating $! If &r. Wade is right% any connection that is apparent bet#een eating of &! If &r. Wade is right% any apparent connection bet#een eating '! ,hould &r. Wade have been right% any connection apparent bet#een eating

/4(.

If industrial pollution continues to deplete the o9one layer% the resulting increase in ultraviolet radiation #ill endanger human health% causing a rise in the incidence of skin cancer and eye disease% and perhaps even threatening global ecological systems. A! and perhaps even threatening "! and may even threaten $! and even a possible threat to &! as #ell as possibly threatening '! as #ell as a possible threat to

/4/.

If the ne# airboat does #hat it is to be doingHtravel at high speeds undeterred by sandbars% crocodile-infested mudflats% or marshy hippo hauntsHit could revolutioni9e transport on the (%)22-mile-long $ongo Diver. A! If the ne# airboat does #hat it is to be doing "! If the ne# airboat does #hat it is supposed to do $! If it does as the ne# airboat is supposed to do &! &oing #hat it is the ne# airboat is supposed to do '! &oing #hat the ne# airboat is to be doing

/44.

If the proposed e<penditures for gathering information abroad are reduced even further% international ne#s reports have been and #ill continue to diminish in number and 5uality. A! have been and #ill continue to diminish "! have and #ill continue to diminish $! #ill continue to diminish% as they already did% &! #ill continue to diminish% as they have already% '! #ill continue to diminish

/47.

If the reporter #ould have kno#n the landlord3s side of the story% she #ould not have #ritten an article so favorable to the 01-year-old tenant. A! #ould have kno#n the landlord3s side of the story% she #ould not have #ritten "! #ould of kno#n the landlord3s side of the story% she #ould not of #ritten $! had kno#n the landlord3s side of the story% she #ould not have #ritten

JI

&! had kno#n the landlord3s side of the story% she #ould not have #rote '! kne# the landlord3s side of the story% she #ould not have #ritten /4:. Ignoring the admonitions of his staff% the chief financial officer accepted the advice of the consulting company because he believed that the standardi9ed accounting procedures #ould prove not only ine<pensive but reliable indicators of economic performance. A! he believed that the standardi9ed accounting procedures #ould prove not only ine<pensive but "! the standardi9ed accounting procedures #ill prove both ine<pensive and also $! he believed the standardi9ed accounting procedures #ould prove themselves to be both ine<pensive and &! he believed that the standardi9ed accounting procedures #ould prove to be both ine<pensive and '! standardi9ed accounting procedures #ill prove his belief that they are both ine<pensive and /4*. Iguanas have been an important food source in .atin America since prehistoric times% and it is still pri9ed as a game animal by the campesinos% #ho typically cook the meat in a heavily spiced ste#. A! it is still pri9ed as a game animal "! it is still pri9ed as game animals $! they are still pri9ed as game animals &! they are still pri9ed as being a game animal '! being still pri9ed as a game animal /40. In 17(* Fing >enry BIII sought to have his marriage to ;ueen $atherine annulled so as to marry Anne "oleyn. A! so as to marry "! and so could be married to $! to be married to &! so that he could marry '! in order that he #ould marry /4). In 1*)1 Dobert $arter III% one of the #ealthiest plantation o#ners in Birginia% stunned his family% friends% and neighbors by filing a deed of emancipation% setting free the more than 722 slaves #ho #ere legally considered his property. A! setting free the more than 722 slaves #ho #ere legally considered "! setting free more than the 722 slaves legally considered as $! and set free more than 722 slaves% #ho #ere legally considered as &! and set free more than the 722 slaves #ho #ere legally considered '! and he set free the more than 722 slaves #ho #ere legally considered as

SC Strategy

JJ

/72.

In 1)((% #hen =ruman #as almost forty years old% he #as living in his mother-in-la#3s house% #atching the haberdashery store he opened three years earlier go bankrupt% and he faced a future #ith no visible prospects. A! opened three years earlier go bankrupt% and he faced "! opened three years earlier go bankrupt and faced $! had opened three years earlier go bankrupt% and he #as facing &! had opened three years earlier go bankrupt% and facing '! #as opening three years earlier going bankrupt% and facing

/71.

In 1)() relatively small declines in the market ruined many speculators having bought on marginE they had to sell% and their selling pushed other investors to the brink. A! speculators having bought on marginE they had to sell% and "! speculators #ho had bought on marginE having had to sell% $! speculators #ho had bought on marginE they had to sell% and &! speculators% those #ho had bought on marginE these speculators had to sell% and '! speculators% #ho% having bought on margin and having to sell%

/7(.

In 1)// the rubber% clothing% and shipbuilding industries put into effect a si<-hour #orkday% believing it a seeming permanent accommodation rather than a temporary e<pedient for #hat many observers thought #as an economy made overproductive by advances in technology. A! believing it a seeming permanent accommodation rather than a temporary e<pedient for #hat many observers thought #as "! believing it a seeming permanent accommodation instead of a temporary e<pedient for #hat many observers thought #as $! believing that it #as not a temporary e<pedient but a seeming permanent accommodation to #hat many observers thought of as a &! not as a temporary e<pedient but as a seemingly permanent accommodation to #hat many observers thought #as '! not as a temporary e<pedient but believing it a seemingly permanent accommodation for #hat many observers thought

/7/.

In 1)*/ mortgage payments represented t#enty-one percent of an average thirty-year-old male3s incomeE and forty-four percent in 1)04. A! incomeE and forty-four percent in 1)04 "! incomeE in 1)04 the figure #as forty-four percent $! income% and in 1)04 forty-four percent &! income% forty-four percent in 1)04 #as the figure '! income that rose to forty-four percent in 1)04

/74.

In 1)*0 a national study found that not only had many contractors licensed by a self-policing

$EE

private guild failed to pass 5ualifying e<ams% they in addition falsified their references. A! they in addition falsified their references "! they had their references falsified in addition $! but they had also falsified their references &! they had also falsified their references '! but their references #ere falsified as #ell /77. In 1)*0 only half the #omen granted child support by a court received the amount a#ardedE at least as much as a million and more others had not any support agreements #hatsoever. A! at least as much as a million and more others had not any "! at least as much as more than a million others had no $! more than a million others had not any &! more than a million others had no '! there #as at least a million or more others #ithout any /7:. In 1)02 the +nited ,tates e<ported t#ice as much of its national output of goods as they had in 1)*2. A! t#ice as much of its national output of goods as they had "! double the amount of their national output of goods as they did $! t#ice as much of its national output of goods as it did &! double the amount of its national output of goods as it has '! t#ice as much of their national output of goods as they had /7*. In 1)02% for the first time% the number of foreigners touring the +nited ,tates #ere in e<cess of the number of Americans going abroad. A! #ere in e<cess of the number of Americans "! had an e<cess over the Americans #ho #ere $! e<ceeded the Americans #ho #ere &! numbered more than the Americans '! e<ceeded the number of Americans /70. In 1)0( the median income for married-couple families #ith a #age-earning #ife #as I)%222 more than a family #here the husband only #as employed. A! a family #here the husband only "! of a family #here only the husband $! that for families in #hich only the husband &! a family in #hich only the husband '! those of families in #hich the husband only

SC Strategy

$E$

/7).

In 1))23s% there are more babies born by #omen over thirty years old than under it. A! than under it "! than #ere they under it $! than had been under it &! than #ere the babies '! than those #ere under it

/:2.

In a 7-to-4 decision% the ,upreme $ourt ruled that t#o upstate Ne# @ork counties o#ed restitution to three tribes of Gneida Indians for the unla#ful sei9ure of their ancestral lands in the eighteenth century. A! that t#o upstate Ne# @ork counties o#ed restitution to three tribes of Gneida Indians for the unla#ful sei9ure of "! that t#o upstate Ne# @ork counties o#ed restitution to three tribes of Gneida Indians because of their unla#ful sei9ure of $! t#o upstate Ne# @ork counties to o#e restitution to three tribes of Gneida Indians for their unla#ful sei9ure of &! on t#o upstate Ne# @ork counties that o#ed restitution to three tribes of Gneida Indians because they unla#fully sei9ed '! on the restitution that t#o upstate Ne# @ork counties o#ed to three tribes of Gneida Indians for the unla#ful sei9ure of

/:1.

In a period of time #hen #omen typically have had a narro# range of choices% 8ary "aker 'ddy became a distinguished #riter and the founder% architect% and builder of a gro#ing church. A! In a period of time #hen #omen typically have "! &uring a time in #hich typically #omen have $! =ypically% during a time #hen #omen &! At a time #hen #omen typically '! =ypically in a time in #hich #omen

/:(.

In a plan to stop the erosion of 'ast $oast beaches% the Army $orps of 'ngineers proposed building parallel to shore a break#ater of rocks that #ould rise si< feet above the #aterline and act as a buffer% so that it absorbs the energy of crashing #aves and protecting the beaches. A! act as a buffer% so that it absorbs "! act like a buffer so as to absorb $! act as a buffer% absorbing &! acting as a buffer% absorbing '! acting like a buffer% absorb

/:/.

In a recent poll% 0: percent of the public favored a $lean Air Act as strong or stronger than the present act.

$E'

A! a $lean Air Act as strong or stronger than "! a $lean Air Act that is stronger% or at least so strong as% $! at least as strong a $lean Air Act as is &! a $lean Air Act as strong or stronger than is '! a $lean Air Act at least as strong as /:4. In A.&. /)1% resulting from the destruction of the largest library of the ancient #orld at Ale<andria% later generations lost all but the Iliad and Gdyssey among -reek epics% most of the poetry of 1indar and ,appho% and do9ens of plays by Aeschylus and 'uripides. A! resulting from the destruction of the largest library of the ancient #orld at Ale<andria% "! the destroying of the largest library of the ancient #orld at Ale<andria resulted and $! because of the result of the destruction of the library at Ale<andria% the largest of the ancient #orld% &! as a result of the destruction of the library at Ale<andria% the largest of the ancient #orld% '! Ale<andria3s largest library of the ancient #orld #as destroyed% and the result #as

/:7.

In addition to having more protein than #heat does% the protein in rice is higher 5uality than that in #heat% #ith more of the amino acids essential to the human diet. A! the protein in rice is higher 5uality than that in "! rice has protein of higher 5uality than that in $! the protein in rice is higher in 5uality than it is in &! rice protein is higher in 5uality than it is in '! rice has a protein higher in 5uality than

/::.

In an effort to reduce their inventories% Italian vintners have cut pricesE their #ines have been priced to sell% and they are. A! have been priced to sell% and they are "! are priced to sell% and they have $! are priced to sell% and they do &! are being priced to sell% and have '! had been priced to sell% and they have

/:*.

In ancient times% Nubia #as the principal corridor #here there #ere cultural influences transmitted bet#een "lack Africa and the 8editerranean basin. A! #here there #ere cultural influences transmitted "! through #hich cultural influences #ere transmitted $! #here there #as a transmission of cultural influences &! for the transmitting of cultural influences

SC Strategy

$E*

'! #hich #as transmitting cultural influences /:0. In Aristophanes3 .ysistrata #omen are seen as the means of bringing peace and good sense to a #ar-torn #orld. A! as "! as if they are $! that they #ill be &! that they are '! for being /:). In assessing the problems faced by rural migrant #orkers% the 5uestion of #hether they are better off materially than the urban #orking poor is irrelevant. A! In assessing the problems faced by rural migrant #orkers% the 5uestion of #hether they are better off materially than the urban #orking poor is irrelevant. "! =he 5uestion of #hether the rural migrant #orker is better off materially than the urban #orking poor is irrelevant in assessing the problems that they face. $! A 5uestion that is irrelevant in assessing the problems that rural migrant #orkers face is #hether they are better off materially than the urban #orking poor. &! In an assessment of the problems faced by rural migrant #orkers% the 5uestion of #hether they are better off materially than the urban #orking poor is irrelevant. '! =he 5uestion of #hether the rural migrant #orker is better off materially than the urban #orking poor is irrelevant in an assessment of the problems that they face. /*2. In astronomy the term red shift denotes the e<tent to #hich light from a distant gala<y has been shifted to#ard the red% or long-#ave% end of the light spectrum by the rapid motion of the gala<y a#ay from the 'arth. A! to #hich light from a distant gala<y has been shifted "! to #hich light from a distant gala<y has shifted $! that light from a distant gala<y has been shifted &! of light from a distant gala<y shifting '! of the shift of light from a distant gala<y /*1. In cold-#ater habitats% certain invertebrates and fish convert starches into comple< carbohydrates called glycerols% in effect manufacturing its o#n antifree9e. A! in effect manufacturing its o#n antifree9e "! effectively manufacturing antifree9e of its o#n $! in effect manufacturing their o#n antifree9e &! so that they manufacture their o#n antifree9e '! thus the manufacture of its o#n antifree9e

$E7

/*(.

In contrast to large steel plants that take iron ore through all the steps needed to produce several different kinds of steel% processing steel scrap into a speciali9ed group of products has enabled small mills to put capital into ne# technology and remain economically viable. A! processing steel scrap into a speciali9ed group of products has enabled small mills to put capital into ne# technology and remain "! processing steel scrap into a speciali9ed group of products has enabled small mills to put capital into ne# technology% remaining $! the processing of steel scrap into a speciali9ed group of products has enabled small mills to put capital into ne# technology% remaining &! small mills% by processing steel scrap into a speciali9ed group of products% have been able to put capital into ne# technology and remain '! small mills% by processing steel scrap into a speciali9ed group of products% have been able to put capital into ne# technology and remained

/*/.

In contrast to true hibernators such as #oodchucks and hedgehogs% #hose body temperatures drop close to the free9ing point during the #inter months% the body temperature of bears remains nearly normal throughout their prolonged sleep. A! the body temperature of bears remains nearly normal "! a nearly normal body temperature is maintained by bears $! a bear3s body temperature remains nearly normal &! a bear maintains a body temperature that is nearly normal '! bears maintain a nearly normal body temperature

/*4.

In &ecember of 1)0* an automobile manufacturer pleaded no contest to criminal charges of odometer tampering and agreed to pay more than I1: million in civil damages for cars that #ere test-driven #ith their odometers disconnected. A! cars that #ere test-driven #ith their odometers disconnected "! cars that it had test-driven #ith their disconnected odometers $! its cars having been test-driven #ith disconnected odometers &! having test-driven cars #ith their odometers disconnected '! having cars that #ere test-driven #ith disconnected odometers

/*7.

In developing ne# facilities for the incineration of solid #astes% #e must avoid the danger of shifting environmental problems from landfills polluting the #ater to polluting the air #ith incinerators. A! landfills polluting the #ater to polluting the air #ith incinerators "! landfills polluting the #ater to the air being polluted #ith incinerators $! the pollution of #ater by landfills to the pollution of air by incinerators &! pollution of the #ater by landfills to incinerators that pollute the air

SC Strategy

$E8

'! #ater that is polluted by landfills to incinerators that pollute the air /*:. In 'gypt in the late 1aleolithic period% the climate changed% pastures became deserts% and the inhabitants #ere forced to #ithdra# to the land bordering the Nile from their hunting grounds. A! inhabitants #ere forced to #ithdra# to the land bordering the Nile from their hunting grounds "! inhabitants had been forced to #ithdra# from their hunting grounds to the land that bordered the Nile $! inhabitants #ere forced to #ithdra# from their hunting grounds to the land bordering the Nile &! inhabitants having been forced to% #ithdre# from their hunting grounds to the land that bordered the Nile '! inhabitants #ithdre#% because they #ere forced to% from their hunting grounds to the land bordering the Nile /**. In 'ngland the #ell-dressed gentleman of the eighteenth century protected their clothing #hile having their #ig po#dered by poking their head through a device that resembled the stocks. A! gentleman of the eighteenth century protected their clothing #hile having their #ig po#dered by poking their head "! gentleman of the eighteenth century protected his clothing #hile having his #ig po#dered by poking his head $! gentleman of the eighteenth century protected their clothing #hile having their #igs po#dered by poking their heads &! gentlemen of the eighteenth century protected his clothing #hile having his #ig po#dered by poking his head '! gentlemen of the eighteenth century protected their clothing #hile having his #ig po#dered by poking his head /*0. In feudal 'urope% urban areas developed from clusters of houses #here peasants lived and commuted to farmlands in the countryside% unlike homesteading policies in the American West that re5uired residency on the land itself in order to obtain eventual o#nership. A! In feudal 'urope% urban areas developed from clusters of houses #here peasants lived and commuted to farmlands in the countryside% unlike homesteading policies in the American West that "! In feudal 'urope% urban areas developed from clusters of houses #here peasants lived and from #hich they commuted to farmlands in the countryside% but in the American West homesteading policies $! +nlike feudal 'urope #here urban areas developed from clusters of houses #here peasants lived and commuted to farmlands in the countryside% the American West3s homesteading policies &! +nlike feudal 'urope #here urban areas developed from clusters of houses #here peasants lived and commuted to farmlands in the countryside% the homesteading policies of the

$E9

American West '! +rban areas developed from clusters of houses #here peasants lived from #hich they commuted to farmlands in the countryside in feudal 'urope% unlike the American West #here homesteading policies /*). In good years% the patch#ork of green fields that surround the ,an Coa5uin Balley to#n bustles #ith farm #orkers% many of them in the area 6ust for the season. A! surround the ,an Coa5uin Balley to#n bustles #ith farm #orkers% many of them "! surrounds the ,an Coa5uin Balley to#n bustles #ith farm #orkers% many of #hom are $! surround the ,an Coa5uin Balley to#n bustles #ith farm #orkers% many of #ho are &! surround the ,an Coa5uin Balley to#n bustle #ith farm #orkers% many of #hich '! surrounds the ,an Coa5uin Balley to#n bustles #ith farm #orkers% many are /02. In her recently published study% Dubin asserts that most #omen do not suffer from the empty nest syndromeE they are% in fact% relieved #hen their children depart. A! they are% in fact% relieved #hen their children depart "! and they are% in fact% relieved #hen their children departed $! they are% in fact% relieved #hen their children departed &! in fact% they are relieved #hen their children departed '! they are relieved at the departure of the children% in fact /01. In his eagerness to find a city #orthy of 1riam% the -erman archaeologist ,chliemann cut through =roy and uncovered a civili9ation a thousand years older as #as the city >omer3s heroes kne#. A! older as #as the city >omer3s heroes kne# "! more ancient than the city kno#n to >omer3s heroes $! older than #as the city kno#n to >omer3s heroes &! more ancient of a city than >omer3s heroes kne# '! older of a city than #as the one kno#n to >omer3s heroes /0(. In his research paper% &r. ?rosh% medical director of the 1ayne Whitney $linic% distinguishes mood s#ings% #hich may be violent #ithout their being grounded in mental disease% from genuine manic-depressive psychosis. A! mood s#ings% #hich may be violent #ithout their being grounded in mental disease% from genuine manic-depressive psychosis "! mood s#ings% perhaps violent #ithout being grounded in mental disease% and genuine manicdepressive psychosis $! bet#een mood s#ings% #hich may be violent #ithout being grounded in mental disease% and genuine manic-depressive psychosis &! bet#een mood s#ings% perhaps violent #ithout being grounded in mental disease% from

SC Strategy

$E;

genuine manic-depressive psychosis '! genuine manic-depressive psychosis and mood s#ings% #hich may be violent #ithout being grounded in mental disease /0/. In his #ork% .eon ?orrest is more reminiscent of >enry 8iller3s obsessive narratives and =oni 8orrison3s mythic languages than Cames Coyce3s internal e<plorations. A! In his #ork% .eon ?orrest is more reminiscent of "! .eon ?orrest #rites more like $! .eon ?orrest3s #ork is more reminiscent of &! .eon ?orrest reminds one more of '! .eon ?orrest3s #ork more resembles that of /04. In >olland% a larger percentage of the gross national product is spent on defense of their coasts from rising seas than is spent on military defense in the +nited ,tates. A! In >olland% a larger percentage of the gross national product is spent on defense of their coasts from rising seas than is spent on military defense in the +nited ,tates. "! In >olland they spend a larger percentage of their gross national product on defending their coasts from rising seas than the +nited ,tates does on military defense. $! A larger percentage of >olland3s gross national product is spent on defending their coasts from rising seas than the +nited ,tates spends on military defense. &! >olland spends a larger percentage of its gross national product defending its coasts from rising seas than the military defense spending of the +nited ,tates. '! >olland spends a larger percentage of its gross national product on defending its coasts from rising seas than the +nited ,tates does on military defense. /07. In its most recent approach% the comet $rommelin passed the 'arth at about the same distance and in about the same position% some (7 degrees above the hori9on% that >alley3s comet #ill pass the ne<t time it appears. A! that >alley3s comet #ill pass "! that >alley3s comet is to be passing $! as >alley3s comet &! as #ill >alley3s comet '! as >alley3s comet #ill do /0:. In Capan elderly people are treated #ith far greater respect than most Western countries. A! most Western countries "! most Western countries do $! most Western countries are &! they do in most Western countries

$EI

'! they are in most Western countries /0*. In Capan% a government advisory committee called for the breakup of Nippon =elephone and =elegraph $ompany% the largest telephone company in the #orld% so it #ould be t#o local phone companies and one long-distance provider. A! In Capan% a government advisory committee called for the breakup of Nippon =elephone and =elegraph $ompany% the largest telephone company in the #orld% so it #ould be "! =he breakup of the #orld3s largest telephone company% Nippon =elephone and =elegraph $ompany% #as called for by a government advisory committee in Capan% so it #ould be $! A government advisory committee in Capan called for the breakup of Nippon =elephone and =elegraph $ompany% the #orld3s largest telephone company% into &! =he breakup of Nippon =elephone and =elegraph $ompany% the #orld3s largest telephone company% #as called for by a government advisory committee in Capan% so it #ould be '! $alled for by a government advisory committee% the breakup of Nippon =elephone and =elegraph $ompany in Capan% the #orld3s largest telephone company% #as to be into /00. In Cune of 1)0*% =he "ridge of =rin5uetaille% Bincent van -ogh3s vie# of an iron bridge over the Dhone sold for I(2.( million and it #as the second highest price ever paid for a painting at auction. A! Dhone sold for I(2.( million and it #as "! Dhone% #hich sold for I(2.( million% #as $! Dhone% #as sold for I(2.( million% &! Dhone #as sold for I(2.( million% being '! Dhone% sold for I(2.( million% and #as /0). In Cune of 1)0)% 1rinceton =o#nship approved a developer3s plans to build /22 houses on a large portion of the (12-acre site of the "attle of 1rinceton% one of only eight Devolutionary War battlefields that had remained undeveloped. A! one of only eight Devolutionary War battlefields that had remained undeveloped "! one of eight of the only Devolutionary War battlefields that have remained undeveloped $! one of the only eight undeveloped Devolutionary War battlefields that remains &! only one of eight Devolutionary War battlefields to remain undeveloped '! only one of the eight remaining undeveloped Devolutionary War battlefields /)2. In Cune% 1)01% si< teenagers in the village of 8ed6ugor6e% @ugoslavia% claimed to have had visions of the Birgin 8ary% #ho they say has continued to appear to them over the ensuing years. A! claimed to have had visions of the Birgin 8ary% #ho "! claimed to have visions of the Birgin 8ary% #hom $! claimed to have had visions of the Birgin 8ary% #hom

SC Strategy

$EJ

&! claimed to have visions of the Birgin 8ary% #ho '! had claimed to have had visions of the Birgin 8ary% #hom /)1. In large doses% analgesics that #ork in the brain as antagonists to certain chemicals have caused psychological disturbances in patients% #hich may limit their potential to relieve severe pain. A! #hich may limit their potential to relieve "! #hich may limit their potential for relieving $! #hich may limit such analgesics3 potential to relieve &! an effect that may limit their potential to relieve '! an effect that may limit the potential of such analgesics for relieving /)(. In metal#ork one advantage of adhesive-bonding over spot-#elding is that the contact% and hence the bonding% is effected continuously over a broad surface instead of a series of regularly spaced points #ith no bonding in bet#een. A! instead of "! as opposed to $! in contrast #ith &! rather than at '! as against being at /)/. In one of the bloodiest battles of the $ivil War% fought at ,harpsburg% 8aryland% on ,eptember 1*% 10:(% four times as many Americans #ere killed as #ould later be killed on the beaches of Normandy during &-&ay. A! Americans #ere killed as "! Americans #ere killed than $! Americans #ere killed than those #ho &! more Americans #ere killed as there '! more Americans #ere killed as those #ho /)4. In one of the most stunning reversals in the history of marketing% the $oca-$ola company in Culy 1)07 yielded to thousands of irate consumers demanding that it should bring back the original $oke formula. A! demanding that it should "! demanding it to $! and their demand to &! #ho demanded that it '! #ho demanded it to

$$E

/)7.

In presenting his modus vivendi proposal% .ansing implied that the American government accepted the -erman vie# that armed merchant vessels #ere #arshipsA ho#ever% #hen the proposal #as dropped by the Wilson administration% it seemed to be reverting to the "ritish vie# on this 5uestion. A! #hen the proposal #as dropped by the Wilson administration% it "! after it #as dropped% the Wilson administration $! by dropping the proposal% the Wilson administration &! the Wilson administration dropped the proposal #hen it '! #hen they dropped the proposal% the Wilson administration

/):.

In recent years cattle breeders have increasingly used crossbreeding% in part that their steers should ac5uire certain characteristics and partly because crossbreeding is said to provide hybrid vigor. A! in part that their steers should ac5uire certain characteristics "! in part for the ac5uisition of certain characteristics in their steers $! partly because of their steers ac5uiring certain characteristics &! partly because certain characteristics should be ac5uired by their steers '! partly to ac5uire certain characteristics in their steers

/)*.

In reference to the current hostility to#ard smoking% smokers fre5uently e<pressed an<iety that their prospects for being hired and promoted are being stunted by their habit. A! In reference to the current hostility to#ard smoking% smokers fre5uently e<pressed an<iety that "! Deferring to the current hostility to#ard smoking% smokers fre5uently e<pressed an<iety about $! When referring to the current hostility to#ard smoking% smokers fre5uently e<press an<iety about &! With reference to the current hostility to#ard smoking% smokers fre5uently e<pressed an<iety about '! Deferring to the current hostility to#ard smoking% smokers fre5uently e<press an<iety that

/)0.

In spite of continuing national trends to#ard increased consumption of specialty foods% agronomists in the 8id#est foresee a gradual reversion to the raising of agricultural staplesA feed corn and hard red #heat. A! a gradual reversion to the raising of agricultural staples "! that a gradual reversion back #ill feature the raising of agricultural staples $! a gradual reversion back to the raising of agricultural staples again &! a gradual reversion to raise agricultural staples '! a gradual reversion into the raising of agricultural staples

/)).

In spite of federal subsidi9ing of public transportation systems massively and ma6or local

SC Strategy

$$$

efforts to persuade the public to use public transportation% mass transit has been steadily losing patronage to the private automobile for the past thirty years. A! In spite of federal subsidi9ing of public transportation systems massively and ma6or local efforts to persuade "! In spite of massive federal subsidi9ing of public transportation systems and ma6or efforts locally at persuading $! &espite massive federal subsidies to public transportation systems and the making of ma6or efforts locally to persuade &! &espite massive federal subsidies to public transportation systems and ma6or local efforts to persuade '! &espite massive federal subsidies to public transportation systems and making ma6or local efforts at persuading 422. In such #orks as his 100) masterpiece% A &ash for the =imber% ?rederic Demington caught the public3s fancy by portraying those moments of conflict that defined the West3s romantic heroes. A! In such #orks as his 100) masterpiece% A &ash for the =imber% ?rederic Demington caught the public3s fancy "! In such #orks as ?rederic Demington3s 100) masterpiece% A &ash for the =imber% the public3s fancy #as caught $! ?rederic Demington% catching the public3s fancy in such #orks as the 100) masterpiece A &ash for the =imber% did it &! =he fancy of ?rederic Demington3s public #as caught in his 100) masterpiece% A &ash for the =imber% '! =he public3s fancy #as caught by ?rederic Demington in such #orks as his 100) masterpiece% A &ash for the =imber% 421. In terms of physics% the characteristic feature of the roller coaster is that the cars3 potential energy% gained through their being lifted by a chain drive through the 'arth3s gravity to the top of the first drop% has been converted to kinetic energy by the time the ride ends. A! cars3 potential energy% gained through their being lifted by a chain drive "! cars3 potential energy% a gain achieved as they are lifted by a chain drive $! potential energy from the cars3 being lifted by a chain drive &! potential energy of the cars% gained as a chain drive lifts them '! potential energy gained by the cars% being achieved #hile a chain drive lifts them 42(. In the 1)723s astronomers #ere divided bet#een those #ho believed the universe began in a cosmic e<plosion the big bang! #ith those #ho favored the model of an eternal and infinite steady-state universe. A! universe began in a cosmic e<plosion the big bang! #ith

$$'

"! universe began #ith a cosmic e<plosion the big bang! and $! universe had a beginning a cosmic e<plosion the big bang! or &! universe3s beginning #as a cosmic e<plosion the big bang! or '! universe3s beginning #as a cosmic e<plosion the big bang! against 42/. In the 1)72s% #hen the $hrysler $orporation sponsored a live television sho# about the assassination of Abraham .incoln% it forbade the actors to mention .incoln3s name or the name of the ?ord =heater because it did not #ant to plug the competition. A! it forbade the actors to mention .incoln3s name or the name of the ?ord =heater because it "! they forbade the actors from mentioning .incoln or the ?ord =heater because they $! it #as forbidden to mention .incoln or the ?ord =heater as they &! the actors #ere forbidden to mention .incoln3s name or the name of the ?ord =heater because they '! the actors #ere forbidden from mentioning .incoln or the ?ord =heater% since they 424. In the 1)023s the rate of increase of the minority population of the +nited ,tates #as nearly t#ice as fast as the 1)*23s. A! t#ice as fast as "! t#ice as fast as it #as in $! t#ice #hat it #as in &! t#o times faster than that of '! t#o times greater than 427. In the face of #idespread concern about environmental #aste% compact disc manufacturers are attempting to find a replacement for the disposable plastic bo< in #hich they package their product. A! the disposable plastic bo< in #hich they "! the disposable plastic bo< #here they $! a disposable plastic bo< in #hich to &! disposable plastic bo<es inside #hich they '! the disposable plastic bo<es in #hich to 42:. In the fall of 1)07% only 12 percent of the #omen entering college planned to ma6or in education% #hile (0 percent chose business% making it the most popular ma6or for #omen as #ell as for men. A! as #ell as for men "! as #ell as the men $! and men too &! and men as #ell

SC Strategy

$$*

'! and also men 42*. In the initial planning stages% the condominium corporation took into account only the concerns of its prospective clients% not those of surrounding homeo#ners. A! the condominium corporation took into account only the concerns of its prospective clients "! the condominium corporation has only taken into account the concerns of their prospective clients $! the condominium corporation only took their prospective clients3 concerns into account &! the concerns of its prospective clients only #ere taken into account by the condominium corporation '! prospective clients had their concerns only taken into account by the condominium corporation 420. In the last fe# years% the number of convicted criminals given community service sentences% #hich allo# the criminals to remain unconfined #hile they perform specific 6obs benefiting the public% have risen dramatically. A! sentences% #hich allo# the criminals to remain unconfined #hile they perform specific 6obs benefiting the public% have "! sentences% performing specific 6obs that benefit the public #hile being allo#ed to remain unconfined% have $! sentences% performing specific 6obs beneficial to the public #hile they are allo#ed to remain unconfined% have &! sentences #hich allo# them to remain unconfined in their performing of specific 6obs beneficial to the public has '! sentences allo#ing them to remain unconfined #hile performing specific 6obs that benefit the public has 42). In the last ten years% the dropout rate among "lack high school students fell substantially over the past decade% #hile the number of "lacks #ho attend college is more than t#ice #hat it #as. A! fell substantially over the past decade% #hile the number of "lacks #ho attend college is more than t#ice #hat it #as "! fell substantially% #hile the number of "lacks attending college is more than double #hat it #as at that time $! has fallen substantially% #hile the number of "lacks attending college has more than doubled &! has fallen substantially over the past decade% #hile the number of "lacks attending college is more than t#ice #hat it #as at that time '! has fallen substantially over the past decade% #hile the number of "lacks #ho are attending college are more than double #hat they #ere 412. In the last t#enty years% despite the chauvinism of 'uropean connoisseurs% $alifornia #ines are respected throughout the #orld.

$$7

A! are respected "! are becoming better respected $! #hich have gained respect &! have gained respect '! have since become respected 411. In the late seventh century% in a dispute over #hether the 1rophet 8uhammad3s son-in-la#% Ali% should carry on as the fourth caliph% 8uhammad3s successor% Islam split into t#o branches% the ,unnis and the ,hiites. A! over #hether the 1rophet 8uhammad3s son-in-la#% Ali% should carry on as the fourth caliph% 8uhammad3s successor "! over if Ali% the 1rophet 8uhammad3s son-in-la#% #as going to carry on and be the fourth caliph% 8uhammad3s successor $! over #hether Ali% the 1rophet 8uhammad3s son-in-la#% #as going to carry on and be the fourth caliph% 8uhammad3s successor &! as to #hether the fourth caliph% 8uhammad3s successor% is to be the 1rophet 8uhammad3s son-in-lo#% Ali '! concerning if the fourth caliph% 8uhammad3s successor% #as to be the 1rophet 8uhammad3s son-in-la#% Ali 41(. In the main% incidents of breakdo#ns in nuclear reactors have not resulted from lapses of high technology but commonplace inade5uacies in plumbing and #iring. A! not resulted from lapses of high technology but "! resulted not from lapses of high technology but from $! resulted from lapses not of high technology but &! resulted from lapses not of high technology but have stemmed from '! resulted not from lapses of high technology but have stemmed from 41/. In the mid-1)(23s the >a#thorne Works of the Western 'lectric $ompany #as the scene of an intensive series of e<periments that #ould investigate changes in #orking conditions as to their effects on #orkers3 performance. A! that #ould investigate changes in #orking conditions as to their effects on #orkers3 performance "! investigating the effects that changes in #orking conditions #ould have on #orkers3 performance $! for investigating #hat are the effects in #orkers3 performance that changes in #orking conditions #ould cause &! that investigated changes in #orking conditions3 effects on #orkers3 performance '! to investigate #hat the effects changes in #orking conditions #ould have on #orkers3

SC Strategy

$$8

performance 414. In the mid-1):23s a ne#ly installed radar #arning system mistook the rising of the moon as a massive missile attack by the ,oviets. A! rising of the moon as a massive missile attack by the ,oviets "! rising of the moon for a massive ,oviet missile attack $! moon rising to a massive missile attack by the ,oviets &! moon as it #as rising for a massive ,oviet missile attack '! rise of the moon as a massive ,oviet missile attack 417. In the minds of many people living in 'ngland% before Australia #as Australia% it #as the antipodes% the opposite pole to civili9ation% an obscure and unimaginable place that #as considered the end of the #orld. A! before Australia #as Australia% it #as the antipodes "! before there #as Australia% it #as the antipodes $! it #as the antipodes that #as Australia &! Australia #as #hat #as the antipodes '! Australia #as #hat had been kno#n as the antipodes 41:. In the most common procedure for harvesting forage crops such as alfalfa% as much as (2 percent of the leaf and small-stem material% #hich is the most nutritious of all the parts of the plant% shattered and fell to the ground. A! #hich is the most nutritious of all the parts of the plant% shattered and fell "! the most nutritious of all parts of the plant% shatter and fall $! the parts of the plant #hich #ere most nutritious% #ill shatter and fall &! the most nutritious parts of the plant% shatters and falls '! parts of the plant #hich are the most nutritious% have shattered and fallen 41*. In the ,oviet +nion the attorney3s role is often played by the 6udge% #ho not only reserves time to hear citi9ens3 legal complaints and also prepares their cases should the claims be valid. A! and also prepares their cases should the claims be "! but also does the preparation of their cases if the claims should be $! and their cases are prepared if the claims are &! but also prepares their cases if the claims are '! and prepares their cases if the claims are 410. In the te<tbook publishing business% the second 5uarter is historically #eak% because revenues are lo# and marketing e<penses are high as companies prepare for the coming school year. A! lo# and marketing e<penses are high as companies prepare "! lo# and their marketing e<penses are high as they prepare

$$9

$! lo# #ith higher marketing e<penses in preparation &! lo#% #hile marketing e<penses are higher to prepare '! lo#% #hile their marketing e<penses are higher in preparation 41). In the traditional Capanese household% most clothing could be packed flatly% and so it #as not necessary to have elaborate closet facilities. A! flatly% and so it #as not necessary to have elaborate closet facilities "! flat% and so elaborate closet facilities #ere unnecessary $! flatly% and so there #as no necessity for elaborate closet facilities &! flat% there being no necessity for elaborate closet facilities '! flatly% as no elaborate closet facilities #ere necessary 4(2. In the +nited ,tates% trade unions encountered far more intense opposition against their struggle for social legitimacy than the organi9ed labor movements of most other democratic nations. A! against their struggle for social legitimacy than "! in their struggle for social legitimacy than did $! against their struggle for social legitimacy as &! in their struggle for social legitimacy as did '! #hen they struggled for social legitimacy than has 4(1. In theory% international civil servants at the +nited Nations are prohibited from continuing to dra# salaries from their o#n governmentsE in practice% ho#ever% some governments merely substitute living allo#ances for their employees3 paychecks% assigned by them to the +nited Nations. A! for their employees3 paychecks% assigned by them "! for the paychecks of their employees #ho have been assigned $! for the paychecks of their employees% having been assigned &! in place of their employees3 paychecks% for those of them assigned '! in place of the paychecks of their employees to have been assigned by them 4((. In three centuriesHfrom 1272 to 1/72Hseveral million tons of stone #ere 5uarried in ?rance for the building of eighty cathedrals% five hundred large churches% and some tens of thousands of parish churches. A! for the building of eighty cathedrals% five hundred large churches% and some "! in order that they might build eighty cathedrals% five hundred large churches% and some $! so as they might build eighty cathedrals% five hundred large churches% and some &! so that there could be built eighty cathedrals% five hundred large churches% and '! such that they could build eighty cathedrals% five hundred large churches% and

SC Strategy

$$;

4(/.

In virtually all types of tissue in every animal species% dio<in induces the production of en9ymes that are the organism3s trying to metaboli9e% or render harmless% the chemical that is irritating it. A! trying to metaboli9e% or render harmless% the chemical that is irritating it "! trying that it metaboli9e% or render harmless% the chemical irritant $! attempt to try to metaboli9e% or render harmless% such a chemical irritant &! attempt to try and metaboli9e% or render harmless% the chemical irritating it '! attempt to metaboli9e% or render harmless% the chemical irritant

4(4.

Income in a single year is a very poor guide to income and #ealth over even a fe# years% much less a lifetimeE in the longer run% a ta< on #hat people spend is therefore not much different than a ta< on their income. A! than a ta< on their income "! from a ta< on #hat they earn $! than ta<ing income &! from the income ta< '! than a ta< on #hat people earn

4(7.

Increases in the cost of energy% turmoil in the international money markets% and the steady erosion of the dollar have altered the investment strategies of +nited ,tates corporations more radically than those of foreign corporations. A! altered the investment strategies of +nited ,tates corporations more radically than those of "! altered the investment strategies of +nited ,tates corporations more radically than $! altered the investment strategies of +nited ,tates corporations more radically than they have &! radically altered the investment strategies of +nited ,tates corporations more than '! radically altered the investment strategies of +nited ,tates and

4(:.

India is country #ith at least fifty ma6or regional languages% of #hom fourteen have official recognition. A! of #hom fourteen have official recognition "! fourteen that have official recognition $! fourteen of #hich are officially recogni9ed &! fourteen that are officially recogni9ed '! among #hom fourteen have official recognition

4(*.

Industrial pollution has done seriously and possibly irreversible damage to the bron9e horses on the facade of the cathedral of ,t. 8ark .a Benica. A! has done seriously and possibly irreversible damage to "! did damage that is seriously and possibly irreversible to

$$I

$! damaged% serious and possibly irreversibly &! has done serious and possibly irreversible damage to '! did damage% serious and possibly irreversible 4(0. Inflation has made many Americans reevaluate their assumptions about the futureE they still e<pect to live better than their parents have% but not so #ell as they once thought they could. A! they still e<pect to live better than their parents have "! they still e<pect to live better than their parents did $! they still e<pect to live better than their parents had &! still e<pecting to live better than their parents had '! still e<pecting to live better than did their parents 4(). Inflation in medical costs slo#ed in 1)0: for the fifth consecutive year but #ere still about 72 percent greater than the rate of price increases for other items included in the consumer price inde<. A! Inflation in medical costs slo#ed in 1)0: for the fifth consecutive year but #ere "! Inflation in medical costs slo#ed for the fifth consecutive year in 1)0: but #as $! In 1)0: inflation in medical costs #ere slo#ed for the fifth consecutive year but #ere &! 1)0: #as the fifth consecutive year in #hich inflation in medical costs slo#ed but #as '! 1)0: #as the fifth consecutive year that inflation in medical costs #ere slo#ed% but they #ere 4/2. Initiated five centuries after 'uropeans arrived in the Ne# World on $olumbus &ay 1))(% 1ro6ect ,'=I pledged a I122 million investment in the search for e<traterrestrial intelligence. A! Initiated five centuries after 'uropeans arrived in the Ne# World on $olumbus &ay 1))(% 1ro6ect ,'=I pledged a I122 million investment in the search for e<traterrestrial intelligence. "! Initiated on $olumbus &ay 1))(% five centuries after 'uropeans arrived in the Ne# World% a I122 million investment in the search for e<traterrestrial intelligence #as pledged by 1ro6ect ,'=I. $! Initiated on $olumbus &ay 1))(% five centuries after 'uropeans arrived in the Ne# World% 1ro6ect ,'=I pledged a I122 million investment in the search for e<traterrestrial intelligence. &! 1ledging a I122 million investment in the search for e<traterrestrial intelligence% the initiation of 1ro6ect ,'=I five centuries after 'uropeans arrived in the Ne# World on $olumbus &ay 1))(. '! 1ledging a I122 million investment in the search for e<traterrestrial intelligence five centuries after 'uropeans arrived in the Ne# World% on $olumbus &ay 1))(% the initiation of 1ro6ect ,'=I took place. 4/1. Innovative companies are not only skillful in developing ne# products but also in finding ne# #ays to promote old ones. A! are not only skillful in developing ne# products but also in finding

SC Strategy

$$J

"! are skillful not only in developing ne# products but also in finding $! not only are skillful in developing ne# products% they also find &! not only develop ne# products% but they are also skillful in finding '! are both skillful in the development of ne# products and in finding 4/(. Intar% the oldest >ispanic theater company in Ne# @ork% has moved a#ay from the ,panish classics and no# it dra#s on the #orks both of contemporary >ispanic authors #ho live abroad and of those in the +nited ,tates. A! no# it dra#s on the #orks both of contemporary >ispanic authors #ho live abroad and of those "! no# dra#s on the #orks of contemporary >ispanic authors% both those #ho live abroad and those #ho live $! it dra#s on the #orks of contemporary >ispanic authors no#% both those living abroad and #ho live &! dra#s no# on the #orks both of contemporary >ispanic authors living abroad and #ho are '! dra#s on the #orks no# of both contemporary >ispanic authors living abroad and those 4//. Intended primarily to stimulate family summer travel% the ne# airfare% #hich allo#s both an adult and a child to fly for the price of one ticket% and also shortens the advance-purchase re5uirement for family travel to a minimum of seven days rather than fourteen. A! and also shortens the advance-purchase re5uirement for family travel to a minimum of seven days rather than "! and also lessens the advance-purchase re5uirement for family travel to a seven-day minimum from $! also shortens the advance-purchase re5uirement for family travel to a minimum of seven days rather than that of &! also lessens the advance-purchase re5uirement for family travel to a seven-day minimum from '! also shortens the advance-purchase re5uirement for family travel to a minimum of seven days rather than 4/4. International sporting events need not be fiscal disasters% the financial success of the 1))( Glympic -ames demonstrates that fact. A! the financial success of the 1))( Glympic -ames demonstrates that fact "! for e<ample% the 1))( Glympic -ames #ere financially successful $! like the financial success of the 1))( Glympic -ames demonstrates &! a fact demonstrated by the 1))( Glympic -ames% #hich #ere financially successful '! as the financial success of the 1))( Glympic -ames demonstrates 4/7. Inuits of the "ering ,ea #ere in isolation from contact #ith 'uropeans longer than Aleuts or Inuits of the North 1acific and northern Alaska.

$'E

A! in isolation from contact #ith 'uropeans longer than "! isolated from contact #ith 'uropeans longer than $! in isolation from contact #ith 'uropeans longer than #ere &! isolated from contact #ith 'uropeans longer than #ere '! in isolation and #ithout contacts #ith 'uropeans longer than 4/:. ID, provision (74-" re5uires that an , corporation #ith assets of greater than I(22%222 send W-( forms to their full- and part-time employees on or before Can. /1. A! that an , corporation #ith assets of greater than I(22%222 send W-( forms to their full- and part-time employees on or before Can. /1 "! an , corporation #ith assets of greater than I(22%222 send W-( forms to their full- and parttime employees on or before Can. /1 $! that an , corporation #ith assets of greater than I(22%222 send W-( forms to its full- and parttime employees on or before Can. /1 &! an , corporation #ith assets of greater than I(22%222 send W-( forms to their full- and parttime employees on Can. /1 or before '! an , corporation #ith assets of greater than I(22%222 send W-( forms to its full- and part-time employees on or before Can. /1 4/*. It appears illogical to some people that West -ermany% #hich bans such seeming lesser evils as la#n-mo#ing on ,undays% still has some 4%222 miles of high#ay #ith no speed limit. A! #hich bans such seeming lesser evils as "! #hich bans such seemingly lesser evils as $! #hich is banning such seeming lesser evils like &! banning such evils that seem lesser% for e<ample '! banning such seeming lesser evils like 4/0. It has been estimated that the annual cost to the +nited ,tates of illiteracy in lost industrial output and ta< revenues is at least I(2 billion a year. A! the annual cost to the +nited ,tates of illiteracy in lost industrial output and ta< revenues is at least I(2 billion a year "! the annual cost of illiteracy to the +nited ,tates is at least I(2 billion a year because of lost industrial output and ta< revenues $! illiteracy costs the +nited ,tates at least I(2 billion a year in lost industrial output and ta< revenues &! I(2 billion a year in lost industrial output and ta< revenues is the annual cost to the +nited ,tates of illiteracy '! lost industrial output and ta< revenues cost the +nited ,tates at least I(2 billion a year because of illiteracy

SC Strategy

$'$

4/).

It is a special feature of cell aggregation in the developing nervous system that in most regions of the brain the cells not only adhere to one another and also adopt some preferential orientation. A! to one another and also adopt "! one to the other% and also they adopt $! one to the other% but also adopting &! to one another but also adopt '! to each other% also adopting

442.

It is an oversimplified vie# of cattle raising to say that all one has to do #ith cattle is leave them alone #hile they feed themselves% corral them% and to drive them to market #hen the time is ripe. A! all one has to do #ith cattle is leave them alone #hile they feed themselves% corral them% and to "! all one has to do #ith cattle is to leave them alone to feed themselves% to corral them% and $! all one has to do #ith cattle is leave them alone #hile they feed themselves and then corral them and &! the only thing that has to be done #ith cattle is leave them alone #hile they feed themselves% corral them% and '! the only thing that has to be done #ith cattle is to leave them alone #hile they feed themselves% to corral them% and

441.

It is as difficult to prevent crimes against property as those that are against a person. A! those that are against a "! those against a $! it is against a &! preventing those against a '! it is to prevent those against a

44(.

It is characteristic of the 8etropolitan 8useum of Art% as of virtually every great American museum% the taste of local collectors has played at least as large a part in the formation of their collections as has the 6udgments of the art historian. A! of virtually every great American museum% the taste of local collectors has played at least as large a part in the formation of their collections as has "! of virtually every great American museum% that the taste of local collectors has played at least as large a part in the formation of their collections as has $! it is of virtually every great American museum% that the taste of local collectors has played at least as large a part in the formation of its collections as have &! it is of virtually every great American museum% that the taste of local collectors have played at least as large a part in the formation of its collections as have

$''

'! it is of virtually every great American museum% the taste of local collectors has played at least as large a part in the formation of its collections as has 44/. It is not likely that the competitive imbalance that no# e<ists bet#een Capan #ith all ma6or industrial nations #ill be redressed during the foreseeable future. A! #ith all ma6or industrial nations #ill be redressed during "! #ith all other ma6or industrial nations #ill be redressed #ithin $! #ith all other ma6or industrial nations #ill be redressed during &! and all ma6or industrial nations #ill be redressed during '! and all other ma6or industrial nations #ill be redressed #ithin 444. It is possible that Native Americans originally have migrated to the Western >emisphere over a bridge of land that once e<isted bet#een ,iberia and Alaska. A! have migrated to the Western >emisphere over a bridge of land that once e<isted "! #ere migrating to the Western >emisphere over a bridge of land that e<isted once $! migrated over a bridge of land to the Western >emisphere that once e<isted &! migrated to the Western >emisphere over a bridge of land that once e<isted '! #ere migrating to the Western >emisphere over a bridge of land e<isting once 447. It is #ell kno#n in the supermarket industry that ho# items are placed on shelves and the fre5uency of inventory turnovers can be crucial to profits. A! the fre5uency of inventory turnovers can be "! the fre5uency of inventory turnovers is often $! the fre5uency #ith #hich the inventory turns over is often &! ho# fre5uently is the inventory turned over are often '! ho# fre5uently the inventory turns over can be 44:. It may be another fifteen years before spacecraft from 'arth again venture to 8ars% a planet no# kno#n to be cold% dry% and probably lifeless. A! again venture to 8ars% a planet no# kno#n to be "! venture to 8ars again% a planet no# kno#n for being $! #ill venture to 8ars again% a planet no# kno#n as being &! venture again to 8ars% a planet that is kno#n no# to be '! #ill again venture to 8ars% a planet kno#n no# as being 44*. It may be that by focusing primarily on a patient3s mental condition and on vague and often very speculative psychodynamic factors% predictions about the patient3s future behavior decrease in accuracy. A! predictions about the patient3s future behavior decrease in accuracy "! mental health professionals lessen the accuracy of their predictions about the patient3s future

SC Strategy

$'*

behavior $! the accuracy of predictions about the patient3s future behavior lessens &! a decrease in accuracy is seen in predictions about the patient3s future behavior can be predicted less accurately '! the patient3s future behavior can be predicted less accurately 440. It may someday be #orth#hile to try to recover uranium from sea#ater% but at present this process is prohibitively e<pensive. A! It may someday be #orth#hile to try to recover uranium from sea#ater "! ,omeday% it may be #orth#hile to try and recover uranium from sea#ater $! =rying to recover uranium out of sea#ater may someday be #orth#hile &! =o try for the recovery of uranium out of sea#ater may someday be #orth#hile '! Decovering uranium from sea#ater may be #orth#hile to try to do someday 44). It seems likely that a number of astronomical phenomena% such as the formation of planetary nebulas% may be caused by the interaction #here t#o stars orbit each other at close range. A! may be caused by the interaction #here t#o stars orbit each other "! may be caused by the interaction bet#een t#o stars that each orbit the other $! are because of the interaction bet#een t#o stars that orbit each other &! are caused by the interaction of t#o stars #here each is orbiting the other '! are caused by the interaction of t#o stars orbiting each other 472. It takes a deft balance bet#een savings discipline% investment kno#ledge% risk taking% luck% and time to raise a million dollars through investments. A! It takes a deft balance bet#een "! &eft balancing is needed bet#een $! &eftly balanced% it takes &! It takes a deft balance of '! A deft balance is #hat one needs among 471. It #as an increase in reported cases of malaria along the -ulf $oast that in 1)(1 led the health authorities3 granting a permit for e<perimentation #ith human sub6ects to the group that later #ould be called by the name of +nimedco. A! authorities3 granting a permit for e<perimentation #ith human sub6ects to the group that later #ould be called by the name of "! authorities3 granting a permit for e<perimentation #ith human sub6ects to the group that later to be called by the name of $! authorities3 granting a permit for e<perimentation using human sub6ects to the group that later #ould be called by the name of

$'7

&! authorities to grant a permit for human e<perimentation to the group later called '! authorities to grant a permit for human e<perimentation to the group that later #ould be called by the name of 47(. It #as because she #as plagued by in6uries that tennis celebrity =racy Austin% #ho #on the Italian and +.,. Gpens at age 1: in 1)*)% sat out the years bet#een 1)04 to 1)00% occasionally #orking as a television commentator. A! It #as because she #as plagued by in6uries that tennis celebrity =racy Austin% #ho #on the Italian and +.,. Gpens at age 1: in 1)*)% sat out the years bet#een "! After #inning the Italian and +.,. Gpens in 1)*) at the age of 1:% in6uries plagued tennis celebrity =racy Austin and forced her to sit out from $! =racy Austin% the tennis celebrity #ho% plagued by in6uries% #on the Italian and +.,. Gpens at age 1: in 1)*)% sat out from &! 1lagued by in6uries% tennis celebrity =racy Austin% #ho #on the Italian and +.,. Gpens at age 1: in 1)*)% sat out the years from '! =ennis celebrity =racy Austin% #ho #on the Italian and +.,. Gpens at age 1: in 1)*)% #as plagued by in6uries #hen she sat out bet#een 47/. It #as the loss of revenue from declines in tourism that in 1)/7 led the ,audi authorities3 granting a concession for oil e<ploration to the company that #ould later be kno#n by the name of Aramco. A! authorities3 granting a concession for oil e<ploration to the company that #ould later be kno#n by the name of "! authorities3 granting a concession for oil e<ploration to the company later to be kno#n as named $! authorities granting a concession for oil e<ploration to the company that #ould later be kno#n by the name of &! authorities to grant a concession for oil e<ploration to the company that later #ill be kno#n as being '! authorities to grant a concession for oil e<ploration to the company later to be kno#n as 474. Capan received huge sums of capital from the +nited ,tates after the ,econd World War% using it to help build a modern industrial system. A! Capan received huge sums of capital from the +nited ,tates after the ,econd World War% using it to help build "! Capan received huge sums of capital from the +nited ,tates after the ,econd World War and used it to help in building $! Capan used the huge sums of capital it received from the +nited ,tates after the ,econd World War to help build &! Capan3s huge sums of capital received from the +nited ,tates after the ,econd World War

SC Strategy

$'8

#ere used to help it in building '! Deceiving huge sums of capital from the +nited ,tates after the ,econd World War% Capan used it to help build 477. Coachim Daff and -iacomo 8eyerbeer are e<amples of the kind of composer #ho receives popular acclaim #hile living% often goes into decline after death% and never regains popularity again. A! often goes into decline after death% and never regains popularity again "! #hose reputation declines after death and never regains its status again $! but #hose reputation declines after death and never regains its former status &! #ho declines in reputation after death and #ho never regained popularity again '! then has declined in reputation after death and never regained popularity 47:. Coan of Arc% a young ?rench#oman #ho claimed to be divinely inspired% turned the tide of 'nglish victories in her country by liberating the city of Grleans and she persuaded $harles BII of ?rance to claim his throne. A! she persuaded $harles BII of ?rance to claim his throne "! persuaded $harles BII of ?rance in claiming his throne $! persuading that the throne be claimed by $harles BII of ?rance &! persuaded $harles BII of ?rance to claim his throne '! persuading that $harles BII of ?rance should claim the throne 47*. Cohn ,mith provides information on the conditions that lead #omen to a gynecologist% and he notes that these conditions can% and sometimes are% used in the promotion of surgery #hich is not needed. A! and he notes that these conditions can% and sometimes are% used in the promotion of surgery #hich is not needed "! noting that these conditions can be% and sometimes are% used to promote needless surgery $! and notes that they could and sometimes are used unnecessarily in promoting surgery &! sometimes promoting needless surgery '! #hich they use sometimes to promote unnecessary surgery 470. Coplin3s faith in his opera =remonisha #as unshakableE in 1)11 he published the score at his o#n e<pense and decided on staging it himself. A! on staging it himself "! that he himself #ould do the staging $! to do the staging of the #ork by himself &! that he himself #ould stage it '! to stage the #ork himself

$'9

47).

Cudge "onham denied a motion to allo# members of the 6ury to go home at the end of each day instead of to confine them to a hotel. A! to allo# members of the 6ury to go home at the end of each day instead of to confine them to "! that #ould have allo#ed members of the 6ury to go home at the end of each day instead of confined to $! under #hich members of the 6ury are allo#ed to go home at the end of each day instead of confining them in &! that #ould allo# members of the 6ury to go home at the end of each day rather than confinement in '! to allo# members of the 6ury to go home at the end of each day rather than be confined to

4:2.

It could be argued that the most significant virtue of a popular democracy is not the right to 12 participate in the selection of leaders% but rather that it affirms our importance in the scheme of things. A! but rather that it affirms "! but rather its affirmation of $! but rather it3s affirmation in terms of &! but instead of that% its affirming that '! affirming rather

4:1.

Cudge ?orer recogni9es that the American 6udicial system provides more safeguards for accused persons than does the legal system of any other country% but she believes there is a great disparity bet#een the systems of 6ustice accorded rich and poor. A! for accused persons than does the legal system of any other country "! to accused persons as the legal system of any country $! for accused persons as the legal system of any country &! to accused persons as the legal system of any other country '! for accused persons than the legal system of any country

4:(.

Cudge .ois ?orer3s study asks #hy do some litigants have a preferred status over others in the use of a public resource% the courts% #hich in theory are available to all but in fact are une5ually distributed among rich and poor. A! do some litigants have a preferred status over others in the use of a public resource% the courts% #hich in theory are available to all but in fact are une5ually distributed among "! some litigants have a preferred status over others in the use of a public resource% the courts% #hich in theory are available to all but in fact are une5ually distributed bet#een $! do some litigants have a preferred status over another in the use of a public resource% the courts% in theory available to all but in fact are une5ually distributed among &! some litigants have a preferred status to another in the use of a public resource% the courts% in

SC Strategy

$';

theory available to all but in fact not e5ually distributed bet#een '! does one litigant have a preferred status over the other in the use of a public resource% the courts% in theory available to all but in fact they are not e5ually distributed among 4:/. Cudicial rules in many states re5uire that the identities of all prosecution #itnesses are made kno#n to defendants so they can attempt to rebut the testimony% but the $onstitution e<plicitly re5uires only that the defendant have the opportunity to confront an accuser in court. A! that the identities of all prosecution #itnesses are made kno#n to defendants so they can attempt to rebut "! that the identities of all prosecution #itnesses be made kno#n to defendants so that they can attempt to rebut $! that the defendants should kno# the identities of all prosecution #itnesses so they can attempt a rebuttal of &! the identities of all prosecution #itnesses should be made kno#n to defendants so they can attempt rebutting '! making kno#n to defendants the identities of all prosecution #itnesses so that they can attempt to rebut 4:4. Cust as a #riter trying to understand shtetl life might read ,halom Aleichem or Isaac "ashevis ,inger% in the same #ay #riters trying to understand "lack life in the American ,outh might #ell listen to records by the 8ississippi &elta bluesman $harlie 1atton. A! in the same #ay #riters trying to understand "lack life in the American ,outh might #ell listen to records "! in the same #ay #riters #ho try and understand "lack life in the American ,outh might #ell listen to record $! so a #riter trying to understand black life in the American ,outh might #ell listen to records &! so do #riters try and understand "lack life in the American ,outh and might #ell listen to a record '! then #riters trying to understand "lack life in the American ,outh could #ell listen to records 4:7. Cust as plant species native to regions #ith bro#sing mammals evolved many natural antibro#ser defenses such as sharp spines and to<ic chemicals!% so humans in malarial regions have evolved do9ens of chemical defenses against malaria. A! so humans in malarial regions have evolved do9ens of chemical defenses against malaria "! humans in malarial regions have been evolving do9ens of chemical defenses against malaria $! there has been% in malarial regions% an evolution of do9ens of human chemical defenses against malaria &! do9ens of chemical defenses against malaria have been evolved by humans in malarial regions '! similarly% in malarial regions% humans have evolved do9ens of chemical defenses against malaria

$'I

4::.

Cust as reading ,amuel 1epys3s diary gives a student a sense of the seventeenth centuryHof its te<ture and psycheHso Cane ?reed3s guileless child narrator takes the operagoer inside turn-ofthe-century Bienna. A! so Cane ?reed3s guileless child narrator takes the operagoer "! so listening to Cane ?reed3s guileless child narrator takes the operagoer $! so the guileless child narrator of Cane ?reed takes the operagoer &! listening to Cane ?reed3s guileless child narrator takes the operagoer '! Cane ?reed3s guileless child narrator takes the operagoer to her opera

4:*.

Cust as the 'uropean countries of the early eighteenth century sought to e<ploit the resources of our continent% so too are #e no# attempting to e<tract energy and minerals from the ocean bed. A! Cust as the 'uropean countries of the early eighteenth century sought to e<ploit the resources of our continent% so too "! =he 'uropean countries of the early eighteenth century sought to e<ploit the resources of our continent% and in a similar #ay $! .ike the case of the 'uropean countries of the early eighteenth century #ho sought to e<ploit the resources of our continent% so too &! As in the e<ploitation of the resources of our continent by 'uropean countries of the early eighteenth century '! ,imilar to the 'uropean countries #hich sought in the early eighteenth century to e<ploit the resources of our continent

4:0.

Cust because Fing Alfred occupied and fortified .ondon in 00: did not mean that he also #on the loyalty of its citi9ensA the invading &anes #ere #ell a#are of this #eakness and used it to their advantage in 0)/. A! Cust because Fing Alfred occupied and fortified .ondon in 00: did not mean that he "! =he fact that Fing Alfred had occupied and fortified .ondon in 00: did not mean that he had $! Cust because Fing Alfred occupied and fortified .ondon in 00:% it did not mean he &! =he fact that Fing Alfred occupied and fortified .ondon in 00:% it did not mean that he '! Cust because Fing Alfred had occupied and fortified .ondon in 00:% it did not mean he

4:).

Fansas Depublican Nancy Fassebaum% one of only t#o #omen in the +.,. ,enate in 1))(% said she did not so much #ish for more #omen senators but more moderate Depublican ones. A! did not so much #ish for more #omen senators but more moderate Depublican ones "! #ished not so much for more senators #ho #ere #omen than moderate Depublicans $! did not #ish so much for more #omen senators as for more moderate Depublicans &! did not #ish for more #omen senators so much as moderate Depublicans '! #ished for more senators #ho are moderate Depublicans than #omen

4*2.

.acking information about energy use% people tend to overestimate the amount of energy used

SC Strategy

$'J

by e5uipment% such as lights% that are visible and must be turned on and off and underestimate that used by unobtrusive e5uipment% such as #ater heaters. A! e5uipment% such as lights% that are visible and must be turned on and off and underestimate that "! e5uipment% such as lights% that are visible and must be turned on and off and underestimate it #hen $! e5uipment% such as lights% that is visible and must be turned on and off and underestimate it #hen &! visible e5uipment% such as lights% that must be turned on and off and underestimate that '! visible e5uipment% such as lights% that must be turned on and off and underestimate it #hen 4*1. .ast spring a $olorado health department survey of *( playgrounds in private child-care centers found unsafe conditions in )7 percent of them and they ranged from splinters to e5uipment near collapse. A! unsafe conditions in )7 percent of them and they ranged "! conditions in )7 percent #ere unsafe and ranging $! the ranging of unsafe conditions in )7 percent of them to be &! that )7 percent had unsafe conditions ranging '! that )7 percent of them had conditions that #ere unsafeE the range #as 4*(. .ast year% land values in most parts of the pinelands rose almost so fast% and in some parts even faster than #hat they did outside the pinelands. A! so fast% and in some parts even faster than #hat they did "! so fast% and in some parts even faster than% those $! as fast% and in some parts even faster than% those &! as fast as% and in some parts even faster than% those '! as fast as% and in some parts even faster than #hat they did 4*/. .a#makers are e<amining measures that #ould re5uire banks to disclose all fees and account re5uirements in #riting% provide free cashing of government checks% and to create basic savings accounts to carry minimal fees and re5uire minimal initial deposits. A! provide free cashing of government checks% and to create basic savings accounts to carry "! provide free cashing of government checks% and creating basic savings accounts carrying $! to provide free cashing of government checks% and creating basic savings accounts that carry &! to provide free cashing of government checks% creating basic savings accounts to carry '! to provide free cashing of government checks% and to create basic savings accounts that carry 4*4. .egend has it that #hen the 8oha#k Coseph "rant #as presented to -eorge III in 1**:% he proudly refused to kiss the Fing3s hand% inasmuch as he regarded himself an ally% not a sub6ect.

$*E

A! as he regarded himself an ally% not "! that he regarded himself to be an ally rather than $! as he considered himself an ally% not &! that he considered himself to be ally instead of '! as he considered himself as an ally rather than 4*7. .egislation in the $anadian province of Gntario re5uires of both public and private employers that pay be the same for 6obs historically held by #omen as for 6obs re5uiring comparable skill that are usually held by men. A! that pay be the same for 6obs historically held by #omen as for 6obs re5uiring comparable skill that are "! that pay for 6obs historically held by #omen should be the same as for a 6ob re5uiring comparable skills $! to pay the same in 6obs historically held by #omen as in 6obs of comparable skill that are &! to pay the same regardless of #hether a 6ob #as historically held by #omen or is one demanding comparable skills '! to pay as much for 6obs historically held by #omen as for a 6ob demanding comparable skills 4*:. .ess detrimental than the effects of bacterial transformation is the effects of bacterial deteriorationA spoilage of food% metals corroding% decay of #ood% and other undesirable alterations of substances. A! is the effects of bacterial deteriorationA spoilage of food% metals corroding "! are the effects of bacterial deteriorationA spoilage of food% metals corroding $! is the effects of bacterial deterioration% #hich include spoilage of food% metals corroding &! are the effects of bacterial deterioration% #hich includes spoilage of food% corrosion of metals '! are the effects of bacterial deteriorationA spoilage of food% corrosion of metals 4**. .ike Auden% the language of Cames 8errill is chatty% arch% and conversationalHgiven to comple< syntactic flights as #ell as to prosaic free-verse strolls. A! .ike Auden% the language of Cames 8errill "! .ike Auden% Cames 8errill3s language $! .ike Auden3s% Cames 8errill3s language &! As #ith Auden% Cames 8errill3s language '! As is Auden3s the language of Cames 8errill 4*0. .ike "yron at 8issolonghi% Cack .ondon #as slo#ly killed by the mistakes of the medical men #ho treated him. A! .ike "yron "! .ike "yron3s death

SC Strategy

$*$

$! Cust as "yron died &! ,imilar to "yron '! As did "yron 4*). .ike 'dvard -rieg% #hom the ,candinavians long refused to recogni9e% the Italians3 disregard for Berdi persisted for a decade after his critical acclaim in ?rance and Austria. A! .ike 'dvard -rieg% #hom the ,candinavians long refused to recogni9e% "! .ike 'dvard -rieg% #ho the ,candinavians long refused to recogni9e% $! Cust as 'dvard -rieg #as long refused recognition by the ,candinavians% &! Cust as the ,candinavians long refused to recogni9e 'dvard -rieg% so '! .ike the ,candinavians3 long refusal to recogni9e 'dvard -rieg% 402. .ike >aydn% ,chubert #rote a great deal for the stage% but he is remembered principally for his chamber and concert-hall music. A! .ike >aydn% ,chubert "! .ike >aydn% ,chubert also $! As has >aydn% ,chubert &! As did >aydn% ,chubert also '! As >aydn did% ,chubert also 401. .ike Cohn 8c1hee3s #orks% Ann "eattie painstakingly assembles in her #orks an interesting and complete #orld out of hundreds of tiny details about a seemingly uninteresting sub6ect. A! .ike Cohn 8c1hee3s #orks% Ann "eattie painstakingly assembles in her #orks "! .ike Cohn 8c1hee% Ann "eattie3s #orks painstakingly assemble $! .ike Cohn 8c1hee% Ann "eattie painstakingly assembles in her #orks &! Cust as Cohn 8c1hee3s% so Ann "eattie3s #orks painstakingly assemble '! Cust as Cohn 8c1hee% Ann "eattie painstakingly assembles in her #orks 40(. .ike many others of his generation of Native American leaders% Coseph "rant lived in t#o #orldsE born into an Iro5uois community and instructed in traditional Iro5uois #ays% he also received an education from 'nglish-speaking teachers. A! .ike many others of his generation of Native American leaders% Coseph "rant lived in t#o #orldsE "! .ike many others of his generation of Native American leaders% living in t#o #orlds% Coseph "rant #as $! .ike many another of his generation of Native American leaders% Coseph "rant% living in t#o #orlds% #as &! As #ith many others of his generation of Native American leaders% living in t#o #orlds% Coseph "rant #as

$*'

'! As #ith many another of his generation of Native American leaders% Coseph "rant lived in t#o #orldsE 40/. .ike many self-taught artists% 1erle >essing did not begin to paint until she #as #ell into middle age. A! .ike "! As have $! Cust as #ith &! Cust like '! As did 404. .ike other educators #ho prefer to substitute anthologies of short stories or collections of popular essays to dull basal readers% 8s. "urton emphasi9es ho# important it is to en6oy good literature. A! to dull basal readers% 8s. "urton emphasi9es ho# important it is to en6oy "! for dull basal readers% 8s. "urton emphasi9es the importance of en6oying $! to dull basal readers% 8s. "urton emphasi9es that it is important to en6oy &! for dull basal readers% 8s. "urton3s emphasis is that it is important to en6oy '! to dull basal readers% 8s. "urton3s emphasis is on the importance of en6oying 407. .ike Dousseau% =olstoi rebelled against the unnatural comple<ity of human relations in modern society. A! .ike Dousseau% =olstoi rebelled "! .ike Dousseau% =olstoi3s rebellion #as $! As Dousseau% =olstoi rebelled &! As did Dousseau% =olstoi3s rebellion #as '! =olstoi3s rebellion% as Dousseau3s% #as 40:. .ike the 10)2s 1opulists #ho e<alted the rural myth% so urban leaders of the 1))2s are trying to glorify the urban myth. A! .ike the 10)2s 1opulists #ho e<alted the rural myth% "! Cust as the 1opulists of the 10)2s e<alted the rural myth% $! =he 1opulists of the 10)2s having e<alted the rural myth% &! Cust like the rural myth #as e<alted by the 1opulists of the 10)2s '! 1opulists of the 10)2s #ere e<alting the rural myth% and 40*. .ike the color-discriminating apparatus of the human eye% insects3 eyes depend on recording and comparing light intensities in three regions of the electromagnetic spectrum. A! insects3 eyes depend on "! an insect eye depends on

SC Strategy

$**

$! that of insects depend on the &! that of an insect3s eye depends on '! that of an insect3s is dependent on the 400. .ike the government that came before it% #hich set ne# records for gro#th% laisse9-faire capitalism is the cornerstone of the ne# government. A! laisse9-faire capitalism is the cornerstone of the ne# government "! the cornerstone of the ne# government is laisse9-faire capitalism $! laisse9-faire capitalism is the ne# government3s cornerstone &! the ne# government has made laisse9-faire capitalism its cornerstone '! the ne# government has a laisse9-faire cornerstone of capitalism 40). .ike the one reputed to live in .och Ness% also an inland lake connected to the ocean by a river% inhabitants of the area around .ake $hamplain claim sightings of a long and narro# sea monster. A! .ike the one reputed to live in .och Ness% also an inland lake connected to the ocean by a river% inhabitants of the area around .ake $hamplain claim sightings of a long and narro# sea monster. "! Inhabitants of the area around .ake $hamplain claim sightings of a long and narro# sea monster similar to the one reputed to live in .och Ness% #hich% like .ake $hamplain% is an inland lake connected to the ocean by a river. $! Inhabitants of the area around .ake $hamplain claim sightings of a long and narro# sea monster similar to .och Ness3s% #hich% like .ake $hamplain% is an inland lake connected to the ocean by a river. &! .ike .och Ness3 reputed monster% inhabitants of the area around .ake $hamplain% also an inland lake connected to the ocean by a river% claim sightings of a long and narro# sea monster. '! ,imilar to that reputed to live in .och Ness% inhabitants of the area around .ake $hamplain% also an inland lake connected to the ocean by a river% claim sightings of a long and narro# sea monster. 4)2. .ike their male counterparts% #omen scientists are above average in terms of intelligence and creativity% but unlike men of science% their female counterparts have had to #ork against the grain of occupational stereotyping to enter a man3s #orld. A! their female counterparts have had to #ork "! their problem is #orking $! one thing they have had to do is #ork &! the handicap #omen of science have had is to #ork '! #omen of science have had to #ork

$*7

4)1.

.ocal residents claim that ,an Antonio% =e<as% has more good 8e<ican American restaurants than any city does in the +nited ,tates. A! any city does "! does any other city $! other cities do &! any city '! other cities

4)(.

.os Angeles has a higher number of family d#ellings per capita than any large city. A! a higher number of family d#ellings per capita than any large city "! higher numbers of family d#ellings per capita than any other large city $! a higher number of family d#ellings per capita than does any other large city &! higher numbers of family d#ellings per capita than do other large cities '! a high per capita number of family d#ellings% more than does any other large city

4)/.

.o#er-level management function most effectively #hen they have thorough training and unimpeded communication #ith #orkers% supervisors% and one another. A! management function most effectively #hen they have "! managers function most effectively #hen they have $! management functions most effectively #hen they have &! managers function most effectively #hen they have received '! management functions most effectively #hen it has

4)4.

8achines replacing human labor% there #as #ide anticipation that the #ork#eek #ould continue to become shorter. A! 8achines replacing human labor% there #as #ide anticipation that "! When machines replaced human labor% there #as #ide anticipation $! As machines replaced human labor% it #as #idely anticipated that &! Insofar as machines replaced human labor% it #as #idely anticipated '! >uman labor being replaced by machines% there #as #ide anticipation that

4)7.

8adagascar #as one of the last habitable areas of the earth to undergo human settlement% #hich has made it an ideal site for researching rare flora and fauna. A! 8adagascar #as one of the last habitable areas of the earth to undergo human settlement% #hich has made it "! 8adagascar #as one of the last habitable areas of the earth to have undergone human settlement% and that has made it $! 8adagascar under#ent human settlement as one of the last habitable areas of the earth% #hich makes it

SC Strategy

$*8

&! 8adagascar% one of the last habitable areas of the earth% under#ent human settlement% making it '! "ecause 8adagascar #as one of the last habitable areas of the earth to undergo human settlement% it is 4):. 8anifestations of Islamic political militancy in the first period of religious reformism #ere the rise of the Wahhabis in Arabia% the ,anusi in $yrenaica% the ?ulani in Nigeria% the 8ahdi in the ,udan% and the victory of the +suli mu6tahids in ,hiite Iran and Ira5. A! 8anifestations of Islamic political militancy in the first period of religious reformism #ere the rise of the Wahhabis in Arabia% the ,anusi in $yrenaica% the ?ulani in Nigeria% the 8ahdi in the ,udan% and "! 8anifestations of Islamic political militancy in the first period of religious reformism #ere sho#n in the rise of the Wahhabis in Arabia% the ,anusi in $yrenaica% the ?ulani in Nigeria% the 8ahdi in the ,udan% and also $! In the first period of religious reformism% manifestations of Islamic political militancy #ere the rise of the Wahhabis in Arabia% of the ,anusi in $yrenaica% the ?ulani in Nigeria% the 8ahdi in the ,udan% and &! In the first period of religious reformism% manifestations of Islamic political militancy #ere sho#n in the rise of the Wahhabis in Arabia% the ,anusi in $yrenaica% the ?ulani in Nigeria% the 8ahdi in the ,udan% and '! In the first period of religious reformism% Islamic political militancy #as manifested in the rise of the Wahhabis in Arabia% the ,anusi in $yrenaica% the ?ulani in Nigeria% and the 8ahdi in the ,udan% and in 4)*. 8any investors base their choice bet#een bonds and stocks on comparing bond yields to the dividends available on common stocks. A! bet#een bonds and stocks on comparing bond yields to "! among bonds and stocks on comparisons of bond yields to $! bet#een bonds and stocks on comparisons of bond yields #ith &! among bonds and stocks on comparing bond yields and '! bet#een bonds and stocks on comparing bond yields #ith 4)0. 8any of them chiseled from solid rock centuries ago% the mountainous regions of northern 'thiopia are dotted #ith hundreds of monasteries. A! 8any of them chiseled from solid rock centuries ago% the mountainous regions of northern 'thiopia are dotted #ith hundreds of monasteries. "! $hiseled from solid rock centuries ago% the mountainous regions of northern 'thiopia are dotted #ith many hundreds of monasteries. $! >undreds of monasteries% many of them chiseled from solid rock centuries ago% are dotting the mountainous regions of northern 'thiopia.

$*9

&! =he mountainous regions of northern 'thiopia are dotted #ith hundreds of monasteries% many of #hich are chiseled from solid rock centuries ago. '! =he mountainous regions of northern 'thiopia are dotted #ith hundreds of monasteries% many of them chiseled from solid rock centuries ago. 4)). 8any policy e<perts say that shifting a portion of health-benefit costs back to the #orkers helps to control the employer3s costs% but also helps to limit medical spending by making patients more careful consumers. A! helps to control the employer3s costs% but also helps "! helps the control of the employer3s costs% and also $! not only helps to control the employer3s costs% but also helps &! helps to control not only the employer3s costs% but '! not only helps to control the employer3s costs% and also helps 722. 8any psychologists and sociologists no# contend that the deliberate and even brutal aggression integral to some forms of competitive athletics increase the likelihood of imitative violence that erupts among cro#ds of spectators dominated by young adult males. A! increase the likelihood of imitative violence that erupts "! increase the likelihood that there #ill be an eruption of imitative violence $! increase the likelihood of imitative violence erupting &! increases the likelihood for imitative violence to erupt '! increases the likelihood that imitative violence #ill erupt 721. 8any scholars regard the pre->ispanic civili9ations of 1eru as the most impressive in ,outh America. A! regard the pre->ispanic civili9ations of 1eru as "! regard the pre->ispanic civili9ations of 1eru to be $! regard the pre->ispanic civili9ations of 1eru to have been &! consider that the pre->ispanic civili9ations of 1eru are '! consider the pre->ispanic civili9ations of 1eru as 72(. 8any #riters of modern 'nglish have ac5uired careless habits that damage the clarity of their prose% but these habits can be broken if they are #illing to take the necessary trouble. A! but these habits can be broken "! but these habits are breakable $! but they can break these habits &! #hich can be broken '! e<cept that can be broken 72/. 8argaret $ourtney-$larke has traveled to remote d#ellings in the =ransvaal to photograph

SC Strategy

$*;

the art of Ndebele #omen% #hose murals are brilliantly colored% their geometrical symmetries embellished #ith old and ne# iconography and in a style that varies from #oman to #oman and house to house. A! #hose murals are brilliantly colored% their geometrical symmetries embellished #ith old and ne# iconography and in a style that varies from #oman to #oman and house to house "! #hose murals are brilliantly colored% their geometrical symmetries are embellished #ith old and ne# iconography% and their style is varying among #omen and houses $! #hose murals are brilliantly colored% their geometrical symmetries are embellished #ith old and ne# iconography% and they are in styles that vary from #oman to #oman and house to house &! #ith murals brilliantly colored% their geometrical symmetries embellished #ith old and ne# iconography% and their style varies among #omen and houses '! #ith murals that are brilliantly colored% their geometrical symmetries embellished #ith old and ne# iconography% and their styles vary among #omen and houses 724. 8arketing researchers have found that% because many residents of the ,outheast do not share the same ethnic heritage as Northeasterners% the t#o varieties of commercially prepared colesla# most popular in Ne# @ork $ity and "oston are virtually ignored by consumers in Dichmond and Daleigh. A! because many residents of the ,outheast do not share the same ethnic heritage as "! because many residents of the ,outheast do not share the same ethnic heritage #ith $! because many residents of the ,outheast do not have the same ethnic heritage as &! due to many residents of the ,outheast not sharing the same ethnic heritage #ith '! #hereas many residents of the ,outheast do not share the same ethnic heritage as 727. 8igraine% the most debilitating common form of headache% afflicts perhaps 10 million Americans% #ho collectively lose :4 million #orkdays a year% and they cost the nation I72 billion in medical e<penses and lost #ork time. A! year% and they cost the nation I72 billion in medical e<penses and lost "! year and thus costs the nation I72 billion in medical e<penses and lost $! year% so as to cost the nation I72 billion in medical e<penses and lost &! year that costs the nation I72 billion in lost medical e<penses and '! year% #hich thus cost the nation I72 billion in lost medical e<penses and 72:. 8innesota is the only one of the contiguous forty-eight states that still has a si9able #olf population% and #here this predator remains the archenemy of cattle and sheep. A! that still has a si9able #olf population% and #here "! that still has a si9able #olf population% #here $! that still has a si9able population of #olves% and #here

$*I

&! #here the population of #olves is still si9ableE '! #here there is still a si9able population of #olves and #here 72*. 8odern critics are amused by early scholars3 categori9ing =acitus3s -ermania as an ethnographic treatise. A! scholars3 categori9ing =acitus3s -ermania as "! scholars3 categori9ing =acitus3s -ermania as if $! scholars% categori9ing of =acitus3s -ermania as &! scholars #ho categori9e =acitus3s -ermania as '! scholars #ho categori9e =acitus3s -ermania if 720. 8onitoring heart patients3 e<ercise% as #ell as athletes e<ercising% is no# done by small transmitters broadcasting physiological measurements to nearby recording machines. A! 8onitoring heart patients3 e<ercise% as #ell as athletes e<ercising% is no# done by small transmitters broadcasting physiological measurements to nearby recording machines. "! 8onitoring the e<ercise of heart patients% as #ell as athletes e<ercising% is no# done by small transmitters broadcasting physiological measurements to nearby recording machines. $! ,mall transmitters broadcasting physiological measurements to nearby recording machines are no# used to monitor the e<ercise of both heart patients and athletes. &! "roadcasting physiological measurements to nearby recording machines% small transmitters are no# used to monitor heart patients3 e<ercise% as #ell as athletes e<ercising. '! "oth athletes e<ercising and heart patients3 e<ercise are no# monitored by small transmitters broadcasting physiological measurements to nearby recording machines. 72). 8ore ancient 'gyptian temples #ere constructed in the reign of Damses II as in any other. A! as in any other "! as any other $! as in others &! than others '! than in any other 712. 8ore and more in recent years% cities are stressing the arts as a means to greater economic development and investing millions of dollars in cultural activities% despite strained municipal budgets and fading federal support. A! to greater economic development and investing "! to greater development economically and investing $! of greater economic development and invest &! of greater development economically and invest '! for greater economic development and the investment of

SC Strategy

$*J

711.

8ore than ever% paper is e<pected to be recycled this year% due to ne# mandatory recycling la#s in municipalities across the nation. A! 8ore than ever% paper is e<pected to be recycled this year "! It is e<pected that more paper than ever #ill be recycled this year than previously and that is $! =he paper e<pected to be recycled this year is more than ever &! =he amount of paper that #ill be recycled this year is e<pected to be greater than ever '! A great increase in the amount of paper that #ill be recycled this year is

71(.

8ore than five thousand years ago% $hinese scholars accurately described the flo# of blood as a continuous circle controlled by the heart% but it #ent unnoticed in the West. A! but it #ent "! but it #as $! although it #as &! but the discovery #ent '! although the discovery #as

71/.

8ore than thirty years ago &r. "arbara 8c-$lintock% the Nobel 1ri9e #inner% reported that genes can 6ump% as pearls moving mysteriously from one necklace to another. A! as pearls moving mysteriously from one necklace to another "! like pearls moving mysteriously from one necklace to another $! as pearls do that move mysteriously from one necklace to others &! like pearls do that move mysteriously from one necklace to others '! as do pearls that move mysteriously from one necklace to some other one

714.

8ost $orporations pay at least t#ice as much to full-time employees% if the value of benefits% sick days% and paid vacation days are included in earnings% than to part-time employees% #hose hourly #ages are often higher than those of their full-time colleagues. A! are included in earnings% than "! are included in earnings% as $! is included in earnings% than they pay &! is included in earnings% as is paid '! is included in earnings% as they pay

717.

8ost energy analysts no# agree that the costs of building and maintaining nuclear reactors are too high for nuclear po#er to likely prove cheaper than coal or oil in the long run. A! too high for nuclear po#er to likely "! high enough for nuclear po#er to be unlikely to $! high enough that it is unlikely nuclear po#er #ill &! so high that nuclear po#er is unlikely to

$7E

'! so high as to be unlikely that nuclear po#er #ill 71:. 8ost large companies prefer customi9ed computer soft#are because it can be molded to fit the #ay a company does business% #hen off-the-shelf soft#are often re5uires the company to alter its procedures to fit the soft#are. A! #hen "! since $! #hereas &! because '! insofar as 71*. 8ost nations regard their airspace as e<tending up#ard as high as an aircraft can flyE no specific altitude% ho#ever% has been officially recogni9ed as a boundary. A! as e<tending "! as the e<tent $! to be an e<tent &! to be an e<tension '! to e<tend 710. 8ost North $arolina ski resorts broadcast music onto the slopesE skiers can choose among hard rock% soft pop% and beautiful music slopes% there are no slopes #ithout music. A! skiers can choose among hard rock% soft pop% and beautiful music slopes% there are "! because skiers can choose hard rock% soft pop% or beautiful music% there are $! ho#ever% skiers can choose among hard rock% soft pop% beautiful music% and &! although skiers can choose among hard rock% soft pop% and beautiful music slopes% there are '! skiers can choose among hard rock% soft pop% beautiful music slopes% but 71). 8ost primates are immune to feline rhinovirus% but a specific group% this being the golden maca5ues% are highly susceptible to the organism. A! a specific group% this being the golden maca5ues% are highly susceptible to "! a specific group% the golden maca5ues% is highly susceptible to $! one group% specifically golden maca5ues% is highly susceptible of &! a specific group% the golden maca5ues% #hich are highly susceptible to '! a specific group% the golden maca5ues% #hich are highly susceptible of 7(2. 8ost state constitutions no# mandate that the state budget be balanced each year. A! mandate that the state budget be balanced "! mandate the state budget to be balanced $! mandate that the state budget #ill be balanced

SC Strategy

$7$

&! have a mandate for a balanced state budget '! have a mandate to balance the state budget 7(1. 8ost teen-agers #ho #ork for pay hold 6obs that re5uire fe# skills% little responsibility% and also no hope for career advancement. A! little responsibility% and also "! little responsibility% and #ith $! little responsibility% and offer &! carry little responsibility% and '! carry little responsibility% and offer 7((. 8ost victims of infectious mononucleosis recover after a fe# #eeks of listlessness% but an unlucky fe# may suffer for years. A! but an unlucky fe# may suffer "! and an unlucky fe# have suffered $! that an unlucky fe# might suffer &! that a fe# being unlucky may suffer '! but a fe# #ho% being unlucky% suffered 7(/. 8s. $hambers is among the forecasters #ho predict that the rate of addition to arable lands #ill drop #hile those of loss rise. A! those of loss rise "! it rises for loss $! those of losses rise &! the rate of loss rises '! there are rises for the rate of loss 7(4. 8s. Wright tries to get inside Iran to understand ho# it #orks% the role it has played in the 8iddle 'ast and its intricate relationship #ith the +nited ,tates% #hich is comple< and depends heavily on understanding of the ?arsi language. A! #hich is comple< and depends "! because it is comple< and depends $! but they are comple< and depend &! #hich are comple< and depend '! a task that is comple< and depends 7(7. Native American burial sites dating back 7%222 years indicate that the residents of 8aine at that time #ere part of a #idespread culture of Algon5uian-speaking people. A! #ere part of a #idespread culture of Algon5uian-speaking people

$7'

"! had been part of a #idespread culture of people #ho #ere Algon5uian-speaking $! #ere people #ho #ere part of a #idespread culture that #as Algon5uian-speaking &! had been people #ho #ere part of a #idespread culture that #as Algon5uian-speaking '! #ere a people #hich had been part of a #idespread% Algon5uian-speaking culture 7(:. Neanderthals had a vocal tract that resembled those of the apes and so #ere probably #ithout language% a shortcoming that may e<plain #hy they #ere supplanted by our o#n species. A! Neanderthals had a vocal tract that resembled those of the apes "! Neanderthals had a vocal tract resembling an ape3s $! =he vocal tracts of Neanderthals resembled an ape3s &! =he Neanderthal3s vocal tracts resembled the apes3 '! =he vocal tracts of the Neanderthals resembled those of the apes 7(*. Never before had ta<payers confronted so many changes at once as they had in the =a< Deform Act of 1)0:. A! so many changes at once as they had in "! at once as many changes as $! at once as many changes that there #ere #ith &! as many changes at once as they confronted in '! so many changes at once that confronted them in 7(0. Never before in the history of music have musical superstars been able to command so e<traordinary fees of the kind they do today. A! so e<traordinary fees of the kind they do today "! so e<traordinary fees as they are today $! such e<traordinary fees as they do today &! such e<traordinary fees of the kind today3s have '! so e<traordinary a fee of the kind they can today 7(). Ne# hardy varieties of rice sho# promise of producing high yields #ithout the costly re5uirements of irrigation and application of commercial fertili9er by earlier high-yielding varieties. A! re5uirements of irrigation and application of commercial fertili9er by earlier high-yielding varieties "! re5uirements by earlier high-yielding varieties of application of commercial fertili9er and irrigation $! re5uirements for application of commercial fertili9er and irrigation of earlier high-yielding varieties &! application of commercial fertili9er and irrigation that #as re5uired by earlier high-yielding

SC Strategy

$7*

varieties '! irrigation and application of commercial fertili9er that #ere re5uired by earlier high-yielding varieties 7/2. Ne# Cersey3s is one of the five highest number of reported cases of .yme disease in the +nited ,tates. A! Ne# Cersey3s is one of the five highest number of reported "! Ne# Cersey3s is one of the five highest numbers in reporting $! Ne# Cersey has a report of one of the five highest numbers of &! Ne# Cersey has one of the five highest numbers of reported '! Ne# Cersey reports one of the five highest number of 7/1. Ne# techni5ues in thermal-scanning photography% a process that records radiation from surface areas% makes it possible to study the effects of calefaction% or #arming% of a river in greater detail than ever before. A! makes it possible to study the effects of calefaction% or #arming% of a river in greater detail than ever before "! make it possible to study% in greater detail% the effects of calefaction% or #arming% of a river than ever before $! have made it possible to study in greater detail than ever before the effects of calefaction% or #arming% of a river &! make possible the study of the effects of calefaction% or #arming% of a river in greater detail than it ever #as before '! has made it more possible than ever before to study in greater detail the effects of calefaction% or #arming% of a river

7/(.

Ne# theories propose that catastrophic impacts of asteroids and comets may have caused reversals in the 'arth3s magnetic field% the onset of ice ages% splitting apart continents 02 million years ago% and great volcanic eruptions. A! splitting apart continents "! the splitting apart of continents $! split apart continents &! continents split apart '! continents that #ere split apart

7//.

Ne#comers to southern $alifornia are surprised to discover that% in spite of the temperate climate% heating bills are as high% or higher than% those in most 8id#estern or eastern cities. A! as high% or higher than% those in "! as high% or higher than%

$77

$! as high as% or higher than% &! at least as high as those in '! no lo#er than 7/4. No less an authority than Walter $ronkite has reported that half of all Americans never read a book. A! No less an authority than "! Nonetheless an authority $! Nevertheless authoritarian &! Not less an authority than '! An authority not less than

7/7.

No state la# forbids an employer from re6ecting a 6ob applicant or to dismiss an employee based on the results of a lie detector test. A! an employer from re6ecting a 6ob applicant or to dismiss "! an employer to re6ect a 6ob applicant or dismiss $! that employers re6ect a 6ob applicant or dismiss &! the re6ection by an employer of a 6ob applicant or dismissal of '! re6ection by employers of a 6ob applicant or dismissal of

7/:.

None of the attempts to specify the causes of crime e<plains #hy most of the people e<posed to the alleged causes do not commit crimes and% conversely% #hy so many of those not so e<posed have. A! have "! has $! shall &! do '! could

7/*.

Not all employment selection mechanisms that have a disparate effect% that is% that screens out a percentage of minorities or #omen disproportionate to their presence in the relevant labor market% are unla#ful. A! that is% that screens out a percentage of minorities or #omen disproportionate to "! #hich means% that screens out a percentage of minorities or #omen disproportionate #ith $! #hich means% that screen out a percentage of minorities or #omen disproportionate #ith &! that is% that screen out a percentage of minorities or #omen disproportionate to '! that is% that screens out a percentage of minorities or #omen disproportionate #ith

7/0.

Not since -alileo suffered the scurvy humor of the In5uisition has a religious organi9ation

SC Strategy

$78

so effectively curbed the ability of a ma6or scientist that he could pursue a theory. A! has a religious organi9ation so effectively curbed the ability of a ma6or scientist that he could pursue "! did a religious organi9ation so effectively curb the ability of a ma6or scientist that he could pursue $! has a religious organi9ation so effectively curbed the ability of a ma6or scientist to pursue &! did a religious organi9ation so effectively curb the ability of a ma6or scientist to pursue '! has a religious organi9ation so effectively curbed #hether a ma6or scientist had the ability that he could be pursuing 7/). Not until the 'nlightenment% some (22 years ago% had society seriously 5uestioned the right of the state that it could e<ecute its citi9ens. A! had society seriously 5uestioned the right of the state that it could e<ecute "! did society seriously 5uestioned the right of the state that it could e<ecute $! had society seriously 5uestioned the right of the state for the e<ecuting of &! did society seriously 5uestion the right of the state to e<ecute '! had society seriously 5uestioned #hether the state had a right that it could e<ecute 742. No#here in 1rakta is the influence of modern 'uropean architecture more apparent than their government buildings. A! more apparent than their "! so apparent as their $! more apparent than in its &! so apparent than in their '! as apparent as it is in its 741. Nuclear fusion is the force that po#ers the ,un% the stars% and hydrogen bombs% merging the nuclei of atoms and not splitting them apart% as in nuclear reactors. A! merging the nuclei of atoms and not splitting them apart% as in nuclear reactors "! merging the nuclei of atoms instead of splitting them apart% like nuclear reactors $! merging the nuclei of atoms rather than splitting them apart% as nuclear reactors do &! and merges the nuclei of atoms but does not split them apart% as is done in unclear reactors '! and merges the nuclei of atoms% unlike atomic reactors that split them apart 74(. Gberlin $ollege in Ghio #as a renegade institution at its 10// founding for deciding to accept both men and #omen as students. A! at its 10// founding for deciding to accept "! for the decision at its 10// founding to accept $! #hen it #as founded in 10// for its decision to accept

$79

&! in deciding at its founding in 10// to accept '! by deciding at its founding in 10// on the acceptance of 74/. Gf all the possible disasters that threaten American agriculture% the possibility of an adverse change in climate is maybe the more difficult for analysis. A! is maybe the more difficult for analysis "! is probably the most difficult to analy9e $! is maybe the most difficult for analysis &! is probably the more difficult to analy9e '! is% it may be% the analysis that is most difficult 744. Gf all the #ild animals in their area% none #as more useful to the &ela#are tribes than the Birginia #hite tailed deerA it #as a source of meat% and its hide #as used for clothing% its antlers and bones for tools% and its sine#s and gut for bindings and glue. A! deerA it #as a source of meat% and its hide #as used for clothing% its antlers and bones for tools% and its sine#s and gut "! deerA it #as a source of meat% and its hide used for clothing% #ith its antlers and bones for tools% and its sine#s and gut used $! deerA #hich #as a source of meat% #ith its hide used for clothing% antlers and bones for tools% as #ell as its sine#s and gut used &! deerA #hich% as #ell as being a source of meat% its hide #as used for clothing% its antlers and bones for tools% and its sine#s and gut #ere '! deerA #ith% as #ell as being a source of meat% its hide used for clothing% its antlers and bones for tools% and its sine#s and gut 747. Gf the t#in supernovas% the most recently discovered is the largest% but it is the small one that has irrevocably altered astronomists3 ideas about the origins of the universe. A! the most recently discovered is the largest% but it is the small one "! the one most recently discovered is the largest% but it is the small one $! the most recently discovered is larger% but it is the smaller &! the more recently discovered one is the large one but the small one '! the more recently discovered is the larger% but it is the smaller one 74:. Gften visible as smog% o9one is formed in the atmosphere from hydrocarbons and nitrogen o<ides% t#o ma6or pollutants emitted by automobiles% react #ith sunlight. A! o9one is formed in the atmosphere from "! o9one is formed in the atmosphere #hen $! o9one is formed in the atmosphere% and #hen &! o9one% formed in the atmosphere #hen

SC Strategy

$7;

'! o9one% formed in the atmosphere from 74*. Gn stage% the force of $arrick3s personality and the vividness of his acting disguised the fact he #as% #hich his surviving velvet suit sho#s% a short man. A! he #as% #hich his surviving velvet suit sho#s% "! he #as% and it is his surviving velvet suit that sho#s it% $! of him being% as his surviving velvet suit sho#s% &! that he #as% as his surviving velvet suit sho#s% '! sho#n in his surviving velvet suit% that he #as 740. Gn the -reat 1lains% nineteenth-century settlers used mud and grass to build their homes% doing it #ithout timber and nails. A! settlers used mud and grass to build their homes% doing it #ithout "! settlers used mud and grass to build their homes% did it #ithout $! settlers used mud and grass to build their homes% making them #hile not having &! settlers used mud and grass to build their homes% making do #ithout '! settlers3 homes #ere built of mud and grass% making do #ithout 74). Gnce an apolitical confrontation of the #orld3s best athletes% the ,ummer Glympics have been increasingly politici9ed in recent years as the superpo#ers have denied many of the best athletes3 access to competition. A! been increasingly politici9ed "! been increasing politici9ation $! been of increasing politici9ation &! politici9ed% increasingly% '! increased politici9ation 772. Gnce common throughout the Western plains% black-footed ferrets are thought to have declined in number as a result of the poisoning of prairie dogs% their prey. A! black-footed ferrets are thought to have declined in number as "! it is thought that the decline in number of black-footed ferrets is $! the decline in the number of black-footed ferrets is thought of as &! that black-footed ferrets have declined in their numbers is thought to be '! the numbers of the black-footed ferret are thought to have declined as 771. Gnce positioned in space% the >ubble ,pace =elescope #ill capture light from sources t#enty times fainter compared to those that can be detected by ground-based instruments. A! compared to those that can be detected "! compared to those they can detect

$7I

$! than that can be detected &! than those that can be detected '! than those detecting 77(. Gnce the economic and social usefulness of the motor car #as demonstrated and #ith its superiority to the horse being proved% much of the early hostility to it in rural regions disappeared. A! and #ith its superiority to the horse being "! and its superiority over the horse had been $! and its superiority to the horse &! its superiority over the horse '! #ith its superiority to the horse having been 77/. Gnce they had seen the report from the medical e<aminer% the investigators did not doubt #hether the body recovered from the river #as the man #ho had attempted to escape from the state prison. A! did not doubt #hether the body recovered from the river #as "! have no doubt #hether the body recovered from the river #as $! had not doubted that the body recovered from the river #as &! have no doubt #hether the body recovered from the river #as that of '! had no doubt that the body recovered from the river #as that of 774. Gne legacy of 8adison Avenue3s recent campaign to appeal to people fifty years old and over is the reali9ation that as a person ages% their concerns change as #ell. A! the reali9ation that as a person ages% their "! the reali9ation that as people age% their $! to reali9e that #hen a person ages% his or her &! to reali9e that #hen people age% their '! reali9ing that as people age% their 777. Gne noted economist has made a comparison of the ?ederal Deserve and an automobile as racing through a tunnel% bouncing first off one #all% then the otherE the car may get #here it is going% but people may be hurt in the process. A! made a comparison of the ?ederal Deserve and an automobile as racing through a tunnel% bouncing "! made a comparison bet#een the ?ederal Deserve and an automobile racing through a tunnel% bouncing $! compared the ?ederal Deserve #ith an automobile as racing through a tunnel and #hich bounced &! compared the ?ederal Deserve to an automobile racing through a tunnel% bouncing

SC Strategy

$7J

'! compared the ?ederal Deserve #ith an automobile that races through a tunnel and it bounces 77:. Gne of Arthur Cessop3s first acts as president of the ?>A #as to deny a re5uest from the private sector that federal home loan programs should be e<panded to cover houses not conforming to civil construction codes. A! should be e<panded to cover houses not conforming to "! be e<panded to cover houses not conforming to $! should be e<pand to cover houses not conforming #ith &! #ould have been e<panded to cover houses not conforming to '! had to be e<panded to cover houses not conforming #ith 77*. Gne of every t#o ne# businesses fail #ithin t#o years. A! fail "! fails $! should fail &! may have failed '! has failed 770. Gne of four babies are no# born to mothers aged thirty years or more% compared #ith 6ust one of si< born in 1)*7. A! of four babies are no# born to mothers aged thirty years or more% compared #ith 6ust one of si< born "! of four babies is no# born to a mother #hose age is thirty or older% compared to 6ust one of si< babies #ho #ere born $! baby in four are no# born to mothers aged thirty or older% compared to 6ust one in si< &! baby in four is no# born to a mother aged thirty or older% compared #ith 6ust one in si<

'! baby in four is no# born to mothers aged thirty years or more% compared to 6ust one in si< 77). Gne of Donald Deagan3s first acts as 1resident #as to rescind 1resident $arter3s directive that any chemical banned on medical grounds in the +nited ,tates be prohibited from sale to other countries. A! that any chemical banned on medical grounds in the +nited ,tates be prohibited from sale to other countries "! that any chemical be prohibited from sale to other countries that #as banned on medical grounds in the +nited ,tates $! prohibiting the sale to other countries of any chemical banned on medical grounds in the +nited ,tates

$8E

&! prohibiting that any chemical banned on medical grounds in the +nited ,tates is sold to other countries '! that any chemical banned in the +nited ,tates on medical grounds is prohibited from being sold to other countries 7:2. Gne of the most po#erful driving forces behind recycling is the threat of legislation that #ould re5uire companies that they take more responsibility for the disposal of its products. A! that they take more responsibility for the disposal of its products "! that they should take more responsibility for disposing of products $! having to take more responsibility for disposing of their products &! to take more responsibility for the disposal of their products '! taking more responsibility for their product3s disposal 7:1. Gne pervasive theory e<plains the introduction of breakfast cereals in the early 1)22s as a result of the gro#ing number of automobiles% #hich led to a decline in horse o#nership and a subse5uent grain glutE by persuading people to eat #hat had previously been horse feed% market e5uilibrium #as restored. A! by persuading people to eat #hat had previously been horse feed% market e5uilibrium #as restored "! persuading people to eat #hat had previously been horse feed restored market e5uilibrium $! by persuading people to eat #hat had previously been horse feed% it restored market e5uilibrium &! the persuasion of people to eat #hat had previously been horse feed restored market e5uilibrium '! market e5uilibrium #as restored #hen people #ere persuaded to eat former horse feed 7:(. Gne reason #hy more young people lose their virginity during the summer than at other times of the year undoubtedly is because school vacations give adolescents more free time. A! Gne reason #hy more young people lose their virginity during the summer than at other times of the year undoubtedly is because "! If young people lose their virginity more during the summer than other seasons% it is undoubtedly because $! Gne undoubtable reason that young people lose their virginity more during the summer than other times is &! Gne reason more young people lose their virginity during the summer than at other times of the year is undoubtedly that '! @oung people lose their virginity more often during the summer undoubtedly because% for one reason% 7:/. Gne vie# of the economy contends that a large drop in oil prices should eventually lead to

SC Strategy

$8$

lo#ering interest rates% as #ell as lo#ering fears about inflation% a rally in stocks and bonds% and a #eakening of the dollar. A! lo#ering interest rates% as #ell as lo#ering fears about inflation% "! a lo#ering of interest rates and of fears about inflation% $! a lo#ering of interest rates% along #ith fears about inflation% &! interest rates being lo#ered% along #ith fears about inflation% '! interest rates and fears about inflation being lo#ered% #ith 7:4. Gpened in 1:0/% the Ashmolean #as the first public museum in -reat "ritain% #hile earlier the "odleian has been the first truly public library. A! #hile earlier the "odleian has been "! #here the "odleian earlier is $! 6ust as earlier the "odleian had been &! as the earlier "odleian has been '! the "odleian earlier being 7:7. Gpening #ith tributes to 6a99-age divas like "essie ,mith and closing #ith Foko =aylor3s electrified gravel-and-thunder songs% the program #ill trace the blues3 vigorous matriarchal line over more than 72 years. A! the program #ill trace "! the program shall trace $! there #ill be a program tracing &! it is a program that traces '! it #ill be a program tracing 7::. Grgani9ed in 1):: by the ?ish and Wildlife ,ervice% the "reeding "ird ,urvey uses annual roadside counts along established routes for monitoring of population changes of as many as% or of more than (72 bird species% including 102 songbirds. A! for monitoring of population changes of as many as% or of "! to monitor population changes of as many% or $! to monitor changes in the populations of &! that monitors population changes of '! that monitors changes in populations of as many as% or 7:*. Griginally developed for detecting air pollutants% a techni5ue called proton-induced <-ray emission% #hich can 5uickly analy9e the chemical elements in almost any substance #ithout destroying it% is finding uses in medicine% archaeology% and criminology. A! Griginally developed for detecting air pollutants% a techni5ue called proton-induced <-ray emission% #hich can 5uickly analy9e the chemical elements in almost any substance #ithout

$8'

destroying it% "! Griginally developed for detecting air pollutants% having the ability to analy9e the chemical elements in almost any substance #ithout destroying it% a techni5ue called proton induced <ray emission $! A techni5ue originally developed for detecting air pollutants% called proton-induced <-ray emission% #hich can 5uickly analy9e the chemical elements in almost any substance #ithout destroying it% &! A techni5ue originally developed for detecting air pollutants% called proton-induced <-ray emission% #hich has the ability to analy9e the chemical elements in almost any substance 5uickly and #ithout destroying it% '! A techni5ue that #as originally developed for detecting air pollutants and has the ability to analy9e the chemical elements in almost any substance 5uickly and #ithout destroying the substance% called proton-induced <-ray emission% 7:0. Griginally published in 1)72% ,ome =ame -a9elle #as "arbara 1ym3s first novel% but it does not read like an apprentice #ork. A! does not read like an apprentice #ork "! seems not to read as an apprentice #ork $! does not seem to read as an apprentice #ork #ould &! does not read like an apprentice #ork does '! reads unlike an apprentice #ork 7:). Gur rate of teenage pregnancies is among the highest in the industriali9ed #orld% being e<ceeded only by $hile% >ungary% Domania% $uba% and "ulgaria. A! is among the highest in the industriali9ed #orld% being e<ceeded only by "! is among the highest in the industriali9ed #orld% e<ceeded only by that of $! are among the highest in the industriali9ed #orld% only e<ceeded by &! is among the highest in the industriali9ed #orld% and e<ceeds only '! are among the highest in the industriali9ed #orld% and they e<ceed those of only 7*2. Gut of America3s fascination #ith all things anti5ue have gro#n a market for bygone styles of furniture and fi<tures that are bringing back the chaise lounge% the overstuffed sofa% and the cla#footed bathtub. A! things anti5ue have gro#n a market for bygone styles of furniture and fi<tures that are bringing "! things anti5ue has gro#n a market for bygone styles of furniture and fi<tures that is bringing $! things that are anti5ues has gro#n a market for bygone styles of furniture and fi<tures that bring &! anti5ue things have gro#n a market for bygone styles of furniture and fi<tures that are bringing

SC Strategy

$8*

'! anti5ue things has gro#n a market for bygone styles of furniture and fi<tures that bring 7*1. Gver five thousand years ago% archaeologists have discovered that 8esopotamian cooks #ere preparing chicken #ith coriander and tamarind% 6ust as 'gyptian cooks are still doing today. A! Gver five thousand years ago% archaeologists have discovered that "! Gver five thousand years ago% archaeologists had discovered that $! Archaeologists have discovered that% over five thousand years ago% &! Gver five thousand years ago% as archaeologists have discovered it% '! Archaeologists have discovered% over five thousand years ago% that 7*(. 1ablo 1icasso% the late ,panish painter% credited African art #ith having had a strong influence on his #ork. A! #ith having had "! for its having $! to have had &! for having '! in that it had 7*/. 1aleontologists believe that fragments of a primate 6a#bone unearthed in "urma and estimated at 42 to 44 million years old provide evidence of a crucial step along the evolutionary path that led to human beings. A! at 42 to 44 million years old provide evidence of "! as being 42 to 44 million years old provides evidence of $! that it is 42 to 44 million years old provides evidence of #hat #as &! to be 42 to 44 million years old provide evidence of '! as 42 to 44 million years old provides evidence of #hat #as 7*4. 1arliament did not accord full refugee benefits to t#elve of the recent immigrants because it believed that to do it re#ards them for entering the country illegally. A! to do it re#ards "! doing it re#ards $! to do this #ould re#ard &! doing so #ould re#ard '! to do it #ould re#ard

7*7.

1ensions are no# vie#ed as a deferred payment of salary% money a #orker is compelled to put a#ay to take care of one3s later years. A! a #orker is compelled to put a#ay to take care of one3s

$87

"! that a #orker is compelled to put a#ay to take care of oneself in $! a #orker is compelled to put a#ay to take care of oneself in &! #orkers are compelled to put a#ay to take care of them in '! #orkers are compelled to put a#ay to take care of themselves in 7*:. 1eople have discovered the principles of solar energy #henever fuel becomes scarce and e<pensive but #ill forget them every time a ne# source of cheap energy is developed. A! have discovered the principles of solar energy #henever fuel becomes scarce and e<pensive but #ill forget "! have discovered the principles of solar energy #henever fuel has become scarce and e<pensive but they forget $! discovered the principles of solar energy every time fuel becomes scarce and e<pensive% forgetting &! discover the principles of solar energy every time fuel became scarce and e<pensive% but they forget '! discover the principles of solar energy #henever fuel becomes scarce and e<pensive but forget 7**. 1eople #ho inherit the sickle cell anemia gene from only one parent seem to be resistant to malaria% an evolutionary advantage that may e<plain #hy a genetic condition so debilitating to many individuals has survived in the human population. A! seem to be resistant to malaria% "! seemingly are resistant to malaria% $! seem to be resistant to malaria and have &! seemingly are resistant to malaria and to have '! are% it seems% resistant to malaria% and they have 7*0. 1ersonal trainers and fitness coaches are to the narcissistic 1)023s 6ust like a private fencing master and dancing teacher #as to an earlier time. A! 6ust like a private fencing master and dancing teacher #as "! as have been a private fencing master and dancing teacher $! #hat private fencing masters and dancing teachers #ere &! #hat private fencing masters and dancing teachers are '! 6ust the same as private fencing masters and dancing teachers had been 7*). 1ersons% suffering from a deficiency of the blood en9yme -:1&% eat fava beans and discover that they trigger hemolytic anemia. A! 1ersons% suffering from a deficiency of the blood en9yme -:1&% eat fava beans and discover that they trigger hemolytic anemia. "! When persons suffering a deficiency of en9yme -:1&% a blood en9yme% eat fava beans% they discover that it triggers hemolytic anemia.

SC Strategy

$88

$! If people suffer deficiencies from the blood en9yme -:1&% they discover that fava beans trigger hemolytic anemia #hen eaten. &! >emolytic anemia is triggered by people #ith a deficiency of the blood en9yme -:1&% #ho discover it eating fava beans. '! 1ersons #ho suffer from a deficiency of the blood en9yme -:1& discover that eating fava beans triggers hemolytic anemia. 702. 1hysics professors% the Arnmore .aboratories and Arnmore Desearch ?acilities #ere founded by .eo and ?ontove Arnmore in 1)0) after ten years of fundraising. A! the Arnmore .aboratories and Arnmore Desearch ?acilities #ere founded by .eo and ?ontove Arnmore in 1)0) after ten years of fundraising "! .eo and ?ontove Arnmore founded the Arnmore .aboratories and Arnmore Desearch ?acilities in 1)0)% after ten years of fundraising $! after ten years of fundraising% the Arnmore .aboratories and Arnmore Desearch ?acilities #ere founded by .eo and ?ontove Arnmore in 1)0) &! the Arnmore .aboratories and Arnmore Desearch ?acilities #ere founded in 1)0) by .eo and ?ontove Arnmore after ten years of fundraising '! .eo and ?ontove Arnmore founded after ten years of fundraising the .aboratories and Arnmore Desearch ?acilities in 1)0) 701. 1iaget3s research revealed that children can learn to count long before the recognition that a pint of #ater poured from a small glass into a large one remains the same amount of #ater. A! the recognition that "! they can recogni9e that $! they #ould recogni9e &! they could have the recognition of '! having the recognition of 70(. 1lausible though it sounds% the #eakness of the hypothesis is that it does not incorporate all relevant evidence. A! 1lausible though it sounds% the #eakness of the hypothesis "! 'ven though it sounds plausible% the #eakness of the hypothesis $! =hough plausible% the hypothesis3 #eakness &! =hough the hypothesis sounds plausible% its #eakness '! =he #eakness of the hypothesis #hich sounds plausible 70/. 1oor management% outdated technology% competition from overseas% and steel3s replacement to materials like aluminum and fiber-reinforced plastics have all been cited as causes for the decline of the +nited ,tates steel industry. A! steel3s replacement to materials like

$89

"! the replacement of steel by such materials as $! the replacing of steel #ith materials of &! the replacing of steel by means of materials like '! to replace steel by materials such as 704. 1resenters at the seminar% one #ho is blind% #ill demonstrate adaptive e5uipment that allo#s visually impaired people to use computers. A! one #ho "! one of them #ho $! and one of them #ho &! one of #hom '! one of #hich 707. 1romotions% retirements% deaths% and other actions approved by the board of directors at its 8ay meeting #ill be reported in the Culy 17 issue of the company paper. A! 1romotions% retirements% deaths% and other actions approved by the board of directors at its 8ay meeting #ill be reported in the Culy 17 issue of the company paper. "! 1romotions% retirements% and other actions #hich have been approved at the 8ay meeting of the board of directors along #ith deaths% #ith be reported in the Culy 17 issue of the company paper. $! =o be reported in the Culy 17 issue of the company paper are the promotions% retirements% deaths% and other actions #hich #ere approved at the board of directors3 8ay meeting. &! 8eeting in 8ay% the promotions% retirements% and other actions approved by the board of directors% including obituaries% #ill be reported in the Culy 17 issue of the company paper. '! =he Culy 17 issue of the company paper #ill report on promotions% retirements% and other actions approved by the board of directors at its 8ay meetingE the paper #ill also include obituaries. 70:. 1rompted by ne# evidence that the health risk posed by radon gas is far more serious than #as previously thought% property o#ners are being advised by authorities to test all d#ellings belo# the third floor for radon gas and to make repairs as needed. A! property o#ners are being advised by authorities to "! property o#ners are advised by authorities that they should $! authorities are advising property o#ners to &! authorities are advising property o#ners they '! authorities3 advice to property o#ners is they should 70*. 1roponents of artificial intelligence say they #ill be able to make computers that can understand 'nglish and other human languages% recogni9e ob6ects% and reason as an e<pert doesH computers that #ill be used to diagnose e5uipment breakdo#ns% deciding #hether to authori9e a

SC Strategy

$8;

loan% or other purposes such as these. A! as an e<pert doesHcomputers that #ill be used to diagnose e5uipment breakdo#ns% deciding #hether to authori9e a loan% or other purposes such as these "! as an e<pert does% #hich may be used for purposes such as diagnosing e5uipment breakdo#ns or deciding #hether to authori9e a loan $! like an e<pertHcomputers that #ill be used for such purposes as diagnosing e5uipment breakdo#ns or deciding #hether to authori9e a loan &! like an e<pert% the use of #hich #ould be for purposes like the diagnosis of e5uipment breakdo#ns or the decision #hether or not a loan should be authori9ed '! like an e<pert% to be used to diagnose e5uipment breakdo#ns% deciding #hether to authori9e a loan or not% or the like 700. 1sychologists no# contend that the #ay adults think and feel are determined as much by their peers in early childhood than by their parents. A! are determined as much by their peers in early childhood than by their "! are determined as much by peers in early childhood as do their $! is determined as much by their early childhood peers as by their &! have been determined by childhood peers as much as their '! #as determined as much by one3s peers in childhood as by one3s 70). 1ublished during the late eighteenth century% &iderot3s factual 'ncyclopedia and his friend Boltaire3s fictional $andide #ere the cause of such a sensational scandal% and both men prudently chose to embark on e<tended vacations in nearby Austria. A! &iderot3s factual 'ncyclopedia and his friend Boltaire3s fictional $andide #ere the cause of such a sensational scandal% and "! &iderot and his friend Boltaire3s caused such a sensational scandal #ith their factual 'ncyclopedia and fictional $andide% respectively% that $! &iderot3s factual 'ncyclopedia and his friend Boltaire3s fictional $andide #ere the cause of a scandal so sensational that &! the scandal caused by &iderot3s factual 'ncyclopedia and his friend Boltaire3s fictional $andide #as so sensational '! a factual 'ncyclopedia by &iderot and the fictional $andide% by his friend Boltaire% caused a sensational scandal% #hich 7)2. 1ublished in >arlem% the o#ner and editor of the 8essenger #ere t#o young 6ournalists% $handler G#en and A. 1hilip Dandolph% #ho #ould later make his reputation as a labor leader. A! 1ublished in >arlem% the o#ner and editor of the 8essenger #ere t#o young 6ournalists% $handler G#en and A. 1hilip Dandolph% #ho #ould later make his reputation as a labor leader. "! 1ublished in >arlem% t#o young 6ournalists% $handler G#en and A. 1hilip Dandolph% #ho

$8I

#ould later make his reputation as a labor leader% #ere the o#ner and editor of the 8essenger. $! 1ublished in >arlem% the 8essenger #as o#ned and edited by t#o young 6ournalists% A. 1hilip Dandolph% #ho #ould later make his reputation as a labor leader% and $handler G#en. &! =he 8essenger #as o#ned and edited by t#o young 6ournalists% $handler G#en and A. 1hilip Dandolph% #ho #ould later make his reputation as a labor leader% and published in >arlem. '! =he o#ner and editor being t#o young 6ournalists% $handler G#en and A. 1hilip Dandolph% #ho #ould later make his reputation as a labor leader% the 8essenger #as published in >arlem. 7)1. ;uasars are so distant that their light has taken billions of years to reach the 'arthE conse5uently% #e see them as they #ere during the formation of the universe. A! #e see them as they #ere during "! #e see them as they had been during $! #e see them as if during &! they appear to us as they did in '! they appear to us as though in 7)(. ;uasars% at billions of light-years from 'arth the most distant observable ob6ects in the universe% believed to be the cores of gala<ies in an early stage of development. A! believed to be "! are believed to be $! some believe them to be &! some believe they are '! it is believed that they are 7)/. Dalph 'ilison and Amiri "araka both argued that music #as perhaps the ultimate e<pression of Afro-American culture% that it #as the one vector of African culture that there #as no possibility to eradicate. A! that it #as the one vector of African culture that there #as no possibility to eradicate "! the one vector of African culture that could not be eradicated $! for it #as the one vector of African culture% and that it #as impossible to eradicate &! a vector of African culture that there #as no possibility to eradicate '! as being the one vector that could not be eradicated from African culture 7)4. Danked as one of the most important of 'urope3s young play#rights% ?ran9 Oaver Froet9 has #ritten forty playsE his #orksHtranslated into over thirty languagesHare produced more often than any contemporary -erman dramatist. A! than any "! than any other

SC Strategy

$8J

$! than are any &! than those of any other '! as are those of any 7)7. Dather than continue to produce most of the items necessary for subsistence% a gro#ing number of farm families during the first decades of the nineteenth century began to speciali9e in the production of grain or cotton and to use the cash proceeds from selling their crops for buying necessities. A! selling their crops for buying "! the sales of their crops for buying $! their selling of crops so as to buy &! their selling crops for buying of '! the sale of their crops to buy 7):. Deal similar to a >olly#ood movie set #ith nothing behind the building fronts% the country3s apparent ne#-found affluence masks a very different realityA most citi9ens are not living at all #ell. A! Deal similar to "! 8uch as if it #as $! As though &! .ike '! Cust as 7)*. Deared apart from each other% a recent +nited ,tates study sho#ed striking similarities in identical t#ins% including many idiosyncrasies of behavior. A! Deared apart from each other% a recent +nited ,tates study sho#ed striking similarities in identical t#ins% including many idiosyncrasies of behavior. "! Deared apart from each other% striking similarities bet#een identical t#ins that include many idiosyncrasies of behavior #ere sho#n in a recent +nited ,tates study. $! A recent +nited ,tates study sho#ed striking similarities in identical t#ins reared apart from each other that include many idiosyncrasies of behavior. &! According to a recent +nited ,tates study% identical t#ins reared apart from each other sho#ed striking similarities% including many idiosyncrasies of behavior. '! According to a recent +nited ,tates study% identical t#ins sho#ed striking similarities reared apart from each other% including many idiosyncrasies of behavior. 7)0. Decent essays by -arrison Feillor% the humorist and host of the public radio sho# A 1rairie >ome $ompanion% describes #hat might happen if >uns% -oths% and Bisigoths #ere to invade $hicago. A! describes #hat might happen if >uns% -oths% and Bisigoths #ere to invade

$9E

"! describes #hat #ould happen if >uns% -oths% and Bisigoths invaded $! describes #hat #ould happen if >uns% -oths% and Bisigoths #ould have invaded &! describe #hat might happen if >uns% -oths% and Bisigoths #ere to invade '! describe #hat happen if >uns% -oths% and Bisigoths invade 7)). Decent e<cavations suggest that the ancient peoples of the Italian peninsula merged the cult of &amiaHa goddess of fertility and the harvestH#ith Benus. A! #ith Benus "! and Benus $! #ith that of Benus &! and Benus3 '! and Benus3 cult :22. Decently discovered fossil remains strongly suggest that the Australian egg-laying mammals of today are a branch of the main stem of mammalian evolution rather than developing independently from a common ancestor of mammals more than ((2 million years ago. A! rather than developing independently from "! rather than a type that developed independently from $! rather than a type #hose development #as independent of &! instead of developing independently from '! instead of a development that #as independent of :21. Decently implemented shift-#ork e5uations based on studies of the human sleep cycle have reduced sickness% sleeping on the 6ob% fatigue among shift #orkers% and have raised production efficiency in various industries. A! fatigue among shift #orkers% and have raised "! fatigue among shift #orkers% and raised $! and fatigue among shift #orkers #hile raising &! lo#ered fatigue among shift #orkers% and raised '! and fatigue among shift #orkers #as lo#ered #hile raising :2(. Decently there has been increased debate over if a budget surplus should go to#ards lo#er ta<es or increased spending on social programs. A! over if a budget surplus should go to#ards lo#er ta<es or increased spending "! over #hether a budget surplus should go to#ards lo#ering ta<es or increasing spending $! about a budget surplus going to#ards lo#er ta<es or increasing spending &! about if lo#er ta<es should come from a budget surplus or spending increases '! concerning a budget surplus and its going to#ards lo#er ta<es or increased spending :2/. Dembrandt so treasured his collection of Islamic portraits that #hen forced to sell them in

SC Strategy

$9$

order to raise money% he first made copies of more than t#enty. A! he first made copies of more than t#enty "! first he made copies of more than t#enty $! more than t#enty #ere copied &! copies of more than t#enty #ere made '! he copies more than t#enty of them first :24. Depelled by bodily punishments such as maiming and branding% the idea of penitentiaries #ere reforms of the penal system by ;uakers. A! the idea of penitentiaries #ere reforms of the penal system by ;uakers "! penitentiaries #ere ideas for reform of the penal system suggested by ;uakers $! ;uakers suggested the penitentiary as a reform of the penal system &! ;uakers suggested that the penal system be reformed as penitentiaries '! the penitentiary #as suggested to be a reform of the penal system by ;uakers :27. Deporting that one of its many problems had been the recent e<tended sales slump in #omen3s apparel% the seven-store retailer said it #ould start a three-month li5uidation sale in all of its stores. A! its many problems had been the recent "! its many problems has been the recently $! its many problems is the recently &! their many problems is the recent '! their many problems had been the recent :2:. Desearch during the past several decades on the nature of language and the processes that produce and make it understandable has revealed great comple<ity instead of underlying simplicity. A! that produce and make it understandable has revealed great comple<ity instead of underlying simplicity "! of producing and understanding it have revealed not underlying simplicity but great comple<ity $! by #hich it is produced and understood has revealed not underlying simplicity but great comple<ity &! by #hich it is produced and understood have revealed great comple<ity rather than underlying simplicity '! by #hich one produces and understands it have revealed great comple<ity instead of underlying simplicity :2*. Desearchers at $ornell +niversity have demonstrated that homing pigeons can sense changes in the earth3s magnetic field% see light #aves that people cannot see% detect lo#-fre5uency sounds

$9'

from miles a#ay% sense changes in air pressure% and can identify familiar odors. A! sense changes in air pressure% and can identify familiar odors "! can sense changes in air pressure% and can identify familiar odors $! sense changes in air pressure% and identify familiar odors &! air pressure changes can be sensed% and familiar odors identified '! air pressure changes are sensed% and familiar odors identified :20. Desearchers have 5uestioned the use of costly and e<perimental diagnostic tests to identify food allergies% such as milk% that supposedly disrupt normal behavior. A! to identify food allergies% such as "! to identify food allergies% like $! to identify food allergies% such as to &! for identifying food allergies% like that of '! for identifying food allergies% such as for :2). Devered by an ill-informed citi9enry% the &uke of @ork #as feted opulently for several months before there #as denunciation and e<ile. A! there #as denunciation and e<ile "! he #as to be denounced #ith e<ile $! being denounced and e<iled &! denunciation and his e<ile '! being e<iled% having been denounced :12. Dising inventories% #hen unaccompanied correspondingly by increases in sales% can lead to production cutbacks that #ould hamper economic gro#th. A! #hen unaccompanied correspondingly by increases in sales% can lead "! #hen not accompanied by corresponding increases in sales% possibly leads $! #hen they #ere unaccompanied by corresponding sales increases% can lead &! if not accompanied by correspondingly increased sales% possibly leads '! if not accompanied by corresponding increases in sales% can lead :11. Doy Wilkins #as among the last of a generation of civil rights activists #ho led the nation through decades of change so profound many young Americans are not able to imagine% even less to remember% #hat segregation #as like. A! so profound many young Americans are not able to imagine% even less to remember "! so profound that many young Americans cannot imagine% much less remember $! so profound many young Americans cannot imagine nor even less remember &! of such profundity many young Americans cannot imagine% even less can they remember '! of such profundity that many young Americans are not able to imagine% much less to

SC Strategy

$9*

remember :1(. Dules banning cancer-causing substances from food apply to ne# food additives and not to natural constituents of food because their use as additives is entirely avoidable. A! their use as additives is "! as additives% their use is $! the use of such additives is &! the use of such additives are '! the use of them as additives is :1/. ,ales of +nited ,tates manufactured goods to nonindustriali9ed countries rose to I1:* billion in 1))(% #hich is 14 percent more than the previous year and largely offsets #eak demand from 'urope and Capan. A! #hich is 14 percent more than the previous year "! #hich is 14 percent higher than it #as the previous year $! 14 percent higher than the previous year3s figure &! an amount that is 14 percent more than the previous year #as '! an amount that is 14 percent higher than the previous year3s figure :14. ,alt deposits and moisture threaten to destroy the 8ohen6o-&aro e<cavation in 1akistan% the site of an ancient civili9ation that flourished at the same time as the civili9ations in the Nile delta and the river valleys of the =igris and 'uphrates. A! that flourished at the same time as the civili9ations "! that had flourished at the same time as had the civili9ations $! that flourished at the same time had &! flourishing at the same time as those did '! flourishing at the same time as those #ere :17. ,amuel ,e#all vie#ed marriage% as other seventeenth-century colonists% like a property arrangement rather than an emotional bond based on romantic love. A! ,amuel ,e#all vie#ed marriage% as other seventeenth-century colonists% like a property arrangement rather than "! As did other seventeenth-century colonists% ,amuel ,e#all vie#ed marriage to be a property arrangement rather than vie#ing it as $! ,amuel ,e#all vie#ed marriage to be a property arrangement% like other seventeenth-century colonists% rather than vie#ing it as &! 8arriage to ,amuel ,e#all% like other seventeenth-century colonists% #as vie#ed as a property arrangement rather than '! ,amuel ,e#all% like other seventeenth-century colonists% vie#ed marriage as a property arrangement rather than

$97

:1:.

,artre believed each individual is responsible to choose one course of action over another one% that it is the choice that gives value to the act% and that nothing that is not acted upon has value. A! each individual is responsible to choose one course of action over another one "! that each individual is responsible for choosing one course of action over another $! that each individual is responsible% choosing one course of action over another &! that each individual is responsible to choose one course of action over the other '! each individual is responsible for choosing one course of action over other ones

:1*.

,chool desegregation has #orked #ell in "uffalo% Ne# @ork% in part because parents and teachers #ere given ma6or roles in designing the city3s magnet schools% because e<tra federal funds #ere allocated to make each school uni5ue% and because the federal 6udge enforced desegregation orders. A! because e<tra federal funds #ere allocated to make each school uni5ue "! because of the allocation of e<tra federal funds that make each school uni5ue $! because each school is made uni5ue by allocating it e<tra federal funds &! e<tra federal funds #ere allocated in order to make each school uni5ue '! e<tra federal funds #ere allocated for making each school uni5ue

:10.

,chool integration plans that involve busing bet#een suburban and central-city areas have contributed% according to a recent study% to significant increases in housing integration% #hich% in turn% reduces any future need for busing. A! significant increases in housing integration% #hich% in turn% reduces "! significant integration increases in housing% #hich% in turn% reduces $! increase housing integration significantly% #hich% in turn% reduces &! increase housing integration significantly% in turn reducing '! significantly increase housing integration% #hich% in turn% reduce

:1).

,cientific interest in providing suitable habitats for bottom-d#elling animals such as river clams arises not because they are important sources of human food but from their role as an integral link in the A5uatic food chain. A! not because they are important sources of human food "! although they are not an important sources of human food $! not in that they are important as human sources of food &! not from their importance as a source of human food '! not from being important sources of food for human beings

:(2.

,cientists believe that unlike the males of most species of moth% the male #histling moths of Nambung% Australia% call female moths to them by the use of acoustical signals% but not olfactory ones% and they attract their mates during the day% rather than at night.

SC Strategy

$98

A! by the use of acoustical signals% but not olfactory ones% and they attract "! by the use of acoustical signals instead of using olfactory ones% and attracting $! by using acoustical signals% not using olfactory ones% and by attracting &! using acoustical signals% rather than olfactory ones% and attract '! using acoustical signals% but not olfactory ones% and attracting :(1. ,cientists calculated that the asteroid% traveling at 4:%222 miles an hour% is on an elliptical path that orbits the ,un once a year and regularly brings it back to#ard 'arth. A! hour% is on an elliptical path that orbits the ,un once a year and regularly brings it "! hour% is orbiting the ,un once a year on an elliptical path that regularly brings it $! hour% once a year orbits the ,un% regularly bringing it on an elliptical path &! hour and orbiting the ,un once a year on an elliptical path% regularly bringing it '! hour% orbits the ,un on an elliptical path once a year and that regularly brings it :((. ,cientists have observed large concentrations of heavy-metal deposits in the upper t#enty centimeters of "altic ,ea sediments% #hich are consistent #ith the gro#th of industrial activity there. A! "altic ,ea sediments% #hich are consistent #ith the gro#th of industrial activity there "! "altic ,ea sediments% #here the gro#th of industrial activity is consistent #ith these findings $! "altic ,ea sediments% findings consistent #ith its gro#th of industrial activity &! sediments from the "altic ,ea% findings consistent #ith the gro#th of industrial activity in the area '! sediments from the "altic ,ea% consistent #ith the gro#th of industrial activity there :(/. ,cientists have recently discovered #hat could be the largest and oldest living organism on 'arth% a giant fungus that is an inter#oven filigree of mushrooms and rootlike tentacles spa#ned by a single fertili9ed spore some 12%222 years ago and e<tending for more than /2 acres in the soil of a 8ichigan forest. A! e<tending "! e<tends $! e<tended &! it e<tended '! is e<tending :(4. ,cientists have suggested that once every 12 million years or so a truly colossal ob6ect from space cuts through the atmosphere and slams into 'arth% sending up a global pall of dust that blots out the ,un% alters the climate% and changes the course of evolution by killing off many plant and animal species. A! sending up a global pall of dust that blots

$99

"! thus sending up a global pall of dust to blot $! thereby sending up a global pall of dust to blot &! and that sends up a global pall of dust% blotting '! #hich sends up a global pall of dust% blots :(7. ,cientists #ho studied the famous gold field kno#n as ,erra 1elada concluded that the rich lode #as not produced by the accepted methods of ore formation but that s#arms of microbes over millions of years concentrated the gold from 6ungle soils and rivers and rocks. A! not produced by the accepted methods of ore formation but that s#arms of microbes over millions of years "! not produced by the accepted methods of ore formation but instead s#arms of microbes over millions of years that $! not produced by the accepted methods of ore formation but s#arms of microbes over millions of years that &! produced not by the accepted methods of ore formation but by s#arms of microbes that over millions of years '! produced not by the accepted methods of ore formation but that s#arms of microbes over millions of years :(:. ,corched by fire% stained by #ater% and inscribed in cramped hand#riting% the seventeenthcentury &utch documents on the beginnings of Ne# @ork $ity #ere long ignored by historians% depending instead on 'nglish sources for information. A! the seventeenth-century &utch documents on the beginnings of Ne# @ork $ity #ere long ignored by historians% depending "! the seventeenth-century &utch documents on the beginnings of Ne# @ork $ity #ere long ignored by historians% #ho depended $! historians long ignored the seventeenth-century &utch documents on the beginnings of Ne# @ork $ity% depending &! historians long ignored the seventeenth-century &utch documents on the beginnings of Ne# @ork $ity and depended '! historians long ignored the seventeenth-century &utch documents on the beginnings of Ne# @ork $ity% they depended :(*. ,culptor Ale<ander $alder% #ho often made use of old pieces of 6unk in his art and also believed in recycling at homeE he once turned a broken goblet into a dinner bell and a cake mold into a lamp. A! ,culptor Ale<ander $alder% #ho often made use of old pieces of 6unk in his art and also believed in recycling at homeE he "! Ale<ander $alder% for #hom old pieces of 6unk #as often made into sculpture% believed in recycling at home and

SC Strategy

$9;

$! A believer in recycling at home% sculptor Ale<ander $alder often made use for old pieces of 6unk in his artE he &! Ale<ander $alder% for #hom sculpture #as often made from old pieces of 6unk% also believed in recycling at home% for e<ample% he '! ,culptor Ale<ander $alder% #ho often made use of old pieces of 6unk in his art% also believed in recycling at homeE he :(0. ,ection 1/ d! of the ,ecurities '<change Act of 1)/4 re5uires anyone #ho buys more than 7 percent of a company3s stock make a public disclosure of the purchase. A! make "! #ill also make $! to make &! must make '! must then make :(). ,eeming to be one of the fe# corporations diversified enough to survive the recession% many shareholders ignored the drop in third-5uarter profits and invested even more heavily in 'mco. A! ,eeming to be "! As if $! In that they seemed &! Although it seemed '! "ecause it seemed to be :/2. ,eeming to be the only organi9ation fighting for the rights of poor people in the ,outh% >osea >udson% a laborer in Alabama% 6oined the $ommunist party in 1)/1. A! ,eeming to be "! As $! In that they seemed &! ,ince it seemed '! "ecause it seemed to be :/1. ,eismologists studying the earth5uake that struck northern $alifornia in Gctober 1)0) are still investigating some of its mysteriesA the une<pected po#er of the seismic #aves% the up#ard thrust that thre# one man straight into the air% and the strange electromagnetic signals detected hours before the temblor. A! the up#ard thrust that thre# one man straight into the air% and the strange electromagnetic signals detected hours before the temblor "! the up#ard thrust that thre# one man straight into the air% and strange electromagnetic signals #ere detected hours before the temblor $! the up#ard thrust thre# one man straight into the air% and hours before the temblor strange

$9I

electromagnetic signals #ere detected &! one man #as thro#n straight into the air by the up#ard thrust% and hours before the temblor strange electromagnetic signals #ere detected '! one man #ho #as thro#n straight into the air by the up#ard thrust% and strange electromagnetic signals that #ere detected hours before the temblor :/(. ,elling several hundred thousand copies in si< months% the publication of 8aple .eaf Dag in 10)) #as an instant hit% helping to establish ,cott Coplin as the preeminent ragtime composer. A! ,elling several hundred thousand copies in si< months% the publication of 8aple .eaf Dag in 10)) #as an instant hit% helping to establish ,cott Coplin as the preeminent ragtime composer. "! =he publication in 10)) of 8aple .eaf Dag #as an instant hitA in si< months they sold several hundred thousand copies and it helped establish ,cott Coplin as the preeminent ragtime composer. $! >elping to establish ,cott Coplin as the preeminent ragtime composer #as the publication of 8aple .eaf Dag in 10))% #hich #as an instant hitA it sold several hundred thousand copies in si< months. &! 8aple .eaf Dag #as an instant hitA it helped establish ,cott Coplin as the preeminent ragtime composer% published in 10)) and selling several hundred thousand copies in si< months. '! 1ublished in 10))% 8aple .eaf Dag #as an instant hit% selling several hundred thousand copies in si< monthsA it helped establish ,cott Coplin as the preeminent ragtime composer. ://. ,enator .asker has proposed legislation re5uiring that employers should retain all older #orkers indefinitely or sho# 6ust cause for dismissal. A! that employers should retain all older #orkers "! that all older #orkers be retained by employers $! the retaining by employers of all older #orkers &! employers3 retention of all older #orkers '! employers to retain all older #orkers :/4. ,enior e<ecutives had a larger percentage increase in pay in 1))2 than the #ages of other salaried #orkers. A! ,enior e<ecutives had a larger percentage increase in pay in 1))2 than "! =he percentage of senior e<ecutives3 pay increase in 1))2 #as larger than for $! =he 1))2 increase in pay for senior e<ecutives #as larger in terms of percentage than &! In 1))2 senior e<ecutives had a larger pay increase in terms of percentage than did '! =he pay of senior e<ecutives increased in 1))2 by a larger percentage than did :/7. ,everal recent studies suggest that a child born into a family #hose members have allergies

SC Strategy

$9J

#ill probably themselves develop allergies follo#ing the onset of a minor viral infection. A! a child born into a family #hose members have allergies #ill probably themselves develop allergies "! children born into families #hose members have allergies #ill probably themselves develop allergies $! a child born into a family the members of #hich have allergies #ill probably develop an allergy &! in those families #here members have allergies% children #ill probably develop allergies themselves '! children born into a family in #hich there are allergies #ill themselves probably develop an allergy :/:. ,everal senior officials spoke to the press on condition that they not be named in the story. A! that they not be named "! that their names #ill not be used $! that their names are not used &! of not being named '! they #ill not be named :/*. ,everal studies have found that the coronary patients #ho e<ercise most actively have half or less than half the chance of dying of a heart attack as those #ho are sedentary. A! have half or less than half the chance of dying of a heart attack as those #ho are sedentary "! have half the chance% or less% of dying of a heart attack than those #ho are sedentary do $! have half the chance that they #ill die of a heart attack% or less% than those #ho are sedentary do &! are at least fifty percent less likely to die of a heart attack as those #ho are sedentary '! are at least fifty percent less likely than those #ho are sedentary to die of a heart attack :/0. ,everal years ago the diet industry introduced a variety of appetite suppressants% but some of these drugs caused stomach disorders severe enough to have them banned by the ?ood and &rug Administration. A! stomach disorders severe enough to have them "! stomach disorders that #ere severe enough so they #ere $! stomach disorders of such severity so as to be &! such severe stomach disorders that they #ere '! such severe stomach disorders as to be :/). ,evere and increasing numerous critics are pointing to deficiencies in the "ritish legal system% deficiencies that seem to deny a proper defense to many clients #ho are charged #ith crimes.

$;E

A! ,evere and increasing numerous critics are pointing to deficiencies in the "ritish legal system% deficiencies that seem "! ,evere and increasing numerous critics point to deficiencies in the "ritish legal system% deficiencies seeming $! ,evere and increasingly numerous critics are pointing to deficiencies in the "ritish legal system that seem &! ,everely and increasingly numerous critics point to deficiencies in the "ritish legal system seeming '! ,everely and increasingly numerous critics are pointing to deficiencies in the "ritish legal system that seem :42. ,he #as a child prodigy% and $lara ,chumann developed into one of the greatest pianists of her time. A! ,he #as a child prodigy% and $lara ,chumann "! A child prodigy% $lara ,chumann $! $hild prodigy that she #as% $lara ,chumann &! $lara ,chumann has been a child prodigy% and she '! "eing a child prodigy% $lara ,chumann :41. ,imilar to rising interest rates% consumer and producer prices have been rising. A! ,imilar to rising interest rates% consumer and producer prices have been rising. "! $onsumer and producer prices have been rising% as have interest rates. $! As interest rates are rising% so have consumer and producer prices. &! $onsumer and producer prices have been rising% like interest rates do. '! $onsumer and producer prices% as interest rates% have been rising. :4(. ,ince 1):7 there are four times as many "lack college students enrolled% and the one million "lack people in college today represent 11 percent of all college students. A! ,ince 1):7 there are four times as many "lack college students enrolled "! =he enrollment of "lack college students #as only one-fourth in 1):7 $! =he enrollment of "lack college students has increased four times from 1):7 on &! ;uadrupling since 1):7% there are no# four times as many "lack college students enrolled '! =he enrollment of "lack college students has 5uadrupled since 1):7 :4/. ,ince 1)*2 the number of "lacks elected to state and federal offices in the +nited ,tates has multiplied nearly four times. A! has multiplied nearly four times "! has almost 5uadrupled $! has almost multiplied by four

SC Strategy

$;$

&! is almost four times as great '! is nearly fourfold #hat it #as :44. ,ince 1)*7 the number of #omen in upper-level management in American corporations have increased by (7 percentE female e<ecutives3 salaries% ho#ever% still lag behind those of their male counterparts. A! have increased by (7 percentE female e<ecutives3 salaries% ho#ever% still lag "! has increased by (7 percent% ho#ever much their salaries lag $! have increased (7 percentE female e<ecutives3 salaries% ho#ever% still have lagged &! has increased by (7 percentE female e<ecutives3 salaries% ho#ever% still lag '! have increased (7 percentE their salaries% ho#ever% still lag :47. ,ince 1)01% #hen the farm depression began% the number of acres overseen by professional farm-management companies have gro#n from 40 million to nearly 7) million% an area that is about $olorado3s si9e. A! have gro#n from 40 million to nearly 7) million% an area that is about $olorado3s si9e "! have gro#n from 40 million to nearly 7) million% about the si9e of $olorado $! has gro#n from 40 million to nearly 7) million% an area about the si9e of $olorado &! has gro#n from 40 million up to nearly 7) million% an area about the si9e of $olorado3s '! has gro#n from 40 million up to nearly 7) million% about $olorado3s si9e :4:. ,ince 1)0: enrollments of African Americans% American Indians% and >ispanic Americans in fulltime engineering programs in the +nited ,tates has steadily increased% #hile the number of other students #ho enter the field has fallen. A! has steadily increased% #hile the number of other students #ho enter the field has fallen "! has steadily increased% #hile other students entering the field have declined in number $! increased steadily% #hile there #as a decline in the number of other students entering the field &! have steadily increased% #hile the number of other students entering the field has fallen '! have steadily increased% #hile that of other students #ho enter the field fell :4*. ,ince 1)0: #hen the &epartment of .abor began to allo# investment officers3 fees to be based on ho# the funds they manage perform% several corporations began paying their investment advisers a small basic fee% #ith a contract promising higher fees if the managers perform #ell. A! investment officers3 fees to be based on ho# the funds they manage perform% several corporations began "! investment officers3 fees to be based on the performance of the funds they manage% several corporations began $! that fees of investment officers be based on ho# the funds they manage perform% several corporations have begun &! fees of investment officers to be based on the performance of the funds they manage% several

$;'

corporations have begun '! that investment officers3 fees be based on the performance of the funds they manage% several corporations began :40. ,ince chromosome damage may be caused by viral infections% medical <-rays% and e<posure to sunlight% it is important that the chromosomes of a population to be tested for chemically induced damage be compared #ith those of a control population. A! to be tested for chemically induced damage be compared #ith "! being tested for damage induced chemically are compared #ith $! being tested for chemically induced damage should be compared to &! being tested for chemically induced damage are to be compared to '! that is to be tested for chemically induced damage are to be comparable #ith :4). ,ince conscious patients often died of shock on the operating table% the invention of anesthesia #as essential to the development of surgery as the invention of the propeller #as to po#ered flight. A! the invention of anesthesia #as essential to the development of surgery as the invention of the propeller #as to po#ered flight "! inventing anesthesia #as as essential for the development of surgery as the invention of the propeller #as for po#ered flight $! the invention of anesthesia #as as essential in the development of surgery much as the invention of the propeller had been for po#ered flight &! the invention of anesthesia #as as essential to the development of surgery as the invention of the propeller #as to po#ered flight '! the invention of anesthesia #as essential to the development of surgery% so #as the invention of the propeller essential to po#ered flight :72. ,ince savings banks have to use short-term deposits to finance long-term fi<ed-rate mortgage loans% they sometimes lose money #hen there is a rise in short-term rates and% on the other hand% they are unable to raise the rates on their mortgages. A! #hen there is a rise in short-term rates and% on the other hand% they are unable to raise "! #hen short-term rates rise and they are unable to raise $! #hen a rise in short-term rates occurs and% correspondingly% there is no rise possible in &! #ith a rise in short-term rates% and they are unable to raise '! #ith short-term rates on the rise and no rise possible in :71. ,ince the 1)/23s aircraft manufacturers have tried to build airplanes #ith frictionless #ings% shaped so smoothly and perfectly that the air passing over them #ould not become turbulent. A! #ings% shaped so smoothly and perfectly "! #ings% #ings so smooth and so perfectly shaped

SC Strategy

$;*

$! #ings that are shaped so smooth and perfect &! #ings% shaped in such a smooth and perfect manner '! #ings% #ings having been shaped smoothly and perfectly so :7(. ,ince the movie #as released seventeen +?Gs have been sighted in the state% #hich is more than had been sighted in the past ten years together. A! #hich is more than had been sighted "! more than had been sighted $! more than they had sighted &! more than had reported sightings '! #hich is more than had reported sightings :7/. ,lips of the tongue do not necessarily reveal concealed beliefs or intentions but rather are the result from the competition bet#een various processing mechanisms in the brain. A! but rather are the result from "! and instead are the result from $! being rather the result of &! and rather result from '! but rather result from :74. ,o poorly educated and trained are many young recruits to the +nited ,tates #ork force that many business e<ecutives fear this country #ill lose its economic preeminence. A! ,o poorly educated and trained are many young recruits to the +nited ,tates #ork force that "! As poorly educated and trained as many young recruits to the +nited ,tates #ork force are $! "ecause of many young recruits to the +nited ,tates #ork force #ho are so poorly educated and trained. &! =hat many young recruits to the +nited ,tates #ork force are so poorly educated and trained is #hy '! 8any young recruits to the +nited ,tates #ork force #ho are so poorly educated and trained e<plains #hy :77. ,ocrates could have fled from Athens after he #as sentenced to death% but he refused to do it. A! ,ocrates could have fled from Athens after he #as sentenced to death% but he refused to do it. "! After he #as sentenced to death% ,ocrates might have fled from Athens% but he refused to do it. $! After he #as sentenced to death% ,ocrates could have fled from Athens% but he refused to do so. &! After he #as sentenced to death% ,ocrates could have done so after he #as sentenced to death. '! ,ocrates could have fled from Athens but refused to after he #as sentenced to death.

$;7

:7:.

,ome analysts contend that true capitalism e<ists only #hen the o#nership of both property and the means of production is regarded as an inalienable right of an individual3s% and it is not a license granted by government and revokable at #him. A! is regarded as an inalienable right of an individual3s% and it is not "! are regarded as individuals3 inalienable rights% and that it not be $! is regarded as an individual3s inalienable right% not as &! are regarded as an individual3s inalienable rights% not #hen they are '! is regarded as the inalienable rights of an individual% not #hen it is

:7*.

,ome analysts of retirement problems insist that the only #ay to provide e5uity and protection for all Americans is to amalgamate all retirement-income systems% including ,ocial ,ecurity% into one central system. A! insist that the only #ay to provide e5uity and protection for all Americans is to amalgamate "! have insisted the only #ay e5uity and protection can be provided for all Americans is amalgamating $! insist the only #ay to provide e5uity and protection for all Americans is the amalgamation of &! are insistent that the only #ay e5uity and protection can be provided for all Americans is the amalgamation of '! insist that the only #ay for the provision of e5uity and protection for all Americans is to amalgamate

:70.

,ome analysts point out that because people are becoming accustomed to a steady inflation rate of four to five percent% businesses found that they could raise prices according to this amount #ithout thereby provoking strong public reaction. A! found that they could raise prices according to this amount #ithout thereby provoking "! found that they #ere capable of raising prices by this amount and not provoke $! find that they are capable of raising prices by this amount and not provoke &! are finding that they can raise prices by this amount #ithout provoking '! are finding that they can raise prices according to this amount and #ill not thereby provoke

:7).

,ome bat caves% like honeybee hives% have residents that take on different duties such as defending the entrance% acting as sentinels and to sound a #arning at the approach of danger% and scouting outside the cave for ne# food and roosting sites. A! acting as sentinels and to sound "! acting as sentinels and sounding $! to act as sentinels and sound &! to act as sentinels and to sound '! to act as a sentinel sounding

::2.

,ome biographers have not only disputed the common notion that 'dgar Allan 1oe drank to

SC Strategy

$;8

e<cess but also 5uestioned #hether he drank at all. A! have not only disputed the common notion that 'dgar Allan 1oe drank to e<cess but also 5uestioned #hether he drank "! not only have disputed the common notion that 'dgar Allan 1oe drank to e<cess but also over #hether he drank $! have disputed not only the common notion that 'dgar Allan 1oe drank to e<cess but also #hether he may not have drunk &! not only have disputed the common notion that 'dgar Allan 1oe drank to e<cess but also 5uestioned #hether or not he had drunk '! have disputed the common notion not only that 'dgar Allan 1oe drank to e<cess but also 5uestioned #hether he may not have drunk ::1. ,ome buildings that #ere destroyed and heavily damaged in the earth5uake last year #ere constructed in violation of the city3s building code. A! ,ome buildings that #ere destroyed and heavily damaged in the earth5uake last year #ere "! ,ome buildings that #ere destroyed or heavily damaged in the earth5uake last year had been $! ,ome buildings that the earth5uake destroyed and heavily damaged last year have been &! .ast year the earth5uake destroyed or heavily damaged some buildings that have been '! .ast year some of the buildings that #ere destroyed or heavily damaged in the earth 5uake had been ::(. ,ome historians of science have argued that science moves for#ard not so much because of the insights of great thinkers but because of more mundane developments% such as improved tools and technologies. A! because of the insights of great thinkers but because of "! because of the insights of great thinkers as the result of $! because of the insights of great thinkers as because of &! through the insights of great thinkers but through '! through the insights of great thinkers but result from ::/. ,ome of the tenth-century stave churches of Nor#ay are still standing% demonstrating that #ith sound design and maintenance% #ooden buildings can last indefinitely. A! standing% demonstrating that #ith sound design and maintenance% #ooden buildings can last indefinitely "! standing% demonstrating ho# #ooden buildings% #hen they have sound design and maintenance% can last indefinitely $! standingE they demonstrate if a #ooden building has sound design and maintenance it can last indefinitely &! standing% and they demonstrate #ooden buildings can last indefinitely #hen there is sound

$;9

design and maintenance '! standing% and they demonstrate ho# a #ooden building can last indefinitely #hen it has sound design and maintenance ::4. ,ome psychiatric studies indicate that among distinguished artists the rates of manic depression and ma6or depression are ten to thirteen times as prevalent as in the population at large. A! the rates of manic depression and ma6or depression are ten to thirteen times as prevalent as in "! the rates of manic depression and ma6or depression are ten to thirteen times more prevalent than in $! the rates of manic depression and ma6or depression are ten to thirteen times more prevalent #hen compared to &! manic depression and ma6or depression are ten to thirteen times as prevalent #hen compared to '! manic depression and ma6or depression are ten to thirteen times more prevalent than in ::7. ,ome scientists have been critical of the laboratory tests conducted by the ?ederal &rug Administration on the grounds that the amounts of suspected carcinogens fed to animals far e<ceeds those that humans could consume. A! far e<ceeds those that humans could consume "! e<ceeds by far those humans can consume $! far e<ceeds those humans are able to consume &! e<ceed by far those able to be consumed by humans '! far e<ceed those that humans could consume :::. ,outh Forea has #itnessed the #orld3s most dramatic gro#th of $hristian congregationsE church membership is e<panding by :.: percent a year% fully t#o-thirds of the gro#th coming from conversions rather than the population increasing. A! coming from conversions rather than the population increasing "! coming from conversions rather than increases in the population $! coming from conversions instead of the population3s increasing &! is from conversions instead of population increases '! is from conversions rather than increasing the population ::*. ,panning more than fifty years% ?riedrich 8uller began his career in an unpromising apprenticeship as a ,anskrit scholar and culminated in virtually every honor that 'uropean governments and learned societies could besto#. A! 8uller began his career in an unpromising apprenticeship as "! 8uller3s career began in an unpromising apprenticeship as $! 8uller3s career began #ith the unpromising apprenticeship of being &! 8uller had begun his career #ith the unpromising apprenticeship of being

SC Strategy

$;;

'! the career of 8uller has begun #ith an unpromising apprenticeship of ::0. ,t. Cohn3s% Ne#foundland% lies on the same latitude as 1aris% ?rance% but in spring ,t. Cohn3s residents are less likely to be sitting at outdoor cafes than to be bracing themselves against arctic chills% shoveling sno#% or seeking shelter from a raging northeast storm. A! residents are less likely to be sitting at outdoor cafes than to be bracing themselves against arctic chills% shoveling sno#% or seeking "! residents are less likely to sit at outdoor cafes% and more to brace themselves against arctic chills% shovel sno#% or be seeking $! residents are less likely to be sitting at outdoor cafes% and more likely to be bracing themselves against arctic chills% shoveling sno#% or to be seeking &! residents% instead of their sitting at outdoor cafes% they are more likely to brace themselves against arctic chills% shovel sno#% or seek '! residents% instead of sitting at outdoor cafes% are more likely to brace themselves against arctic chills% shovel sno#% or to be seeking ::). ,table interest rates on long-term bonds are the financial market3s vote of confidence in the ?ederal Deserve keeping in control of inflation. A! in the ?ederal Deserve keeping in control of inflation "! that the ?ederal Deserve #ill keep inflation under control $! for the ?ederal Deserve% that it #ould keep control of inflation &! that inflation #ill be kept control of by the ?ederal Deserve '! that inflation #ould be kept control of by the ?ederal Deserve :*2. ,tars like the sun can continue to shine steadily for billions of years because its light and heat are produced by nuclear fusion% in #hich titanic releases of energy result in the loss of only tiny amounts of mass. A! its light and heat are produced by nuclear fusion% in #hich "! nuclear fusion produces its light and heat% in #hich $! nuclear fusion produces their light and heat% #here &! its light and heat are produced by nuclear fusion% #here '! their light and heat are produced by nuclear fusion% in #hich :*1. ,tate officials report that soaring rates of liability insurance have risen to force cutbacks in the operations of everything from local governments and school districts to day-care centers and recreational facilities. A! rates of liability insurance have risen to force "! rates of liability insurance are a force for $! rates for liability insurance are forcing &! rises in liability insurance rates are forcing

$;I

'! liability insurance rates have risen to force

:*(.

,tudents in the metropolitan school district lack math skills to such a large degree as to make it difficult to absorb them into a city economy becoming ever more dependent on informationbased industries. A! lack math skills to such a large degree as to make it difficult to absorb them into a city economy becoming "! lack math skills to a large enough degree that they #ill be difficult to absorb into a city3s economy that becomes $! lack of math skills is so large as to be difficult to absorb them into a city3s economy that becomes &! are lacking so much in math skills as to be difficult to absorb into a city3s economy becoming '! are so lacking in math skills that it #ill be difficult to absorb them into a city economy becoming

:*/.

,tudies of circadian rhythms can be applied to problems ranging from chronic insomnia in fre5uent transcontinental air travelers to #orkers in all-night plants being inefficient. A! to #orkers in all-night plants being inefficient "! to#ard #orkers in all-night plants being inefficient $! to #orkers in all-night plants #ho are inefficient &! to combat #orker inefficiency in all-night plants '! to decreased efficiency among #orkers in all-night plants

:*4.

,tudies of the human sleep-#ake cycle have practical relevance for matters ranging from duty assignments in nuclear submarines and air-traffic control to#ers to the staff of shifts in (4hour factories. A! to the staff of "! to those #ho staff $! to the staffing of &! and staffing '! and the staff of

:*7.

,tudies sho# that young people #ith higher-than-average blood pressure and their families have a history of high blood pressure are more likely than others to develop a severe form of the condition. A! and their families have a history of high blood pressure "! #hose families have a history of high blood pressure $! and a history of high blood pressure runs in the family &! #hose families have a history of high blood pressure running in them

SC Strategy

$;J

'! #ith a history of high blood pressure running in their family :*:. ,ulfur dio<ide% a ma6or contributor to acid rain% is an especially serious pollutant because it diminishes the respiratory system3s ability to deal #ith all other pollutants. A! an especially serious pollutant because it diminishes the respiratory system3s ability to deal "! an especially serious pollutant because of diminishing the respiratory system3s capability of dealing $! an especially serious pollutant because it diminishes the capability of the respiratory system in dealing &! a specially serious pollutant because it diminishes the capability of the respiratory system to deal '! a specially serious pollutant because of diminishing the respiratory system3s ability to deal :**. ,unspots% vortices of gas associated #ith strong electromagnetic activity% are visible as dark spots on the surface of the ,un but have never been sighted on the ,un3s poles or e5uator. A! are visible as dark spots on the surface of the ,un but have never been sighted on "! are visible as dark spots that never have been sighted on the surface of the ,un $! appear on the surface of the ,un as dark spots although never sighted at &! appear as dark spots on the surface of the ,un% although never having been sighted at '! appear as dark spots on the ,un3s surface% #hich have never been sighted on :*0. =elevision programs developed in con6unction #ith the marketing of toys% #hich #as once prohibited by federal regulations% are thriving in the free market conditions permitted by the current ?ederal $ommunications $ommission. A! =elevision programs developed in con6unction #ith the marketing of toys% #hich #as once prohibited by federal regulations% are "! =elevision programs developed in con6unction #ith the marketing of toys% a practice that federal regulations once prohibited% is $! &eveloping television programs in con6unction #ith the marketing of toys% as once prohibited by federal regulations% is &! ?ederal regulations once prohibited developing television programs in con6unction #ith the marketing of toys% but they are '! ?ederal regulations once prohibited developing television programs in con6unction #ith the marketing of toys% but such programs are :*). =emporary-employment agencies benefit not only from the increasing demand for clerical #orkers but also the higher profits made #hen highly paid professionals are placed% re5uests for #hom have increased in the recent #ave of corporate takeovers. A! the higher profits made #hen highly paid professionals are placed% re5uests for #hom "! the higher profits that are made in the placement of highly paid professionals% re5uests for

$IE

#hom $! from the re5uests for highly paid professionals% #ho make higher profits for the agencies #hen placed and #hose re5uests &! from highly paid professionals% #hose placement makes higher profits for the agencies and #hose re5uests '! from the higher profits made in placing highly paid professionals% re5uests for #hom :02. =enor -eorge ,hirley sang more than (2 leading roles at the 8etropolitan Gpera since having his debut there as ?ernando in $osifan tulle on Gctober (4% 1):1. A! sang more than (2 leading roles at the 8etropolitan Gpera since having "! sang more than (2 leading roles at the 8etropolitan Gpera since having had $! has sung more than (2 leading roles at the 8etropolitan Gpera since &! has sung more than (2 leading roles at the 8etropolitan Gpera after '! has sung more than (2 leading roles at the 8etropolitan Gpera subse5uently to :01. =eratomas are unusual forms of cancer because they are composed of tissues such as tooth and bone not normally found in the organ in #hich the tumor appears. A! because they are composed of tissues such as tooth and bone "! because they are composed of tissues like tooth and bone that are $! because they are composed of tissues% like tooth and bone% tissues &! in that their composition% tissues such as tooth and bone% is '! in that they are composed of tissues such as tooth and bone% tissues :0(. =e<tbooks for the used book sale should be in good condition and should have no #riting in them or be underlined. A! and should have no #riting in them or be underlined "! and should not have #riting in them or not be underlined $! and contain no #riting or underlining &! #ithout containing #riting nor be underlined '! #ithout having any #riting or no underlining in them :0/. =hat educators have not anticipated the impact of microcomputer technology can hardly be said that it is their faultA Alvin =offler% one of the most prominent students of the future% did not even mention microcomputers in ?uture ,hock% published in 1)*2. A! =hat educators have not anticipated the impact of microcomputer technology can hardly be said that it is their fault "! =hat educators have not anticipated the impact of microcomputer technology can hardly be said to be at fault $! It can hardly be said that it is the fault of educators #ho have not anticipated the impact of

SC Strategy

$I$

microcomputer technology &! It can hardly be said that educators are at fault for not anticipating the impact of microcomputer technology '! =he fact that educators are at fault for not anticipating the impact of microcomputer technology can hardly be said :04. =hat the ne# managing editor rose from the publication3s soft ne#s sections to a leadership position is more of a landmark in the industry than her being a #oman. A! her being a #oman "! being a #oman is $! her #omanhood &! that she #as a #oman '! that she is a #oman :07. =he 1)th-century proponents of the school of thought kno#n as mechanism held that life process are not the products of some mysterious life force% but are the same chemical and physical processes that operate in inorganic systems% #hich is still a sub6ect of debate bet#een biologists today. A! systems% #hich is still a sub6ect of debate bet#een "! systems% #hich are still debated bet#een $! systems still debated among &! systems% a theory still debated by '! systems% a theory still debated bet#een :0:. =he aim of the ne# regulations is to make it easier for prospective homeo#ners to have funds available for energy improvement in their ne# houses. A! to make it easier for prospective homeo#ners to have "! to make easier for prospective homeo#ners the having of $! making it easier for prospective homeo#ners so they can have &! that prospective homeo#ners more easily can have '! for prospective homeo#ners to more easily have :0*. =he airline industry is cutting its lo#est discount fares more #idely% more substantially% and earlier this year than it normally does at the end of the summer% a time during #hich travel usually decreases and the industry uses some lo#er fares for the attraction of passengers. A! during #hich travel usually decreases and the industry uses some lo#er fares for the attraction of "! during #hich travel usually decreases and therefore the industry #ill use some lo#er fares for the attraction of $! in #hich travel usually decreases and in #hich the industry therefore uses some lo#er fares

$I'

attracting &! #hen travel usually decreases and the industry uses some lo#er fares to attract '! #hen travel usually decreases and therefore the industry #ill use lo#er fares for the attraction of :00. =he American 8edical Association has argued that the rapidly rising costs associated #ith malpractice litigation are driving doctors from the profession and that reform of the tort system is imperative for bringing malpractice insurance premiums under control. A! that reform of the tort system is imperative for bringing malpractice insurance premiums "! that reform of the tort system is imperative if malpractice insurance premiums are to be brought $! that reform of the tort system is imperative to bring malpractice insurance premiums &! reform of the tort system is necessary in bringing malpractice insurance premiums '! the tort system needs to be reformed so that malpractice insurance premiums are brought :0). =he animosity bet#een those #ho regulate and those #ho are regulated% never more pronounced than in recent debates over environmentalism and pollution control. A! =he animosity bet#een those #ho regulate and those #ho are regulated% never "! =he animosity bet#een those #ho regulate and those #ho are regulated% never being $! =he animosity bet#een those #ho regulate and those #ho are regulated has never been &! "et#een those #ho regulate and those #ho are regulated% such animosity #as never '! "et#een those #ho regulate and bet#een those regulated% such animosity has never been :)2. =he argument that the dominant form of family structure today is not the nuclear family% but rather it is a family #hich is modified and e<tended% is based on a number of factsA the e<istence of three-generational families% the amount of vertical and hori9ontal communication bet#een family subunits% and the e<tent to #hich family members offer assistance to one another. A! family% but rather it is a family #hich is modified and e<tended% is based on a number of factsA the e<istence of three-generational families "! family% but is instead a family that is modified and e<tended% is based on a number of factsA three-generational families e<ist $! family% #hich has been modified and e<tended% is based on a number of factsA the e<istence of three-generational families &! family but a modified e<tended family is based on a number of factsA the e<istence of threegenerational families '! family% but also a modified and e<tended family% is based on a number of factsA threegenerational families e<ist :)1. =he aristocratic values e<pressed in the #ritings of 8arguerite @ourcenar place her #ithin the ?rench classical tradition% as does her passionate interest in history% particularly Doman history.

SC Strategy

$I*

A! as does "! so do $! as do &! so is the case #ith '! similarly% does :)(. =he attorney turned do#n the la# firm3s offer of a position because she suspected that it #as meant merely to fill an affirmative action 5uota #ith no commitment to minority hiring and eventually promoting. A! 5uota #ith no commitment to minority hiring and eventually promoting "! 5uota% having no commitment to minority hiring and eventually promoting $! 5uota and did not reflect a commitment to minority hiring and eventual promotion &! 5uota% not reflecting a commitment to minority hiring and eventual promotion '! 5uota% not one that reflected that minority hiring and eventual promotion #as a commitment :)/. =he Audubon ,ociety and other conservation groups% concerned over #hat they have perceived to be the serious threatening of the environment as posed by the policies of the government% are preparing for a ma6or political effort. A! have perceived to be the serious threatening of the environment as "! perceived as the serious threat to the environment as $! perceive being the serious threat to the environment &! are perceiving as the serious threatening of the environment '! perceive as the serious threat to the environment :)4. =he auto industry has e<perienced one of its most significant trends in the last 72 years% #hich is the migration of motorists from passenger cars to minivans% sport utility vehicles% and pickups. A! =he auto industry has e<perienced one of its most significant trends in the last 72 years% #hich is "! Gf the trends the auto industry e<perienced in the last 72 years has been one of the most significant $! In the last 72 years% one of the most significant trends that the auto industry has been e<periencing has been &! Gne of the most significant trends that the auto industry has e<perienced in the last 72 years is '! In the last 72 years% the auto industry e<perienced one of the most significant trends that it has had% that of :)7. =he average #eekly #age nearly doubled in the 1)*23s% rising from I114 to I((2% yet the average #orker ended the decade #ith a decrease in #hat their pay may buy.

$I7

A! #ith a decrease in #hat their pay may buy "! #ith #hat #as a decrease in #hat they #ere able to buy $! having decreased that #hich they could buy &! decreasing in purchasing po#er '! #ith a decrease in purchasing po#er :):. =he "aldrick 8anufacturing $ompany has for several years follo#ed a policy aimed at decreasing operating costs and improving the efficiency of its distribution system. A! aimed at decreasing operating costs and improving "! aimed at the decreasing of operating costs and to improve $! aiming at the decreasing of operating costs and improving &! the aim of #hich is the decreasing of operating costs and improving '! #ith the aim to decrease operating costs and to improve :)*. =he bank ackno#ledged that they are and #ill continue to e<perience difficulties as it attempts to deal #ith the precipitous fall of the dollar against the yen and the dislocations reflected in the stock market decline. A! they are and #ill continue to e<perience difficulties as it attempts "! they are and #ill continue to e<perience difficulties as they attempt $! it is and #ill continue to e<perience difficulties as it attempts &! it is e<periencing and #ill continue to e<perience difficulties as they make an attempt '! its difficulties are likely to continue as it attempts :)0. =he bank holds I/ billion in loans that are seriously delin5uent or in such trouble that they do not e<pect payments #hen due. A! they do not e<pect payments #hen "! it does not e<pect payments #hen it is $! it does not e<pect payments to be made #hen they are &! payments are not to be e<pected to be paid #hen '! payments are not e<pected to be paid #hen they #ill be :)). =he black hole has entered the popular imagination as an ob6ect too massive that neither light nor matter can escape its gravitational pull. A! too massive that neither light nor matter can escape its "! too massive for either allo#ing light or matter to escape its $! massive enough that either light or matter cannot escape their &! so massive that neither light nor matter could escape their '! so massive that neither light nor matter can escape its

SC Strategy

$I8

*22.

=he boom in agricultural e<ports in the early 1)*23s emptied +nited ,tates grain bins and many #ere led to thinking that overproduction #as no# a problem of the past. A! many #ere led to thinking that overproduction #as no# "! many had been led to thinking of overproduction as if it #ere $! the thought this led to #as that overproduction had become &! led many to the thought of overproduction as if it #ere '! led many to think that overproduction had become

*21.

=he brain is something of a stimulus reduction system% a means to reduce% in order to comprehend% the nearly infinite amount of stimuli that reach the senses at any given moment. A! a means to reduce% in order to comprehend% the nearly infinite amount "! a means to reduce% in order to comprehend% the nearly infinite number $! the means of reducing for comprehending the nearly infinite number &! the means that reduces% in order to comprehend% the nearly infinite amount '! the means for reducing in order to comprehend the nearly infinite amount

*2(.

=he "ritish Admiralty and the War Gffice met in 8arch 10)( to consider a possible Dussian attempt to sei9e $onstantinople and ho# they #ould have to act militarily to deal #ith them. A! ho# they #ould have to act militarily to deal #ith them "! ho# to deal #ith them if military action #ould be necessary $! #hat #ould be necessary militarily for dealing #ith such an event &! #hat military action #ould be necessary in order to deal #ith such an event '! the necessity of #hat kind of military action in order to take for dealing #ith it

*2/.

=he "ritish sociologist and activist "arbara Wootton once noted as a humorous e<ample of income maldistribution that the elephant that gave rides to children at the Whipsnade Koo #as earning annually e<actly #hat she then earned as director of adult education for .ondon. A! that the elephant that gave rides to children at the Whipsnade Koo #as earning "! that the elephant% giving rides to children at the Whipsnade Koo% had been earning $! that there #as an elephant giving rides to children at the Whipsnade Koo% and it earned &! the elephant that gave rides to children at the Whipsnade Koo and #as earning '! the elephant giving rides to children at the Whipsnade Koo and that it earned

*24.

=he brochure notes that in the seminar the importance that communication is a t#o-#ay process #ill be emphasi9ed. A! importance that communication is a t#o-#ay process #ill be emphasi9ed "! importance of communication as a t#o-#ay process #ill be emphasi9ed $! importance of communication being a t#o-#ay process #ill be the emphasis &! fact #ill be emphasi9ed that communication is a t#o-#ay process and of importance

$I9

'! emphasis #ill be that communication being a t#o-#ay process is important *27. =he cameras of the Boyager II spacecraft detected si< small% previously unseen moons circling +ranus% #hich doubles to t#elve the number of satellites no# kno#n as orbiting the distant planet. A! #hich doubles to t#elve the number of satellites no# kno#n as orbiting "! doubling to t#elve the number of satellites no# kno#n to orbit $! #hich doubles to t#elve the number of satellites no# kno#n in orbit around &! doubling to t#elve the number of satellites no# kno#n as orbiting '! #hich doubles to t#elve the number of satellites no# kno#n that orbit *2:. =he capital of "osnia->er9egovina% ,ara6evo3s population on the eve of the ?irst World War #as 71%)1). A! "osnia->er9egovina% ,ara6evo3s population "! "osnia->er9egovina is ,ara6evo% #hose population $! "osnia->er9egovina is ,ara6evo% #ith a population &! "osnia->er9egovina% ,ara6evo having a population that '! "osnia->er9egovina% the population of ,ara6evo *2*. =he caterpillar of the geometrid moth strikes #hen special tactile hairs on its body are disturbed% after capturing its prey% holds the victim so that it cannot escape. A! strikes #hen special tactile hairs on its body are disturbed% "! striking #hen special tactile hairs on its body are disturbed% but $! #hich strikes #hen special tactile hairs on its body are disturbed% &! #hich% striking #hen special tactile hairs on its body are disturbed% '! strikes #hen special tactile hairs on its body are disturbed and% *20. =he $oast guard is conducting tests to see #hether pigeons can be trained to help find survivors of #recks at sea. A! to see #hether pigeons can be trained to help find "! to see #hether pigeons can be trained as help to find $! to see if pigeons can be trained for helping to find &! that see if pigeons are able to be trained in helping to find '! that see #hether pigeons are able to be trained for help in finding *2). =he $oast -uard recently redesigned channel markers in the $hesapeake "ay to accommodate large numbers of ospreys% #hose huge nests formerly obstructed the lights. A! numbers of ospreys% #hose huge nests "! numbers of ospreys% their huge nests

SC Strategy

$I;

$! amounts of ospreys% the huge nests of #hich &! nests of ospreys they '! numbers of ospreys% and their huge nests *12. =he colori9ation of black-and-#hite films by computers is defended by those #ho o#n the film rights% for the process can mean increased revenues for themE many others in the film industry% ho#ever% contend that the techni5ue degrades ma6or #orks of art% #hich they liken to putting lipstick on a -reek statue. A! #hich they liken to putting lipstick on a -reek statue "! #hich they liken to a -reek statue #ith lipstick put on it $! #hich they liken to lipstick put on a -reek statue &! likening it to a -reek statue #ith lipstick put on it '! likening it to putting lipstick on a -reek statue *11. =he $ommerce &epartment announced that the economy gre# during the second 5uarter at a *.7 percent annual rate% #hile inflation eased #hen it might have been e<pected for it to rise. A! it might have been e<pected for it to rise "! it might have been e<pected to rise $! it might have been e<pected that it should rise &! its rise might have been e<pected '! there might have been an e<pectation it #ould rise *1(. =he commission ackno#ledged that no amount of money or staff members can ensure the safety of people #ho live in the vicinity of a nuclear plant% but it approved the installation because it believed that all reasonable precautions had been taken. A! no amount of money or staff members "! neither vast amounts of money nor staff members $! neither vast amounts of money nor numbers of staff members &! neither vast amounts of money nor a large staff '! no matter ho# large the staff or ho# vast the amount of money *1/. =he commission has directed advertisers to restrict the use of the #ord natural to foods that do not contain color or flavor additives% chemical preservatives% or nothing that has been synthesi9ed. A! or nothing that has been "! or that has been $! and nothing that is &! or anything that has been '! and anything

$II

*14.

=he commission proposed that funding for the park3s development% #hich could be open to the public early ne<t year% is obtained through a local bond issue. A! that funding for the park3s development% #hich could be open to the public early ne<t year% is "! that funding for development of the park% #hich could be open to the public early ne<t year% be $! funding for the development of the park% perhaps open to the public early ne<t year% to be &! funds for the park3s development% perhaps open to the public early ne<t year% be '! development funding for the park% #hich could be open to the public early ne<t year% is to be

*17.

=he commissior3s office of compliance% inspections% and investigations plans to intensify its scrutiny of stock analysts to investigate not only #hether research is an independent function at brokerage firms% but also #hether conflicts result #hen analysts o#n the stocks they #rite about or #hen they are paid for their #ork by a firm3s investment. A! to investigate not only #hether research is an independent function at brokerage firms% but also #hether conflicts result #hen analysts o#n the stocks they #rite about or #hen they are "! to investigate not only #hether research is an independent function at brokerage firms% but also if conflicts result #hen analysts o#n the stocks they #rite about or they are $! to not only investigate #hether or not research is an independent function at brokerage firms% but also if conflicts result #hen analysts o#n the stocks they #rite about or are &! not only to investigate #hether or not research is an independent function at brokerage firms% but also #hether conflicts result #hen analysts o#n the stocks they #rite about or are '! not only to investigate #hether research is an independent function at brokerage firms% but also #hether conflicts result #hen analysts o#n the stocks they #rite about or #hen

*1:.

=he company announced that its profits declined much less in the second 5uarter than analysts had e<pected it to and its business #ill improve in the second half of the year. A! had e<pected it to and its business #ill improve "! had e<pected and that its business #ould improve $! e<pected it #ould and that it #ill improve its business &! e<pected them to and its business #ould improve '! e<pected and that it #ill have improved its business

*1*.

=he company is negotiating to sell its profitable credit card subsidiary% #hich it plans to use money from to ac5uire some of the mortgage-servicing operations that are being sold by troubled savings institutions. A! subsidiary% #hich it plans to use money from "! subsidiary% from #hich it plans to use money $! subsidiary% and it plans the use of money from that &! subsidiary and plans to use money from that sale

SC Strategy

$IJ

'! subsidiary and plans the use of money from that sale *10. =he comple< ta< dispute bet#een the $overed "ridge 8all and >arris =o#nship is not likely to be ad6udicated for several years% and% in the meantime% both sides are intent on creating difficulties for the other. A! both sides are intent on creating difficulties for the other "! both sides are intent on creating difficulties for each other $! each side is intent on creating difficulties for the other &! each side is intent on creating difficulties for one another '! the sides are both intent on creating difficulties for each other *1). =he computer soft#are being designed for a pro6ect studying Native American access to higher education #ill not only meet the needs of that study% but also has the versatility and po#er of facilitating similar research endeavors. A! but also has the versatility and po#er of facilitating "! but also have the versatility and po#er to facilitate $! but it also has the versatility and po#er to facilitate &! and also have the versatility and po#er of facilitating '! and it also has such versatility and po#er that it can facilitate *(2. =he concept of the grand 6ury dates from the t#elfth century% #hen >enry II of 'ngland ordered panels of common citi9ens should prepare lists of #ho #ere their communities3 suspected criminals. A! should prepare lists of #ho #ere their communities3 suspected criminals "! #ould do the preparation of lists of their communities3 suspected criminals $! preparing lists of suspected criminals in their communities &! the preparing of a list of suspected criminals in their communities '! to prepare lists of suspected criminals in their communities *(1. =he current administration% being #orried over some foreign trade barriers being removed and our e<ports failing to increase as a result of deep cuts in the value of the dollar% has formed a group to study #ays to sharpen our competitiveness. A! being #orried over some foreign trade barriers being removed and our e<ports failing "! #orrying over some foreign trade barriers being removed% also over the failure of our e<ports $! #orried about the removal of some foreign trade barriers and the failure of our e<ports &! in that they #ere #orried about the removal of some foreign trade barriers and also about the failure of our e<ports '! because of its #orry concerning the removal of some foreign trade barriers% also concerning the failure of our e<ports

$JE

*((.

=he data collected by #eather airplanes that fly into the heart of a hurricane are useful mainly for gauging the storm3s structure and strength% not for the speed and the path of their movement. A! not for the speed and the path of their movement "! not for the speed and path of its movement $! not the speed and path of its movement &! and not the speed and path of their movements '! and not for the speed and the path of its movements

*(/.

=he debate over bilingual education centers on the issue of #hether the +nited ,tates should foster the idea of single common language% an idea% some believe% that has in the past been crucial in binding diverse constituencies together. A! been crucial in binding diverse constituencies together "! been crucial as a binding together of diverse constituencies $! been crucial to bind together constituencies that are diverse &! become crucial in binding together diverse constituencies '! become crucial to bind together constituencies that are diverse

*(4.

=he decision by one of the nation3s largest banks to admit to I/ billion in potential losses on foreign loans could mean less lending by commercial banks to developing countries and increasing the pressure on multigovernment lenders to supply the funds. A! increasing the pressure "! the increasing pressure $! increased pressure &! the pressure increased '! the pressure increasing

*(7.

=he decisions of Cohn 8arshall% the fourth chief 6ustice% have had a greater influence than any chief 6ustice in history. A! than any chief 6ustice in history "! historically than any other chief 6ustice $! than have those of any chief 6ustice in history &! in history as any other chief 6ustice has had '! than those of any other chief 6ustice in history

*(:.

=he delin5uency rates on mortgages for office buildings% hotels% shopping malls% and other commercial properties held by the nation3s insurance companies in recent months have increased sharply% leading to predictions that foreclosures on these types of loan could double over the ne<t three years. A! in recent months have increased sharply% leading to predictions that foreclosures on these

SC Strategy

$J$

types of loan could double over the ne<t three years "! have increased sharply in recent months% leading to predictions that foreclosures on these types of loans could double over the ne<t three years $! have increased sharply in recent months% leading to predictions that doubling of foreclosures on these types of loans could occur over the ne<t three years &! has increased sharply in recent months% and lead to predictions that over the ne<t three years doubling of foreclosures on this type of loan could occur '! in recent months has increased sharply% and this leads to predictions that foreclosures on that type of loan in the ne<t three years could double *(*. =he demand for airplane mechanics is e<pected to gro# about ten percent a year in the ne<t decade% largely because ne# federal rules calling for ma6or modifications to older planes and because the airlines are adding hundreds of ne# 6ets. A! ne# federal rules calling for ma6or modifications to older planes "! ne# federal rules% #hich call for ma6or modifications to older planes $! ne# federal rules call for ma6or modifications to older planes &! ma6or modifications to older planes called for by ne# federal rules '! ma6or modifications to older planes are called for according to ne# federal rules *(0. =he department defines a private passenger vehicle as one registered to an individual #ith a gross #eight of less than 0%222 pounds. A! as one registered to an individual #ith a gross #eight of less than 0%222 pounds "! to be one that is registered to an individual #ith a gross #eight of less than 0%222 pounds $! as one that is registered to an individual and that has a gross #eight of less than 0%222 pounds &! to have a gross #eight less than 0%222 pounds and being registered to an individual '! as having a gross #eight of less than 0%222 pounds and registered to an individual *(). =he development of a ne# 6umbo rocket that is e<pected to carry the +nited ,tates into its ne<t phase of space e<ploration #ill be able to deliver a heavier load of instruments into orbit than the space shuttle and at a lo#er cost. A! =he development of a ne# 6umbo rocket that is e<pected to carry the +nited ,tates into its ne<t phase of space e<ploration #ill be able to deliver a heavier load of instruments into orbit than the space shuttle and at a lo#er cost. "! =he development of a ne# 6umbo rocket is e<pected to carry the +nited ,tates into its ne<t phase of space e<ploration and be able to deliver a heavier load of instruments into orbit at a lo#er cost than the space shuttle. $! =he ne# development of a 6umbo rocket% #hich is e<pected to carry the +nited ,tates into its ne<t phase of space e<ploration% #ill be able to deliver a heavier load of instruments into orbit at a lo#er cost than the space shuttle.

$J'

&! A ne#ly developed 6umbo rocket% #hich is e<pected to carry the +nited ,tates into its ne<t phase of space e<ploration% #ill be able to deliver a heavier load of instruments into orbit than the space shuttle can% and at a lo#er cost. '! A ne#ly developed 6umbo rocket% #hich is e<pected to carry the +nited ,tates into its ne<t phase of space e<ploration% #ill be able to deliver a heavier load of instruments into orbit than the space shuttle and to cost less. */2. =he diet of the ordinary -reek in classical times #as largely vegetarianHvegetables% fresh cheese% oatmeal% and meal cakes% and meat rarely. A! and meat rarely "! and meat #as rare $! #ith meat as rare &! meat a rarity '! #ith meat as a rarity */1. =he direction in #hich the 'arth and the other solid planetsH8ercury% Benus% and 8arsH spins #ere determined from collisions #ith giant celestial bodies in the early history of the ,olar ,ystem. A! spins #ere determined from "! spins #ere determined because of $! spins #as determined through &! spin #as determined by '! spin #as determined as a result of */(. =he distinctive hereditary tartans that are alleged to be #orn since anti5uity by members of the >ighland clans #ere in fact designed by ,cottish #oolen manufacturers in the eighteenth and nineteenth centuries. A! that are alleged to be #orn "! alleged to have been #orn $! that are #orn% it #as alleged &! alleged as having been #orn '! that are #orn% allegedly *//. =he distribution of mass #ithin the core of the 'arth% like the mantle that surrounds the core% has been deduced from the orbital behavior of the 'arth and the motions of satellites controlled by the gravity of the 'arth. A! the mantle that surrounds the core "! that #ithin the mantle surrounding the core $! that of the mantle surrounding the core &! the mantle the core surrounds

SC Strategy

$J*

'! the distribution of mass #ithin the mantle that surrounds the core */4. =he domesticated camel% #hich some scholars date around the t#elfth century ".$.% #as the key to the development of the spice trade in the ancient #orld. A! =he domesticated camel% #hich some scholars date "! =he domesticated camel% #hich some scholars having thought to occur $! &omesticating the camel% dated by some scholars at &! =he domestication of the camel% thought by some scholars to have occurred '! =he camel3s domestication% dated by some scholars to have been */7. =he earnings of #omen are #ell belo# that of men in spite of educational differences that are diminishing bet#een the se<es. A! #ell belo# that of men in spite of educational differences that are diminishing "! much belo# that of men3s despite educational differences diminishing $! much belo# men in spite of diminishing educational differences &! #ell belo# those of men in spite of diminishing educational differences '! belo# men3s despite their educational differences that are diminishing */:. =he economic forces #hich may affect the ne# public offering of stock include sudden do#nturns in the market% hedging and other investor strategies for preventing losses% loosening the interest rates in Washington% and fearing that the company may still be undercapitali9ed. A! loosening the interest rates in Washington% and fearing that the company may still be undercapitali9ed "! loosening the interest rates in Washington% and a fear of the company still being undercapitali9ed $! a loosening of the interest rates in Washington% and fearing that the company may still be undercapitali9ed &! a loosening of the interest rates in Washington% and a fear of the still undercapitali9ed company '! a loosening of the interest rates in Washington% and a fear that the company may still be undercapitali9ed */*. =he efforts of the bipartisan committee #ere marked as much by frustration as it #as by success. A! as it #as by "! and also by $! as by &! and e5ually by

$J7

'! as there #as */0. =he 'mperor Augustus% it appears% commissioned an ideali9ed sculptured portrait% the features of #hich are so unrealistic as to constitute #hat one scholar calls an artificial face. A! so unrealistic as to constitute "! so unrealistic they constituted $! so unrealistic that they have constituted &! unrealistic enough so that they constitute '! unrealistic enough so as to constitute */). =he end of the eighteenth century sa# the emergence of pri9e-stock breeding% #ith individual bulls and co#s receiving a#ards% fetching unprecedented prices% and e<cited enormous interest #henever they #ere put on sho#. A! e<cited "! it e<cited $! e<citing &! #ould e<cite '! it had e<cited *42. =he endurance and consistency of baseball star .ou -ehrig% kno#n as =he Iron >orse% are legendary. A! =he endurance and consistency of baseball star .ou -ehrig% kno#n as =he Iron >orse% are legendary. "! =he endurance and consistency of .ou -ehrig% a baseball star kno#n as =he Iron >orse% is legendary. $! Fno#n as =he Iron >orse% the endurance and consistency of .ou -ehrig% the baseball star% is legendary. &! =he reason baseball star .ou -ehrig is kno#n as =he Iron >orse is because of his legendary endurance and consistency. '! Fno#n as =he Iron >orse% baseball star .ou -ehrig3s endurance and consistency are legendary. *41. =he energy source on Boyager ( is not a nuclear reactor% in #hich atoms are actively broken apartE rather a kind of nuclear battery that uses natural radioactive decay to produce po#er. A! apartE rather "! apart% but rather $! apart% but rather that of &! apart% but that of '! apartE it is that of

SC Strategy

$J8

*4(.

=he evolution of the technology of microelectronics over the past decade has been so rapid that it is sometimes called a revolution. A! has been so rapid that it is sometimes "! has been rapid enough sometimes to be $! has been rapid enough it is sometimes &! is so rapid it has sometimes been '! is so rapid that it is sometimes

*4/.

=he e<hibition of art from Nubians% the site of a "lack civili9ation that goes back to the fourth millennium ".$.% makes clear the Nubians combined artistic elements from 'gypt to that of sub,aharan Africa. A! the Nubians combined artistic elements from 'gypt to that "! that the Nubians combined artistic elements from 'gypt to that $! the Nubians combined artistic elements from 'gypt #ith that &! that the Nubians combined artistic elements from 'gypt #ith those '! that Nubians combined artistic elements from 'gypt and those

*44.

=he e<igencies of dramatic art% as sho#n even by the history plays of ,hakespeare% makes the foreshortening of dramati9ed historical events inevitable. A! makes the foreshortening of dramati9ed historical events inevitable "! made dramati9ed historical events inevitably foreshortened $! make the foreshortening of dramati9ed historical events inevitable &! has inevitably foreshortened dramati9ed historical events '! inevitably foreshortens dramati9ed historical events

*47.

=he e<pected rise in the price of oil could be a serious impact to industriali9ed nations and severely diminish the possibility to have an economy free of inflation. A! be a serious impact to industriali9ed nations and severely diminish the possibility to have "! seriously impact on industriali9ed nations and severely impede the possibility to have $! seriously impact on industriali9ed nations and severely impede the possibility of having &! have a serious impact on industriali9ed nations and severely impede the possibility to have '! have a serious impact on industriali9ed nations and severely diminish the possibility of having

*4:.

=he e<traordinary diary of William .yon 8acken9ie Fing% prime minister of $anada for over t#enty years% revealed that this most bland and circumspect of men #as a mystic guided in both public and private life by omens% messages received at sMances% and signs from heaven. A! that this most bland and circumspect of men #as a mystic guided in both public and "! that this most bland and circumspect of men #as a mystic and also guided both in public as #ell as

$J9

$! this most bland and circumspect of men #as a mystic and that he #as guided in both public and &! this most bland and circumspect of men #as a mystic and that he #as guided in both public as #ell as '! this most bland and circumspect of men to have been a mystic and that he guided himself both in public as #ell as *4*. =he fastest of mammals% cheetah3s bodies are geared to accelerate from one to forty miles per hour in less than t#o seconds and reach speeds of seventy miles per hour. A! =he fastest of mammals% cheetah3s bodies are geared to "! =he fastest of mammals% the body of cheetah3s bodies is able to $! ?aster than other mammals% the body of the cheetah is geared to &! =he fastest of mammals% the cheetah can '! =he cheetah% the fastest of mammals% have bodies that can *40. =he fear of rabies is #ell foundedE fe# people are kno#n to recover from the disease after the appearance of the clinical symptoms. A! fe# people are kno#n to recover from the disease after the appearance of the clinical symptoms "! fe# people are kno#n to have recovered from the disease once the clinical symptoms have appeared $! there are fe# kno#n people #ho have recovered from the disease once the clinical symptoms have appeared &! after the clinical symptoms appear% there are fe# kno#n people #ho have recovered from the disease '! recovery from the disease is kno#n for only a fe# people after the clinical symptoms appear *4). =he federal government re5uires hospitals to tell a 8edicare patient of their legal right of challenge their discharge if they feel they are being sent home prematurely. A! hospitals to tell a 8edicare patient of their "! hospital to tell 8edicare patients that they have a $! hospitals to tell 8edicare patients that there is a &! that hospitals tell a 8edicare patient of their '! that hospitals tell a 8edicare patient that they have a *72. =he ?ederal Deserve "oard3s reduction of interest rates on loans to financial institutions is both an ackno#ledgement of past economic trends and an effort to influence their future direction. A! reduction of interest rates on loans to financial institutions is both an ackno#ledgement of past economic trends and an effort "! reduction of interest rates on loans to financial institutions is an ackno#ledgement both of past

SC Strategy

$J;

economic trends as #ell as an effort $! reduction of interest rates on loans to financial institutions both ackno#ledge past economic trends and attempt &! reducing interest rates on loans to financial institutions is an ackno#ledgement both of past economic trends and an effort '! reducing interest rates on loans to financial institutions both ackno#ledge past economic trends as #ell as attempt *71. =he ?ederalist papers% a strong defense of the +nited ,tates $onstitution and important as a body of #ork in political science as #ell% represents the handi#ork of three different authors. A! and important as a body of #ork in political science as #ell% represents "! as #ell as an important body of #ork in political science% represent $! and also a body of #ork of importance in political science is representing &! an important body of #ork in political science and has been representative of '! and as political science an important body of #ork too% represent *7(. =he figure of the 6aguar% being a recurring symbol #ithin Glmec art% is prominent among the hieroglyphics inscribed on a monument that #as discovered in the 8e<ican state of Beracru9. A! being a recurring symbol #ithin "! a symbol having recurred #ithin $! a recurring symbol in &! having been a symbol that recurred in '! recurring as it is% a symbol in *7/. =he filibuster% a parliamentary device that slo#s the snail3s pace that prevails even in the best of times in congressional sessions and tests the endurance of everyone associated #ith it% seems more and more an anachronism in the age of telecommunications. A! sessions and tests the endurance of everyone associated #ith it% seems "! sessions and tests the endurance of everyone #ho is associated #ith it% seeming to be $! sessions% tests the endurance of everyone associated #ith it% seems &! sessions% that tests the endurance of everyone associated #ith it and seems '! sessions% testing the endurance of everyone associated #ith it and seeming *74. =he financial crash of Gctober 1)0* demonstrated that the #orld3s capital markets are integrated more closely than never before and events in one part of the global village may be transmitted to the rest of the villageHalmost instantaneously. A! integrated more closely than never before and "! closely integrated more than ever before so $! more closely integrated as never before #hile

$JI

&! more closely integrated than ever before and that '! more than ever before closely integrated as *77. =he first decision for most tenants living in a building undergoing being converted to cooperative o#nership is if to sign a no-buy pledge #ith the other tenants. A! being converted to cooperative o#nership is if to sign "! being converted to cooperative o#nership is #hether they should be signing $! being converted to cooperative o#nership is #hether or not they sign &! conversion to cooperative o#nership is if to sign '! conversion to cooperative o#nership is #hether to sign *7:. =he five hundred largest manufacturing firms in the +nited ,tates produce goods that are #orth almost as much as that of the four hundred thousand firms that remain. A! that are #orth almost as much as that of the four hundred thousand firms that remain "! of a #orth that is almost as much as that of the four hundred thousand firms that remain $! almost as much in #orth as those of the remaining four hundred thousand firms &! almost as much in #orth as that of the remaining four hundred thousand firms '! #orth almost as much as those of the remaining four hundred thousand firms *7*. =he ?orbidden $ity in "ei6ing% from #hich the emperors ruled by heavenly mandate% #as a site #hich a commoner or foreigner could not enter #ithout any permission% on pain of death. A! #hich a commoner or foreigner could not enter #ithout any permission% "! #hich a commoner or foreigner could enter #ithout any permission only $! #hich no commoner or foreigner could enter #ithout permission% &! #hich% #ithout permission% neither commoner or foreigner could only enter% '! #hich% to enter #ithout permission% neither commoner or foreigner could do% *70. =he golden crab of the -ulf of 8e<ico has not been fished commercially in great numbers% primarily on account of living at great depthsH(%722 to /%222 feet do#n. A! on account of living "! on account of their living $! because it lives &! because of living '! because they live *7). =he -orton-&odd bill re5uires that a bank disclose to their customers ho# long they #ill delay access to funds from deposited checks. A! that a bank disclose to their customers ho# long they #ill delay access to funds from deposited checks

SC Strategy

$JJ

"! a bank to disclose to their customers ho# long they #ill delay access to funds from a deposited check $! that a bank disclose to its customers ho# long it #ill delay access to funds from deposited checks &! a bank that it should disclose to its customers ho# long it #ill delay access to funds from a deposited check '! that banks disclose to customers ho# long access to funds from their deposited check is to be delayed *:2. =he group called the =eton ,iou< inhabits parts of North and ,outh &akotaE their language and customs differ% ho#ever% from the @ankton% ,isseton% and &akota ,iou<. A! their language and customs differ% ho#ever% from "! its language and customs differ% ho#ever% from $! ho#ever% they have different language and customs than &! ho#ever% their language and customs differ from those of '! its language and customs differ% ho#ever% from those of *:1. =he gro#ing demand for housing% traffic congestion% and longer commuting trips has all but eliminated the cost advantage of o#ning a house in many rural communities. A! =he gro#ing demand for housing% traffic congestion% and longer commuting trips has "! =raffic congestion% the gro#ing demand for housing% and longer commuting trips has $! .onger commuting trips% traffic congestion% and the gro#ing demand for housing has &! =raffic congestion% longer commuting trips% and the gro#ing demand for housing have '! =he gro#ing demand for housing% as #ell as traffic congestion and longer commuting trips% have *:(. =he gro#th of the railroads led to the abolition of local times% #hich #as determined by #hen the sun reached the observer3s meridian and differing from city to city% and to the establishment of regional times. A! #hich #as determined by #hen the sun reached the observer3s meridian and differing "! #hich #as determined by #hen the sun reached the observer3s meridian and #hich differed $! #hich #ere determined by #hen the sun reached the observer3s meridian and differing &! determined by #hen the sun reached the observer3s meridian and differed '! determined by #hen the sun reached the observer3s meridian and differing

*:/.

=he guiding principles of the ta< plan released by the =reasury &epartment could have even a greater significance for the economy than the particulars of the plan. A! even a greater significance for the economy than

'EE

"! a significance that is even greater for the economy than $! even greater significance for the economy than have &! even greater significance for the economy than do '! a significance even greater for the economy than have *:4. =he gyrfalcon% an Arctic bird of prey% has survived a close brush #ith e<tinctionE its numbers are no# five times greater than #hen the use of &&= #as sharply restricted in the early 1)*23s. A! e<tinctionE its numbers are no# five times greater than "! e<tinctionE its numbers are no# five times more than $! e<tinction% their numbers no# fivefold #hat they #ere &! e<tinction% no# #ith fivefold the numbers they had '! e<tinction% no# #ith numbers five times greater than *:7. =he herbicide Gry9alin #as still being produced in 1)*)% three years after the #ives of #orkers producing the chemical in Densselaer% Ne# @ork% #ere found to have borne children #ith heart defects or miscarriages% and none of their pregnancies #as normal. A! to have borne children #ith heart defects or miscarriages% and none of their pregnancies #as "! to have had children born #ith heart defects or miscarriages% and none of the pregnancies #as $! either to have had children #ith heart defects or miscarriages% #ithout any of their pregnancies being &! either to have had miscarriages or to have borne children #ith heart defectsE none of the pregnancies #as '! either to have had miscarriages or children born #ith heart defects% #ithout any of their pregnancies being *::. =he human gro#th hormone% made by the pituitary gland% is secreted during sleep in higher concentrations than #hen a#ake. A! during sleep in higher concentrations than #hen a#ake "! #hen sleeping in higher concentrations than #aking hours $! in higher concentrations during sleeping than #aking &! in higher concentrations during sleep than during #aking hours '! in higher concentrations #hen asleep than #hen a#ake *:*. =he increased popularity and availability of televisions has led to the decline of regional dialects% language variations #hich originate from diverse ethnic and cultural heritages and perpetuated by geographic isolation. A! #hich originate from diverse ethnic and cultural heritages and perpetuated "! that originated from diverse ethnic and cultural heritages and perpetuated $! originated from diverse ethnic and cultural heritages and perpetuated

SC Strategy

'E$

&! originating from diverse ethnic and cultural heritages and perpetuated '! originating from diverse ethnic and cultural heritages and perpetuating *:0. =he investor #ho is uncertain about the future is more likely to put money into blue-chip stocks or treasury bills than into gold. A! than into "! than they do $! than they are &! as into '! as *:). =he Iro5uois #ere primarily planters% but supplementing their cultivation of mai9e% s5uash% and beans #ith fishing and hunting. A! but supplementing "! and had supplemented $! and even though they supplemented &! although they supplemented '! but #ith supplementing **2. =he key to control over the 'urasian steppes lay in the nomad3s ability to use the horse both as a means of transport but also as an effective military tool. A! but also as "! or as $! and as &! or '! and also **1. =he labor agreement permits staff reductions through attrition #ith increased pension benefits and a special early-retirement program for speeding it up. A! attrition #ith increased pension benefits and a special early-retirement program for speeding it up "! attrition and provides increased pension benefits and a special early-retirement program to speed the attrition process $! attrition% #hich #ill be speeded up by providing increased pension benefits and a special early-retirement program &! attrition% #hich% by their providing increased pension benefits and a special early-retirement program% #ill speed the process '! attrition% #hich provides increased pension benefits and a special early-retirement program for speeding the attrition process

'E'

**(.

=he lack of complete historical records from the mid-to-late 10223s have made some "lack inventions difficult to trace to their originators. A! have made some "lack inventions difficult to trace to their originators "! have made for difficulties in tracing some inventions by "lacks to their originators $! have made it difficult to trace some inventions by "lacks to their originators &! has made it difficult to trace some inventions to their "lack originators '! has made it difficult in tracing some "lack inventions to their originators

**/.

=he last #ild Indian in North America% according to anthropologist Alfred Froeber% #as the lone survivor of $alifornia3s lost @ahi tribe% #hich staggered out of the mountains near .assen 1eak in 1)1(% deep in mourning for the last of his companions% e<pecting to be butchered and eaten by #hite ranchers. A! #hich "! #ho $! that &! the survivor having '! having

**4.

=he la#yer for the defense charged that she suspected the police of having illegally taped her confidential conversations #ith her client and then used the information obtained to find evidence supporting their murder charges. A! used the information obtained to find evidence supporting "! used such information as they obtained to find evidence supporting $! used the information they had obtained to find evidence that #ould support &! of using the information they had obtained to find evidence that #ould support '! of using such information as they obtained to find evidence that #ould be supportive of

**7.

=he main reasons for the phenomenal gro#th of agribusiness over the last fifteen years are because of elevated land prices% the basing of retail marketing net#orks on produce imported from other states% and decreased competition from developers. A! because of elevated land prices% the basing of retail marketing net#orks "! a result of elevated land prices% retail marketing net#orks based $! that land prices are elevated% retail marketing net#orks are based &! elevated land prices% retail marketing net#orks based '! because land prices are elevated% the basing of retail marketing net#orks

**:.

=he ma6or areas of medicine in #hich lasers are effective is in the cutting and closing of blood vessels% and in the destruction of tumors. A! is in the cutting and closing of blood vessels% and in the destruction

SC Strategy

'E*

"! are the cutting and closing of blood vessels% and also the case of destroying $! are the cutting% closing of blood vessels% and in the destroying &! are the cutting and closing of blood vessels% and the destruction '! is in the cutting and closing of blood vessels% and the destroying ***. =he man #as al#ays a#are% sometimes proudly and sometimes resentfully% that he #as a small-to#n 8id#esterner #ho #as thrust into a #orld that #as dominated by #ealthier% bettereducated% and more polished people than him. A! #ho #as thrust into a #orld that #as dominated by #ealthier% better-educated% and more polished people than him "! #ho had been thrust into a #orld that #as dominated by more #ealthy% educated% and polished people than him $! #ho had been thrust into a #orld dominated by #ealthier% better-educated% and people more polished than he #as &! thrust into a #orld dominated by more #ealthy% educated% and polished people than him '! thrust into a #orld dominated by #ealthier% better-educated% and more polished people than he **0. =he medieval scholar made almost no attempt to investigate the anatomy of plants% their mechanisms of gro#th% nor the #ays #here each #as related to the other. A! nor the #ays #here each #as related to the other "! nor ho# each #as related to some other $! or the #ay #here one is related to the ne<t &! or the #ays in #hich they are related to one another '! or the #ays that each related to some other **). =he metabolic rate of sharks is lo# compared #ith the rates of most other fishes. A! #ith the rates of most other fishes "! to most other fishes3 rate $! to that of rates for most other fishes &! to most other fishes '! #ith most other fishes *02. =he mistakes children make in learning to speak tell linguists more about ho# they learn language than the correct forms they use. A! ho# they learn language than "! ho# one learns language than $! ho# children learn language than do &! learning language than '! their language learning than do

'E7

*01.

=he moderni9ation program for the steel mill #ill cost appro<imately 71 million dollars% #hich it is hoped can be completed in the late 1)023s. A! =he moderni9ation program for the steel mill #ill cost appro<imately 71 million dollars% #hich it is hoped can be completed in the late 1)023s. "! =he moderni9ation program for the steel mill% hopefully completed in the late 1)023s% #ill cost appro<imately 71 million dollars. $! 8oderni9ing the steel mill% hopefully to be completed in the late 1)023s% #ill cost appro<imately 71 million dollars. &! =he program for moderni9ing the steel mill% #hich can% it is hoped% be completed in the late 1)023s and cost appro<imately 71 million dollars. '! 8oderni9ing the steel mill% a program that can% it is hoped% be completed in the late 1)023s% #ill cost appro<imately 71 million dollars.

*0(.

=he most common reasons for an employee3s un#illingness to accept a transfer are that mortgage rates are high% housing in the ne# location costs more% and the difficulty of selling the old home. A! that mortgage rates are high% housing in the ne# location costs more% and the difficulty of selling the old home "! that mortgage rates are high% housing in the ne# location costs more% and that it is difficult to sell the old home $! high mortgage rates% the greater cost of housing in the ne# location% and that the old home is difficult to sell &! high mortgage rates% the greater cost of housing in the ne# location% and it is difficult to sell the old home '! high mortgage rates% the greater cost of housing in the ne# location% and the difficulty of selling the old home

*0/.

=he most favorable locations for the gro#th of glaciers% rather than being the cold% dry polar regions% #ould be instead the cool% moist middle latitudes% #here there is abundant precipitation and #here it is cold enough to allo# some sno# to accumulate year by year. A! glaciers% rather than being the cold% dry polar regions% #ould be instead the cool% moist middle latitudes "! glaciers are not the cold% dry polar regions but the cool% moist middle latitudes $! glaciers are the cool% moist middle latitudes rather than the cold% dry polar regions &! glaciers% instead of being the cold% dry polar regions% #ould be the cool% moist middle latitudes '! glaciers are% instead of the cold% dry polar regions% rather the cool% moist middle latitudes

*04.

=he nation3s three military academies have seen a dramatic rise in applications% one fueled by a resurgence of patriotism% increasing tuition costs at private colleges% and improved recruiting by the academies.

SC Strategy

'E8

A! one fueled by a resurgence of patriotism% increasing tuition costs at private colleges% and improved recruiting by the academies "! one fueled by a resurgence of patriotism% tuition costs that have increased at private colleges% and academies improving their recruiting $! one fueled by a resurgence of patriotism% private colleges that increased their tuition costs% and recruiting improvements by the academies &! fueled by a resurgence of patriotism% tuition costs increasing at private colleges% and academies improving their recruiting '! fueled by a resurgence of patriotism% increasing tuition costs at private colleges% and academies improving their recruiting *07. =he National .abor Delations Act e<pressly forbids unions from engaging in secondary boycotts against companies not directly involved in a labor dispute. A! unions from engaging in "! the engagement by unions of $! unions to engage in &! unions from becoming engaged #ith '! that unions engage upon *0:. =he National =ransportation ,afety "oard has recommended the use of fail-safe mechanisms on airliner cargo door latches assuring the doors are properly closed before takeoff and to prevent them from popping open in flight. A! assuring the doors are properly closed "! for the assurance of proper closing $! assuring proper closure &! to assure closing the doors properly '! to assure that the doors are properly closed *0*. =he nephe# of 1liny the 'lder #rote the only eye#itness account of the great eruption of Besuvius in t#o letters to the historian =acitus. A! =he nephe# of 1liny the 'lder #rote the only eye#itness account of the great eruption of Besuvius in t#o letters to the historian =acitus. "! =o the historian =acitus% the nephe# of 1liny the 'lder #rote t#o letters% being the only eye#itness accounts of the great eruption of Besuvius. $! =he only eye#itness account is in t#o letters by the nephe# of 1liny the 'lder #riting to the historian =acitus an account of the great eruption of Besuvius. &! Writing the only eye#itness account% 1liny the 'lder3s nephe# accounted for the great eruption of Besuvius in t#o letters to the historian =acitus.

'E9

'! In t#o letters to the historian =acitus% the nephe# of 1liny the 'lder #rote the only eye#itness account of the great eruption of Besuvius. *00. =he ne# contract forbids a strike by the transportation union. A! forbids a strike by the transportation union "! forbids the transportation union from striking $! forbids that there be a strike by the transportation union &! #ill forbid the transportation union from striking '! #ill forbid that the transportation union strikes *0). =he ne# regulations mandate that a company allo#s their retiring employees #ho #ould other#ise lose group health care coverage to continue the same insurance at their o#n e<pense for a specified period. A! that a company allo#s their retiring employees #ho #ould other#ise lose group health care coverage to continue "! companies to allo# their retiring employees #ho #ould other#ise lose group health care coverage that they can continue $! that a company allo# its retiring employees #ho #ould other#ise lose group health care coverage to continue &! companies allo#ing a retiring employee #hose group health care coverage #ould other#ise be lost the continuation of '! companies to allo# a retiring employee #hose group health care coverage #ould other#ise be lost he continuation of *)2. =he number of mountain gorillas is declining #ith such rapidity that the population is one-half in the t#enty years bet#een a count made by -eorge ,challer in 1):2 and the one made by &ian ?ossey in 1)02. A! #ith such rapidity that the population is one-half "! #ith such rapidity that the population #as one-half $! so rapidly the population divided in half &! so rapidly that the population #as halved '! in such rapidity that the population is halved *)1. =he number of undergraduate degrees in engineering a#arded by colleges and universities in the +nited ,tates increased by more than t#ice from 1)*0 to 1)07. A! increased by more than t#ice "! increased more than t#o times $! more than doubled &! #as more than doubled '! had more than doubled

SC Strategy

'E;

*)(.

=he odds are about 4 to 1 against surviving a takeover offer% and many business consultants therefore advise that a company3s first line of defense in eluding offers like these be to even refuse to take calls from likely corporate raiders. A! that a company3s first line of defense in eluding offers like these be to even refuse "! that a company3s first line of defense in eluding such offers be to refuse even $! a company defending itself against offers of this kind that% as a first line of defense% they should even refuse &! companies #hich are defending themselves against such an offer that% as a first line of defense% they should even refuse '! that the first line of defense for a company #ho is eluding offers like these is the refusal even

*)/.

=he official languages are of India% >indi% and of 1akistan% +rdu% but neither are spoken by a ma6ority of the population. A! =he official languages are of India% >indi% and of 1akistan% +rdu% but neither are "! =he official languages are of India% >indi% and of 1akistan% +rdu% but neither is $! Gfficially% the languages are >indi for India and for 1akistan% +rdu% but neither are &! =he official language of India is >indi% and that of 1akistan is +rdu% but neither is '! =he official language of India is >indi% and +rdu in 1akistan% but none is

*)4.

=he Glympic -ames helped to keep peace among the pugnacious states of the -reek #orld in that a sacred truce #as proclaimed during the festival3s month. A! #orld in that a sacred truce #as proclaimed during the festival3s month "! #orld% proclaiming a sacred truce during the festival3s month $! #orld #hen they proclaimed a sacred truce for the festival month &! #orld% for a sacred truce #as proclaimed during the month of the festival '! #orld by proclamation of a sacred truce that #as for the month of the festival

*)7.

=he only #ay for gro#ers to salvage fro9en citrus is to process them 5uickly into 6uice concentrate before they rot #hen #armer #eather returns. A! to process them 5uickly into 6uice concentrate before they rot #hen #armer #eather returns "! if they are 5uickly processed into 6uice concentrate before #armer #eather returns to rot them $! for them to be processed 5uickly into 6uice concentrate before the fruit rots #hen #armer #eather returns &! if the fruit is 5uickly processed into 6uice concentrate before they rot #hen #armer #eather returns '! to have it 5uickly processed into 6uice concentrate before #armer #eather returns and rots the fruit

'EI

*):.

=he ordinance is intended to force householders to separate such ha9ardous #aste like pesticides% batteries% fertili9ers% and oil-based paints from the general stream of household trash. A! to separate such ha9ardous #aste like "! that they should separate such ha9ardous #aste like $! separating such ha9ardous #astes as &! that they should separate such ha9ardous #astes as '! to separate such ha9ardous #astes as

*)*.

=he original building and loan associations #ere organi9ed as limited life funds% #hose members made monthly payments on their share subscriptions% then taking turns dra#ing on the funds for home mortgages. A! subscriptions% then taking turns dra#ing "! subscriptions% and then taking turns dra#ing $! subscriptions and then took turns dra#ing &! subscriptions and then took turns% they dre# '! subscriptions and then dre#% taking turns

*)0.

=he original employees hired% #ho had been there over t#enty years% #ere fiercely loyal to the firm% and it offered no retirement benefits or profit sharing to any employees. A! and it offered no "! and it offered neither $! still it offered no &! though it offered no '! though it offered neither

*)).

=he o#ner of ,teele3s -rocery in Gsage% Ghio% saved I:22 monthly on heat during the #inter by putting all his refrigerator air compressors together in an insulated compartment% then installing t#o fans and a duct that carried #aste heat from the compressors into the main part of the store. A! then installing t#o fans and a duct that carried "! then he installing t#o fans and a duct that carried $! then t#o fans #ere installed #ith a duct that carried &! installing t#o fans% and then carrying through a duct '! installing t#o fans% and a duct carrying

022.

=he 1arthenon #as a church from 1(24 until 147:% #hen Athens #as taken by -eneral 8ohammed the $on5ueror% the =urkish sultan% #ho established a mos5ue in the building and used the Acropolis as a fortress. A! #ho established a mos5ue in the building and used the Acropolis as

SC Strategy

'EJ

"! #ho% establishing a mos5ue in the building% used the Acropolis like $! #ho% #hen he had established a mos5ue in the building% used the Acropolis like &! #ho had established a mos5ue in the building% using the Acropolis to be '! establishing a mos5ue in the building and using the Acropolis as 021. =he pattern of #hisker spot on the face of a male lion% like human fingerprints% are a lifelong means of identification% since they are both uni5ue and unchanging. A! like human fingerprints% are a lifelong means of identification% since they are both uni5ue and unchanging "! like human fingerprints% is a lifelong means of identification% since it is both uni5ue and unchanging $! like human fingerprints% is a means of identification for life% being both uni5ue and unchanging &! since they are both uni5ue and unchanging% are% like human fingerprints% are a means of identification for life '! both uni5ue and unchanging% are% like human fingerprints% a lifelong means of identification 02(. =he peaks of a mountain range% acting like rocks in a streambed% produce ripples in the air flo#ing over themA the resulting flo# pattern% #ith crests and troughs that remain stationary although the air that forms them is moving rapidly% are kno#n as standing #aves. A! crests and troughs that remain stationary although the air that forms them is moving rapidly% are "! crests and troughs that remain stationary although they are formed by rapidly moving air% are $! crests and troughs that remain stationary although the air that forms them is moving rapidly% is &! stationary crests and troughs although he air that forms them is moving rapidly% are '! stationary crests and troughs although they are formed by rapidly moving air% is 02/. =he period #hen the great painted caves at .ascau< and Altamira #ere occupied by +pper 1aleolithic people has been established by carbon-14 dating% but #hat is much more difficult to determine are the reason for their decoration% the use to #hich primitive people put the caves% and the meaning of the magnificently depicted animals. A! has been established by carbon-14 dating% but #hat is much more difficult to determine are "! has been established by carbon-14 dating% but #hat is much more difficult to determine is $! have been established by carbon-14 dating% but #hat is much more difficult to determine is &! have been established by carbon-14 dating% but #hat is much more difficult to determine are '! are established by carbon-14 dating% but that #hich is much more difficult to determine is 024. =he physical structure of the human eye enables it to sense light of #avelengths up to 2.2227 millimetersE infrared radiation% ho#ever% is invisible because its #avelengthH2.1 millimetersHis too long to be registered by the eye.

'$E

A! infrared radiation% ho#ever% is invisible because its #avelengthH2.1 millimetersHis too long to be registered by the eye "! ho#ever% the #avelength of infrared radiationH2.1 millimetersHis too long to be registered by the eye making it invisible $! infrared radiation% ho#ever% is invisible because its #avelengthH2.1 millimetersHis too long for the eye to register it &! ho#ever% because the #avelength of infrared radiation is 2.1 millimeters% it is too long for the eye to register and thus invisible '! ho#ever% infrared radiation has a #avelength of 2.1 millimeters that is too long for the eye to register% thus making it invisible 027. =he plot of =he "ostonians centers on the rivalry bet#een Glive $hancellor% an active feminist% #ith her charming and cynical cousin% "asil Dansom% #hen they find themselves dra#n to the same radiant young #oman #hose talent for public speaking has #on her an ardent follo#ing. A! rivalry bet#een Glive $hancellor% an active feminist% #ith her charming and cynical cousin% "asil Dansom "! rivals Glive $hancellor% an active feminist% against her charming and cynical cousin% "asil Dansom $! rivalry that develops bet#een Glive $hancellor% an active feminist% and "asil Dansom% her charming and cynical cousin &! developing rivalry bet#een Glive $hancellor% an active feminist% #ith "asil Dansom% her charming and cynical cousin '! active feminist% Glive $hancellor% and the rivalry #ith her charming and cynical cousin "asil Dansom 02:. =he president of the block association tried to convince her neighbors they should 6oin forces to prevent crime in the neighborhood rather than continuing to be victimi9ed. A! they should 6oin forces to prevent crime in the neighborhood rather than continuing to be victimi9ed "! that they should 6oin forces to prevent crime in the neighborhood rather than continue to be victimi9ed $! about 6oining forces to prevent crime in the neighborhood instead of continuing to be victimi9ed &! for the 6oining of forces to prevent crime in the neighborhood rather than continue to be victimi9ed '! to 6oin forces to prevent crime in the neighborhood rather than continuing to be victimi9ed 02*. =he prime lending rate is a key rate in the economyA not only are the interest rates on most loans to small and medium-si9ed businesses tied to the prime% but also on a gro#ing number of

SC Strategy

'$$

consumer loans% including home e5uity loans. A! not only are the interest rates on most loans to small and medium-si9ed businesses tied to the prime% but also on "! tied to the prime are the interest rates not only on most loans to small and medium-si9ed businesses% but also on $! the interest rates not only on most loans to small and medium-si9ed businesses are tied to the prime% but also &! not only the interest rates on most loans to small and medium-si9ed businesses are tied to the prime% but also on '! the interest rates are tied to the prime% not only on most loans to small and medium-si9ed businesses% but also 020. =he proposed health care bill #ould increase government regulation of health insurance% establish standards that #ould guarantee #ider access to people #ith past health problems and to #orkers changing 6obs #ho other#ise could be uncovered for months. A! establish standards that #ould guarantee #ider access to people #ith past health problems and to #orkers changing 6obs #ho "! establishing standards that #ould guarantee #ider access to people #ith past health problems and to #orkers #ho are changing 6obs and $! to establish standards that #ould guarantee #ider access to people #ith past health problems and to #orkers #ho change 6obs that &! for establishing standards that #ould guarantee #ider access for people #ith past health problems and #orkers changing 6obs #ho '! for the establishment of standards that #ould guarantee #ider access for people #ith past health problems and #orkers #ho are changing 6obs that 02). =he proposed urban development 9ones do not represent a ne# principleE it #as employed in Gperation "ootstrap in 1uerto Dico. A! do not represent a ne# principleE it "! represent not a ne# principle% but one that $! are not a ne# principleE the same one &! are not a ne# principle% but one that '! are not ne# in principleE it 012. =he prospect of a ne# #ave of automobile imports has prompted domestic manufacturers to reduce staff% close plants% and offering buyers financial incentives so they stay competitive. A! reduce staff% close plants% and offering buyers financial incentives so they "! reduce staff% close plants% and offer financial incentives to buyers in order to $! reducing staff% closing plants% and the offer of financial incentives to buyers so they can

'$'

&! staff reductions% closing plants% and offering buyers financial incentives in order to '! a reduction of staff% plant closings% and offering financial incentives to buyers to 011. =he psychologist William Cames believed that facial e<pressions not only provide a visible sign of an emotion% actually contributing to the feeling itself. A! emotion% actually contributing to the feeling itself "! emotion but also actually contributing to the feeling itself $! emotion but also actually contribute to the feeling itself &! emotionE they also actually contribute to the feeling of it '! emotionE the feeling itself is also actually contributed to by them 01(. =he ;uechuans believed that all things participated in both the material level and the mystical level of reality% and many individual ;uechuans claimed to have contact #ith it directly #ith an ichana dream! e<perience. A! contact #ith it directly #ith "! direct contact #ith it by #ay of $! contact #ith the latter directly through &! direct contact #ith the latter by means of '! contact directly #ith the mystical level due to 01/. =he 5uestion of #hether to divest themselves of stock in companies that do business in ,outh Africa is particularly troublesome for the nation3s 11: private "lack colleges because their economic bases are often more fragile than most predominantly White colleges. A! than "! than those of $! than is so of &! compared to '! compared to those of 014. =he recent surge in the number of airplane flights has clogged the nation3s air-traffic control system% to lead to 77 percent more delays at airports% and prompts fears among some officials that safety is being compromised. A! to lead to 77 percent more delays at airports% and prompts "! leading to 77 percent more delay at airports and prompting $! to lead to a 77 percent increase in delay at airports and prompt &! to lead to an increase of 77 percent in delays at airports% and prompted '! leading to a 77-percent increase in delays at airports and prompting 017. =he record of the past is al#ays incomplete% and the historian #ho #rites about it inevitably reflects the preoccupations of their o#n time.

SC Strategy

'$*

A! the historian #ho #rites about it inevitably reflects "! the historian #riting about it #ill inevitably reflect $! a historian #riting about it inevitably reflects &! #riting about it% it is inevitable for historians to reflect '! historians in #riting about it inevitably reflect 01:. =he relationship bet#een corpulence and disease remain controversial% although statistics clearly associate a reduced life e<pectancy #ith chronic obesity. A! remain controversial% although statistics clearly associate a reduced life e<pectancy #ith "! remain controversial% although statistics clearly associates a reduced life e<pectancy #ith $! remain controversial% although statistics clearly associates reduced life e<pectancy to &! remains controversial% although statistics clearly associate a reduced life e<pectancy #ith '! remains controversial% although statistics clearly associates reduced life e<pectancy to 01*. =he report on the gross national productHthe nation3s total production of goods and services Hsho#ed that second-5uarter inflation #as some#hat lo#er than a previous estimation and the savings rate slightly higher. A! a previous estimation and the savings rate slightly higher "! a previous estimation and #ith a slightly higher savings rate $! a previous estimate and that the savings rate is slightly higher &! previously estimated and a slightly higher savings rate '! previously estimated and that the savings rate #as slightly higher 010. =he reports from the &epartment of $ommerce indicated that the economy had gro#n at an annual rate much higher than most economists had predicted may occur. A! had predicted may occur "! had predicted $! predicted the occurrence of &! predicted may occur '! predicted 01). =he residents3 opposition to the spraying program has rekindled an old debate among those #ho oppose the use of pesticides and those #ho feel that the pesticides are necessary to save the trees. A! among those #ho oppose the use of pesticides and "! bet#een those #ho oppose the use of pesticides and $! among those opposing the use of pesticides #ith &! bet#een those #ho oppose the use of pesticides #ith '! among those opposing the use of pesticides and

'$7

0(2.

=he rise in the $ommerce &epartment3s inde< of leading economic indicators suggest that the economy should continue its e<pansion into the coming months% but that the mi<ed performance of the inde<3s individual components indicates that economic gro#th #ill proceed at a more moderate pace than in the first 5uarter of this year. A! suggest that the economy should continue its e<pansion into the coming months% but that "! suggest that the economy is to continue e<pansion in the coming months% but $! suggests that the economy #ill continue its e<panding in the coming months% but that &! suggests that the economy is continuing to e<pand into the coming months% but that '! suggests that the economy #ill continue to e<pand in the coming months% but

0(1.

=he rising of costs of data-processing operations at many financial institutions has created a gro#ing opportunity for independent companies to provide these services more efficiently and at lo#er cost. A! =he rising of costs "! Dising costs $! =he rising cost &! "ecause the rising cost '! "ecause of rising costs

0((.

=he root systems of most flo#ering perennials either become too cro#ded% #hich results in loss in vigor% and spread too far out#ard% producing a bare center. A! #hich results in loss in vigor% and spread "! resulting in loss in vigor% or spreading $! #ith the result of loss of vigor% or spreading &! resulting in loss of vigor% or spread '! #ith a resulting loss of vigor% and spread

0(/.

=he Dorsch9ch test is gaining ne# respect as a diagnostic tool because it takes only one hour to e<pose behavior and thought processes that may be unlikely to emerge in other procedures or #eeks of ordinary intervie#ing. A! that may be unlikely to emerge in other procedures or #eeks of ordinary intervie#ing "! #hose emergence is unlikely in other procedures or #eeks of ordinary intervie#s $! that might not emerge in other procedures or in #eeks of ordinary intervie#s &! that may not emerge under other procedures or #eeks of ordinary intervie#s '! unlikely not to emerge during #eeks of ordinary intervie#ing or in other procedures

0(4.

=he rules that govern political contributions are less stringent in local elections than they are in national elections because they typically involve smaller amounts of money and present less opportunity for abuse.

SC Strategy

'$8

A! =he rules that govern political contributions are less stringent in local elections than they are in national elections because they typically involve smaller amounts of money and present less opportunity for abuse. "! "ecause they typically involve smaller amounts of money and present less opportunity for abuse% the rules that govern political contributions are less stringent in local elections than the rules are less stringent in local elections than the rules are in national elections. $! =he rules that govern political contributions are less stringent in local elections than national elections because they typically involve smaller amounts of money and present less opportunity for abuse. &! "ecause local elections typically involve smaller amounts of money and present less opportunity for abuse than national elections% the rules that govern local political contributions are less stringent than national contributions. '! =he rules that govern political contributions are less stringent in local elections than they are in national elections because local elections typically involve smaller amounts of money and present less opportunity for abuse. 0(7. =he sale of government surplus machinery #ill begin at ) a.m. and continue until the supply lasts. A! #ill begin at ) a.m. and continue until the supply lasts "! begins at ) a.m.% continuing until the supply lasts $! #ill begin at ) a.m. and% until the supply lasts% #ill continue &! begins at ) a.m. and% as long as the supply may last% it continues '! #ill begin at ) a.m. and continue as long as the supply lasts 0(:. =he school board ruling mandating that physically handicapped students be placed in regular classroom settings #henever possible also assured all children #ho have a reading problem of special aid. A! be placed in regular classroom settings #henever possible also assured all children #ho have a reading problem "! should be placed in regular classroom settings #henever possible also assures all children that have a reading problem $! are placed in regular classroom settings #henever possible also assures those children #ho are having reading problems &! be placed in regular classroom settings #henever possible also assured children #ith reading problems '! should be placed in regular classroom settings #henever possible also has assured all those children #ith a reading problem 0(*. =he science of economics% #hich for four decades #as dominated by Feynesians% #ho at first stressed the government3s role in stimulating the economy% but #ho #ere ultimately led a#ay

'$9

from solutions based on government intervention. A! economics% #hich for four decades #as "! economics that #as to be $! economics% one #hich has% for four decades% been &! economics is one that for four decades has been '! economics% for four decades% is one that #as 0(0. =he ,enate approved immigration legislation that #ould grant permanent residency to millions of aliens currently residing here and if employers hired illegal aliens they #ould be penali9ed. A! if employers hired illegal aliens they #ould be penali9ed "! hiring illegal aliens #ould be a penalty for employers $! penali9e employers #ho hire illegal aliens &! penali9ing employers hiring illegal aliens '! employers to be penali9ed for hiring illegal aliens 0(). =he speculative fever of the Doaring =#enties infected rich and poor alikeE vast 5uantities of people #ere dangerously overe<tended% credit #as absurdly easy to obtain% and most brokerage houses re5uired only ten percent cash for stocks bought on margin. A! rich and poor alikeE vast 5uantities of people #ere dangerously overe<tended "! both rich and poor alikeE large amounts of people dangerously overe<tended themselves $! rich and poor alikeE great numbers of people #ere dangerously overe<tended &! both rich and poor alikeE vast amounts of people dangerously overe<tended themselves '! both rich and poorE great 5uantities of people #ere dangerously overe<tended 0/2. =he spraying of pesticides can be carefully planned% but accidents% #eather conditions that could not be foreseen% and pilot errors often cause much larger deposits of spray than they had anticipated. A! #eather conditions that could not be foreseen% and pilot errors often cause much larger deposits of spray than they had "! #eather conditions that cannot be foreseen% and pilot errors often cause much larger deposits of spray than $! unforeseeable #eather conditions% and pilot errors are the cause of much larger deposits of spray than they had &! #eather conditions that are not foreseeable% and pilot errors often cause much larger deposits of spray than '! unforeseeable #eather conditions% and pilot errors often cause much larger deposits of spray than they had 0/1. =he study undertaken by &epartment of the Interior #ill involve e<amination and tagging of

SC Strategy

'$;

the $alifornia condor in order to obtain information about their daily movements% foraging habits% and sites #here they nest. A! their daily movements% foraging habits% and sites #here they nest "! their daily movements% foraging habits% and their nesting sites $! its daily movements% foraging habits% and nesting sites &! its daily movements% foraging habits% and about nesting sites '! daily movements% foraging habits% and sites in #hich there are nests 0/(. =he supply of oil being finite has become an economical and political consideration of the first magnitude for all modern industrial nations. A! =he supply of oil being finite has become an economical "! =he finite supply of oil has become an economical $! =hat the supply of oil is finite has become an economical &! =he supply of oil being finite has become an economic '! =hat the supply of oil is finite has become an economic 0//. =he ,upreme $ourt3s concern #ith legitimacy is not for the sake of the court but the nation to #hich it is responsible. A! but the nation to #hich it is responsible "! but for the sake of the nation to #hich it is responsible $! so much as the nation it is responsible to &! as the nation it is responsible to '! but the nation 0/4. =he suspect in the burglary #as advised of his right to remain silent% told he could not leave% and #as interrogated in a detention room. A! of his right to remain silent% told he could not leave% and #as "! of his right to remain silent% told he could not leave% and $! of his right to remain silent and that he could not leave and &! that he had a right to remain silent% could not leave% and #as '! that he had a right to remain silent% that he could not leave% and #as 0/7. =he task force is revie#ing the company3s hiring practices for the determination of #hether they are meeting the re5uirements set by the Gffice of '5ual Gpportunity. A! for the determination of #hether they are meeting the re5uirements set by the Gffice of '5ual Gpportunity "! for the determining of #hether or not it meets the re5uirement set by the Gffice of '5ual Gpportunity $! for the determining of #hether the re5uirements set by the Gffice of '5ual Gpportunity are

'$I

being met or not &! determining #hether the re5uirements set by the Gffice of '5ual Gpportunity are met '! to determine #hether they meet the re5uirements set by the Gffice of '5ual Gpportunity 0/:. =he technical term pagination is a process that leaves editors% instead of printers% assemble the page images that become the metal or plastic plates used in printing. A! is a process that leaves editors% instead of printers% assemble "! refers to a process that allo#s editors% rather than printers% to assemble $! is a process leaving the editors% rather than printers% to assemble &! refers to a process #hich allo#s editors% but not to printers% the assembly of '! has reference to the process leaving to editors% instead of the printer% assembling 0/*. =he themes that Dita &ove e<plores in her poetry is universal% encompassing much of the human condition #hile occasionally deals #ith racial issues. A! is universal% encompassing much of the human condition #hile occasionally deals "! is universal% encompassing much of the human condition% also occasionally it deals $! are universal% they encompass much of the human condition and occasionally deals &! are universal% encompassing much of the human condition #hile occasionally dealing '! are universal% they encompass much of the human condition% also occasionally are dealing 0/0. =he underlying physical principles that control the midair gyrations of divers and gymnasts are the same as the body orientation controlling astronauts in a #eightless environment. A! as the body orientation controlling "! as the body orientation #hich controls $! as those controlling the body orientation of &! ones to control the body orientation of '! ones used in controlling the body orientation of 0/). =he +nited ,tates government employs a much larger proportion of #omen in trade negotiations than any government. A! a much larger proportion of #omen in trade negotiations than any "! a much larger proportion of #omen in trade negotiations than does any other $! much larger proportions of #omen in trade negotiations than has any &! proportions of #omen in trade negotiations that are much larger than any '! proportions of #omen in trade negotiations that are much larger than any other 042. =he +nited ,tates petroleum industry3s cost to meet environmental regulations is pro6ected at ten percent of the price per barrel of refined petroleum by the end of the decade. A! =he +nited ,tates petroleum industry3s cost to meet environmental regulations is pro6ected at

SC Strategy

'$J

ten percent of the price per barrel of refined petroleum by the end of the decade. "! =he +nited ,tates petroleum industry3s cost by the end of the decade to meet environmental regulations is estimated at ten percent of the price per barrel of refined petroleum. $! "y the end of the decade% the +nited ,tates petroleum industry3s cost of meeting environmental regulations is pro6ected at ten percent of the price per barrel of refined petroleum. &! =o meet environmental regulations% the cost to the +nited ,tates petroleum industry is estimated at ten percent of the price per barrel of refined petroleum by the end of the decade. '! It is estimated that by the end of the decade the cost to the +nited ,tates petroleum industry of meeting environmental regulations #ill be ten percent of the price per barrel of refined petroleum. 041. =he unprecedented increases in the prime lending rate this year has probably been brought about by business community3s uncertainty about the 1resident3s position on the budget deficit. A! in the prime lending rate this year has "! this year in the prime lending rate has $! this year in the prime lending rate having &! in the prime lending rate this year had '! in the prime lending rate this year have 04(. =he unskilled #orkers at the Allenby plant reali9ed that their hourly rate of I4.11 to I4.*7 #as better than many nearby factory #ages. A! many nearby factory #ages "! many #ages in nearby factories $! #hat are offered by many nearby factories &! it is in many nearby factories '! that offered by many nearby factories 04/. ago. A! e5ually e<tensive or more so than ten years ago "! e5ual to or more e<tensive than ten years ago $! as e<tensive as ten years ago or more &! e5ual to% if not more% than ten years ago '! as e<tensive as it #as ten years ago% if not more so 044. =he use of gravity #aves% #hich do not interact #ith matter in the #ay electromagnetic #aves do% hopefully #ill enable astronomers to study the actual formation of black holes and neutron stars. A! in the #ay electromagnetic #aves do% hopefully #ill enable =he use of chemical pesticides in this country is e5ually e<tensive or more so than ten years

''E

"! in the #ay electromagnetic #aves do% #ill% it is hoped% enable $! like electromagnetic #aves% hopefully #ill enable &! like electromagnetic #aves% #ould enable% hopefully '! such as electromagnetic #aves do% #ill% it is hoped% enable 047. =he utility company has announced that it #ill permanently close its +nit I nuclear po#er plant% the first plant that had been built by private industry and the model for a generation of modern nuclear reactors. A! that had been built by private industry and "! built by private industry and #hich #as $! to be built by private industry and #hich #as &! built by private industry and '! to have been built by private industry and #as 04:. =he visiting pharmacologists concluded that the present amalgam of $hinese and Western medicine is probably as good% or better than% any system that might be devised for the patients #ho are treated at the Nan Fai hospital in =ian-6ing. A! as good% or better than% any system that might be devised for the patients #ho are "! as good% or better% than any system that might be devised for patients being $! as good% or better than% any system that might be devised for patients #hich are being &! good as% or even better than% any other system that may be devised for the patients #ho are '! as good as% or better than% any other system that might be devised for the patients 04*. =he voluminous personal papers of =homas Alva 'dison reveal that his inventions typically sprang to life not in a flash of inspiration but evolved slo#ly from previous #orks. A! sprang to life not in a flash of inspiration but evolved slo#ly "! sprang to life not in a flash of inspiration but #ere slo#ly evolved $! did not spring to life in a flash of inspiration but evolved slo#ly &! did not spring to life in a flash of inspiration but had slo#ly evolved '! did not spring to life in a flash of inspiration but they #ere slo#ly evolved 040. =he Wallerstein study indicates that even after a decade young men and #omen still e<perience some of the effects of a divorce occurring #hen a child. A! occurring #hen a child "! occurring #hen children $! that occurred #hen a child &! that occurred #hen they #ere children '! that has occurred as each #as a child 04). =he Western #orld3s love affair #ith chocolate is #ell-documentedA fe# people have been

SC Strategy

''$

kno#n to have tasted it for the first time #ithout re5uesting more. A! fe# people have been kno#n to have tasted it "! fe# having been kno#n to taste it $! it has been tasted by fe# people &! fe# people have been kno#n to taste it '! fe# people having tasted it 072. =he #inds that ho#l across the -reat 1lains not only blo# a#ay valuable topsoil% thereby reducing the potential crop yield of a tract of land% and also damage or destroy young plants. A! and also damage or destroy "! as #ell as damaging or destroying $! but they also cause damage or destroy &! but also damage or destroy '! but also causing damage or destroying 071. =he #ork of mathematician Doger 1enrose in the early 1)*2s% on the geometry of #hat are called aperiodic tiles% turned out to describe the architecture of a previously unkno#n class of crystals. A! #hat are called aperiodic tiles% turned out to describe "! #hat is called aperiodic tiles% describes $! aperiodic tiles% describing &! so-called aperiodic tiles% describe '! aperiodic tiles% it turned out to describe 07(. =here are more than forty ne#spapers published in the cities of Ferala% a state on the 8alabar $oast% #hich reflects the fact that Feralans are by far India3s most literate citi9ens. A! #hich reflects "! and that number reflects $! #hich reflect &! that number reflects '! that reflects 07/. =here has been a /2- to 42-fold increase in the incidence of malaria caused by increasing mos5uito resistance against pesticides. A! increase in the incidence of malaria caused by increasing mos5uito resistance against "! increase in the incidence of malaria because of increasing resistance of mos5uitoes to $! increasing malaria incidence because of increasing resistance of mos5uitoes to &! incidence of malaria increase caused by increasing mos5uito resistance against '! incidence of malaria increase because of increased mos5uito resistance to

'''

074.

=here is ample evidence% derived from the lore of traditional folk medicine% that naturally occurring antibiotics are usually able to be modified to make them a more effective drug. A! are usually able to be modified to make them a more effective drug. "! are usually able to be modified to make them more effective drugs $! are usually able to be modified% #hich makes them more effective drugs &! can usually be modified to make them a more effective drug '! can usually be modified to make them more effective drugs

077.

=here is gro#ing demand in the state for initiative and referendum% a procedure that allo#s voters to propose and pass la#s% as #ell as to repeal them. A! allo#s voters to propose and pass la#s% as #ell as to repeal them "! allo#s voters to propose% pass% and to repeal la#s $! allo#s voters to propose% to pass% and repeal la#s &! #ill allo# the voter to propose% pass% as #ell as to repeal la#s '! #ill allo# la#s to be proposed% passed% as #ell as repealed by voters

07:.

=here is no consensus on #hat role% if any% is played by acid rain in slo#ing the gro#th or damaging forests in the eastern +nited ,tates. A! slo#ing the gro#th or damaging "! the damage or the slo#ing of the gro#th of $! the damage to or the slo#ness of the gro#th of &! damaged or slo#ed gro#th of '! damaging or slo#ing the gro#th of

07*.

=here is speculation that increasing cold #eather #as #hat may have been responsible for the Anasa9i move from 8esa Berde to sites in other canyons. A! that increasing cold #eather #as #hat may have been "! #hether increasing cold #eather #as #hat #as $! that increasingly cold #eather #as #hat had been &! #hether increasingly cold #eather may have been #hat #as '! that increasingly cold #eather may have been

070.

=here is substantial evidence that certain forms of solar energy either no# or #ithin a fe# years #ill be economically competitive #ith conventional sources of heat and po#er. A! either no# or #ithin a fe# years #ill be economically competitive #ith conventional sources of heat and po#er "! #ill either be economically competitive #ith conventional sources of heat and po#er #ithin a fe# years or are so no# $! #ill be economically competitive #ith conventional sources of heat and po#er either no# or

SC Strategy

''*

#ithin a fe# years &! either are no# economically competitive #ith conventional sources of heat and po#er or #ill be so #ithin a fe# years '! are either no# or #ill be #ithin a fe# years economically competitive #ith conventional sources of heat and po#er 07). =homas 'akins3 po#erful style and his choices of sub6ectHthe advances in modern surgery% the discipline of sport% the strains of individuals in tension #ith society or even #ith themselvesH #as as disturbing to his o#n time as it is compelling for ours. A! #as as disturbing to his o#n time as it is "! #ere as disturbing to his o#n time as they are $! has been as disturbing in his o#n time as they are &! had been as disturbing in his o#n time as it #as '! have been as disturbing in his o#n time as 0:2. =hose #ho come to church #ith a predisposition to religious belief #ill be happy in an auditorium or even a storefront% and there is no doubt that religion is sometimes better served by adapted spaces of this kind instead of by some of the buildings actually designed for it. A! adapted spaces of this kind instead of by some of the buildings actually designed for it "! adapted spaces like these rather than some of the buildings actually designed for them $! these adapted spaces instead of by some of the buildings actually designed for it &! such adapted spaces rather than by some of the buildings actually designed for them '! such adapted spaces than by some of the buildings actually designed for it 0:1. =hose #ho enter marathons soon learn that% to succeed in these grueling competitive events% runners must be in e<cellent condition% have unshakable self-confidence% and% most important of all% kno# ho# to pace yourself. A! kno# ho# to pace yourself "! is kno#ing ho# to pace yourself $! kno# ho# to pace themselves &! you must pace yourself '! they must kno# ho# to pace themselves 0:(. =hose #ith a cynical turn of mind might speculate if the ne# corporation% eager for profit% might not have started the rumor that caused its competitor to declare bankruptcy. A! speculate if the ne# corporation% eager for profit% might not have started "! speculate if the ne# corporation% eager for profit% had not started $! speculate if% in its eagerness for profit% the ne# corporation started &! #onder as to #hether% in its eagerness for profit% the ne# corporation did not start

''7

'! #onder #hether the ne# corporation% eager for profit% had started 0:/. =hough the term graphic design may suggest laying out corporate brochures and annual reports% they have come to signify #idely ranging #ork% from package designs and company logotypes to signs% book 6ackets% computer graphics% and film titles. A! suggest laying out corporate brochures and annual reports% they have come to signify #idely ranging "! suggest laying out corporate brochures and annual reports% it has come to signify a #ide range of $! suggest corporate brochure and annual report layout% it has signified #idely ranging &! have suggested corporate brochure and annual report layout% it has signified a #ide range of '! have suggested laying out corporate brochures and annual reports% they have come to signify #idely ranging 0:4. =hree out of every four automobile o#ners in the +nited ,tates also o#n a bicycle. A! =hree out of every four automobile o#ners in the +nited ,tates also o#n a bicycle. "! Gut of every four% three automobile o#ners in the +nited ,tates also o#ns a bicycle. $! "icycles are o#ned by three out of every four o#ners of automobiles in the +nited ,tates. &! In the +nited ,tates% three out of every four automobile o#ners o#ns bicycles. '! Gut of every four o#ners of automobiles in the +nited ,tates% bicycles are also o#ned by three. 0:7. =hrough the years% the e<5uisitely subtle flavors and superb richness of Fenyan coffee has attracted an international follo#ing of discerning consumers. A! the e<5uisitely subtle flavors and superb richness of Fenyan coffee has "! the coffee of Fenya% #ith its e<5uisitely subtle and superbly rich flavors% have $! the e<5uisitely subtle% superbly rich flavors of Fenyan coffee are #hat has &! Fenyan coffee3s superb richness and e<5uisite subtlety of flavor has '! the e<5uisitely subtle flavors and superb richness of Fenyan coffee have 0::. =iny 5uantities of more than thirty rare gases% most of them industrial by-products% threaten to #arm the 'arth3s atmosphere even more rapidly than carbon dio<ide during the ne<t fifty years. A! to #arm the 'arth3s atmosphere even more rapidly than carbon dio<ide during the ne<t fifty years "! to #arm the 'arth3s atmosphere even more rapidly over the ne<t fifty years than carbon dio<ide #ill $! during the ne<t fifty years to #arm the 'arth3s atmosphere even more rapidly than carbon dio<ide &! a #arming of the 'arth3s atmosphere during the ne<t fifty years even more rapid than carbon

SC Strategy

''8

dio<ide3s '! a #arming of the 'arth3s atmosphere even more rapid than carbon dio<ide3s #ill be over the ne<t fifty years 0:*. =o compare the lightning-fast genius of play#right =om ,toppard #ith the pedestrian efforts of some of his contemporaries is to compare the e<5uisite bou5uet of a fine #ine #ith that of ordinary grape 6uice. A! =o compare the lightning-fast genius of play#right =om ,toppard #ith the pedestrian efforts of some of his contemporaries is to compare the e<5uisite bou5uet of a fine #ine #ith that of ordinary grape 6uice. "! =o compare the lightning-fast genius of play#right =om ,toppard #ith the pedestrian efforts of some of his contemporaries is comparing the e<5uisite bou5uet of a fine #ine #ith that of ordinary grape 6uice. $! $omparing the lightning-fast genius of play#right =om ,toppard #ith the pedestrian efforts of some of his contemporaries is to compare the e<5uisite bou5uet of a fine #ine #ith ordinary grape 6uice. &! $omparing the lightning-fast genius of play#right =om ,toppard #ith the pedestrian efforts of some of his contemporaries is like comparing the e<5uisite bou5uet of a fine #ine #ith ordinary grape 6uice. '! =o compare the lightning-fast genius of play#right =om ,toppard #ith the pedestrian efforts of some of his contemporaries is to compare a fine #ine3s bou5uet #ith ordinary grape 6uice3s bou5uet. 0:0. =o ensure consistently high 5uality in its merchandise% the chain of retail stores became involved in every aspect of their suppliers3 operations% dictating not only the number of stitches and the #idth of the hem in every garment as #ell as the profit margins of those suppliers. A! their suppliers3 operations% dictating not only the number of stitches and the #idth of the hem in every garment as #ell as "! its suppliers3 operations% dictating not only the number of stitches and the #idth of the hem in every garment as #ell as $! their suppliers3 operations% dictating not only the number of stitches and the #idth of the hem in every garment but also &! its suppliers3 operations% dictating not only the number of stitches and the #idth of the hem in every garment but also '! their suppliers3 operations% dictating the number of stitches% the #idth of the hem in every garment% and 0:). =o help preserve ancient 'gyptian monuments threatened by high #ater tables% a ,#edish engineering firm has proposed installing pumps% perhaps solar po#ered% to lo#er the underground #ater level and dig trenches around the bases of the stone #alls. A! to lo#er the underground #ater level and dig trenches

''9

"! to lo#er the underground #ater level and to dig trenches $! to lo#er the underground #ater level and digging trenches &! that lo#er the underground #ater level and that trenches be dug '! that lo#er the underground #ater level and trench digging 0*2. =o maintain a high demand for their product% the manufacturers first took over the marketing and sales functions previously performed by outside agentsE ne<t% they began changing their advertising campaigns monthly to keep pace #ith the rapid changes in consumers3 lives. A! they began changing "! this began changing $! the former began changing &! to begin changing '! to change 0*1. =o read of Abigail Adams3 lengthy separation from her family% her difficult travels% and her constant battles #ith illness is to feel intensely ho# harsh life #as even for the so-called aristocracy of Devolutionary times. A! =o read of "! Deading about $! >aving read about &! Gnce one reads of '! =o have read of 0*(. =o speak habitually of the truly needy is gradually instilling the notion that many of those #ho are 6ust called needy actually have ade5uate resourcesE such a conclusion is un#arranted. A! =o speak habitually of the truly needy is gradually instilling the notion "! =o speak habitually of the truly needy is instilling the notion gradually $! =o speak habitually of the truly needy is gradually to instill the notion &! ,peaking habitually of the truly needy is to instill the gradual notion '! ,peaking habitually of the truly needy is instilling the gradual notion 0*/. =o spread the #ord about mortgage servicers% a fact sheet that outlines one3s legal rights if you get caught in a mortgage serving mess has been put together by the ?ederal =rade $ommission. A! a fact sheet that outlines one3s legal rights if you get caught in a mortgage serving mess has been put together by the ?ederal =rade $ommission "! an outline of one3s legal rights has been put together by the ?ederal =rade $ommission if one gets caught up in a mortgaging mess on a fact sheet $! should you get caught in a mortgaging mess% a fact sheet outlining your legal rights has been

SC Strategy

'';

put together by the ?ederal =rade $ommission &! there is a fact sheet put together by the ?ederal =rade $ommission% #hich outlines one3s legal rights in a mortgage servicing mess '! the ?ederal =rade $ommission has put together a fact sheet that outlines your legal rights if you get caught in a mortgage servicing mess 0*4. =oday% because of improvements in agricultural technology% the same amount of acreage produces double the apples that it has in 1)12. A! double the apples that it has "! t#ice as many apples as it did $! as much as t#ice the apples it has &! t#o times as many apples as there #ere '! a doubling of the apples that it did 0*7. =oday3s technology allo#s manufacturers to make small cars more fuel-efficient no# than at any time in their production history. A! small cars more fuel-efficient no# than at any time in their "! small cars that are more fuel-efficient than they #ere at any time in their $! small cars that are more fuel-efficient than those at any other time in &! more fuel-efficient small cars than those at any other time in their '! more fuel-efficient small cars no# than at any time in 0*:. =oo old to bear arms himself% ?rederick &ouglass served as a recruiting agent% traveled through the North to e<hort "lack men to 6oin the +nion army. A! traveled through the North to e<hort "! and he traveled through the North and e<horted $! and traveling through the North e<horted &! traveling through the North and e<horted '! traveling through the North and e<horting 0**. =raveling the back roads of >ungary% in 1)27 "Mla "artNk and KoltPn FodPly began their pioneering #ork in ethnomusicology% and they #ere armed only #ith an 'dison phonograph and insatiable curiosity. A! =raveling the back roads of >ungary% in 1)27 "Mla "artNk and KoltPn FodPly began their pioneering #ork in ethnomusicology% and they #ere armed only "! In 1)27% "Mla "artNk and KoltPn FodPly% traveling the back roads of >ungary% began their pioneering #ork in ethnomusicology% and they #ere only armed $! In 1)27 "Mla "artNk and KoltPn FodPly began their pioneering #ork in ethnomusicology% traveling the back roads of >ungary armed only

''I

&! >aving traveled the back roads of >ungary% in 1)27 "Mla "artNk and KoltPn FodPly began their pioneering #ork in ethnomusicologyE they #ere only armed '! "Mla "artNk and KoltPn FodPly% in 1)27 began their pioneering #ork in ethnomusicology% traveling the back roads of >ungary% arming themselves only 0*0. =rying to learn some of the basics of programming is the same as to tinker #ith a car #hen one is a teenagerA some people end up going to engineering school% and others% t#enty years later% remember nothing of the e<perience. A! the same as to tinker #ith a car #hen one is a teenager "! similar to a teenager tinkering #ith a car $! like tinkering #ith a car as a teenager &! the same as a teenager tinkering #ith a car '! like the teenager3s tinkering #ith a car 0*). =#o ne# studies indicate that many people become obese more due to the fact that their bodies burn calories too slo#ly than overeating. A! due to the fact that their bodies burn calories too slo#ly than overeating "! due to their bodies burning calories too slo#ly than to eating too much $! because their bodies burn calories too slo#ly than that they are overeaters &! because their bodies burn calories too slo#ly than because they eat too much '! because of their bodies burning calories too slo#ly than because of their eating too much 002. =#o valence states of uranium% one #ith a deficit of four electrons and the other one #ith a deficit of si<% occurs in nature and contributes to the diversity of uranium3s behavior. A! the other one #ith a deficit of si<% occurs in nature and contributes "! the other one a deficit of si<% occur in nature and contribute $! the other #ith a deficit of si<% occurs in nature and contributes &! the other #ith a deficit of si<% occur in nature and contribute '! one #ith si<% occurs in nature and contributes 001. =#o #eek notice being given to employers before leaving a 6ob is the generally accepted protocol. A! =#o #eek notice being given to employers before leaving "! -iving notice to employers of t#o #eeks before having to leave $! =#o #eek3s notice to give to employers before leaving &! -iving notice to employers t#o #eeks before leaving '! =o give t#o #eeks3 #orth of notice before having to leave 00(. +nder a provision of the $onstitution that #as never applied% $ongress has been re5uired to call a convention for considering possible amendments to the document #hen formally asked to

SC Strategy

''J

do it by the legislatures of t#o-thirds of the states. A! #as never applied% $ongress has been re5uired to call a convention for considering possible amendments to the document #hen formally asked to do it "! #as never applied% there has been a re5uirement that $ongress call a convention for consideration of possible amendments to the document #hen asked to do it formally $! #as never applied% #hereby $ongress is re5uired to call a convention for considering possible amendments to the document #hen asked to do it formally &! has never been applied% #hereby $ongress is re5uired to call a convention to consider possible amendments to the document #hen formally asked to do so '! has never been applied% $ongress is re5uired to call a convention to consider possible amendments to the document #hen formally asked to do so 00/. +nder Napoleon the ?rench #ere not able to organi9e an ade5uate supply system% and it #as a ma6or cause of the failure of their invasion of Dussia. A! +nder Napoleon the ?rench #ere not able to organi9e an ade5uate supply system% and it "! =he ?rench being unable to organi9e an ade5uate supply system under Napoleon $! ?or the ?rench under Napoleon% to be unable to organi9e an ade5uate supply system &! =he inability of the ?rench under Napoleon to organi9e an ade5uate supply system '! =he ?rench inability under Napoleon of organi9ing an ade5uate supply system. 004. +nder the ne# corporate insurance policy% #hen an employer is charged for damages to a third party #holly or largely as a result of actions by an employee% he is entitled to recoup the amount of the damages. A! he is entitled to recoup "! the employer is entitled to recoup $! he or she is entitled to recoup &! he is entitled to recoup for '! the employer is entitled to recoup for 007. +nder the restructuring% the huge organi9ation that operates the company3s basic businesses #ill be divided into five groups% each #ith its o#n e<ecutive. A! each #ith its o#n e<ecutive "! all having their o#n e<ecutive $! each having their o#n e<ecutive &! #ith its o#n e<ecutive for each '! every one #ith an e<ecutive of their o#n 00:. +nder the ,afe &rinking Water Act% the 'nvironmental 1rotection Agency is re5uired either to approve individual state plans for controlling the discharge of #astes into underground #ater or that they enforce their o#n plan for states #ithout ade5uate regulations.

'*E

A! that they enforce their "! for enforcing their $! they should enforce their &! it should enforce its '! to enforce its 00*. +nlike a funded pension system% in #hich contributions are invested to pay future beneficiaries% a pay-as-you-go approach is the foundation of ,ocial ,ecurity. A! a pay-as-you-go approach is the foundation of ,ocial ,ecurity "! the foundation of ,ocial ,ecurity is a pay-as-you-go approach $! the approach of ,ocial ,ecurity is pay-as-you-go &! ,ocial ,ecurity3s approach is pay-as-you-go '! ,ocial ,ecurity is founded on a pay-as-you-go approach 000. +nder the 1)0: ta< la#% interest payments on a refinanced home loan are deductible only if the amount of the loan does not e<ceed the purchase price of the home% the cost of improvements% and any additional amount borro#ed against the home to pay for medical or educational e<penses. A! any additional amount borro#ed against the home to pay for medical or educational e<penses "! borro#ing any additional amount against the home for payment of medical or educational e<penses $! also borro#ing any additional amount against the home to pay for medical or educational e<penses &! any additional payment of medical or educational e<penses that #ere borro#ed against the home '! any additional payment borro#ed against the home for medical or educational e<penses 00). +nlike a hurricane% #hich can be observed from #ithin% a tornado is so small that such a study has not been practical. A! that such a study has not been practical "! that studying it that #ay has not been impractical $! for such studies as this to have been impractical &! as to not make such a study practical '! as to be impractical of study 0)2. +nlike a typical automobile loan% #hich re5uires a fifteen- to t#enty-percent do#n payment% the lease-loan buyer is not re5uired to make an initial deposit on the ne# vehicle. A! the lease-loan buyer is not re5uired to make "! #ith lease-loan buying there is no re5uirement of $! lease-loan buyers are not re5uired to make

SC Strategy

'*$

&! for the lease-loan buyer there is no re5uirement of '! a lease-loan does not re5uire the buyer to make 0)1. +nlike auto insurance% the fre5uency of claims does not affect the premiums for personal property coverage% but if the insurance company is able to prove e<cessive loss due to o#ner negligence% it may decline to rene# the policy. A! +nlike auto insurance% the fre5uency of claims does not affect the premiums for personal property coverage "! +nlike #ith auto insurance% the fre5uency of claims do not affect the premiums for personal property coverage $! +nlike the fre5uency of claims for auto insurance% the premiums for personal property coverage are not affected by the fre5uency of claims &! +nlike the premiums for auto insurance% the premiums for personal property coverage are not affected by the fre5uency of claims '! +nlike #ith the premiums for auto insurance% the premiums for personal property coverage is not affected by the fre5uency of claims 0)(. +nlike computer skills or other technical skills% there is a disinclination on the part of many people to recogni9e the degree to #hich their analytical skills are #eak. A! +nlike computer skills or other technical skills% there is a disinclination on the part of many people to recogni9e the degree to #hich their analytical skills are #eak. "! +nlike computer skills or other technical skills% #hich they admit they lack% many people are disinclined to recogni9e that their analytical skills are #eak. $! +nlike computer skills or other technical skills% analytical skills bring out a disinclination in many people to recogni9e that they are #eak to a degree. &! 8any people% #illing to admit that they lack computer skills or other technical skills% are disinclined to recogni9e that their analytical skills are #eak. '! 8any people have a disinclination to recogni9e the #eakness of their analytical skills #hile #illing to admit their lack of computer skills or other technical skills. 0)/. +nlike -ertrude ,tein% '9ra 1ound% and other e<patriates% William $arlos Williams insisted that poets honor their o#n regions and employ specifically American rhythms. A! +nlike -ertrude ,tein% '9ra 1ound% and other e<patriates% William $arlos Williams insisted "! Although -ertrude ,tein% '9ra 1ound% and other e<patriates did not% William $arlos Williams3 insistence #as $! $ontrary to -ertrude ,tein% '9ra 1ound% and other e<patriates% it #as William $arlos Williams #ho insisted &! As opposed to #hat -ertrude ,tein% '9ra 1ound% and other e<patriates did% William $arlos Williams #as to insist '! While -ertrude ,tein% '9ra 1ound% and other e<patriates did not% William $arlos Williams #as

'*'

insistent 0)4. +nlike in other transportation industries% there are no minimum standards set by government or industry for 5ualifying for being an engineer on a train. A! +nlike in other transportation industries% there are no minimum standards set by government or industry for 5ualifying for being "! +nlike other transportation industries that have minimum standards set by government or industry% there are none for 5ualifying to be $! Although the government or industry usually sets minimum standards for transportation industries% no such one has been set for 5ualifying as to being &! Although the government or industry usually sets minimum standards for transportation industries% no such standard has been set for 5ualifying to be '! Although there are usually minimum standards set by government or industry for transportation industries% there is none for 5ualifying and being 0)7. +nlike most #arbler species% the male and female blue-#inged #arbler are very difficult to tell apart. A! +nlike most #arbler species% the male and female blue-#inged #arbler are very difficult to tell apart. "! +nlike most #arbler species% the gender of the blue-#inged #arbler is very difficult to distinguish. $! +nlike those in most #arbler species% the male and female blue-#inged #arblers are very difficult to distinguish. &! It is very difficult% unlike in most #arbler species% to tell the male and female blue-#inged #arbler apart. '! "lue-#inged #arblers are unlike most species of #arbler in that it is very difficult to tell the male and female apart. 0):. +nlike other arachnids% #hich have their nerve cells evenly distributed along their bodies% the scorpion3s nerve cells are clustered in its head% like a mammal3s. A! bodies% the scorpion3s nerve cells are clustered in its head% like a mammal3s "! bodies% the scorpion3s head had a cluster of nerve cells% as a mammal does $! body% the scorpion has a cluster of nerve cells in its head% as a mammal does &! body% nerve cells are clustered in the scorpion3s head% like a mammal3s '! body% a cluster of nerve cells is in the scorpion3s head% like a mammal3s 0)*. +nlike ,choenberg3s t#elve-tone system that dominated the music of the post#ar period% "artok founded no school and left behind only a handful of disciples. A! ,choenberg3s t#elve-tone system that dominated "! ,choenberg and his t#elve-tone system #hich dominated

SC Strategy

'**

$! ,choenberg% #hose t#elve-tone system dominated &! the t#elve-tone system of ,choenberg that has dominated '! ,choenberg and the t#elve-tone system% dominating 0)0. +nlike that of human beings% #ho #aste a#ay #hen they go #ithout food for long periods% hibernating bears e<ist for months on only their e<cess fat. A! +nlike that of human beings% #ho #aste a#ay #hen they go "! +nlike human beings% #ho #aste a#ay #hen they go $! +nlike human beings% #asting a#ay #hen going &! &issimilar to human beings% #asting a#ay #hen '! .acking similarity to human beings% #ho #aste a#ay #hen 0)). +nlike that of the Native Americans of "ritish $olumbia% the 1lains% and the ,outh#est% those of 1uget ,ound lived in relatively small% autonomous villages. A! +nlike that of "! +nlike those of $! +nlike &! In contrast to that of '! &issimilar to )22. +nlike the acid smoke of cigarettes% pipe tobacco% cured by age-old methods% yields an alkaline smoke too irritating to be dra#n into the lungs. A! +nlike the acid smoke of cigarettes% pipe tobacco% cured by age-old methods% yields an alkaline smoke "! +nlike the acid smoke of cigarettes% pipe tobacco is cured by age-old methods% yielding an alkaline smoke $! +nlike cigarette tobacco% #hich yields an acid smoke% pipe tobacco% cured by age-old methods% yields an alkaline smoke &! &iffering from cigarettes3 acid smoke% pipe tobacco3s alkaline smoke% cured by age-old methods% is '! =he alkaline smoke of pipe tobacco differs from cigarettes3 acid smoke in that it is cured by age-old methods and is )21. +nlike the lives of 1ushkin% -ogol% =olstoi% and &ostoevski% sub6ects of other =royat biographies% $hekhov belongs to the t#entieth century% an age of fretfulness and melancholy skepticism. A! +nlike the lives of 1ushkin% -ogol% =olstoi% and &ostoevski% sub6ects of other =royat biographies% $hekhov belongs "! $hekhov% unlike the other =royat biographies of 1ushkin% -ogol% =olstoi% and &ostoevski% belongs

'*7

$! =he life of $hekhov% unlike the lives of the sub6ects of other =royat biographies% 1ushkin% -ogol% =olstoi% and &ostoevski% belongs &! $hekhov and his life% unlike that of the other =royat biographiesH1ushkin% -ogol% =olstoi% and &ostoevski% belong '! =he life of $hekhov% unlike that of other =royat biographies of 1ushkin% -ogol% =olstoi% and &ostoevski% belongs )2(. +nlike the ,hiites% #ho constitute the other ma6or branch of Islam% the ,unnites do not a#ait the 8ahdi as a messenger from -od% nor do they endo# him #ith divine 5ualities or immunity from failure in 6udgment. A! nor do they endo# him "! but they do not endo# him $! neither do they endo# him &! and they neither endo# him '! #hile endo#ing him neither )2/. +nlike the +nited ,tates% Capanese unions appear reluctant to organi9e lo#er-paid #orkers. A! +nlike the +nited ,tates% Capanese unions appear reluctant to organi9e "! +nlike those in the +nited ,tates% Capanese unions appear reluctant to organi9e $! In Capan% unlike the +nited ,tates% unions appear reluctant to organi9e &! Capanese unions% unlike the +nited ,tates% appear reluctant to organi9e '! Capanese unions% unlike those in the +nited ,tates% appear reluctant about organi9ing )24. +nlike the +nited ,tates% #here farmers can usually depend on rain or sno# all year long% the rains in most parts of ,ri .anka are concentrated in the monsoon months% Cune to ,eptember% and the skies are generally clear for the rest of the year. A! +nlike the +nited ,tates% #here farmers can usually depend on rain or sno# all year long% the rains in most parts of ,ri .anka "! +nlike the +nited ,tates farmers% #ho can usually depend on rain or sno# all year long% the rains in most parts of ,ri .anka $! +nlike those of the +nited ,tates% #here farmers can usually depend on rain or sno# all year long% most parts of ,ri .anka3s rains &! In comparison #ith the +nited ,tates% #hose farmers can usually depend on rain or sno# all year long% the rains in most parts of ,ri .anka '! In the +nited ,tates% farmers can usually depend on rain or sno# all year long% but in most parts of ,ri .anka the rains )27. +nlike their counterparts in other Western democracies% the American labor movement has never embraced revolutionary ideologies calling for the ultimate transformation of the economic order.

SC Strategy

'*8

A! +nlike their counterparts in other Western democracies% the American labor movement has never "! +nlike that of their counterparts in other Western democracies% the American labor movement has never $! +nlike its counterpart in other Western democracies% the American labor movement never have &! +nlike that of its counterparts in other Western democracies% the American labor movement never has '! +nlike its counterparts in other Western democracies% the American labor movement has never )2:. +nlike transplants bet#een identical t#ins% #hose genetic endo#ment is the same% all patients receiving hearts or other organs must take antire6ection drugs for the rest of their lives. A! +nlike transplants bet#een identical t#ins% #hose genetic endo#ment is the same "! "esides transplants involving identical t#ins #ith the same genetic endo#ment $! +nless the transplant involves identical t#ins #ho have the same genetic endo#ment &! Aside from a transplant bet#een identical t#ins #ith the same genetic endo#ment '! Gther than transplants bet#een identical t#ins% #hose genetic endo#ment is the same )2*. +ntil 5uite recently% American presidents lived in a #orld in #hich the public and private realms of their lives #ere largely separate% and the press cooperated in maintaining the distinction% and Americans 6udged national leaders #ithout receiving% or e<pecting% intimate information about them. A! and the press cooperated in maintaining the distinction% and "! #here the press cooperated in maintaining the distinction% and #here $! for the press cooperated to maintain the distinction and &! the press cooperated to maintain the distinction% for '! in #hich the press cooperated in maintaining the distinction% and in #hich )20. +pset by the recent do#nturn in production numbers during the first half of the year% the possibility of adding #orker incentives #as raised by the board of directors at its 5uarterly meeting. A! the possibility of adding #orker incentives #as raised by the board of directors at its 5uarterly meeting "! the addition of #orker incentives #as raised as a possibility by the board of directors at its 5uarterly meeting $! added #orker incentives #as raised by the board of directors at its 5uarterly meeting as a possibility &! the board of directors raised at its 5uarterly meeting the possibility of #orker incentives being added

'*9

'! the board of directors% at its 5uarterly meeting% raised the possibility of adding #orker incentives )2). +rban officials #ant the census to be as accurate and complete as possible for the reason that the amount of lo#-income people in a given area affect the distribution of about fifty billion dollars a year in federal funds. A! for the reason that the amount of lo#-income people in a given area affect "! for the reason because the amount of lo#-income people in a given area effects $! in that the amount of lo#-income people in given areas effect &! because the number of lo#-income people in a given area affects '! because the numbers of lo#-income people in given areas effects )12. +sing a &oppler ultrasound device% fetal heartbeats can be detected by the t#elfth #eek of pregnancy. A! +sing a &oppler ultrasound device% fetal heartbeats can be detected by the t#elfth #eek of pregnancy. "! ?etal heartbeats can be detected by the t#elfth #eek of pregnancy% using a &oppler ultrasound device. $! &etecting fetal heartbeats by the t#elfth #eek of pregnancy% a physician can use a &oppler ultrasound device. &! "y the t#elfth #eek of pregnancy% fetal heart-beats can be detected using a &oppler ultrasound device by a physician. '! +sing a &oppler ultrasound device% a physician can detect fetal heartbeats by the t#elfth #eek of pregnancy. )11. +sing the techni5ues employed by genetic engineering% a ne# species of microorganism has been developed by laboratory scientists to aid in cleaning up oil spills by digesting the oil. A! +sing the techni5ues employed by genetic engineering% a ne# species of microorganism has been developed by laboratory scientists to "! +sing the techni5ues employed by genetic engineering% a ne# species of microorganism that #as developed by laboratory scientists #ill $! +sing the techni5ues of genetic engineering laboratory scientists have developed a ne# species of microorganism to &! 'mploying the techni5ues of genetic engineering there has been a development by laboratory scientists of a ne# species of microorganism that #ill '! 'mploying the techni5ues of genetic engineering% a ne# species of microorganism that #as developed by laboratory scientists #ill )1(. Bery popular from 1)22 until the 1)(23s% the rene#ed interest in ceiling fans began #hen the energy crisis in 1)*4 forced homeo#ners to look for alternative methods of heating and cooling.

SC Strategy

'*;

A! Bery popular from 1)22 until the 1)(23s% the rene#ed interest in ceiling fans began "! =he rene#ed interest in ceiling fans% #hich #ere very popular from 1)22 until the 1)(23s% began $! After they #ere very popular from 1)22 until the 1)(23s% the rene#ed interest in ceiling fans #as beginning &! $eiling fans #ere very popular from 1)22 until the 1)(23s% #ith rene#ed interest beginning in them '! ?rom 1)22 until the 1)(23s ceiling fans #ere very popular% and no# the rene#ed interest in them has begun )1/. Biolence in the stands at soccer matches has gotten so pronounced in several 'uropean countries that some stadiums have adopted ne# rules that aim to identify fans of visiting teams and that seat them in a separate area. A! to identify fans of visiting teams and that seat them "! to identify fans of visiting teams and seat them $! to identify fans of visiting teams for seating &! at identifying fans of visiting teams so as to seat them '! at identifying fans of visiting teams and that seat them )14. Birtually undisturbed for the last three centuries on their starkly beautiful islands near the edge of the Arctic $ircle% the inhabitants of the .ofotens have evolved folk#ays and a life-style that bring #armth to their harsh environment. A! the inhabitants of the .ofotens have evolved folk#ays and a life-style that bring #armth "! the inhabitants of the .ofotens have evolved folk#ays and a life-style that brings #armth $! evolving folk#ays and a life-style #ere evolved by the .ofotens inhabitants to bring #armth &! evolving folk#ays and a life-style brought #armth to the .ofotens3 inhabitants as #ell as '! #armth-bringing folk#ays and life-styles have been evolved by the inhabitants of the .ofotens )17. Bisitors to the park have often looked up into the leafy canopy and sa# monkeys sleeping on the branches% #hose arms and legs hang like socks on a clothesline. A! sa# monkeys sleeping on the branches% #hose arms and legs hang "! sa# monkeys sleeping on the branches% #hose arms and legs #ere hanging $! sa# monkeys sleeping on the branches% #ith arms and legs hanging &! seen monkeys sleeping on the branches% #ith arms and legs hanging '! seen monkeys sleeping on the branches% #hose arms and legs have hung )1:. Warning that computers in the +nited ,tates are not secure% the National Academy of ,ciences has urged the nation to revamp computer security procedures% institute ne# emergency response teams% creating a special nongovernment organi9ation to take charge of computer

'*I

security planning. A! creating a special nongovernment organi9ation to take "! creating a special nongovernment organi9ation that takes $! creating a special nongovernment organi9ation for taking &! and create a special nongovernment organi9ation for taking '! and create a special nongovernment organi9ation to take )1*. Water and resource management problems #ill be at the head of the legislature3s list of concerns for the coming session. A! Water and resource management problems "! 1roblems of managing #ater and resources $! 1roblems in the management of #ater and other resources &! 1roblems of #ater and other resource management '! Desource management problems% including #ater )10. What brought the automobile company back from the verge of bankruptcy shortly after the ,econd World War #as a special% governmentally sanctioned price increase allo#ed during a period of #age and price controls. A! What brought "! =he thing that brought $! =hat #hich brought &! "ringing '! What has brought )1). What #as as remarkable as the development of the compact disc has been the use of the ne# technology to revitali9e% in better sound than #as ever before possible% some of the classic recorded performances of the pre-.1 era. A! What #as as remarkable as the development of the compact disc "! =he thing that #as as remarkable as developing the compact disc $! No less remarkable than the development of the compact disc &! &eveloping the compact disc has been none the less remarkable than '! &evelopment of the compact disc has been no less remarkable as )(2. When $ongress reconvenes% some ne#ly elected members from rural states #ill try and establish tighter restrictions for the amount of grain farmers are to be allo#ed to gro# and to encourage more aggressive sales of +nited ,tates farm products overseas. A! and establish tighter restrictions for the amount of grain farmers are to be allo#ed to gro# and to encourage "! and establish tighter restrictions on the amount of grain able to be gro#n by farmers and

SC Strategy

'*J

encouraging $! establishing tighter restrictions for the amount of grain farmers are allo#ed to gro# and to encourage &! to establish tighter restrictions on the amount of grain capable of being gro#n by farmers and encouraging '! to establish tighter restrictions on the amount of grain farmers #ill be allo#ed to gro# and to encourage )(1. When evidence of financial #rongdoing by an elected official surfaces% it is the electorate #ho must decide #hether the evidence #arrants censuring him or ousting him from office. A! #hether the evidence #arrants censuring him or ousting him "! if there is evidence that #arrants a censure or an ousting of him $! #hether or not the evidence #arrants the censuring or ousting of him &! if there is evidence that #arrants censuring him or his ousting '! if the evidence #ould #arrant that he be censured or that he be ousted )((. When rates #ere raised in 1)07% postal service officials predicted they #ould make further rate increases unnecessary for at least three years. A! they #ould make further rate increases unnecessary "! they #ould mean that further rate increases #ould not be needed $! that it #ould not be necessary for further rate increases &! that the increase #ould make further rate increases unnecessary '! further rate increases #ill not be needed )(/. When the prime lending rates #ent up in 1)0*% economists determined they #ould cause interest rates to rise and then decline over the ensuing five-year period. A! they #ould cause interest rates to rise and then decline "! they #ould mean that interest rates #ould rise and then decline $! that they #ill cause interest rates to rise and then decline &! that the increase #ould cause interest rates to rise and then decline '! that the increase #ould cause interest rates3 rising and subse5uent declining )(4. When the techni5ue kno#n as gene-splicing #as invented in the early 1)*23s% it #as feared that scientists might inadvertently create an Andromeda strain% a microbe never before seen on 'arth that might escape from the laboratory and it #ould kill vast numbers of humans #ho #ould have no natural defenses against it. A! it #ould kill vast numbers of humans #ho #ould have no natural defenses against it "! it might kill vast numbers of humans #ith no natural defenses against it $! kill vast numbers of humans #ho #ould have no natural defenses against it

'7E

&! kill vast numbers of humans #ho have no natural defenses against them '! kill vast numbers of humans #ith no natural defenses against them )(7. Where once the union had ac5uiesced to the pre6udices of its 'nglish-speaking members by supporting the imposition of an alien ta< on immigrant #orkers% after 10)* the +nited 8ine Workers made a determined effort to enlist Italians and ,lavs in its ranks. A! Where once the union had ac5uiesced to the pre6udices of its 'nglish-speaking members by supporting "! Where once the union ac5uiesced to it 'nglish-speaking members3 pre6udice for the support of $! While once the union had ac5uiesced to the pre6udices of its 'nglish-speaking members in support of &! While once the union ac5uiesced to its 'nglish-speaking members3 pre6udice in supporting '! While once the union had ac5uiesced to the pre6udices of its 'nglish-speaking members in its supporting of )(:. While all states face similar industrial #aste problems% the predominating industries and regulatory environment of the states obviously determines the types and amounts of #aste produced% as #ell as the cost of disposal. A! all states face similar industrial #aste problems% the predominating industries and regulatory environment of the states obviously determines "! each state faces a similar industrial #aste problem% their predominant industries and regulatory environment obviously determine $! all states face a similar industrial #aste problem% their predominating industries and regulatory environment obviously determines &! each state faces similar industrial #aste problems% the predominant industries and regulatory environment of each state obviously determines '! all states face similar industrial #aste problems% the predominant industries and the regulatory environment of each state obviously determine )(*. While depressed property values can hurt some large investors% they are potentially devastating for home-o#ners% #hose e5uityHin many cases representing a life3s savingsHcan plunge or even disappear. A! they are potentially devastating for homeo#ners% #hose "! they can potentially devastate homeo#ners in that their $! for homeo#ners they are potentially devastating% because their &! for homeo#ners% it is potentially devastating in that their '! it can potentially devastate homeo#ners% #hose )(0. While Cackie Dobinson #as a "rooklyn &odger% his courage in the face of physical threats and verbal attacks #as not unlike that of Dosa 1arks% #ho refused to move to the back of a bus in

SC Strategy

'7$

8ontgomery% Alabama. A! not unlike that of Dosa 1arks% #ho refused "! not unlike Dosa 1arks% #ho refused $! like Dosa 1arks and her refusal &! like that of Dosa 1arks for refusing '! as that of Dosa 1arks% #ho refused )(). While larger banks can afford to maintain their o#n data-processing operations% many smaller regional and community banks are finding that the cost associated #ith upgrading data-processing e5uipment and #ith the development and maintenance of ne# products and technical staff are prohibitive. A! cost associated #ith "! costs associated #ith $! costs arising from &! cost of '! costs of )/2. While some academicians believe that business ethics should be integrated into every business course% others say that students #ill take ethics seriously only if it #ould be taught as a separately re5uired course. A! only if it #ould be taught as a separately re5uired course "! only if it is taught as a separate% re5uired course $! if it is taught only as a course re5uired separately &! if it #as taught only as a separate and re5uired course '! if it #ould only be taught as a re5uired course% separately )/1. While some propose to combat #idespread illegal copying of computer programs by attempting to change people3s attitudes to#ard pirating% others by suggesting reducing soft#are prices to decrease the incentive for pirating% and still others by calling for the prosecution of those #ho copy soft#are illegally. A! by suggesting reducing soft#are prices to decrease the incentive for pirating% and still others by calling "! by suggesting the reduction of soft#are prices to decrease the incentive for pirating% and still others call $! suggest the reduction of soft#are prices for decreasing the incentive for pirating% and still others call &! suggest the reduction of soft#are prices to decrease the incentive for pirating% and still others by calling '! suggest reducing soft#are prices to decrease the incentive for pirating% and still others are

'7'

calling )/(. While the base salary for the top five officers of the company did not change from 1))2 to 1))1% cuts #ere made nonsalary compensation% as in allo#ances for overseas assignments and club memberships. A! cuts #ere made nonsalary compensation% as in "! cuts #ere made in such nonsalary compensation as $! cuts #ere made in such nonsalary compensation as those in &! cuts in nonsalary compensation #ere made in areas like '! there #ere cuts made in nonsalary compensation% in areas like )//. While the o#ner of a condominium apartment has free and clear title to the d#elling% o#ners of cooperative apartments have shares in a corporation that o#ns a building and leases apartments to them. A! While the o#ner of a condominium apartment has free and clear title to the d#elling% "! =he o#ner of a condominium apartment has free and clear title to the d#elling% but $! Whereas o#ners of condominium apartments have free and clear title to their d#ellings% &! An o#ner of a condominium apartment has free and clear title to the d#elling% #hereas '! $ondominium apartment o#ners have a title to their d#elling that is free and clear% #hile )/4. William >. Cohnson3s artistic debt to ,candinavia is evident in paintings that range from sensitive portraits of citi9ens in his #ife3s &anish home% Ferteminde% and a#e-inspiring vie#s of f6ords and mountain peaks in the #estern and northern regions of Nor#ay. A! and "! to $! and to &! #ith '! in addition to )/7. Wind resistance created by opening #indo#s #hile driving results in a fuel penalty as great or greater than is incurred by using air conditioning. A! as great or greater than is incurred by using air conditioning "! that is as great or greater than is incurred using air conditioning $! as great as or greater than that of using air conditioning &! at least as great as air conditioning3s '! at least as great as that incurred by using air conditioning )/:. Wisconsin% Illinois% ?lorida% and 8innesota have begun to enforce state#ide bans prohibiting landfills to accept leaves% brush% and grass clippings. A! prohibiting landfills to accept leaves% brush% and grass clippings

SC Strategy

'7*

"! prohibiting that landfills accept leaves% brush% and grass clippings $! prohibiting landfills from accepting leaves% brush% and grass clippings &! that leaves% brush% and grass clippings cannot be accepted in landfills '! that landfills cannot accept leaves% brush% and grass clippings )/*. With a total population of less than t#o hundred and fe#er breeding females than ever before% the American crocodile seemed a decade ago to be in danger of disappearing. A! of less than t#o hundred and fe#er "! lo#er than t#o hundred and less $! lesser than t#o hundred and fe#er &! fe#er than t#o hundred and less '! of fe#er than t#o hundred and of fe#er )/0. With diamonds% as #ith all gems% one should ask for a #ritten description of one3s purchaseE the description may prove useful later if you have reason to believe the 6e#eler misled you. A! one should ask for a #ritten description of one3s purchase "! you should ask for a #ritten description of your purchase $! a #ritten discretion of your purchase is #hat one should ask for &! a #ritten description of one3s purchase is #hat should be asked for '! a #ritten description of your purchase is #hat should be asked for )/). With its abundance of noun inflections% Icelandic is one of several -ermanic languages that is compact #hen #ritten but can lengthen considerably #hen translated into 'nglish. A! is compact #hen #ritten but can lengthen considerably #hen translated into 'nglish "! are compact #hen they are #ritten% but they can lengthen considerably #hen they are translated in 'nglish $! is compact #hen #ritten but can lengthen considerably #hen being translated into 'nglish &! are compact #hen #ritten but can lengthen considerably in 'nglish translation '! is compact #hen it is #ritten but can lengthen considerably #hen translated in 'nglish )42. With its plan to develop seven and a half acres of shore land% $leveland is but one of a large number of communities on the -reat .akes that is looking to its #aterfront as a #ay to improve the 5uality of urban life and attract ne# businesses. A! is looking to its #aterfront as a #ay to improve the 5uality of urban life and attract "! is looking at its #aterfront to improve the 5uality of urban life and attract $! are looking to their #aterfronts to improve the 5uality of urban life and attract &! are looking to its #aterfront as a #ay of improving the 5uality of urban life and attracting '! are looking at their #aterfronts as a #ay they can improve the 5uality of urban life and attract )41. With 6ust several 5uick strokes of the pen% the monkeys #ere dra#n by the artist% capturing

'77

their antics. A! the monkeys #ere dra#n by the artist% capturing their antics "! the artist sketched the monkeys% capturing their antics $! the artist captured the antics of the monkeys% sketching them &! the artist sketched the monkeys and also capturing their antics '! the monkeys and heir antics #ere sketched by the artist )4(. With only 7 percent of the #orld3s population% +nited ,tates citi9ens consume (0 percent of its nonrene#able resources% drive more than one-third of its automobiles% and use (1 times more #ater per capita than 'uropeans do. A! With "! As $! "eing &! &espite having '! Although accounting for )4/. With total sales of less than three hundred thousand dollars and fe#er ne# subscribers than last year% the Ne# 'ngland =heatre $ompany is in danger of losing its building. A! of less than three hundred thousand dollars and fe#er "! lo#er than three hundred thousand dollars and less $! lesser than three hundred thousand dollars and fe#er &! fe#er than three hundred thousand dollars and less '! of fe#er than three hundred thousand dollars and of fe#er )44. Within the boundaries of artistic rivalry lies a sense of familyA the shared genetic inheritance% or accident% that enables musicians to make music. A! Within the boundaries of artistic rivalry lies a sense of family "! Within the boundaries of artistic rivalry lays a sense of family $! A sense of family lies inside of the boundaries of artistic rivalry &! A family sense lies #ithin artistic rivalry3s boundaries '! Within artistic rivalry3s boundaries lays a family sense )47. Without hearing a #ord of #hat is being said or shouted% an e<perienced trader on the floor of the stock e<change can listen to the hum of voices around them and tell #hat is happening. A! Without hearing a #ord of #hat is being said or shouted% an e<perienced trader "! Without hearing a #ord of #hat is being said or shouted% e<perienced traders $! 'ven though the person has not heard a #ord of #hat is being said or shouted% an e<perienced trader &! 'ven #hen the person has not heard a #ord that is being said or shouted% e<perienced traders

SC Strategy

'78

'! In spite of not hearing a #ord of #hat is being said or shouted% an e<perienced trader )4:. Wynton and "ranford 8arsalis% brothers #ho have fused the comple< rhythms of contemporary 6a99 #ith the rollicking musical legacy of their hometo#n% are fitting symbols of the Ne# Grleans 6a99 revival. A! Wynton and "ranford 8arsalis% brothers #ho have fused the comple< rhythms of contemporary 6a99 #ith the rollicking musical legacy of their hometo#n% are fitting symbols of the Ne# Grleans 6a99 revival. "! Wynton and "ranford 8arsalis are brothers% have fused the comple< rhythms of contemporary 6a99 #ith the rollicking musical legacy of their hometo#n% and are fitting symbols of the Ne# Grleans 6a99 revival. $! =he 8arsalis brothers% Wynton and "ranford% have fused the comple< rhythms of contemporary 6a99 #ith the rollicking musical legacy of their hometo#n% #hich is a fitting symbol of the Ne# Grleans 6a99 revival. &! ?using the rhythms of contemporary 6a99% #hich are comple<% #ith the rollicking musical legacy of their hometo#n% Wynton and "ranford 8arsalis are a fitting symbol of the Ne# Grleans 6a99 revival. '! A fitting symbol of the Ne# Grleans 6a99 revival are brothers Wynton and "ranford 8arsalis% #ho fuse the comple< rhythms of contemporary 6a99 #ith the rollicking musical legacy of their hometo#n. )4*. @oung female ballet dancers and gymnasts sometimes fail to maintain good eating habits caused by the desire to be as thin as possible. A! @oung female ballet dancers and gymnasts sometimes fail to maintain good eating habits caused by the desire to be as thin as possible. "! -ood eating habits sometimes fail to be maintained by young female ballet dancers and gymnasts caused by desiring to be as thin as possible. $! "ecause they desire to be as thin as possible% good eating habits are sometimes not maintained by young female ballet dancers and gymnasts. &! "ecause they desire to be as thin as possible% young female ballet dancers and gymnasts sometimes fail to maintain good eating habits. '! @oung female ballet dancers and gymnasts sometimes fail to maintain good eating habits because they desire to be as thin as possible. )40. A discussion of our nation3s foreign policy must begin #ith the fact of there being an independent Western 'urope #hich no# thinks of itself in trans-nationalist terms. A! A discussion of our nation3s foreign policy must begin #ith the fact of there being "! "eginning any discussion of our nation3s foreign policy must be the fact of there being $! Any discussion of our nation3s foreign policy must begin #ith the fact that there is &! Any discussion of our nation3s foreign policy must begin by ackno#ledging the e<istence of

'79

'! =o begin discussing our nation3s foreign policy thee must be an ackno#ledgment of the fact that )4). A large and increasingly influential sector of publishing% (2 percent of all the National and American "ook a#ards since 1)72 have gone to university-press books. A! A large and increasingly influential sector of publishing% (2 percent of all the National and American "ook a#ards since 1)72 have gone to university-press books. "! A large and increasingly influential sector of publishing% university-press books have #on (2 percent of all the National and American "ook a#ards since 1)72. $! Increasingly influential as a large sector of publishing% (2 percent of all the National and American "ook a#ards since 1)72 have gone to university-press books. &! ,ince 1)72% a large and increasingly influential sector of publishing% (2 percent of all the National and American "ook a#ards have gone to university-press books. '! ,ince 1)72% university-press books% a large and increasingly influential sector of publishing% #on (2 percent of all the National and American "ook a#ards from then on. )72. A private house in Ne# @ork $ity is a building o#ned by an individual or individuals having less than eight units and no commercial space. A! a building o#ned by an individual or individuals having less than eight units and no commercial space "! one that an individual or individuals o#n #ith fe#er than eight units and no commercial space $! a building #ith fe#er than eight units% no commercial space% and is o#ned by an individual or individuals &! one that has fe#er than eight units% no commercial space and it is o#ned by an individual or individuals '! one that has fe#er than eight units% is o#ned by an individual or individuals% and has no commercial space )71. A substance from the licorice plant% 72 times s#eeter than sucrose% #as recently discovered% is not only a natural s#eetener but also prevents tooth decay. A! A substance from the licorice plant% 72 times s#eeter than sucrose% #as recently discovered "! A substance% #hich #as recently discovered% from the licorice plant% 72 times s#eeter than sucrose% $! A substance from the licorice plant% #hich #as recently discovered to be 72 times s#eeter than sucrose% &! A substance from the licorice plant% 72 times s#eeter than sucrose % #hich #as recently discovered% '! A recently discovered substance% 72 times s#eeter than sucrose from the licorice plant% )7(. After more than four decades of research and development% a ne# type of 6et engine is being tested that could eventually propel aircraft any#here in the #orld #ithin t#o hours or help boost

SC Strategy

'7;

cargoes into space at significantly lo#er costs than current methods permit. A! tested that could eventually propel aircraft any#here in the #orld #ithin t#o or help "! tested that could eventually have the capability of propelling aircraft any#here in the #orld #ithin t#o hours or to help $! tested% eventually able to propel aircraft any#here in the #orld #ithin t#o hours% or helping &! tested% and it eventually could propel aircraft any#here in the #orld #ithin t#o hours% or helping '! tested% and it could eventually have the capability to propel aircraft any#here in the #orld #ithin t#o hours or help )7/. Although no proof yet e<ists of electromagnetic fields generated by household appliances posing any health threat% mounting scientific evidence has convinced many e<perts that there is cause for concern. A! of electromagnetic fields generated by household appliances posing any health threat "! of electromagnetic fields generated by household appliances that pose any threat to health $! that electromagnetic fields generated by household appliances pose any threat to health &! that poses any threat to health from electromagnetic fields generated by household appliances '! for any health threat posed by electromagnetic fields generated by household appliances )74. Art historians are using a process kno#n as infrared scanning in analy9ing the 8ona .isa to determine if it has been altered since completion and if .eonardo da Binci first sketched the figure in black% as done by many artists of the time. A! if it has been altered since completion and if .eonardo da Binci first sketched the figure in black% as done "! if it had been altered since completion and if .eonardo da Binci first sketched the figure in black% a practice employed $! #hether it has been altered since completion and #hether .eonardo da Binci first sketched the figure in black% a practice employed &! #hether it #as altered since completion and #hether .eonardo da Binci first sketched the figure in black% as #as done '! #hether it had been altered since completion and #hether .eonardo da Binci first sketched the figure in black% a practice done )77. Asset allocators create portfolios% often in the form of mutual funds% #ith the intention to turn in good results in both bull and bear markets. A! #ith the intention "! the intention of #hich is $! intended &! and intending

'7I

'! so intended as )7:. "aker #as perhaps not the most gifted soloist in the orchestra% but the conductor felt #hat #as lacking in his technical skill #as more than made up by the passion #ith #hich he played the music. A! #hat #as lacking in his technical skill #as more than made up by "! #hat he lacked in technical skill #as more than made up by $! #hatever #as lacking in his technical skill #as more than made up by &! #hatever he lacked in technical skill #as more than made up for by '! #hatever he lacked in technical skill he more than made up by )7*. "y la#% a 5ualified physician can only prescribe medicine% protecting the public. A! "y la#% a 5ualified physician can only prescribe medicine% protecting the public. "! "y la#% only a 5ualified physician can prescribe medicine% protecting the public. $! "y la#% only a 5ualified physician can prescribe medicine #hich protects the public. &! In order to protect the public% by la# a 5ualified physician only can prescribe medicine. '! In order to protect the public% by la# only a 5ualified physician can prescribe medicine. )70. $artographers have long struggled #ith the problem of having the spherical 'arth to dra# on a flat sheet of paper. A! having the spherical 'arth to dra# on a flat sheet of paper "! having a flat sheet of paper on #hich to dra# the spherical 'arth $! ho# can one dra# the spherical 'arth on a flat sheet of paper &! ho# they could use a flat sheet of paper to dra# the spherical 'arth '! ho# to dra# the spherical 'arth on a flat sheet of paper )7). $hild care already a solid part of the employee benefits package at many companies% more businesses are focusing on a ne#er family benefit kno#n as elder care% servicing for older dependents. A! $hild care "! With child care $! $hild care as &! "eing the $hild care #as '! With child care3s being ):2. $onstance >orner% chief of the +nited ,tates government3s personnel agency% has recommended that the use of any dangerous or illegal drug in the five years prior to application for a 6ob be grounds for not hiring an applicant. A! the use of any dangerous or illegal drug in the five years prior to application for a 6ob be grounds for not hiring an applicant

SC Strategy

'7J

"! any dangerous or illegal drug% if used in the five years prior to applying for a 6ob% should be grounds for not hiring an applicant $! an applicant3s use of any dangerous or illegal drug in the five years prior to application for a 6ob be grounds not to hire them &! an applicant3s use of any dangerous or illegal drug in the five years prior to applying for a 6ob are grounds that they not be hired '! for five years prior to applying for a 6ob% an applicant3s use of any dangerous or illegal drug be grounds for not hiring them ):1. $onsumer confidence levels% #hich many economists consider an early indication of the economy3s direction% sagged as the stock market tumbled% but not dramatically enough for giving a clear picture of ne# spending patterns. A! tumbled% but not dramatically enough for giving "! tumbled% but not dramatically enough to give $! tumbled% and not so dramatically as to have given &! has tumbled% and not dramatically enough to give '! has tumbled% but not so dramatically as to give ):(. $ontrary to the scholarly #isdom of the 1)723s and early 1):23s that predicted the processes of moderni9ation and rationali9ation #ould gradually undermine it% ethnicity is a #orld#ide phenomenon of increasing importance. A! #ould gradually undermine it "! to be a gradual undermining of it $! #ould be a gradual undermining of ethnicity &! to gradually undermine ethnicity '! gradually undermining it

worked it out as &elow: Contrary to the scholarly wisdom of the $J8E3s and earl $J9E3s that predicted the processes of "oderni0ation and rationali0ation would graduall under"ine it# ethnicit is a worldwide pheno"enon of increasing i"portance. 2$ that predicted the processes $$$ wouldDtoDgraduallyE As so"ething is KpredictedK in past LM KwouldK is re+uire to refer to future fro" past 6$ undermine vs$ undermining As so"ething is KpredicatedK so it will Kunder"ineK % Kunder"iningK i"plies that Kunder"iningK alread stated and is in progress

'8E

/A5 would graduall under"ine it /B5 to &e a gradual under"ining of it /,5 would &e a gradual under"ining of ethnicit % graduall under"ine it /65 to graduall under"ine ethnicit /E5 /"issing !er&5 graduall under"ining it

):/.

?or people #ho have never #orked for a living% any 6ob may instill a valuable sense of self#orth and open doors to better 6obs in the future. A! may instill a valuable sense of self-#orth and open doors to better 6obs in the future "! might instill for them a valuable sense of self- #orth and to open doors to better 6obs in the future $! may% in them% instill a valuable sense of self-#orth% opening their doors to better 6obs in the future &! opening the door later for a better 6ob and giving them a valuable sense of self-#orth no# '! may open the door for a better 6ob later and giving them a valuable sense no# of their self#orth

):4.

In a leveraged buyout% investors borro# huge sums of money to buy companies% hoping to pay off the debt by using the company3s earnings and to profit richly by the later resale of the companies or their divisions. A! by using the company3s earnings and to profit "! by using the companies3 earnings and by profiting $! using the companies3 earnings and profiting &! #ith the company3s earnings% profiting '! #ith the companies3 earnings and to profit

):7.

In many upper-class 'gyptian homes% ?rench #as spoken #ithin the family% 6ust as it had once been among the Dussian aristocracy. A! 6ust as it had once been among the Dussian aristocracy "! 6ust like it once been among the Dussian aristocracy $! 6ust as Dussian aristocracy had once done &! similar to #hat the Dussian aristocracy had once done '! like #hat had once been done by the Dussian aristocracy

)::.

In repousse% a method of sculpture% #orkers lay copper sheets over #ooden molds% and then% using a variety of e<otic hammers% carefully pounding the metal into shape. A! In repousse% a method of sculpture% #orkers lay copper sheets over #ooden molds% and then% using a variety of e<otic hammers% carefully pounding the metal into shape.

SC Strategy

'8$

"! Depousse is method of sculpture in #hich #orkers lay copper sheets over #ooden molds and then% using a variety of e<otic hammers% carefully pound the metal into shape. $! Depousse #orkers% #ho do a method of sculpture by laying copper sheets over #ooden molds% use a variety of e<otic hammers% and then pounding the metal into shape. &! Workers #ho lay copper sheets over #ooden molds use a variety of e<otic hammers and carefully pound the metal into shape in order to do repousse% a method of sculpture. '! .aying copper sheets over #ooden molds% and using a variety of e<otic hammers in order to pound the metal into shape% repousse is a method of sculpture done by #orkers. ):*. In the late nineteenth century Annie "esant #as #idely regarded as one of the greatest living public orators% second only to -ladstone in a culture #here oratory #as the dominant public medium. A! as one of the greatest living public orators% second only "! to be one of the greatest living public orators% secondary only $! that she #as one of the greatest living public orators% secondary only &! as being one of the greater living public orators she #as only second '! to be greater than most other living public orators% and she #as second only ):0. Interest rates on mortgages have declined steadily during the first si< months of this year but virtually remained unchanged during the ne<t three months. A! have declined steadily during the first si< months of this year but virtually remained unchanged "! declined steadily during the first si< months of this year but virtually remain unchanged $! steadily declined during the first si< months of this year but remain virtually unchanged &! declined steadily during the first si< months of this but have remained virtually unchanging3 '! declined steadily during the first si< months of this year but have remained virtually unchanged ):). .eaching% the recovery of copper from the drainage #ater of mines% as a method of the e<traction of minerals% it #as #ell established as early as the eighteenth century% but until about (7 years ago miners did not reali9e that bacteria taken an active part in the process. A! as a method of the e<traction of minerals% it #as #ell established "! as a method of the e<traction of minerals #ell established $! #as a #ell-established method of mineral e<traction &! #as a #ell-established method of e<tracting mineral that #as '! had been a method of mineral e<traction% #ell established )*2. .incoln% discovering in young manhood the secret that the @ankee peddler has learned before him% kne# ho# to use a good story to generate good #ill. A! .incoln% discovering in young manhood the secret that the @ankee peddler has learned before

'8'

him% kne# "! &iscovering in young manhood the secret that the @ankee peddler has learned before him% .incoln kne# $! .incoln% discovering the secret that the @ankee peddler had learned in young manhood before him% kne# &! In young manhood .incoln discovered the secret that the @ankee peddler had learned before himE '! .incoln% discovered in young manhood the secret that the @ankee peddler had learned before him% kne# )*1. 8any economists predict that the ne<t recession% #hen it comes% #ill be caused by ?ederal Deserve action taken to prevent an inflationary upsurge that #ould result if the economy #ere to e<pand at an annual rate of three percent or more. A! taken to prevent an inflationary upsurge that #ould result "! they took for preventing an inflationary upsurge that #ould result $! taken to prevent an inflationary upsurge resulting &! they took to prevent an inflationary upsurge resulting '! taken for preventing an inflationary upsurge that #ill result )*(. 8any environmentalists believe that the #idespread planting of trees% along #ith the conservation of e<isting forests% #ould be one of the surest% easiest% and least e<pensive #ays to begin to halt or even to reverse the buildup of carbon dio<ide in the air. A! one of the surest% easiest% and least e<pensive #ays to begin to halt or even to reverse "! one of the most sure% easy% and least e<pensive #ays to begin a halt or even reverse $! one of the surest% easiest% and least e<pensive #ays that #ould begin halting or even reversing &! a most sure% easy% and ine<pensive #ay beginning the halting and even reversing of '! the most sure% easiest% and ine<pensive #ay that #ould begin halting or even reversing )*/. No matter ho# patiently they e<plain their reasons for confiscating certain items% travelers often treat customs inspectors like #anton poachers rather than government employees. A! travelers often treat customs inspectors like #anton poachers rather than government employees "! travelers often treat customs inspectors as #anton poachers instead of government employees $! travelers often treat customs inspectors as if they #ere not government employees but #anton poachers &! customs inspectors are often treated by travelers as if they #ere #anton poachers rather than government employees '! customs inspectors are often treated not like government employees but #anton poachers by travelers

SC Strategy

'8*

)*4.

Gne noted economist has made a comparison of the ?ederal Deserve and an automobile as racing through a tunnel% bouncing first off one #all% then the otherA the car may get #here it is going% but people may be hurt in the process. A! made a comparison of the ?ederal Deserve and an automobile as racing through a tunnel% bouncing "! made a comparison bet#een the ?ederal Deserve and an automobile racing through a tunnel% bouncing $! compared the federal Deserve #ith an automobile as racing through a tunnel and #hich bounced &! compared the ?ederal Deserve to an automobile racing through a tunnel% bouncing '! compared the ?ederal Deserve #ith an automobile that races through a tunnel and it bounces

)*7.

1uritan fanatics brought to civil and military affairs a coolness of 6udgment and mutability of purpose that some #riters have though inconsistent #ith their religious 9eal% but #hich #as in fact a natural outgro#th of it. A! but #hich #as in fact a natural outgro#th of it "! but #hich #ere in fact a natural outgro#th of it $! but #hich #ere in fact natural outgro#ths of it &! but it #as in fact a natural outgro#th of them '! #hich #as in fact a natural outgro#th of it

)*:.

,eeking to spur science education% elementary and secondary schools #ill receive I(7 million over the ne<t ten years from the National ,cience ?oundation to promote science. A! elementary and secondary schools #ill receive I(7 million over the ne<t ten years from the National ,cience ?oundation to promote science "! I(7 million #ill be spent by the National ,cience ?oundation over the ne<t ten years to promote science in elementary and secondary schools $! over the ne<t ten years I(7 million #ill be spent on elementary and secondary schools by the National ,cience ?oundation for promoting science &! the National ,cience ?oundation is to spend I(7 million over the ne<t ten years for promoting science in elementary and secondary schools '! the National ,cience ?oundation #ill spend I(7 million over the ne<t ten years to promote science in elementary and secondary schools

)**.

,ome archaeologists claim that the tablets found at 'bla could force a revision of current theories on the origins of Cudaism and $hristianity% alter many scriptural interpretations% make all current "ible translations obsolete% and scholars may be re5uired to credit the Gld =estament #ith greater historical accuracy. A! scholars may be re5uired to credit the Gld =estament #ith greater historical accuracy "! crediting the Gld =estament #ith greater historical accuracy may be re5uired of scholars

'87

$! re5uire that scholars are to credit the Gld =estament #ith greater historical accuracy &! crediting the Gld =estament #ith greater historical accuracy may be a scholarly re5uirement '! re5uire scholars to credit the Gld =estament #ith greater historical accuracy )*0. =ektites% #hich may have been propelled to 'arth from lunar volcanoes% are much like the volcanic glass obsidian% but their chemical composition is different than any terrestrial lavaE they contain far less #ater than obsidian does and none of its characteristic microcrystals. A! is different than any terrestrial lavaE they contain "! is different than any terrestrial lava3s% containing $! is different from that of any terrestrial lavaE they contain &! differs from any terrestrial lava in containing '! differs from that of any terrestrial lava3s% containing )*). =he failing of the book lies not in a lack of attention to scientific detail but in the depiction of scenes of life and death in the marine #orld #ith emotional overtones that reduce the credibility of the #ork. A! the depiction of scenes of life and death in the marine #orld #ith emotional overtones that "! fact that it depicts marine #orld scenes of life and death as having emotional overtones that $! depiction of scenes of life and death in the marine #orld% #hose emotional overtones &! depiction of marine #orld scenes of life and death% #hich have emotional overtones and thus '! fact that if depicts scenes of life and death in the marine #orld% #hose emotional overtones )02. =he Immigration ,ervice no# has the discretionary po#er to keep families united even though all their members do not meet the five-year residency re5uirement. A! all their members do not meet the five-year residency re5uirement "! not all their members meet the five-year residency re5uirement $! all their members have not met the re5uirement for a five-year residency &! not all their members have resided for five years% a re5uirement '! all their members have not resided for five years% as re5uired )01. =he .ake 8anyara 1ark in =an9ania affords the visitor #ith une5ualled opportunities to photograph lions playing in trees #ithout the aid of telephoto lenses. A! =he .ake 8anyara 1ark in =an9ania affords the visitor #ith une5ualled opportunities to photograph lions playing in trees #ithout the aid of telephoto lenses. "! =he .ake 8anyara 1ark in =an9ania permits the visitor une5ualled opportunities to photograph lions playing in trees #ithout the aid of telephoto lenses. $! =he .ake 8anyara 1ark in =an9ania gives the visitor the une5ualled opportunity to photograph lions playing in trees #ithout telephoto lenses. &! =he visitor to the .ake 8anyara 1ark in =an9ania has the une5ualled opportunity to

SC Strategy

'88

photograph lions playing in trees #ithout the aid of telephoto lenses. '! 'ven #ithout the aid of telephoto lenses% the visitor to =an9ania3s .ake 8anyara 1ark has an une5ualled opportunity to photograph lions playing in trees. )0(. =he rise in the price of crabmeat and an increase in demand has convinced some ?loridians they should try to harvest and sell a species of large crab that lives deep in the #aters of the -ulf of 8e<ico. A! has convinced some ?loridians they should try to harvest and sell a species of large crab "! has convinced some ?loridians to try harvesting and selling a large crab species $! has convinced some ?loridians that they should try the harvest and sale of a large crab species &! have convinced some ?loridians to try to harvest and sell a species of large crab '! have convinced some ?loridians to try and harvest and sell a large crab species )0/. =he skill and the precision of the Anasa9i% ancient inhabitants of the ,outh#est% in measuring the movements of the ,un and 8oon is evidenced not only at $haco $anyon but at a number of other sites. A! in measuring the movements of the ,un and 8oon is evidenced not only at "! in measuring the movements of the ,un and 8oon are evidenced not only at $! in measuring the movements of the ,un and 8oon is evidenced at not only &! to measure the movements of the ,un and 8oon is evidenced at not only '! to measure the movements of the ,un and 8oon are evidenced not only at )04. =here is ample evidence% derived from the lore of traditional folk medicine% that naturally occurring antibiotics are usually able to be modified to make them a more effective drug. A! are usually able to be modified to make them a more effective drug "! are usually able to be modified to make them more effective drugs $! are usually able to be modified% #hich makes them more effective drug &! can usually be modified to make them a more effective drug '! can usually be modified to make them more effective drugs )07. =hose #ho have visited the -rand $anyon have typically seen layers of sediment in the gaping canyon% #ith different colors that mark the passage of time like the rings in a tree trunk. A! seen layers of sediment in the gaping canyon% #ith different colors that mark "! see layers of sediment in the gaping canyon% #hose different colors mark $! been seeing layers of sediment in the gaping canyon% #hose different colors are markers of &! been able to see layers of sediment in the gaping canyon% #ith different colors marking '! seen layers of sediment in the gaping canyon% marking by different colors
( am not sure***

'89

4 " , the answer given in 2FFFSCGs collection is wrong$ B, C " incorrect " no clear antecedent for pronoun &hose ' " incorrect " wordy " been able to see + " incorrect " unclear sub!ect for mar9ing " layers or the Canyon that is mar'in( the passa(e of time)

)0:.

=urkey3s economy has gro#n prodigiously over the past ten years% averaging an annual increase each year that is about eight percentHone of the highest gro#th rates in the #orld. A! averaging an annual increase each year that is about eight percent "! #ith an annual average each year #hich is about eight percent $! eight percent is #hat it averages out to each year &! #ith an average annual increase of about eight percent '! the average of the annual increase each year is about eight percent

)0*.

With cloning technology% scientists are approaching #hat has long been the ultimate goal of modern husbandryA achieving a consistency of 5uality and production in farm animals as once thought to be limited to manufactured goods. A! achieving a consistency of 5uality and production in farm animals as once thought to be "! achieving farm animals #ith a consistency of 5uality and production as #ere once thought of as $! achieving in farm animals a consistency of 5uality and production that #as once thought to be &! achievement of farm animals #hose consistency of 5uality and production are the same as #hat #ere once thought to he '! achievement of farm animals at a consistency of 5uality and production once thought of as

)00. =om "radley #as mayor of .os Angeles from 1)*/ to 1))/% an era #hen the city had transformed from a collection of suburban neighborhoods to the second-largest city in the +nited ,tates. A! an era #hen the city had transformed "! an era during #hich the city #as transformed $! an era that transformed it &! during #hich era the city transformed '! during #hich the city #as transformed

)0). =he results of the company3s cost-cutting measures are evident in its profits% #hich increased five percent during the first three months of this year after it fell over the last t#o years.

SC Strategy

'8;

A! #hich increased five percent during the first three months of this year after it fell "! #hich had increased five percent during the first three months of this year after it had fallen $! #hich have increased five percent during the first three months of this year after falling &! #ith a five percent increase during the first three months of this year after falling '! #ith a five percent increase during the first three months of this year after having fallen

))2 1art of the proposed increase in state education spending is due to higher enrollment% since the number of students in public schools have gro#n steadily since the mid-1)023s and% at nearly 4* million% are at a record high. A! enrollment% since the number of students in public schools have gro#n steadily since the mid1)023s and% at nearly 4* million% are at "! enrollment% #ith a number of students in public schools gro#ing steadily since the mid-1)023s and% at nearly 4* million% reaching $! enrollmentA since students in public schools have gro#n steadily in number since the mid-1)023s and% at nearly 4* million% have reached &! enrollmentA the number of student in public schools has gro#n steadily since the mid-1)023s and% at nearly 4* million% has reached '! enrollmentA students in public schools have gro#n steadily in number% since the mid-1)023s and% at nearly 4* million% are at

))1. Bivien =homas% #ho had no formal medical training% in struggling against over#helming odds% he became a cardiac surgeon and eventually to receive an honorary doctorate from Cohns >opkins +niversity. A! #ho had no formal medical training% in struggling against over#helming odds% he became "! having had no formal medical training% in struggling against over#helming odds to become $! #ho% having no formal medical training% he struggled against over#helming odds in becoming &! #ho% having had no formal medical training and struggled against over#helming odds% became '! #ho had no formal medical training% struggled against over#helming odds to become

'8I

))(. =he yield of natural gas from Nor#ay3s =roil gas field is e<pected to increase annually until the year (227 and then to stabili9e at si< billion cubic feet a day% #hich #ill allo# such an e<traction rate at least for 72 years3 production. A! (227 and then to stabili9e at si< billion cubic feet a day% #hich #ill allo# such an e<traction rate at least for "! (227 and then to stabili9e at si< billion cubic feet a day% an e<traction rate that #ill allo# at least $! (227 and then stabili9ing at si< billion cubic feet a day% #ith such an e<traction rate at the least allo#ing &! (227% then stabili9ing at si< billion cubic feet a day% allo#ing such an e<traction rate for at least '! (227% then stabili9ing at si< billion cubic feet a day% #hich #ill allo# such an e<traction rate for at least

))/ In archaeology% there must be a balance bet#een e<planation of the value and #orkings of archaeology% revealing the mysteries of past and present cultures% and to promote respect for archaeological sites. A! bet#een e<planation of the value and #orkings of archaeology% revealing the mysteries of past and present cultures% and to promote "! among e<plaining the value and #orkings of archaeology% revealing the mysteries of past and present cultures% and promoting $! bet#een e<plaining the value and #orkings of archaeology% revealing the mysteries of past and present cultures% and #hen promoting &! among e<plaining the value and #orkings of archaeology% the revelation of the mysteries of past and present cultures% and to promote '! bet#een e<plaining archaeology3s value and #orkings% in the revealing the mysteries of past and present cultures% and in promoting

))4. According to a 1)): survey by the National Association of $ollege an +niversity "usiness Gfficers% more than three times as many independent institutions of higher education charge tuition and fees of under I0%222 a year than those that charge over I1:%222. A! than those that charge "! than are charging

SC Strategy

'8J

$! than to charge &! as charge '! as those charging

))7. Nobody kno#s e<actly ho# many languages there are in the #orld% partly because of the difficulty ofJ ! distinguishing bet#een a language and the sub-languages or dialects #ithin it% but those #ho have tried to count typically have found about five thousand. A! and the sub-languages or dialects #ithin it% but those #ho have tried to count typically have found "! and the sub-languages or dialects #ithin them% #ith those #ho have tried counting typically finding $! and the sub-languages or dialects #ithin it% but those #ho have tried counting it typically find &! or the sub-languages or dialects #ithin them% but those #ho tried to count them typically found '! or the sub-languages or dialects #ithin them% #ith those #ho have tried to count typically finding

)):. +nlike most severance packages% #hich re5uire #orkers to stay until the last day scheduled to collect% #orkers at the automobile company are eligible for its severance package even if they find a ne# 6ob before they are terminated. A! the last day scheduled to collect% #orkers at the automobile company are eligible for its severance package "! the last day they are scheduled to collect% #orkers are eligible for it at the automobile company3s severance package $! their last scheduled day to collect% the automobile company offers its severance package to #orkers. &! their last scheduled day in order to collect% the automobile company3s severance package is available to #orkers. '! the last day that they are scheduled to collect% the automobile company3s severance package is available to #orkers.

))* ,tudying the fruit fly% a household nuisance but a time-honored e<perimental sub6ect% has enabled the secrets of ho# embryos develop to begin to be unraveled by scientists.

'9E

A! ,tudying the fruit fly% a household nuisance but a time-honored e<perimental sub6ect% has enabled the secrets of ho# embryos develop to begin to be unraveled by scientists. "! "y the study of the fruit fly% a household nuisance and also a time-honored e<perimental sub6ect% it #as possible for the secrets of ho# embryos develop to begin to be unraveled by scientists. $! "y studying a household nuisance but a time-honored e<perimental sub6ect% the fruit fly enabled scientist to begin to unravel the secrets of ho# embryos develop. &! "y studying the fruit fly% a household nuisance and also a time-honored e<perimental sub6ect% the secrets of ho# embryos develop are beginning to be unraveled by scientist. '! =he study of the fruit fly% a household nuisance but a time-honored e<perimental sub6ect% has enabled scientist to begin to unravel the secrets of ho# embryos develop.

))0A =he Industrial Devolution% making it possible to mass-produce manufactured goods% #as marked by their use of ne# machines% ne# energy sources% and ne# basic materials. A! making it possible to mass-produce manufactured goods% #as marked by their use of "! making possible the mass production of manufactured goods% marked by the use of $! #hich made it possible that manufactured goods #ere mass-produced% #as marked by their using &! #hich made possible the mass-production of manufactured goods% #as marked by the use of '! #hich made the mass production of manufactured goods possible and #as marked by using

))). In Canuary 1))4 an oil barge ran aground off the coast of ,an Cuan% 1uerto Dico% leaking its cargo of *72222 gallons into the ocean% #hile causing the pollution of the city3s beaches. A! leaking its cargo of *72222 gallons into the ocean% #hile causing the pollution of "! #ith its cargo of its cargo of *72222 gallons leaking into the ocean% and it polluted $! and its cargo of its cargo of *72222 gallons leaked into the ocean% polluting &! #hile it leaked its cargo of *72222 gallons into the ocean and caused the pollution of '! so that its cargo of *72222 gallons leaked into the ocean% and they #ere polluting

1222. Capan3s abundant rainfall and the typically mild temperature throughout most of the country

SC Strategy

'9$

have produced a lush vegetation cover and% despite the mountainous terrain and generally poor soils% it has made possible the raising of a variety of crops. A! it has made possible the raising of "! has made possible fro them to raise $! have made it possible to raise &! have made it possible for raising '! thus making it possible for them to raise

1221. ;0A A ne# hair-gro#ing drug is being sold for three times the price% per milligram% as the drug3s maker charges for another product #ith the same active ingredient. A. as ". than $. that &. of #hat '. at #hich

122(. ;12A Authoritative parents are more likely than permissive parents to have children #ho as adolescents are self-confident% high in self-esteem% and responsibly independent.

A. Authoritative parents are more likely than permissive parents to have children #ho as adolescents are self-confident% high in self-esteem% and responsibly independent. ". Authoritative parents #ho are more likely than permissive parents to have adolescent children that are self-confident% high in self-esteem% and responsibly

'9'

independent. $. $hildren of authoritative parents% rather than permissive parents% are the more likely to be self-confident% have a high self-esteem% and to be responsibly independent as adolescents. &. $hildren #hose parents are authoritative rather than being permissive% are more likely to have self-confidence% a high self-esteem% and be responsibly independent #hen they are an adolescent. '. Dather than permissive parents% the children of authoritative parents are the more likely to have self-confidence% a high self-esteem% and to be responsibly independent as an adolescent.

122/. ;1(A A study by the Gcean Wildlife $ampaign urged states to undertake a number of remedies to reverse a decline in the shark population% #hich includes the establishment of si9e limits for shark catches% closing state #aters for shark fishing during pupping season% and re5uiring commercial fishers to have federal shark permits.

A. #hich includes the establishment of si9e limits for shark catches% closing ". #hich includes establishing limits to the si9e of sharks that can be caught% closing $. #hich include the establishment of si9e limits for shark catches% the closing of &. including establishing si9e limits for shark catches% closing '. including the establishing of limits to the si9e of sharks that are caught% the closing of

1224. $oncerns about public health led to the construction bet#een 10*: and 1)24 of three separate se#er systems to serve metropolitan "oston.

SC Strategy

'9*

A. $oncerns about public health led to the construction bet#een 10*: and 1)24 of three separate se#er systems to serve ". $oncerns about public health have led to the construction of three separate se#er systems bet#een 10*: and 1)24 to serve $. $oncerns about public health have led bet#een 10*: and 1)24 to the construction of three separate se#er systems for serving &. =here #ere concerns about public health leading to the construction bet#een 10*: and 1)24 of three separate se#er systems serving '. =here #ere concerns leading bet#een 10*: and 1)24 to the construction of three separate se#er systems for serving

1227. &espite the gro#ing number of people #ho purchase plane tickets online% airline e<ecutives are convinced that% 6ust as one-third of bank customers still prefer human tellers to automatic teller machines% many travelers #ill still use travel agents.

A. gro#ing number of people #ho purchase plane tickets online% airline e<ecutives are convinced that% 6ust as one-third of bank customers still prefer human tellers to automatic teller machines% many travelers #ill ". gro#ing number of people #ho purchase plane tickets online% airline e<ecutives are convinced% 6ust as one-third of bank customers still prefer human tellers to automatic teller machines% that many travelers #ould $. gro#ing number of people purchasing plane tickets online% airline e<ecutives are convinced% 6ust as one-third of bank customers still prefer human tellers as compared to automatic teller machines% many travelers #ill &. fact that the number of people purchasing plane tickets online is gro#ing% airline

'97

e<ecutives are convinced% 6ust as one-third of bank customers still prefer human tellers as compared to automatic teller machines% that many travelers #ould '. fact that the number of people #ho purchase plane tickets online are gro#ing% airline e<ecutives are convinced that% 6ust as one-third of bank customers still prefer human tellers compared #ith automatic teller machines% many travelers #ould

122:. ?ossils of the arm of a sloth found in 1uerto Dico in 1))1% and dated at /4 million years old% made it the earliest kno#n mammal of the -reater Antilles islands.

A. sloth found in 1uerto Dico in 1))1% and dated at /4 million years old% made it the earliest kno#n mammal of ". sloth% that they found in 1uerto Dico in 1))1% has been dated at /4 million years old% thus making it the earliest mammal kno#n on $. sloth that #as found in 1uerto Dico in 1))1% #as dated at /4 million years old% making this the earliest kno#n mammal of &. sloth% found in 1uerto Dico in 1))1% have been dated at /4 million years old% making the sloth the earliest kno#n mammal on '. sloth #hich% found in 1uerto Dico in 1))1% #as dated at /4 million years old% made the sloth the earliest kno#n mammal of

122*. In April 1))*% >illary Dodham $linton hosted an all-day White >ouse scientific conference on ne# findings that indicates a child3s ac5uiring language% thinking% and emotional skills as an active process that may be largely completed before age three.

A. that indicates a child3s ac5uiring language% thinking% and emotional skills as

SC Strategy

'98

". that are indicative of a child ac5uiring language% thinking% and emotional skills as $. to indicate that #hen a child ac5uires language% thinking% and emotional skills% that it is &. indicating that a child3s ac5uisition of language% thinking% and emotional skills is '. indicative of a child3s ac5uisition of language% thinking% and emotional skills as

Você também pode gostar